You are on page 1of 397

ĐỀ SỐ 01 ĐỀ THI THỬ TỐT NGHIỆP THPT

NĂM HỌC: 2020 – 2021


MÔN: TIẾNG ANH
Thời gian làm bài: 60 phút; không kể thời gian phát đề

Mark the letter A, B, C, or D on your answer sheet to indicate the word that differs from the
other three in the position of primary stress in each of the following questions.
Question 1. A. wither B. thesis C. thoughtful D. breath
Question 2. A. traffic B. configuration C. cinema D. camouflage
Mark the letter A, B, C, or D on your answer sheet to indicate the word whose underlined part
differs from the other three in pronunciation in each of the following questions.
Question 3. A. report B. conclude C. deter D. sanction
Question 4. A. advertise B. definite C. composite D. communicate
Mark the letter A, B, C or D on your answer sheet to indicate the correct answer to each of the
following questions.
Question 5. Tim and his friends have founded___________voluntary organisation which helps disabled
people with their transport needs.
A. a B. an C. the D. ∅
Question 6. Dana finally admitted___________by what her father had said the day before.
A. hurt B. hurting C. being hurt D. hurted
Question 7. The man was very brave. He___________, but he chose to stay and fight.
A. must have escaped B. escaped C. had escaped D. could have escaped
Question 8. Air pollution is getting___________serious in big cities such as Hanoi and Beijing.
A. the more and the more B. more and more
C. most and most D. the most and the most
Question 9. The company has made an announcement that any application___________ in after 30th
April shall not be considered.
A. sends B. is sent C. being sent D. sent
Question 10. Only after the teacher had explained the procedure clearly___________to go ahead with the
experiment.
A. the students were allowed B. were the students allowed
C. the students allowed D. did the students allow
Question 11. Her parents are really strict. They rarely let her stay out late, ___________ ?
A. do they B. don’t they C. does she D. doesn’t she
Question 12. Visitors to the local museum are mostly attracted by___________ rocking chair.
A. an old wooden European beautiful B. a beautiful old European wooden

Trang 1
C. an old beautiful wooden European D. a wooden old beautiful European
Question 13. Many astronomers never get back to the Earth because of___________accidents.
A. tragedy B. tragic C. tragically D. tragedies
Question 14. After years of training hard, the athlete finally could___________ her ambition of winning
an Olympic gold medal.
A. reject B. exacerbate C. recognise D. realise
Question 15. The commission estimates that at least seven companies took___________ of the program.
A. advantage B. use C. benefit D. dominance
Question 16. The cinema is no longer as popular as it was in the 1930 s and 1940’s, but it is still an
important___________of entertainment.
A. status B. source C. origin D. prospect
Question 17. After years of derision from the world, the Hindi film industry is achieving its
___________ in the Sun.
A. position B. image C. shade D. place
Question 18. While I was looking through my old albums the other day, I___________this photograph of
my parents’ wedding.
A. took after B. made up C. turned down D. came across
Mark the letter A, B, C, or D on your answer sheet to indicate the underlined part that needs
correction in each of the following questions.
Question 19. Of more than 1,300 volcanoes in the world, only about 600 can classify as active.
A. only B. classify C. volcanoes D. active
Question 20. In spite of the Indian lacked animals, they had the ability to cultivate plants suitable for
daily use.
A. suitable B. lacked animals C. in spite of D. had the ability
Question 21. A professor of economy and history at our university has developed a new theory of the
relationship between historical events and financial crises.
A. financial crises B. relationship C. historical D. economy
Mark the letter A, B, C or D on your answer sheet to indicate the sentence that is closest in meaning
to each of the following questions.
Question 22. John no longer drinks a lot.
A. John rarely drank a lot. B. John used to drink a lot.
C. John didn’t use to drink a lot. D. John now drinks a lot.
Question 23. “Stop smoking or you’ll be ill,” the doctor told me.
A. I was ordered not to smoke to recover from illness.
B. The doctor advised me to give up smoking to avoid illness.
C. The doctor suggested smoking to treat illness.

Trang 2
D. I was warned against smoking a lot of cigarettes.
Question 24. People think that increasing levels of carbon dioxide and other greenhouse gases will
inevitably lead to global warming.
A. Global warming is thought to result in increasing levels of carbon dioxide and other greenhouse
gases.
B. Increasing levels of carbon dioxide and other greenhouse gases is thought to be responsible for
global warming.
C. Increasing levels of carbon dioxide and other greenhouse gases is attributed to global warming.
D. Global warming is blamed for increasing levels of carbon dioxide and other greenhouse gases.

Mark the letter A, B, C or D on your answer sheet to indicate the sentence that best combines each
pair of sentences in the following questions.
Question 25. He was very tired. He agreed to help me with my homework.
A. As tired as was he, he agreed to help me with my homework.
B. Despite being very tired, but he agreed to help me with my homework
C. Tired though he was, but he agreed to help me with my homework.
D. Tired as he was, he agreed to help me with my homework.
Question 26. Mary quarreled with her boyfriend yesterday. She didn’t want to answer his phone
call this morning.
A. Mary’s refusal to answer the phone leads to her argument with her boyfriend yesterday.
B. Much as Mary quarreled with her boyfriend yesterday, she answered his phone call this morning.
C. Having quarreled with her boyfriend yesterday, Mary refused to answer his phone call this morning.
D. Before Mary quarrelled with her boyfriend yesterday, she answered his phone call.
Mark the letter A, B, C or D on your answer sheet to indicate the most suitable response to
complete each of the following exchanges.
Question 27. Sam and David are talking about what to do after work.
- “Do you fancy going to a movie this evening?” - “___________”
A. Not at all. Go ahead. B. I’m sorry. I don’t know that.
C. That would be nice. D. Not so bad. And you?
Question 28. Laura: “What a lovely house you have!”
A. Of course not, it’s not costly. B. Thank you. Hope you will drop in.
C. I think so. D. No problem.
Mark the letter A, B, C, or D on your answer sheet to indicate the word or phrase that is CLOSEST
in meaning to the underlined part in each of the following questions.
Question 29. When the Titanic crashed into the iceberg, the crew quickly sent out distress signals to alert
nearby ships of the disaster and request their help.

Trang 3
A. amusing B. strange C. bold D. help
Question 30. I’m all in favour of ambition but I think when he says he’ll be a millionaire by the time he’s
25, he’s simply crying for the moon.
A. longing for what is beyond the reach B. asking for what is attainable
C. doing something with vigor or intensity D. crying a lot and for a long time
Mark the letter A, B, C, or D on your answer sheet to indicate the word or phrase that is
OPPOSITE in meaning to the underlined part in each of the following questions.
Question 31. I prefer secure jobs because I don’t like keeping on moving and changing all the time.
A. challenging B. demanding C. stable D. safe
Question 32. When he passes the entrance exam, his parents will be walking on air.
A. extremely happy B. extremely light
C. feeling extremely unhappy D. feeling extremely airy
Read the following passage and mark the letter A, B, C or D on your answer sheet to indicate the
correct word for each of the blanks from 33 to 37.
Most adults struggle to recall events from their first few years of life and now scientists have identified
exactly when these childhood memories are lost forever. A new study into childhood amnesia has found
that it tends to take (33)___________around the age of seven.
The rapid decline of memories persisting while children are five and six is owing to the change in the way
memories are formed.
Before the age of seven, children tend to have an (34) ___________form of recall with no sense of time
and place in their memories. In older children, however, the early recollected events tend to be more adult
like in their content and the way they are formed. Faster rate of forgetting in children and higher turnover
of memories means early memories are less likely to survive. (35) ___________ , memories of younger
children tend to lack autobiographical narrative leading to a process known as “retrieval induced
forgetting” (36) ___________the action of remembering causes other information to be forgotten.
Consequently, if childhood memories can survive into the ninth or tenth year of life, they may stay a
chance of (37) ___________it into adulthood.
Question 33. A. affect B. effective C. effect D. effectively
Question 34. A. unbalanced B. immature C. insufficient D. irrational
Question 35. A. Besides B. However C. Therefore D. Otherwise
Question 36. A. whom B. which C. when D. where
Question 37. A. turning B. making C. transferring D. getting
Read the following passage and mark the letter A, B, C, or D on your answer sheet to indicate the
correct answer to each of the questions from 38 to 42.
Being repeatedly late may just be accidental - or could it show a deeply rooted psychological desire to
express your own superiority? when I worked in an office, meetings would often start late, usually

Trang 4
because of a certain individual. Then they would overrun and the whole day lost its shape. But the
individual was high-ranking and self-important: nobody challenged. So, what are the ethics of lateness?
There’s a psychotherapist called Irvin Yalom who argues that all behaviour reflects psychology. Just as
people who like to be on time are motivated by certain deep-seated beliefs, so those who make others wait
are acting out an inner agenda, often based on an acute sense of power. There’s famous footage in which
Silvio Berlusconi kept Angela Merkel waiting while he made a call on his mobile. It speaks volumes.
But that is when all lateness is in one’s control, what about when your train is cancelled or your flight is
delayed or you had to wait longer for the plumber to arrive? In such cases, there’s not a lot of psychology
involved. Or is there? Some people will genuinely worry about the impact it will have on those left
waiting, while others might secretly enjoy the power of their absence.
The essential fact is that lateness means breaking a convention - you can only be late in respect of a
time agreed with other people. Regardless of psychology, it has a social value. And when we treat other
people’s time as less valuable than our own, we treat them as inferior.
Question 38. What is the main idea of the passage?
A. Lateness indicates superiority of important individuals.
B. Social values of individuals are fostered through lateness.
C. Being late can reveal a lot about people’s psychology.
D. People with high sense of power have a tendency to be late.
Question 39. What is the “speaks volumes” in paragraph 2 closest in meaning to?
A. tells a lot B. makes sounds C. holds power D. talks loudly
Question 40. Which of the following is NOT TRUE according to the passage?
A. The way our minds work is responsible for the way we behave.
B. Some people control a situation for their benefit through lateness.
C. Lateness can have an impact on other people who are made to wait.
D. Most people react the same when lateness is out of their control.
Question 41. It can be inferred from the reading passage that___________.
A. Berlusconi made Merkel wait as he wanted to show his importance.
B. it is fashionable to be late to meetings because nobody challenges.
C. the writer’s daily schedule was unaffected by his colleague’s lateness.
D. nothing can be said about one’s psychology if lateness is out of control.
Question 42. What does the word “it” in paragraph 4 refer to?
A. fact B. lateness C. convention D. psychology
Read the following passage and mark the letter A, B, C, or D on your answer sheet to indicate the
correct answer to each of the questions front 43 to 50.
Homeopathy, the alternative therapy created in 1796 by Samuel Hahnemann, and now widely used all
over the world, is based on the belief that the body can be stimulated to heal itself. A central principle of

Trang 5
the “treatment” is that “like cures like”, meaning a substance that causes certain symptoms can also help
to remove those symptoms. Medicines used in homeopathy are created by heavily diluting in water the
substance in question and subsequently shaking the liquid vigorously. They can then be made into tablets
and pills. Practitioners believe that the more a substance is diluted in this way, the greater its power to
treat symptoms.
However, in a new study, a working committee of medical experts at Australia’s National Health and
Medical Research Council (NHMRC) has claimed that homeopathic medicines are only as effective as
placebos at treating illness. Their research, involving the analysis of numerous reports from homeopathy
interest groups and the public, concluded that there is no reliable evidence that homeopathy works.
Moreover, researchers uncovered no fewer than 68 ailments that homeopathic remedies had failed to
treat, including asthma, sleep disturbances, cold and flu, and arthritis.
As a result of the findings, the NHMRC is urging health workers to inform their patients to be wary of
anecdotal evidence that appears to support the effectiveness of homeopathic medicine. “It isn’t possible to
tell whether a health treatment is effective or not simply by considering the experience of one individual
or the beliefs of a health practitioner” says the report. Experts believe that most illnesses said to have been
cured by homeopathy would be cured by the body on its own without taking the medicine. Apparently,
many illnesses are short-lived by their very nature which often leads to people believing that it is the
homeopathy that cures them.
A more serious matter is highlighted by Professor John Dwyer of the University of New South Wales.
As an immunologist, he is concerned about homeopathic vaccinations on offer for diseases such as HIV,
tuberculosis, and malaria, none of which he considers effective. According to Professor John Dwyer, the
concept that homeopathic vaccinations are just as good as traditional vaccinations is delusion, and those
who believe it are failing to protect themselves and their children.
Question 43. Which could be the best title for the passage?
A. “Homeopathy kills” say scientists B. “Avoid homeopathy” say scientists
C. Homeopathy worth trying once D. Unknown effects of homeopathy
Question 44. According to the reading passage, homeopathic medicines are___________ .
A. prepared similarly for different diseases
B. made up of a variety of ingredients
C. suitable for a wide range of symptoms
D. available only in the liquid form
Question 45. What does the word “their” in paragraph 2 refer to?
A. committee B. NHMRC C. medicines D. placebos
Question 46. The Australian study reveals that homeopathy is___________.
A. increasingly popular with the public B. helpful in the case of respiratory infections
C. ineffective in treating many diseases D. of great interest to a certain group of people

Trang 6
Question 47. What is the word “wary” in paragraph 3 closest in meaning to?
A. apprehensive B. reluctant C. cautious D. observant
Question 48. People tend to believe in homeopathy because of___________.
A. advertisements displayed in health center
B. positive feedback from a small number of people
C. reliable proof from recent medical reports
D. results of research into some alternatives therapies
Question 49. What is the word “delusion” in paragraph 4 closest in meaning to?
A. legend B. falsehood C. imagination D. hallucination
Question 50. In Professor John Dwyer’s view, homeopathic vaccinations__________.
A. may bring about bad consequences B. do harm to people’s health
C. might be ridiculously expensive D. are better than nothing at all

Trang 7
Đáp án
1-A 2-C 3-D 4-D 5-A 6-C 7-D 8-B 9-D 10-B
11-A 12-B 13-B 14-D 15-A 16-B 17-D 18-D 19-B 20-C
21-D 22-B 23-B 24-B 25-D 26-C 27-C 28-B 29-D 30-A
31-A 32-C 33-C 34-B 35-A 36-D 37-B 38-C 39-A 40-D
41-A 42-B 43-D 44-A 45-B 46-C 47-C 48-B 49-B 50-A

LỜI GIẢI CHI TIẾT

Question 1: Đáp án A

Đáp án A đúng vì phần gạch chân của đáp án A là âm / ð /. Các đáp án còn lại có phần gạch

chân là âm /θ/.

A. wither /ˈwɪðə(r)/ (v): teo đi/ héo đi

B. thesis /ˈθiːsɪs / (n): luận văn/ luận án

C. thoughtful /ˈθɔːtfl/ (a): chu đáo/ biết nghĩ

D. breath /breθ/ (n): hơi thở

* Mở rộng: khi “th” bắt đầu từ mang nghĩa ngữ pháp thì phát âm là / ð /; ví dụ: they, though.

Khi “th” bắt đầu từ mang nghĩa từ vựng thì phát âm là /θ/; ví dụ: think, thought.

Question 2: Đáp án C

Đáp án C đúng vì phần gạch chân của đáp án C là âm /s/. Các đáp án còn lại có phần gạch chân

là âm /k/.

A. traffic /ˈtræfɪk/ (n)/ (v): giao thông (n)/ buôn bán người (v)

B. configuration /kənˌfɪɡəˈreɪʃn/ (n): cấu hình/ dạng

C. cinema /ˈsɪnəmə/ (n): rạp chiếu phim

D. camouflage /ˈkæməflɑːʒ/ (n)/(v): ngụy trang

Question 3: Đáp án D

Đáp án D đúng vì đáp án D có trọng âm rơi vào âm tiết thứ nhất. Các đáp án còn lại có trọng

âm rơi vào âm tiết thứ hai.

A. report /rɪˈpɔːt/ (v)/ (n): bản báo cáo (n)/ đưa tin (v)

B. conclude /kənˈkluːd/ (v): kết luận / conclude from sth that + clause: từ điều gì đó kết luận

được rằng...

C. deter /dɪˈtɜː(r)/ (v): ngăn cản/ làm nản lòng /deter sb from (doing) sth: ngăn cản ai làm việc gì

đó.
Trang 8
D. sanction /ˈsæŋkʃn/ (v)/ (n): sự chấp thuận, sự trừng phạt (n)/ chấp thuận, trừng phạt (v)

* Mở rộng: Từ hai âm tiết là động từ thì đa phần có trọng âm rơi vào âm tiết thứ hai. Từ kết

thúc bằng đuôi -tion/ -sion có trọng âm rơi vào âm tiết trước nó.

Question 4: Đáp án D

Đáp án D đúng vì đáp án D có trọng âm rơi vào âm tiết thứ hai. Các đáp án còn lại có trọng âm

rơi vào âm tiết thứ nhất.

A. advertise / 'ædvətaɪz/ (v): quảng cáo

B. definite /ˈdefɪnət/ (a): rõ ràng, dứt khoát, chắc chắn.

C. composite /ˈkɒmpəzɪt/ (a)/ (n): kết hợp từ nhiều chất (a)/ hợp chất (n).

D. communicate /kəˈmjuːnɪkeɪt/ (v): giao tiếp/ truyền đạt

Question 5: Đáp án A

Đáp án A – câu hỏi mạo từ

“Voluntary organisation” là danh từ đếm được số ít nên phải có mạo từ đi kèm, loại đáp án D.

Danh từ này được nhắc đến lần đầu nên chưa xác định, loại đáp án C. “Voluntary” bắt đầu

bằng phụ âm, nên chỉ chọn đáp án A.

Tạm dịch: Tim và bạn bè của anh ấy vừa thành lập một tổ chức để hỗ trợ những nhu cầu sử

dụng giao thông của những người khuyết tật.

* Mở rộng: Danh từ đếm được số ít luôn luôn đi kèm với một từ để xác định cho nó như: mạo

từ (a/ an/ the), chỉ định từ (this/ that), sở hữu (my/ your/ his/ Peter’s..)...

Question 6: Đáp án C

Đáp án C - câu hỏi V + V-ing

Động từ “admit” + V-ing: thú nhận việc gì đó. Vì “admit” đi với V-ing nên chỉ giữ lại hai đáp án

B - hurting và D - being hurt.

Động từ “hurt” có hai trường hợp dùng:

1/ “hurt” đứng một mình (nội động từ) - luôn để ở dạng chủ động - và có nghĩa là: cảm thấy

đau. Nghĩa này thường đi với chủ ngữ là các bộ phận cơ thể: My leg hurts (chân tôi cảm thấy

đau).

2/ “hurt” + sb/sth (ngoại động từ) - nếu không có sb/sth theo sau thì để bị động - và có nghĩa là:

làm đau/ gây đau đớn/ gây tổn thương cho ai đó/ cái gì đó.

Câu đề bài cho có “by what her father had said” là một dấu hiệu của bị động, và chủ ngữ ở đây

không phải là bộ phận cơ thể, nên đáp án phù hợp về cả ngữ pháp và ngữ nghĩa là C.

Trang 9
Tạm dịch: Cuối cùng thì Dana cũng thú nhận việc bị tổn thương bởi những gì bố cô ấy nói vào

ngày hôm trước.

* Mở rộng: Ngoại động từ luôn luôn có sb/ sth (tân ngữ) theo sau. Nếu không có hoặc thiếu,

động từ đó sẽ để ở dạng bị động.

Question 7: Đáp án D

Đáp án D - câu hỏi Modal verb + have + P2 (nguyên mẫu hoàn thành)

“Modal Verb + have + P2” được dùng để diễn tả một khả năng ở quá khứ.

“Must have + P2” diễn tả một khả năng chắc chắn xảy ra ở quá khứ, thường dùng để đoán chắc

chắn vì đã có căn cứ.

“Could have + P2” diễn tả một khả năng đáng lẽ có thể xảy ra nhưng không xảy ra, thường

dùng ở câu điều kiện loại III

Hai đáp án “escaped” và “had escaped” là động từ chia theo thì - dùng để kể lại chuyện có thật.

Đề bài cho hành động “escape” (bỏ trốn) không xảy ra, vì người đàn ông trong câu trên “chose

to stay” (chọn ở lại), vì thế nên chỉ có thể chọn đáp án D - could have escaped (đáng lẽ ra có thể

bỏ trốn nhưng không trốn). Ba đáp án còn lại đều diễn tả việc bỏ trốn có xảy ra.

Tạm dịch: Người đàn ông đó rất can đảm. Ông ấy đáng lẽ có thể bỏ trốn, nhưng đã chọn ở lại

và chiến đấu.

* Mở rộng: “could/ would/ should/ might/ need + (not) have + P2” nói chung dùng để giả sử

những khả năng không xảy ra trong thực tế ở quá khứ.

Question 8: Đáp án B

Đáp án B - câu hỏi so sánh

Cấu trúc so sánh kép như sau: “more and more + adj dài” hoặc “adj ngắn -er and adj ngắn -er”.

Theo cấu trúc trên, hai đáp án A và D bị loại. Hơn nữa, “serious” là tính từ dài (hai âm tiết trở

lên), nên chọn đáp án B - more and more.

Tạm dịch: Ô nhiễm không khí đang trở nên ngày càng nghiêm trọng ở các thành phố lớn như

Hà Nội và Bắc Kinh.

Question 9: Đáp án D

Đáp án D - câu hỏi mệnh đề quan hệ rút gọn

Trong câu cho mệnh đề sau “that” có chủ ngữ là “any application”. Chủ ngữ này có động từ

chính là “shall not be considered” ở sau. Nếu muốn thêm một động từ “send” vào trước thì

phải nối bằng “and/ or/ but...” hoặc để ở dạng mệnh đề quan hệ rút gọn.

Trang 10
Trong câu không có “and/ or/ but...” nên phải để “send” ở dạng mệnh để quan hệ rút gọn, tức

là V-ing (phân từ 1) hoặc P2 (phân từ 2). Trong 4 đáp án chỉ có D - sent là P2 (mang nghĩa bị

động) mà không có V-ing nên chọn D.

Tạm dịch: Công ty vừa đưa ra thông báo rằng bất cứ đơn xin việc nào được gửi đến sau ngày 30

tháng Tư đều không được xem xét.

* Mở rộng: Một chủ ngữ chỉ có một động từ đi theo sau nếu như không có “and/ or/ but...” nối

thêm các động từ khác. Nếu chủ ngữ đã có động từ, muốn thêm động từ nữa mà không dùng

phép nối vừa nhắc đến thì phải để ở dạng mệnh đề quan hệ.

Question 10: Đáp án B

Đáp án B - câu hỏi đảo ngữ

“Only + liên từ + mệnh đề” đứng đầu câu là dấu hiệu của đảo ngữ trợ động từ lên trước chủ

ngữ, vì thế nên, hai đáp án A và C bị loại do không có đảo ngữ.

Động từ “allow” (cho phép) là ngoại động từ, sau nó phải có sb/sth (tân ngữ), nhưng trong câu

này không có, nên phải để ở dạng bị động (be + P2). Vậy nên, ta chọn đáp án có bị động và đảo

ngữ là đáp án B.

Tạm dịch: Chỉ sau khi thầy giáo giải thích quy trình một cách cặn kẽ thì sinh viên mới được

phép thực hiện thí nghiệm.

* Mở rộng: Ngoài trường hợp “only + liên từ + mệnh đề” thì ta còn có “only + giới từ + Noun/ V-

ing” cũng là dấu hiệu của đảo ngữ; ví dụ: Only on Sunday do we have a day off (Chỉ vào ngày

Chủ nhật chúng tôi mới được nghỉ).

Question 11: Đáp án A

Đáp án A - câu hỏi đuôi

Câu hỏi đuôi luôn hỏi cho động từ và chủ ngữ ở mệnh đề chính. Trong trường hợp này, chủ

ngữ của mệnh đề chính là “they” nên hai đáp án C và D (hỏi cho She) bị loại.

Câu khẳng định có câu hỏi đuôi dạng phủ định còn câu phủ định có câu hỏi đuôi dạng khẳng

định. Câu đề bài cho có “rarely” là trạng từ phủ định nên nó là câu phủ định. Vì vậy, chọn đáp

án A - do they (có phần hỏi đuôi khẳng định).

Tạm dịch: Bố mẹ cô ấy vô cùng nghiêm khắc. Họ không cho cô ấy ra ngoài quá muộn đâu, có

đúng không?

Question 12: Đáp án B

Trang 11
Đáp án B - câu hỏi trật tự tính từ trước danh từ Các tính từ sắp xếp trước danh từ theo trật tự

“OSASCOMP”, lân lượt theo thứ tự: Opinion (nhận xét, đánh giá), Size (kích cỡ), Age (tuổi tác/

cũ mới), Shape (hình dáng), colour (màu sắc), origin (nguồn gốc), material (nguyên liệu), và

purpose (mục đích sử dụng) + Noun. Bốn tính từ trong câu cho là: beautiful (xinh đẹp -

opinion), old (cũ - age), European (từ Châu Âu - origin), và wooden (làm từ gỗ - material). Theo

đúng trật tự “OSASCOMP” thì đáp án B đúng.

Tạm dịch: Những du khách đến với bảo tàng địa phương này phần lớn đều bị thu hút bởi một

chiếc ghế xích đu cũ xinh đẹp làm từ gỗ và đến từ châu Âu kia.

* Lưu ý: Trật tự tính từ của tiếng Việt và tiếng Anh không giống nhau nên khi dịch sẽ không thể

bảo toàn được trật tự.

Question 13: Đáp án B

Đáp án B - câu hỏi từ loại

“Because of” (bởi vì) là giới từ chỉ nguyên nhân. Sau giới từ là Noun hoặc V-ing.

Trong câu đã cho danh từ (noun) là “accidents”, vậy trước “accidents” cần có một tính từ (adj)

để bổ sung thông tin cho danh từ đó. Vậy nên ta chọn B - tragic (là tính từ).

A. tragedy (n): bi kịch/ sự thảm khốc

B. tragic (a): thảm khốc

C. tragically (adv): một cách thảm khốc

D. tragedies (n): danh từ số nhiều của đáp án A.

Tạm dịch: Rất nhiều phi hành gia không bao giờ quay trở lại trái đất nguyên nhân do những tai

nạn thảm khốc.

* Mở rộng: Ngoài “because of”, còn có “due to”, “thanks to”, “owing to”, và “out of” là các giới

từ chỉ nguyên nhân

Question 14: Đáp án D

Đáp án D - câu hỏi từ vựng

Câu này chọn đáp án đúng phù hợp với nghĩa và kết hợp từ. Đáp án D - realise (đạt được) phù

hợp và đúng với danh từ “ambition” - tham vọng.

A. reject (v): từ chối/ loại bỏ

B. exacerbate (v): làm cho tệ đi

C. recognise (v): nhận ra

D. realise (v): nhận ra/ đạt được một cái gì đó mà mình hy vọng

Trang 12
Tạm dịch: Sau nhiều năm khổ luyện, vận động viên đó cuối cùng đã có thể đạt được tham vọng

giành huy chương vàng Olympic của mình

Question 15: Đáp án A

Đáp án A - câu hỏi thành ngữ

Thành ngữ “take advantage of sb/sth”: tận dụng/ lợi dụng ai hoặc cái gì đó. Xét 4 đáp án, chỉ

chọn được A - advantage.

A. advantage (n): lợi thế

B. use (n)/ (v): công dụng (n) - thường đi trong “Make use of sth - tận dụng cái gì đó/ sử dụng

(v)

C. benefit (n): lợi ích

D. dominance (n): sự thống trị

Tạm dịch: Ủy ban này ước tính rằng có ít nhất 7 công ty đã lợi dụng chương trình này

Question 16: Đáp án B

Đáp án B - câu hỏi từ vựng

Câu này chọn đáp án đúng phù hợp nghĩa và kết hợp từ. Đáp án B - source (nguồn) kết hợp với

“entertainment” thành cụm từ “source of entertainment” - nguồn giải trí - phù hợp về kết hợp

từ và nghĩa.

Các đáp án khác không phù hợp về kết hợp từ và nghĩa.

A. status (n): vị trí xã hội/ tình trạng/ trạng thái

B. source (n): nguồn/ nguồn cơn

C. origin (n): nguồn gốc

D. prospect (n): triển vọng

Tạm dịch: Rạp chiếu phim hiện không còn phổ biến như những năm 1930 và 1940 nữa, nhưng

nó vẫn là một nguồn giải trí quan trọng.

Question 17: Đáp án D

Đáp án D - câu hỏi thành ngữ

Thành ngữ “a place in the Sun” - một vị trí thuận lợi/ vị trí mong muốn/ vị trí có lợi. Xét 4 đáp

án chỉ có thể chọn được đáp án D - place.

A. position (n): tư thế đứng, ngồi/ vị trí/ thứ tự

B. image (n): hình ảnh

C. shade (n): bóng đen do che ánh nắng/ rèm che nắng

Trang 13
D. place (n): nơi/ chỗ

Tạm dịch: Sau nhiều năm bị coi thường bởi thế giới, nền công nghiệp phim Ấn Độ đang đạt

được vị trí mong muốn của mình.

Question 18: Đáp án D

Đáp án D - câu hỏi phrasal verb

Câu hỏi này cần chọn một đáp án có cụm động từ (phrasal verb) phù hợp nghĩa nhất. Chọn D -

came across vì:

A. take after sb: giống ai đó

B. make up sth: bịa ra cái gì đó

C. turn sth down: từ chối cái gì đó

D. came across sth/sb: tình cờ gặp ai/ cái gì đó.

Tạm dịch: Hôm trước, khi tôi đang xem lại mấy album ảnh cũ, tôi tình cờ thấy tấm ảnh đám

cưới này của bố mẹ tôi.

* Mở rộng: “Make up” còn có nghĩa là trang điểm/bịa chuyện/ làm lành (with sb)/ chiếm phần

trăm...

Question 19: Đáp án B

Động từ “classify” - phân loại/ xếp loại - là ngoại động từ, sau nó cần có sb/sth (làm tân ngữ).

Nhưng trong câu này không có tân ngữ, ta phải chia bị động. Đáp án B - classify → be

classified.

Tạm dịch: Trong số hơn 1,300 núi lửa trên toàn thế giới, chỉ có khoảng 600 núi lửa có thể được

xếp vào loại còn hoạt động.

Question 20: Đáp án A

Trước và sau dấu phẩy là hai mệnh đề. Để nối hai mệnh đề lại với nhau, ta dùng liên từ

(conjunction) chứ không dùng giới từ (giới từ chỉ được dùng để nối Noun/ V-ing với một câu

khác). Hay nói cách khác sau “In spite of” phải đi với Noun/ V-ing mà ở đây lại là mệnh đề. Vì

vậy đáp án A - in spite of (là giới từ): mặc dù - là đáp án sai. Đáp án A - in spite of → although/

though/ even though/ even if (liên từ): mặc dù.

Tạm dịch: Mặc dù người dân Ấn Độ đã từng bị thiếu các loại động vật, họ vẫn có khả năng

canh tác các loại cây trồng phù hợp với các nhu cầu sử dụng hằng ngày.

* Mở rộng: Ngoài “in spite of”, còn có “despite” cũng là giới từ và mang nghĩa là “mặc dù”

Question 21: Đáp án D

Trang 14
Đáp án D sai loại từ. Trong câu, “history” (môn lịch sử) được nối song song bằng “and” với

môn kinh tế học mới đúng. Vì thế đáp án D - economy: nền kinh tế - là đáp án sai, cần sửa D -

economy economics (môn kinh tế học).

Tạm dịch: Một giáo sư về lịch sử và kinh tế học trường tôi vừa mới phát triển một lý thuyết mới

về mối quan hệ giữa các sự kiện lịch sử và các lần khủng hoảng tài chính.

Question 22: Đáp án B

Dịch câu đề bài: John bây giờ không còn uống nhiều nữa rồi. (ngụ ý John đã từng uống nhiều).

Dịch đáp án:

A. John ngày xưa không uống nhiều. → sai nghĩa

B. John ngày xưa đã từng thường xuyên uống nhiều. → đúng

C. John ngày xưa không thường uống nhiều. → sai nghĩa

D. John bây giờ uống rất nhiều. → sai nghĩa

Đáp án đúng là B, vì đề bài ngụ ý rằng John đã từng uống nhiều và đáp án B dùng cấu trúc

“used to V” để diễn tả thói quen trong quá khứ.

Ba đáp án còn lại sai nghĩa

Question 23: Đáp án B

Dịch câu đề bài: “Ngưng hút thuốc ngay không thì anh sẽ bị bệnh” - ông bác sỹ bảo tôi. (ngụ ý:

ngưng hút thuốc để tránh bệnh tật) Dịch đáp án:

A. Tôi bị ra lệnh không được hút thuốc lá để khỏi bệnh. → sai nghĩa

B. Ông bác sỹ khuyên tôi ngưng hút thuốc lá để phòng ngừa bệnh tật. → đúng

C. Ông bác sỹ gợi ý cùng hút thuốc để chữa bệnh → sai nghĩa

D. Tôi bị cảnh báo không được hút nhiều thuốc. → sai, thiếu nghĩa

Đáp án đúng là B, vì đề bài ngụ ý rằng nên dừng hút thuốc để tránh bệnh và đáp án B chuyển

câu trực tiếp về câu gián tiếp dạng khuyên bảo theo cấu trúc “advise sb to V”, phù hợp với

nghĩa đó.

Ba đáp án còn lại sai hoặc thiếu nghĩa.

Question 24: Đáp án B

Dịch đề bài: Người ta cho rằng việc tăng mức độ khí CO2 và các loại khí nhà kính khác sẽ chắc

chắn dẫn đến sự nóng lên toàn cầu. (ngụ ý việc tăng mức độ khí CO2 và các khí nhà kính khác

gây ra sự nóng lên toàn cầu).

Dịch đáp án:

Trang 15
A. Sự nóng lên toàn cầu được cho là gây ra sự tăng trong mức độ khí CO2 và các loại khí nhà

kính khác. -→ ngược nghĩa so với đề bài.

B. Việc tăng mức độ khí CO2 và các loại khí nhà kính khác được cho là phải chịu trách nhiệm

cho sự nóng lên toàn cầu đúng

C. Việc tăng mức độ khí CO2 và các loại khí nhà kính khác được quy cho là trách nhiệm của sự

nóng lên toàn cầu. → ngược nghĩa so với đề bài.

D. Sự nóng lên toàn cầu bị vu cho trách nhiệm gây ra việc tăng mức độ khí CO2 và các loại khí

nhà kính khác. → ngược nghĩa so với đề bài.

Đáp án B đúng, vì phù hợp với ý của đề bài.

Ba đáp án còn lại sai vì ngược nghĩa so với đề bài.

* Mở rộng: ở các đáp án, có những cụm từ đáng chú ý và cần học thuộc sau:

1. Result in sth: gây ra/ dẫn tới cái gì đó

2. Be responsible for sth: chịu trách nhiệm (vì đã gây ra) điều gì đó

3. Be attributed to sb/sth: bị quy cho là trách nhiệm của ai/ cái gì.

4. Be blamed for sth: bị vu cho việc gì đó.

Question 25: Đáp án D

Dịch đề bài: Anh ấy lúc đó rất mệt. Anh ấy đồng ý giúp tôi làm bài tập về nhà. (ngụ ý: dù rất

mệt vẫn giúp làm bài tập về nhà)

Đây là câu hỏi về cấu trúc ngữ pháp nên không dịch đáp án mà xét cấu trúc.

Để nối hai mệnh đề có nghĩa nhượng bộ, liên từ “although/ though/ even though/ even if’ - mặc

dù - thường được sử sựng theo cấu trúc sau:

Although/ though/ even though/ even if + S1 + V1+..., S2 + V2 + ....

Nếu không dùng liên từ, có thể dùng hai giới từ là “in spite of” và “despite” - mặc dù - để nối

câu theo cấu trúc sau:

Despite/ in spite of + Noun/ V-ing, S + V +

Ngoài ra, còn có một cấu trúc đặc biệt dành cho tính từ và trạng từ như sau:

Adj/ adv + though/ as + S1 + V1 +..., S2 + V2 + ...

Tất cả các trường hợp nối mệnh đề nhượng bộ kể trên sẽ không được phép đi với “but” hoặc

các từ đồng nghĩa với “but”.

Xét 4 đáp án, ta có:

Trang 16
Đáp án D đúng vì sử dụng cấu trúc đặc biệt “Adj + as + S1 + V1.. ., S2 + V2 +...” Đáp án A sai vì

dùng sai cấu trúc.

Đáp án B và C sai vì có “but”.

Question 26: Đáp án C

Dịch đề bài: Mary cãi nhau với bạn trai ngày hôm qua. Sáng nay cô ấy không muốn trả lời điện

thoại của anh ta. (ngụ ý: hôm qua cãi nhau nên hôm nay không muốn nghe điện thoại).

Dịch đáp án:

A. Việc từ chối trả lời điện thoại của Mary dẫn đến trận cãi nhau với bạn trai ngày hôm qua. →

ngược nghĩa so với đề bài

B. Mặc dù Mary cãi nhau với bạn trai ngày hôm qua nhiều thế nào đi chăng nữa, cô ấy vẫn

nghe điện thoại của bạn trai vào sáng nay. → sai nghĩa

C. Vì đã cãi nhau với bạn trai ngày hôm trước, Mary từ chối nghe điện thoại của anh ta sáng

nay → đúng

D. Trước khi Mary cãi nhau với bạn trai vào hôm qua, cô ấy đã trả lời điện thoại của anh ta. →

sai nghĩa.

Đáp án C đúng, vì đề bài ngụ ý rằng Mary cãi nhau với bạn trai hôm trước nên sáng nay không

nghe điện thoại của anh ta. Đáp án C dùng phân từ hoàn thành (having + P2) để nối hai mệnh

đề cùng chủ ngữ là Mary. Mệnh đề có phân từ hoàn thành ở đây cung cấp thông tin về nguyên

nhân và thời gian cho mệnh đề sau. Đáp án C đúng về nghĩa với câu đề bài cho.

Ba đáp án còn lại sai vì ngược hoặc sai nghĩa so với câu đề bài cho.

Question 27: Đáp án C

“Do you fancy going to a movie this evening?” - em có muốn đi xem phim tối nay không? - đây

là một lời mời.

Đối với lời mời, các câu trả lời thường sẽ là: “That would be nice/ great/ excellent...” hoặc “That

is a great/ nice/ excellent idea” hoặc “I would love/ like to”. Nếu từ chối, vẫn trả lời như trên

nhưng thêm “but...”.

Xét 4 đáp án, chọn đáp án C là phù hợp nhất.

A. Không chút nào hết. Cứ làm đi. → thường dùng để trả lời cho những lời xin phép được làm

điều gì đó có thể gây phiền.

B. Tôi xin lỗi. Tôi không biết điều đó. → dùng để xin lỗi khi lỡ mắc lỗi.

C. Thật tuyệt đấy. → dùng để đồng ý lời mời.

Trang 17
D. Không tệ lắm. Còn bạn? → để trả lời và hỏi lại khi được hỏi về tình trạng sức khỏe, công

việc, học tập.

* Mở rộng: Ngoài câu hỏi ở đề bài, có thể sử dụng những mẫu câu sau để mời: “would you

like...”/ “what about + V-ing/ Noun”/ “How about + V-ing/ Noun”.

Question 28: Đáp án B

“What a lovely house you have!” - Bạn có một cái nhà thật đẹp! - đây là một lời khen.

Đối với những lời khen hay chúc mừng, câu trả lời thường là cám ơn, theo mẫu sau: Thank you/

Thanks + lời mời/ lời nói khách sáo.

Xét 4 đáp án, chọn đáp án B là phù hợp nhất.

A. Dĩ nhiên là không rồi, nó không có đắt đâu.

B. Cám ơn bạn. Mong bạn ghé chơi.

C. Tớ cũng nghĩ thế.

D. Không thành vấn đề

Question 29: Đáp án D

“Distress” (n): nguy cấp/ tình huống cần cứu giúp. Chọn đáp án D - help (n): giúp đỡ - là gần

nghĩa nhất với từ đề bài cho.

A. amusing (a): vui/ giải trí.

B. strange (a): kì lạ/ lạ lùng

C. bold (a): can đảm/ tự tin/ quyết tâm

D. help (n): sự giúp đỡ

Tạm dịch: Khi tàu Titanic đâm vào tảng băng trôi, thủy thủ đoàn nhanh chóng gửi đi những tín

hiệu cầu cứu để thông báo cho các tàu lân cận biết về thảm họa và đề nghị họ giúp đỡ.

Question 30: Đáp án A

“Cry/ ask for the moon” (thành ngữ): đòi hỏi/ mong muốn những thứ không khả thi hoặc khó

khăn, chọn đáp án A - longing for what is beyond the reach: mong muốn những thứ ngoài tầm

với - là gần nghĩa nhất với thành ngữ bài cho.

A. mong muốn những thứ ngoài tầm với

B. đòi hỏi những thứ có thể đạt được

C. thực hiện thứ gì đó với quyết tâm và cường độ cao

D. khóc nhiều và lâu

Trang 18
Tạm dịch: Tôi ủng hộ những hoài bão nhưng tôi nghĩ khi anh ta nói anh ta sẽ trở thành một

triệu phú khi 25 tuổi, anh ấy chỉ đơn giản là mong đợi điều gì đó không khả thi.

Question 31: Đáp án A

“Secure” (a): an toàn/ ổn định. Chọn đáp án A - challenging (a): thử thách/ mạo hiểm - là trái

nghĩa với từ mà bài cho.

A. challenging (a): thử thách/ mạo hiểm

B. demanding (a): khó khăn/ yêu cầu cao

C. stable (a): ổn định/ không biến động

D. safe (a): an toàn

Tạm dịch: Tôi ưu tiên những công việc ổn định bởi tôi không thích việc chuyển nhà và thay đổi

liên tục.

Question 32: Đáp án C

“Be walking on air” (thành ngữ): cực kì vui/ cực kì hạnh phúc, chọn đáp án C - feeling

extremely unhappy: cảm thấy cực kì không vui/ bất hạnh - là trái nghĩa với thành ngữ bài cho.

A. cực kì vui vẻ hạnh phúc

B. cực kì nhẹ

C. cảm thấy cực kì không vui/ bất hạnh

D. cảm thấy vô cùng tự tin

Tạm dịch: Khi anh ấy đỗ đại học, bố mẹ anh ấy sẽ rất vui sướng và hạnh phúc

Question 33: Đáp án C

A. affect (v): tác động

B. effective (a): có hiệu quả

C. effect (n): tác động/ hiệu ứng/ hiệu quả

D. effectively (adv): một each có hiệu quả

Câu hỏi về từ vựng và ngữ pháp. Chọn đáp án C vì cụm từ “take effect” là cụm kết hợp từ

(collocation) và có nghĩa là: bắt đầu có tác dụng, tác động/ có hiệu lực/ ngoài ra, sau “take” cần

có một danh từ làm tân ngữ, nên chỉ có thể chọn C - effect (n).

Trích bài: A new study into childhood amnesia has found that it tends to take effect around the

age of seven.

Tạm dịch: Một nghiên cứu mới về hội chứng quên ở trẻ em đã cho thấy hội chứng này bắt đầu

có tác động khi trẻ vào khoảng 7 tuổi.

Trang 19
Question 34: Đáp án B

A. unbalanced (a): điên loạn/ mất cân bằng

B. immature (a): non nớt/ chưa phát triển đủ

C. insufficient (a): không đủ nhiều/ thiếu

D. irrational (a): phi lý

Câu hỏi về từ vựng, căn cứ vào nghĩa để chọn đáp án B - immature.

Trích bài: Before the age of seven, children tend to have an immature form of recall with no

sense of time and place in their memories.

Tạm dịch: Trước khi lên 7 tuổi, trẻ em thường có một kiểu nhớ lại non nớt, không có một chút

nhận thức nào về thời gian và địa điểm trong trí nhớ của chúng.

Question 35: Đáp án A

A. besides (adv): ngoài ra/ thêm nữa

B. however (adv): tuy nhiên

C. therefore (adv): vì vậy/ thế nên

D. otherwise (adv): nếu không thì → dùng ở câu giả định

Câu hỏi về từ vựng, căn cứ vào nghĩa để chọn đáp án A - besides.

Trích bài: Faster rate of forgetting in children and higher turnover of memories means early

memories are less likely to survive. Besides, memories of younger children tend to lack

autobiographical narrative leading to a process known as “retrieval induced forgetting”

Tạm dịch: Tốc độ quên và tốc độ quay vòng trí nhớ nhanh hơn ở trẻ em đồng nghĩa với việc

những kí ức đầu đời càng ít có khả năng tồn tại lâu. Hơn nữa, kí ức của những trẻ nhỏ hơn

thường thiếu mất những tường thuật về bản thân mình, điều này dẫn tới một quá trình được

biết tới với cái tên “quên để phục hồi”.

Question 36: Đáp án D

A. whom (đại từ): người mà - đóng vai trò làm tân ngữ chỉ người trong mệnh đề quan hệ

B. which (đại từ): cái mà - đóng vai trò làm chủ ngữ và tân ngữ chỉ vật trong mệnh đề quan hệ

C. when (trạng từ): khi mà - đóng vai trò làm trạng ngữ thời gian trong mệnh đề quan hệ

D. where (trạng từ): ở đó - đóng vai trò làm trạng ngữ nơi chốn trong mệnh đề quan hệ

Câu hỏi về ngữ pháp. Chọn đáp án D vì trong mệnh đề quan hệ “___________the action of

remembering causes other information to be forgotten” đã có đủ chủ ngữ, động từ, và tân ngữ

nên chỉ cần có một trạng ngữ nữa điền vào chỗ trống. Ngoài ra, “retrieval induced forgetting”

Trang 20
không phải là danh từ chỉ thời gian, mà nó chỉ một quá trình trừu tượng (có thể coi như một nơi

xảy ra sự việc) nên chọn D- where.

Trích bài: memories of younger children tend to lack autobiographical narrative leading to a

process known as “retrieval induced forgetting” where the action of remembering causes other

information to be forgotten.

Tạm dịch: kí ức của những trẻ nhỏ hơn thường thiếu mất những tường thuật về bản thân mình,

điều này dẫn tới một quá trình được biết tới với cái tên “quên để phục hồi”, ở đó, hành động

nhớ khiến cho các thông tin khác bị quên lãng đi

Question 37: Đáp án B

A. turning (v): quay đi/ xoay chuyển

B. making (v): làm ra/ khiến

C. transferring (v): di dời/ chuyển/ chuyển nhượng

D. getting (v): lấy được/ hiểu được

Câu hỏi từ vựng. Chọn đáp án B - making vì cụm từ “make it” là cụm cố định và có nghĩa là:

thành công trong việc đi đâu đó/ thành công trong việc làm gì đó/ sống sót thành công/ cố gắng

vượt qua khó khăn.

Trích bài: Consequently, if childhood memories can survive into the ninth or tenth year of life,

they may stay a chance of (37)___________it into adulthood.

Tạm dịch: Vì thế cho nên, nếu những kí ức tuổi thơ có thể tồn tại đến năm thứ 9 hoặc thứ 10 của

cuộc đời, chúng rất có thể sẽ có cơ hội sống sót thành công và song hành đến lúc lớn.

Question 38: Đáp án C

Dịch đề bài: Ý chính của đoạn văn này là gì?

A. Việc đến muộn thể hiện sự thượng đẳng của các cá nhân quan trọng.

B. Các giá trị xã hội của những cá nhân được thúc đẩy thông qua việc đi muộn

C. Việc đi muộn có thể tiết lộ nhiều điều về tâm lý của một người.

D. Những người có nhận thức về quyền lực cao có xu hướng đi muộn.

* Hướng dẫn làm bài:

Đối với loại câu hỏi tìm ý chính/ tiêu đề của đoạn vãn, việc cần làm là:

1/ Đọc 2-3 câu đẩu tiên của bài vì những câu đó là câu thể hiện chủ đề của bài viết.

2/ Đọc lướt qua nội dung của bài để chắc chắn mình xác định ý chính đúng.

Trang 21
Câu đầu tiên của bài này là: “Being repeatedly late may just be accidental - or could it show a

deeply rooted psychological desire to express your own superiority?”

Tạm dịch: Việc đi muộn liên tục có thể chỉ là tai nạn tình cờ hay liệu nó có cho thấy được những

ham muốn tâm lý vùi sâu trong mỗi con người để thể hiện sự thượng đẳng của chính mình?

Như vậy, ta thấy ý chính của bài này là những nguyên nhân tâm lý đằng sau việc đi muộn.

Ngoài ra, nội dung các đoạn ở dưới đều nói đến mối liên quan giữa tâm lý và việc đi muộn.

Vậy nên, đáp án C đúng.

Question 39: Đáp án A

Dịch để bài: Cụm từ “speaks volumes” ở đoạn hai gần nghĩa với từ nào nhất?

A. tiết lộ rất nhiều thứ

B. tạo ra âm thanh

C. nắm giữ quyền lực

D. nói chuyện lớn

Câu hỏi về từ vựng. “Speaks volumes” (thành ngữ): nói rõ, tiết lộ rõ. Chọn đáp án A - tells a lot:

tiết lộ rất nhiều thứ - là gần nghĩa nhất với từ bài cho.

Question 40: Đáp án D

Dịch đề bài: Đáp án nào dưới đây không đúng theo như bài đọc?

A. Cách mà hệ thống thần kinh của chúng ta hoạt động chịu trách nhiệm gây ra cách chúng ta

hành xử.

B. Một vài người kiểm soát tình huống vì lợi ích của họ thông qua việc đến muộn.

C. Việc đến muộn có thể có ảnh hưởng đến những người bị bắt phải đợi chờ.

D. Hầu hết mọi người đều phản ứng như nhau khi việc đến muộn họ không kiểm soát được.

* Hướng dẫn làm bài:

Đối với loại câu hỏi tìm ý sai hoặc tìm ý đúng cần phải đọc hết cả 4 đáp án và tìm thông tin

trong bài ủng hộ các đáp án đó. Nếu đáp án nào không có thông tin ủng hộ hoặc ngược lại với

thông tin trong bài, thì đó là đáp án sai. Việc tìm thông tin ủng hộ đáp án phải dựa vào từ khóa

của đáp án và trên bài đọc. Tuyệt đối không được dùng suy luận từ kiến thức của bản thân để

trả lời loại câu hỏi này mà phải dựa hết vào bài đọc.

Trích thông tin trong bài:

1/ All behaviour reflects psychology: tất cả các hành vi đều phản ánh tâm lý. → thông tin ủng

hộ đáp án A.

Trang 22
2/ those who make others wait are acting out an inner agenda, often based on an acute sense of

power: những người khiến người khác phải đợi là đang thể hiện một mục tiêu ngầm, thường là

dựa trên một cảm giác rất mạnh về quyền lực bản thân. → thông tin ủng hộ đáp án B.

3/ Some people will genuinely worry about the impact it will have on those left waiting: một vài

người rõ ràng sẽ lo lắng về những ảnh hưởng mà việc đi muộn gây ra cho những người bị bắt

đợi. → thông tin ủng hộ đáp án C.

4/ Some people will genuinely worry about the impact it will have on those left waiting, while

others might secretly enjoy the power of their absence: một vài người rõ ràng sẽ lo lắng về

những ảnh hưởng mà việc đi muộn gây ra cho những người bị bắt đợi, trong khi đó những

người khác có thể sẽ bí mật tận hưởng sức mạnh của sự vắng mặt của họ. → thông tin không

ủng hộ đáp án D vì có hai kiểu phản ứng chứ không phải hầu hết mọi người phản ứng như

nhau.

Vậy, đáp án D không đúng theo bài → chọn D

Question 41: Đáp án A

Dịch đề bài: Có thể suy ra từ bài đọc rằng………..

A. Berlusconi khiến cho Merkel phải đợi chờ vì ông ta muốn thể hiện tầm quan trọng của mình.

B. Việc đến họp muộn là hợp thời vì không có ai phản đối.

C. Lịch trình làm việc hàng ngày của tác giả không bị ảnh hưởng bởi việc đi muộn của đồng

nghiệp

D. Không gì có thể được rút ra về tâm lý nếu như việc đi muộn là ngoài tâm kiểm soát.

* Hướng dẫn làm bài: Đối với loại câu hỏi suy luận, đọc kĩ 4 đáp án và tìm thông tin hỗ trợ

trong bài. Chắc chắn chỉ có một đáp án được thông tin trong bài hỗ trợ, các đáp án còn lại đều

sai hoặc không có thông tin trong bài. Lưu ý phải tìm thông tin dựa vào từ khóa, và không

dùng kiến thức bản thân để trả lời loại câu hỏi này.

Trích thông tin trong bài:

1/ those who make others wait are acting out an inner agenda, often based on an acute sense of

power. There’s famous footage in which Silvio Berlusconi kept Angela Merkel waiting while he

made a call on his mobile: những người khiến người khác phải đợi là đang thể hiện một mục

tiêu ngầm, thường là dựa trên một cảm giác rất mạnh về quyền lực bản thân. Có những đoạn

phim ngắn nổi tiếng ghi lại cách Silvio Berlusconi bắt Angela Merkel phải đợi trong lúc ông ta

nghe điện thoại. → thông tin ủng hộ đáp án A, bởi đoạn trích dẫn có hai câu, câu sau nói về việc

Trang 23
Berlusconi bắt Merkel đợi chờ như một ví dụ cho giả thiết ở câu trước: những người bắt người

khác đợi thường thể hiện quyền lực mạnh mẽ của mình.

2/ When I worked in an office, meetings would often start late, usually because of a certain

individual: khi tôi còn làm việc ở văn phòng, các buổi họp thường bắt đầu trễ hơn, thường là

bởi vi một cá nhân cụ thể. → thông tin không ủng hộ đáp án C, vì lịch trình làm việc của tác giả

bị ảnh hưởng.

3/ In such cases, there’s not a lot of psychology involved. Or is there? Trong những trường hợp

này, không có chút biểu hiện tâm lý nào liên quan. Ồ, nhưng có đấy. → không ủng hộ đáp án D.

4/ Đáp án B không có thông tin trong bài.

Vậy, đáp án A được thông tin trong bài ủng hộ, chọn đáp án A.

Question 42: Đáp án

Dịch đề bài: Từ “it” ở đoạn 4 nói đến cái gì?

A. sự thật B. sự đến muộn

C. quy ước D. tâm lý

* Hướng dẫn làm bài: Đối với loại câu hỏi này, đọc ngược lại 1-2 câu trước nó để tìm từ mà nó

đang thay thế.

Trích thông tin trong bài: The essential fact is that lateness means breaking a convention -....

Regardless of psychology, it has a social value: Một sự thật thiết yếu chính là việc đi muộn sẽ

phá vỡ các quy ước. Khi không nói đến tâm lý, việc đi muộn cũng mang những giá trị xã hội.

Vậy, it thay cho lateness → chọn B.

Question 43: Đáp án D

Dịch đề bài: Đáp án nào dưới đây có thể là tiêu đề tốt nhất cho bài đọc?

A. “Vi lượng đồng căn gây chết người” các nhà khoa học cho hay

B. “Hãy tránh xa vi lượng đồng căn” các nhà khoa học cho hay

C. Vi lượng đồng căn đáng trải nghiệm một lần

D. Tác dụng chưa rõ ràng của vi lượng đồng căn

* Hướng dẫn làm bài: Đối với loại câu hỏi tìm ý chính/ tiêu đề của đoạn văn, việc cần làm là:

1/ Đọc 2-3 câu đầu tiên của bài vì những câu đó là câu thể hiện chủ đề của bài viết.

2/ Đọc lướt qua nội dung của bài để chắc chắn mình xác định ý chính đúng.

Trang 24
Câu đầu tiên của bài đọc là: Homeopathy, the alternative therapy created in 1796 by Samuel

Hahnemann, and now widely used all over the world, is based on the belief that the body can

be stimulated to heal itself.

Tạm dịch: Vi lượng đồng căn, một liệu pháp chữa bệnh thay thế được sáng tạo năm 1796 bởi

Samuel Hahnemann, và ngày nay được sử dụng rộng rãi trên toàn thế giới, được phát triển dựa

trên niềm tin rằng cơ thể có thể được kích thích để tự chữa bệnh cho nó. Như vậy, bài đọc này

nói về “Vi lượng đồng căn”. Khi đọc qua nội dung các đoạn dưới, có thể thấy tác giả đưa ra

những nghi ngờ về tác dụng của liệu pháp này, và chứng minh rằng nó không có hiệu quả.

Ngoài ra còn có những cảnh báo về nguy cơ của việc lạm

dụng liệu pháp này khi nó ít hiệu quả.

Vì vậy, đáp án D là hợp lý hơn cả.

Question 44: Đáp án A

Dịch đề bài: Theo như bài đọc, các loại thuốc của phương pháp vi lượng đồng căn thì...

A. Được chuẩn bị theo cách tương tự cho các loại bệnh khác nhau

B. Được pha chế từ nhiều nguyên liệu

C. Thích hợp cho nhiều triệu chứng

D. Chỉ có ở dạng lỏng

* Hướng dẫn làm bài: Đối với loại bài tìm thông tin, cần chọn ra một vài từ khóa ở đáp án cũng

như ở cầu hỏi rồi tìm những từ khóa đó ở trên bài đọc. Khi đã tìm được đoạn chứa từ khóa trên

bài đọc, cần đọc kĩ và chọn lọc những thông tin nào ủng hộ đáp án. Sẽ chỉ có một đáp án được

thông tin trong bài ủng hộ, và đó là đáp án đúng.

Trích thông tin trong bài: Medicines used in homeopathy are created by heavily diluting in

water the substance in question and subsequently shaking the liquid vigorously. They can then

be made into tablets and pills.

Tạm dịch: Các loại thuốc dùng trong phương pháp vi lượng đồng căn đều được tạo ra bằng

cách pha loãng thật nhiều loại chất đang bị nghi ngờ đó vào trong nước và sau đó lắc mạnh

dung dịch đó. Những thứ thuốc này có thể được chế thành thuốc viên hoặc thuốc con nhộng

Từ đây có thể thấy, đáp án A được thông tin trong bài ủng hộ.

Đáp án B sai vì mỗi loại thuốc này chỉ được pha từ một chất còn bị nghi ngờ với nước → không

nhiều nguyên liệu.

Đáp án C không được nhắc đến trong bài.

Trang 25
Đáp án D sai vì thuốc có thể ở dạng viên nén

Question 45: Đáp án B

Dịch câu hỏi: Từ “their” ở đoạn 2 nói đến cái gì?

A. ủy ban B. NHMRC

C. các loại thuốc D. các loại giả dược

* Hướng dẫn làm bài: Đối với loại câu hỏi này, đọc ngược lại 1-2 câu trước nó để tìm từ mà nó

đang thay thế.

Trích thông tin trong bài: (NHMRC) has claimed that homeopathic medicines are only as

effective as placebos at treating illness. Their research, involving the analysis of numerous

reports from homeopathy interest groups and the public, concluded that there is no reliable

evidence that homeopathy works.

Tạm dịch: NHMRC vừa khẳng định rằng các loại thuốc của phương pháp vi lượng đồng căn

này chỉ có hiệu quả tương đương với giả dược trong việc chữa bệnh. Nghiên cứu của họ, bao

gồm việc phân tích nhiều bản báo cáo từ những nhóm ủng hộ cho phương pháp vi lượng đồng

cán và từ cộng đồng, kết luận rằng không hề có bằng chứng đáng tin cậy nào cho rằng phương

pháp vi lượng đồng căn thực sự có hiệu quả.

Như vậy, “their” được dùng để nói đến NHMRC → chọn đáp án B.

Question 46: Đáp án C

Dịch đề bài: Nghiên cứu đến từ nước Úc tiết lộ rằng phương pháp vi lượng đồng căn thì...

A. càng ngày càng phổ biến đối với cộng đồng

B. có ích trong các trường hợp bệnh truyền nhiễm hô hấp

C. không có hiệu quả trong việc điều trị nhiều loại bệnh

D. thu hút nhiều sự chú ý của một nhóm người nhất định

* Hướng dẫn làm bài: Đối với loại bài tìm thông tin, cần chọn ra một vài từ khóa ở đáp án cũng

như ở câu hỏi rồi tìm những từ khóa đó ở trên bài đọc. Khi đã tìm được đoạn chứa từ khóa trên

bài đọc, cần đọc kĩ và chọn lọc những thông tin nào ủng hộ đáp án. Sẽ chỉ có một đáp án được

thông tin trong bài ủng hộ, và đó là đáp án đúng.

Trích thông tin trong bài: Their research, involving the analysis of numerous reports from

homeopathy interest groups and the public, concluded that there is no reliable evidence that

homeopathy works. Moreover, researchers uncovered no fewer than 68 ailments that

Trang 26
homeopathic remedies had failed to treat, including asthma, sleep disturbances, cold and flu,

and arthritis.

Tạm dịch: Nghiên cứu của họ, bao gồm việc phân tích nhiều bản báo cáo từ những nhóm ủng

hộ cho phương pháp vi lượng đồng căn và từ cộng đồng, kết luận rằng không hề có bằng

chứng đáng tin cậy nào cho rằng phương pháp vi lượng đồng căn thực sự có hiệu quả. Hơn thế

nữa, các nhà nghiên cứu còn tiết lộ không dưới 68 loại bệnh mà phương pháp vi lượng đồng

căn không thể chữa được, bao gồm hen suyễn, rối loạn giấc ngủ, cảm cúm, và thấp khớp.

Như vậy, nghiên cứu trên chỉ ra rằng phương pháp vi lượng đồng căn không hiệu quả trong

việc chữa trị nhiều loại bệnh → đáp án C phù hợp

Question 47: Đáp án C

Dịch đề bài: Từ “wary” ở đoạn 3 gần nghĩa nhất với từ nào?

A. apprehensive (a): lo sợ/ sợ hãi

B. reluctant (a): miễn cưỡng/ chần chừ

C. cautious (a): cẩn trọng

D. observant (a): tinh ý

Câu hỏi về từ vựng. “Wary” (a): cẩn trọng/ cẩn thận. Chọn đáp án C - cautious: cẩn trọng - là

gần nghĩa nhất với từ mà bài cho.

Question 48: Đáp án B

Dịch đề bài: Mọi người thường có xu hướng tin vào phương pháp vi lượng đồng căn vì...

A. những quảng cáo trình chiếu ở các trung tâm sức khỏe

B. phản hồi tích cực từ một số lượng nhỏ người

C. những bằng chứng đáng tin cậy trong những báo cáo y tế gần đây

D. kết quả của các nghiên cứu về những phương pháp chữa bệnh thay thế.

Trích thông tin trong bài: the NHMRC is urging health workers to inform their patients to be

wary of anecdotal evidence that appears to support the effectiveness of homeopathic medicine.

“It isn’t possible to tell whether a health treatment is effective or not simply by considering the

experience of one individual or the beliefs of a health practitioner

Tạm dịch: NHMRC cũng thúc giục các nhân viên chăm sóc sức khỏe thông báo đến bệnh nhân

của mình và nhắc họ phải cẩn trọng trước những bằng chứng có truyền miệng mà có vẻ như

ủng hộ sự hiệu quả của các loại thuốc vi lượng đồng căn. “Việc cho rằng một phương pháp

Trang 27
chữa bệnh có hiệu quả hay không chỉ bằng việc cần nhắc những kinh nghiệm của một cá nhân

hoặc các niềm tin của một người thực hành chăm sóc sức khỏe là không khả thi.

Từ đây có thể thấy, mọi người tin tưởng vào phương pháp vi lượng đồng căn vì họ nghe những

thông tin truyền miệng về những kinh nghiệm của một số ít người. Chọn đáp án B.

Question 49: Đáp án B

Dịch đề bài: Từ “delusion” ở đoạn 4 gần nghĩa với từ nào nhất?

A. legend (n): huyền thoại

B. falsehood (n): sự đánh lừa

C. imagination (n): sự tưởng tượng

D. hallucination (n): ảo giác

Câu hỏi từ vựng. “Delusion” (n): sự đánh lừa. Chọn đáp án B - falsehood: sự đánh lừa - là gần

nghĩa nhất với từ mà bài cho.

Question 50: Đáp án A

Dịch đề bài: Theo quan điểm của giáo sư John Dwyer, các loại vắc xin vi lượng đồng căn thì...

A. có thể mang tới những hậu quả xấu

B. gây hại tới sức khỏe của mọi người

C. có thể đắt một cách nực cười

D. vẫn tốt hơn là không có gì

Trích thông tin trong bài: According to Professor John Dwyer, the concept that homeopathic

vaccinations are just as good as traditional vaccinations is delusion, and those who believe it are

failing to protect themselves and their children.

Tạm dịch: Theo giáo sư John Dwyer, ý tưởng cho rằng các loại vắc xin vi lượng đồng căn thì tốt

như vắc xin truyền thống là một sự lừa đảo, và những người tin vào nó sẽ không thể bảo vệ nổi

bản thân họ cũng như con cái họ.

Từ đây có thể thấy ông Dwyer coi vắc xin vi lượng đồng căn là một sự lừa đảo, nếu người ta tin

vào nó, chính bản thân họ và con cái họ sẽ không được bảo vệ → gây hậu quả xấu. Chọn đáp án

A.

Trang 28
ĐỀ SỐ 02 ĐỀ THI THỬ TỐT NGHIỆP THPT
NĂM HỌC: 2020 – 2021
MÔN: TIẾNG ANH
Thời gian làm bài: 60 phút; không kể thời gian phát đề

Mark the letter A, B, C, or D on your answer sheet to indicate the word whose underlined part
differs from the other three in pronunciation in each of the following questions.
Question 1. A. general B. generous C. generation D. guillotine
Question 2. A. changes B. increases C. suffocates D. compromises
Mark the letter A, B, C or D on your answer sheet to indicate the word that differs from the rest in
the position of the main stress in each of the following questions.
Question 3. A. deploy B. maintain C. border D. attain
Question 4. A. referee B. kangaroo C. cigarette D. technique
Mark the letter A, B, C or D on your answer sheet to indicate the correct answer to each of the
following questions.
Question 5. After so___________consideration, we have finally arrived at a decision.
A. a lot of B. lots of C. many D. much
Question 6. The author did not intend anybody___________his paintings until they were finished.
A. seeing B. to seeing C. to see D. to be seen
Question 7. Every day, Peters mother drives him to school. However, today, he ___________to school
by his father.
A. was taken B. is being taken C. is taking D. took
Question 8. Linda, along with hundreds of others, ___________unemployed when the factory closes
next month.
A. becomes B. become C. is becoming D. are becoming
Question 9. The customs officers stopped the suspected man and ordered him to show___________.
A. what was he carrying in his bag. B. what he was carrying in his bag
C. what he carries in his bag D. what does he carry in his bag
Question 10. It was an English student, not a French one, ___________was the captain of our school
team.
A. which B. what C. whom D. who
Question 11. The man standing next to our father is a persuasive speaker with a natural
talent___________leadership.
A. for B. at C. in D. of
Question 12. The report form is available on the two___________discs in my drawer.
A. small green round B. round small green
Trang 1
C. green small round D. small round green
Question 13. The drug under examination has shown some___________results when given to
volunteers in some countries.
A. impressed B. impresses C. impressive D. impression
Question 14. During rush hours, the traffic is usually___________and cars are closing up behind each
other.
A. light B. heavy C. substantial D. enormous
Question 15. After suffering severe wounds in the attack, the secret agent decided to
his resignation to the commander.
A. give B. apply C. propose D. offer
Question 16. Her father was really a___________of strength to her when her marriage broke up.
A. source B. sculpture C. figure D. tower
Question 17. When travelling in Europe, we tried to ___________at least an hour in a medieval town to
explore it.
A. stop off B. stop out C. stop up D. stop in
Question 18. Last week, an earthquake which___________6.1 on the Richter scale struck southern
California.
A. read B. measured C. calculated D. estimated
Mark the letter A, B, C, or D on your answer sheet to indicate the underlined part that needs
correction in each of the following questions.
Question 19. Just pulling out of the station the train was that we were supposed to catch.
A. the B. the train was C. were supposed D. pulling out
Question 20. Alice Thornton’s autobiography provides a fascinate account of family life in seventeenth-
century England.
A. a fascinate account B. life C. seventeenth-century D. autobiography
Question 21. The meeting would have finished earlier if there had not been for their interruptions.
A. interruptions B. would have finished
C. had not been for D. earlier
Mark the letter A, B, C or D on your answer sheet to indicate the sentence that is closest in meaning
to each of the following questions.
Question 22. ‘T have never been to Russia. I think I shall go there next year.” said Bill.
A. Bill said that he had never been to Russia and he thought he would go there the next year.
B. Bill said that he would have never been to Russia and he thinks he would go there the next year.
C. Bill said that he had never been to Russia and he thinks he will go there the next year.

Trang 2
D. Bill said that he has never been to Russia and he thinks he would go there the next year.
Question 23. People believed that Jane retired because of her poor health.
A. Jane is believed to have retired because of her poor health.
B. Jane was believed to retire because of her poor health.
C. It is believed that Jane retired because of her poor health.
D. Jane retired because of her poor health was believed.
Question 24. Charles would have won the essay contest if he had typed his paper.
A. Charles won the essay contest in spite of not typing his paper.
B. Charles did not win the essay contest because he did not type his paper
C. Typing his paper made Charles win the essay contest.
D. Charles did not win the essay contest even though he typed his paper.
Mark the letter A, B, C or D on your answer sheet to indicate the sentence that best combines each
pair of sentences in the following questions.
Question 25. The government knows the extent of the problem. The government needs to take
action soon.
A. The government knows the extent of the problem whereas it needs to take action soon.
B. The government knows the extent of the problem so that it needs to take action soon.
C. Knowing the extent of the problem, the government needs to take action soon.
D. The government knows the extent of the problem, or else it needs to take action soon.
Question 26. The substance is very toxic. Protective clothing must be worn at all times.
A. Since the substance is very toxic, so protective clothing must be worn at all times.
B. So toxic is the substance that protective clothing must be worn at all times.
C. The substance is such toxic that protective clothing must be worn at all times.
D. The substance is too toxic to wear protective clothing at all times.
Mark the letter A, B, C or D on your answer sheet to indicate the most suitable response to
complete each of the following exchanges.
Question 27. Dick is thanking Michelle for giving him the gift on his birthday.
Dick: “Thanks for the nice gift you brought to me!”
Michelle: “___________”
A. Welcome! I’m glad you like it
B. Not at all
C. Actually speaking, I myself don’t like it
D. All right! Do you know how much it costs?
Question 28. Ben: “___________.”

Trang 3
Jane: “Never mind.”
A. Sorry for staining your carpet. Let me have it cleaned.
B. Would you mind going to dinner next Sunday?
C. Thank you for being honest with me.
D. Congratulations! How wonderful!
Mark the letter A, B, C, or D on your answer sheet to indicate the word or phrase that is CLOSEST
in meaning to the underlined part in each of the following questions.
Question 29. The first time I met my friend’s parents, I was walking on eggshells because I knew their
political views were very different from mine.
A. was talking nervously B. had a lot of pressure
C. was given many eggs D. had to be very cautious
Question 30. Albert Einstein is lauded as one of the greatest theoretical physicists of all time.
A. acclaimed B. dictated C. described D. ordained
Mark the letter A, B, C, or D on your answer sheet to indicate the word or phrase that is
OPPOSITE in meaning to the underlined part in each of the following questions.
Question 31. His lawyer thought Jack had a good chance of being acquitted at the trial, if no further
evidence was found.
A. found guilty B. declared innocent
C. charged of being faulty D. advised of appealing
Question 32. Recycling and disposal of wastes require sizable expenditure, in such situations, industries
preferred to export their wastes to other countries.
A. minimum B. considerable C. plentiful D. trivial
Read the following passage and mark the letter A, B, C or D on your answer sheet to indicate the
correct word for each of the blanks from 33 to 37.
The United Nations Children's Fund (UNICEF), originally known (33) ___________the United Nations’
International Children's Emergency Fund, was created by the United Nations General Assembly on 11
December 1946, to provide emergency food and healthcare to children and mothers in countries that had
been devastated by World War II. The Polish physician Ludwik Rajchman is widely regarded as the (34)
___________of UNICEF and served as its first chairman from 1946 to 1950, when he had to flee the
United States in the wake of McCarthyism. Rajchman is to this day the only person that served as
UNICEF's Chairman for longer than 2 years. On Rajchman's suggestion, the American Maurice Pate was
appointed its first executive director, serving from 1947 until his death in 1965. In 1950, UNICEF's
mandate was extended to address the long-term needs of children and women in developing countries
everywhere. In 1953 it (35) ___________a permanent part of the United Nations System, and the words

Trang 4
"international" and "emergency" were dropped from the organization's name, though it retained the
original acronym, "UNICEF".
UNICEF (36) ___________on contributions from governments and private donors. UNICEF's total
income for 2015 was US$5,009,557,471. Governments contribute two-thirds of the organization's
resources. Private groups and individuals contribute the rest through national committees. It is estimated
that 92 per cent of UNICEF revenue is distributed to program services. UNICEF's programs emphasize
developing community-level services to (37)___________the health and well-being of children. UNICEF
was awarded the Nobel Peace Prize in 1965 and the Prince of Asturias Award of Concord in 2006.
Question 33. A. for B. as C. such as D. like
Question 34. A. ancestor B. descendant C. pioneer D. founder
Question 35. A. became B. becomes C. becoming D. had become
Question 36. A. focuses B. stands C. concentrates D. relies
Question 37. A. promote B. provide C. widen D. increase
Read the following passage and mark the letter A, B, C, or D on your answer sheet to indicate the
correct answer to each of the questions from 38 to 42.
Birds have evolved many physical attributes that contribute to their flying ability. Wings are important,
but adjustable tails, large hearts and light bones play critical roles
To fly, birds, like airplanes, move air across their wings. Wings are designed so that air above the wings
is forced to move faster than air below the wing. This creates higher pressure under the wings, called lift,
which pushes the bird up. Different wing types evolved for different ways of flying. Prolonged flight
requires long wings and an ability to soar. Other birds need superior maneuverability. Finches and
sparrows have short, broad wings. Faster birds, like hawks have built - in spoilers that reduce turbulence
while flying. This allows a steeper angle of attack without stalling.
Tails have evolved for specialized use. The tail acts like a rudder helping birds steer. Birds brake by
spreading out their tails as they land. This adaptation allows them to make sudden, controlled stops in
essential skill, since most birds need to land on individual branches or on prey.
Flight takes muscle strength. If body builders had wings, they still could not flap hard enough to leave the
ground. Birds have large, specialized hearts that beat much faster than the human heart and provide the
necessary oxygen to the muscles. The breast muscle accounts for 15 percent of the bird’s body weight. On
pigeons, it accounts for a third of their total body weight. Birds carry no excess baggage, they have
hollow feathers and hollow bones with struts inside to maintain strength, like cross beams in a bridge.
Birds fly to find a prey, escape predators, and attract mates-in other words, to survive.
Question 38. what is the main idea of the passage?
A. Wings are the most important physical attribute of birds.

Trang 5
B. Different wing styles evolved for different types of flight.
C. Birds have many specialized features that aid in their survival.
D. Birds fly for many reasons.
Question 39. According to the passage, what causes birds to rise when they start flying?
A. Long wings with hollow feathers B. Higher air pressure below than above the wings
C. Spreading out their tails D. Superior muscle strength
Question 40. The phrase “finches and sparrows” refers to___________.
A. wings B. maneuvers C. ways of flying D. birds
Question 41. According to the passage, what benefit comes from having built-in spoilers?
A. an ability to fly faster B. a steeper angle of diving for prey
C. prolonged flight D. superior maneuverability when climbing
Question 42. What does the author imply about the body builders having wings?
A. If they flapped their wings, they could fly a little.
B. If they had wings, their muscles would be strong enough for flight.
C. If they had wings, their hearts would still not be large for flight.
D. Their wings would total 15 percent of their body weight.
Read the following passage and mark the letter A, B, C, or D on your answer sheet to indicate the
correct answer to each of the questions from 43 to 50.
The Americans with Disabilities Act (ADA) was signed into law in 1990. This law extends civil rights
protection to persons with disabilities in private sector employment, all public services, and in public
accommodations, transportation, and telecommunications. A person with disability is defined as someone
with a mental or physical impairment that substantially limits him or her in a major life activity, such as
walking, talking, working, or self-care. A person with a
disability may also be someone with a past record of such an impairment, for example, someone who no
longer has heart disease but discriminated against because of that history.
The ADA states that employers with fifteen or more employees may not refuse to hire or promote a
person because of a disability if that person is qualified to perform the job. Also, the employer must make
reasonable accommodations that will allow a person with a disability to perform essential functions of the
job. All new vehicles purchased by public transit authorities must be accessible to people with disabilities.
All rail stations must be made accessible, and at least one car per train in existing rail systems must be
made accessible.
It is illegal for public accommodations to exclude or refuse persons with disabilities. Public
accommodations are businesses and services such as restaurants, hotels, grocery stores, and parks. All

Trang 6
new buildings must be made accessible, and existing facilities must remove barriers if the removal can be
accomplished without much difficulty or expense.
The ADA also stipulates that companies offering telephone service to the general public must offer relay
services to individuals who use telecommunications devices for the deaf, twenty- four hours a day, seven
days a week.
Question 43. what is the main purpose of the passage?
A. To describe discrimination against persons with disabilities.
B. To explain the provisions of the Americans with Disabilities Act.
C. To make suggestions for hiring persons with disabilities.
D. To discuss telecommunications devices for the deaf.
Question 44. According to the passage, all of the followings are affected by the Americans with
Disabilities Act EXCEPT___________.
A. someone who has difficulty walking
B. a public transit authority
C. an employer with fewer than fifteen employees
D. a person with a past record of an impairment
Question 45. The word “impairment” in bold in paragraph 1 is closest in meaning to___________.
A. disability B. violation C. aptitude D. danger
Question 46. The author mentions grocery stores as an example of___________.
A. public transit B. barriers
C. private sector employment D. public accommodation
Question 47. The word “facilities” in bold in paragraph 3 refers to___________ .
A. barriers B. buildings C. rail stations D. disabilities
Question 48. The author implies all of the followings EXCEPT___________.
A. the ADA requires people with disabilities to pay for special accommodations
B. the ADA is designed to protect the civil rights of many people
C. public transportation must accommodate the needs of people with disabilities
D. the ADA protects the rights of people with mental impairments
Question 49. The word “stipulates” in bold in paragraph 5 is closest in meaning to___________.
A. supposes B. admits C. states D. requests
Question 50. It can be inferred from the passage that___________ .
A. restaurants can refuse service to people with disabilities
B. every car of a train must be accessible to persons with disabilities
C. the ADA is not well-liked by employers

Trang 7
D. large companies may not discriminate against workers with disabilities

Trang 8
Đáp án
1-D 2-C 3-C 4-D 5-D 6-C 7-B 8-C 9-B 10-D
11-A 12-D 13-C 14-B 15-D 16-D 17-A 18-B 19-B 20-A
21-C 22-A 23-B 24-B 25-C 26-B 27-A 28-A 29-D 30-A
31-A 32-D 33-B 34-D 35-A 36-D 37-A 38-C 39-B 40-D
41-B 42-C 43-B 44-C 45-A 46-D 47-B 48-A 49-C 50-D

LỜI GIẢI CHI TIẾT


Question 1: Đáp án D
Đáp án D đúng vì phần gạch chân của đáp án D là âm /g/. Các đáp án còn lại có phần gạch chân là âm
/dʒ/.
A. general /ˈdʒenrəl/ (a)/(n): tổng quan, tổng quát (a)/ đại tướng (n)
B. generous /ˈdʒenərəs/ (a): hào phóng/ rộng rãi
C. generation /ˌdʒenəˈreɪʃn/ (n): thế hệ
D. guillotine /ˈɡɪlətiːn/ (n): máy chém
Question 2: Đáp án C
Đáp án C đúng vì phần gạch chân của đáp án C là âm /s/. Các đáp án còn lại có phần gạch chân là âm /ɪz/.
A. changes /tʃeɪndʒ/ (n)/(v): số nhiều của danh từ “thay đổi”/ động từ “thay đổi” chia số ít
B. increases /ɪnˈkriːs/ (n)/(v): số nhiều của danh từ “sự tăng lên”/ động từ “tăng lên” chia số ít
C. suffocates /ˈsʌfəkeɪt/ (v): động từ “chết ngạt/ làm chết ngạt” chia số ít
D. compromises /ˈkɒmprəmaɪz/ (n)/ (v): số nhiều của danh từ “sự nhượng bộ”/ động từ “nhượng bộ” chia
số ít
* Mở rộng: Với đuôi -s/es, có ba cách phát âm:
1/ Phát âm là /ɪz/ khi danh từ/ động từ có âm cuối cùng là một trong sáu âm /s/, /z/, /ʃ/, /ʒ/, /tʃ/, /dʒ/. Các
âm này thường được thể hiện bằng các chữ cái sau: s, x, sh, ch, ge, z... ví dụ: kiss /kɪs/ → kisses /kɪsɪz/,
match /mætʃ/ → matches /mætʃɪz/
2/ Phát âm là /s/ khi danh từ/ động từ có âm cuối cùng là các âm vô thanh, trừ ba âm /s/, /ʃ/, /tʃ/. Các âm
vô thanh trừ ba âm này thường được thể hiện bằng các chữ cái sau: p, k, t, th.. ví dụ: stop /stɒp/ → stops
/stɒps/, breath /breθ/ → breaths /breθs/.
3/ Phát âm là /z/ khi danh từ/ động từ có âm cuối cùng là các âm hữu thanh còn lại, trừ ba âm /z/, /ʒ/, /dʒ/.
Các âm này thường được thể hiện bằng các chữ cái thể hiện nguyên âm: u, e, o, a, i, và các chữ cái thể
hiện phụ âm khác. Ví dụ: need /ni:d/ → needs /ni:dz/, employee /ɪmˈplɔɪiː/ → employees /ɪmˈplɔɪiːz/
Question 3: Đáp án C

Trang 9
Đáp án C đúng vì đáp án C có trọng âm rơi vào âm tiết thứ 1. Các đáp án còn lại có trọng âm rơi vào âm
tiết thứ 2.
A. deploy /dɪˈplɔɪ/ (v): triển khai
B. maintain /meɪnˈteɪn/ (v): duy trì/ bảo dưỡng
C. border /ˈbɔːdə(r)/ (n)/(v): biên giới (n)/ tiếp giáp biên giới với (v)
D. attain /əˈteɪn/ (v): lấy được/ đạt được.
* Mở rộng: Những động từ kết thúc với đuôi “-ain” thường có trọng âm rơi vào chính nó.
Đáp án B và D là hai ví dụ cụ thể.
Question 4. Đáp án D.
Đáp án D đúng vì đáp án D có trọng âm rơi vào âm tiết thứ 2. Các đáp án còn lại có trọng âm rơi
vào âm tiết thứ 3.
A. referee /ˌrefəˈriː/ (n): trọng tài/ người tham khảo thông tin
B. kangaroo /ˌkæŋɡəˈruː/ (n): con chuột túi
C. cigarette /ˌsɪɡəˈret/ (n): thuốc lá
D. technique /tekˈniːk/ (n): kỹ thuật
* Mở rộng: Những từ kết thúc với các đuôi “-ade, -ee, -eer, -ese, -ette, -een, -oo, -oon, -aire, -esque, -
ique, -ain, -ever” thường có trọng âm rơi vào chính nó. Các đáp án trong câu này là ví dụ cụ thể.
Question 5. Đáp án D.
Đáp án D - câu hỏi lượng từ
“consideration” - sự cân nhắc - là danh từ không đếm được, vậy nên đáp án C - many: nhiều (đi với danh
từ đếm được số nhiều) là đáp án bị loại
“A lot of/ lots of” đều có nghĩa là rất nhiều, vậy nên hai từ này sẽ không theo sau các trạng từ chỉ mức độ
“so, very, too, rather”. Vì vậy, hai đáp án A - a lot of, và B - lots of cũng bị loại. Đáp án D - much: nhiều
(đi với danh từ không đếm được) - là đáp án đúng.
Tạm dịch: Sau khi cân nhắc rất nhiều, chúng tôi cuối cùng cũng đưa ra được quyết định.
* Mở rộng: “Arrive at a decision/ solution/ compromise” là cách dùng thành ngữ của “arrive at” và có
nghĩa là: đạt được/ tìm ra.
Question 6. Đáp án C
Đáp án C - câu hỏi V + To V
Động từ “Intend” + (sb) to do sth: dự định làm gì/ dự định cho ai làm gì. Vì “intend + sb” đi với To V nên
hai đáp án A - seeing, và đáp án B - to seeing loại vì sai cấu trúc.
Động từ “see” thường có hai cách dùng:
1/ “see” đứng một mình (nội động từ) - luôn ở dạng chủ động - nghĩa là: sử dụng mắt để nhìn.

Trang 10
2/ “see” + sb/ sth (ngoại động từ) - nếu không có sb/sth (tân ngữ) theo sau thì để bị động - nghĩa là: nhìn
thấy sb/sth, cần nhắc/suy nghĩ sb/sth, suy nghĩ sb/ sth.
Câu đề bài cho có “his painting” là “sth” (tân ngữ) của “see” nên trường hợp này phải để ở dạng chủ
động. Vì vậy đáp án D - to be seen (thể bị động) - là đáp án án bị loại. Đáp án đúng là C - to see.
Tạm dịch: Tác giả có dự định không để cho bất cứ ai nhìn thấy những bức tranh của mình trước khi chúng
được hoàn thiện.
* Mở rộng: Ngoại động từ luôn luôn có sb/ sth (tân ngữ) theo sau. Nếu không có hoặc thiếu, động từ đó
sẽ để ở dạng bị động.
Question 7. Đáp án B
Đáp án B - câu bị động
Động từ “take” + sb/sth + to/into sth: đưa ai đó/ cái gì đó đến một địa điểm. Trong cách dùng này, “take”
là ngoại động từ vì có “sb” theo sau làm tân ngữ. Câu đề bài cho lại không có “sb/sth” sau chỗ trống nên
“take” không được để chủ động vì đang thiếu tân ngữ, vậy ta loại đáp án D.
Đề bài cho mệnh đề chứa “take” diễn tả một hành động xảy ra bất thường so với mọi khi. Để diễn tả hành
động xảy ra bất thường so với mọi khi, thì Hiện tại Tiếp diễn được sử dụng. Vậy nên, đáp án đúng là đáp
án B - is being taken - bị động của “take” ở thì hiện tại tiếp diễn.
Tạm dịch: Hàng ngày, mẹ lái xe đưa Peter đi học. Tuy nhiên, hôm nay, anh chàng lại được bố đưa đến
trường.
Question 8. Đáp án C
Đáp án C - câu hỏi hòa hợp chủ vị
Thời gian đề bài cho là “next month” - tháng sau (chỉ tương lai) nên chỉ có thể giữ lại hai đáp án là C - is
becoming, và D - are becoming bởi thì Hiện tại Tiếp diễn cũng được dùng để diễn tả hành động chắc chắn
sẽ xảy ra trong tương lai vì đó là kế hoạch.
Khi cụm “N1 + with/ along with/ as well as/ together with/ accompanied by/ besides/ in addition to + N2”
làm chủ ngữ trong câu, động từ được chia theo N1. Cụ thể, trong câu bài cho “Linda, along with hundreds
of others” có N1 là Linda - danh từ riêng chỉ tên người - là số ít. Vì vậy, đáp án C - is becoming (chia theo
chủ ngữ số ít) là đáp án đúng.
Tạm dịch: Linda, cùng với hàng trăm người khác sẽ trở thành thất nghiệp khi nhà máy đóng cửa vào
tháng tới.
* Mở rộng: “with/ along with/ as well as/ together with/ accompanied by/ besides/ in addition to” là các
giới từ. “Giới từ + N” tạo thành cụm giới từ - đóng vai trò làm Trạng ngữ của câu. Vì vậy, thực chất cụm
“N1 + with/ along with/ as well as/ together with/ accompanied by/ besides/ in addition to + N2” là một
cụm danh từ được hình thành bởi “N1 + Trạng ngữ”. Vậy nên, động từ phải được chia hòa hợp với N1
(danh từ chính của cụm danh từ).

Trang 11
Question 9. Đáp án B
Đáp án B - câu hỏi mệnh đề danh từ/ danh ngữ
Mệnh đề danh từ/ danh ngữ là một mệnh để được dùng như một danh từ trong câu chứ không phải là câu
hỏi nên không có đảo ngữ (đảo trợ động từ lên trước chủ ngữ), nên hai đáp án có đảo ngữ là A - what was
he carrying in his bag, và D - what does he carry in his bag là hai đáp án bị loại.
Đề bài cho động từ ở mệnh đề chính là “stopped” và “ordered” ở thì quá khứ, nên mệnh đề danh từ/ danh
ngữ (là một mệnh đề phụ) cũng phải được chia ở thì quá khứ. Vì vậy, đáp án B - what he was carrying in
his bag (quá khứ tiếp diễn) là đáp án đúng.
Tạm dịch: Các nhân viên hải quan đã chặn người đàn ông bị tình nghi lại và ra lệnh cho ông ta trình ra
những gì ông ta đang mang trong cặp của mình.
* Mở rộng: Mệnh đề phụ trong câu như mệnh đề trạng ngữ, mệnh đề danh từ/ danh ngữ, mệnh đề quan
hệ... thường sẽ tuân thủ theo thì động từ của mệnh đề chính.
Question 10. Đáp án D
Đáp án D - câu hỏi câu chẻ/ câu nhấn mạnh
Câu chẻ/ câu nhấn mạnh thường có cấu trúc: It + be + danh từ HOẶC giới từ + danh từ (cụm giới tử) +
that/ who/ which...
Trong trường hợp này, “an English student” là danh từ chỉ người, nên đáp án A - which (chỉ vật) bị loại.
Vì không có đáp án “that” nên đáp án D - who (chỉ người) là đáp án duy nhất có thể chọn.
Tạm dịch: Chính một sinh viên người Anh chứ không phải người Pháp mới là đội trưởng của đội tuyển
trường chúng tôi.
* Mở rộng: Ở câu chẻ, “that” là từ dùng cho mọi trường hợp. “Which” chỉ dùng làm chủ ngữ và đứng sau
danh từ chỉ vật. “Who” chỉ dùng làm chủ ngữ và đứng sau danh từ chỉ người.
Question 11. Đáp án A
Đáp án A - câu hỏi giới từ
Danh từ “talent” + for: tài năng về lĩnh vực gì đó.
Vì vậy, đáp án A đúng.
Tạm dịch: Người đàn ông đứng cạnh bố chúng tôi là một người nói chuyện rất thuyết phục và có tài lãnh
đạo thiên bẩm.
Question 12. Đáp án D
Đáp án D - câu hỏi trật tự tính từ trước danh từ
Các tính từ sắp xếp trước danh từ theo trật tự “OSASCOMP”, lần lượt theo thứ tự: Opinion (nhận xét,
đánh giá), Size (kích cỡ), Age (tuổi tác/ cũ mới), Shape (hình dáng), colour (màu sắc), origin (nguồn gốc),
material (nguyên liệu), và purpose (mục đích sử dụng) + Noun. Ba tính từ trong câu cho là: small (nhỏ -

Trang 12
size), round (hình tròn - shape), và green (màu xanh lá cây - colour). Theo đúng trật tự “OSASCOMP” thì
đáp án D đúng.
Tạm dịch: Mẫu báo cáo có sẵn trong hai chiếc đĩa tròn nhỏ màu xanh ở trong ngăn kéo của tôi.
* Lưu ý: Trật tự tính từ của tiếng Việt và tiếng Anh không giống nhau nên khi dịch sẽ không thể bảo toàn
được trật tự.
Question 13. Đáp án C
Đáp án C - câu hỏi từ loại
Từ chỉ số lượng “some” bắt đầu cụm danh từ. Đề bài đã cho “results” là danh từ, nên chỗ trống trước
danh từ “results” cần có một tính từ để bổ sung nghĩa cho danh từ “results”. Vì vậy, hai đáp án bị loại là B
- impresses (động từ chia số ít) và D - impression (danh từ) là hai đáp án bị loại.
Còn lại hai đáp án A - impressed và C - impressive là hai tính từ. Đến đây nghĩa của hai từ này sẽ được
cân nhắc.
A - Impressed: bị ấn tượng
C - impressive: ấn tượng/ gây ấn tượng.
Đề bài cho danh từ “results”: kết quả. Xét về nghĩa, đáp án C - impressive phù hợp hơn khi “impressive
results” có nghĩa là “những kết quả ấn tượng”.
Tạm dịch: Loại thuốc đang được kiểm nghiệm đã cho thấy những kết quả ấn tượng khi được phát cho
những người tình nguyện thử ở một vài quốc gia.
* Mở rộng: Các phân từ 1 (V-ing) và phân từ 2 (P2) cũng thường được dùng như tính từ. Khi đó, V-ing là
tính từ mang nghĩa chủ động (gây ra cảm giác....) còn P2 mang nghĩa bị động (bị tác động bởi cảm
giác...). Ví dụ: interesting: gây cảm giác thú vị/ interested: bị làm cho thấy thú vị.
Question 14. Đáp án B
Đáp án B - câu hỏi kết hợp từ (collocation) Danh từ “traffic” - giao thông - thường được kết hợp với các
tính từ: heavy, light, và busy. Vậy nên đáp án A - light và B - heavy là hai đáp án có thể chọn.
Đề bài cho trạng ngữ là “during rush hours” - trong giờ cao điểm. Giờ cao điểm là giờ giao thông đông
đúc, nên đáp án B - heavy là đáp án phù hợp.
A. light (a): nhẹ/ vắng vẻ
B. heavy (a): nặng/ đông đúc
C. substantial (a): nhiều/ đáng kể
D. enormous (a): khổng lồ
Tạm dịch: Trong giờ cao điểm, giao thông thường đông đúc và xe ô tô nối đuôi dồn sát nhau.
Question 15. Đáp án D
Đáp án D - câu hỏi về lựa chọn từ.
Đáp án D - offer (nộp) phù hợp với nghĩa và kết hợp với danh từ “resignation” - đơn xin nghỉ việc.

Trang 13
A. give (v): đưa/ tặng
B. apply (v): ứng tuyển/ áp dụng
C. propose (v): đề xuất
D. offer (v): đề nghị giúp đỡ/ đưa cho/ nộp
Tạm dịch: Sau khi phải chịu đựng những vết thương nghiêm trọng trong vụ tấn công, người mật vụ đó
quyết định nộp đơn xin nghỉ việc lên tổng chỉ huy.
Question 16. Đáp án D
Đáp án D - câu hỏi thành ngữ
Thành ngữ “a tower of strength”: một người làm chỗ dựa, người bảo vệ. Xét 4 đáp án, đáp án D - tower là
phù hợp với thành ngữ.
A. source (n): nguồn/ nguồn cơn
B. sculpture (n): bức tượng
C. figure (n): con số/ nhân vật
D. tower (n): tòa tháp
Tạm dịch: Cha cô ấy thực sự là một chỗ dựa tinh thần khi cuộc hôn nhân của cô ấy tan vỡ.
Question 17. Đáp án A
Đáp án A - câu hỏi phrasal verb
Câu hỏi này cần chọn một đáp án có cụm động từ (phrasal verb) có nghĩa phù hợp nhất. Chọn A - stop off
vì:
A. stop off: ghé thăm dọc được
B. stop out: đi khuya không về nhà
C. stop up: thức khuya
D. stop in: ở nhà, không ra ngoài
Tạm dịch: Khi đi du lịch châu Âu, chúng tôi cố gắng ghé một thị trấn từ thời trung cổ trên đường đi để
khám phá nó.
Question 18. Đáp án B.
Đáp án B - câu hỏi lựa chọn từ
Đáp án B - measured (có số đo/ có độ lớn là) phù hợp về nghĩa trong câu này.
A. read (v): đọc/ viết là
B. measured (v): đo lường/ có số đo, độ lớn là
C. calculated (v): tính toán
D. estimated (v): ước lượng
Tạm dịch: Tuần trước, một trận động đất có độ lớn 6.1 độ Richter ập đến miền nam California.
Question 19. Đáp án B

Trang 14
Kiến thức đảo ngữ đặc biệt: Đảo ngữ được sử dụng khi cần đưa cụm “N + mệnh đề quan hệ” về cuối câu.
Ở câu này “the train was” sai vì kiến thức đảo ngữ đặc biệt được sử dụng. Vì thế nên đáp án B - the train
was → was the train.
Câu đúng: Just pulling out of the station was the train that we were supposed to catch, (có “N + mệnh đề
quan hệ” ở cuối câu).
Tạm dịch: Vừa mới ra khỏi nhà ga là chuyến tàu mà đáng lẽ chúng ta phải bắt.
Question 20. Đáp án A
“A fascinate account” là cụm danh từ, vị trí của “fascinate” phải là tính từ vì “account” là danh từ chính
còn “a” là mạo từ. Vì vậy, A - a fascinate account → a fascinating account, (sửa động từ “fascinate” thành
tính từ “fascinating”).
Tạm dịch: Cuốn tự truyện của Alice Thornton cung cấp một câu chuyện thú vị về cuộc sống gia đình ở
nước Anh vào thế kỉ 17.
Question 21. Đáp án C
Cấu trúc: “if there + were not/ had not been + N” dùng để giả sử một điều gì đó không tồn tại ở câu điều
kiện loại 2 và 3. Trong câu này đáp án C - had not been for → had not been (lược bỏ “for”).
* Mở rộng: “if there were not/ had not been + N” = “if it were not/ had not been for + N” (khi chủ ngữ là
“it”, trước danh từ mới có “for”.
Question 22. Đáp án A
Dịch câu đề bài: “Tôi chưa bao giờ đến Nga. Tôi nghĩ tôi sẽ đi đến đó vào năm tới” Bill nói.
A. Bill nói rằng gã chưa bao giờ đến Nga và nghĩ rằng mình sẽ đi đến đó vào năm sau.
B. không dịch vì sai ngữ pháp - động từ “thinks” không lùi thì.
C. không dịch vì sai ngữ pháp - động từ “thinks” không lùi thì.
D. không dịch vì sai ngữ pháp - động từ “thinks” không lùi thì.
Đáp án A đúng vì tuân thủ các quy tắc chuyển từ câu trực tiếp về câu gián tiếp: lùi các thì hiện tại về thì
quá khứ tương ứng, chuyển các ngôi trực tiếp (I) về ngôi gián tiếp (he), và chuyển trạng ngữ chỉ thời gian
“next year” thành “the next year”.
Ba đáp án còn lại sai vì chưa lùi thì của động từ “thinks” về thì quá khứ tương ứng.
* Mở rộng: Ở câu gián tiếp, các yếu tố về ngôi luôn luôn thay đổi so với câu trực tiếp. Tuy nhiên, các yếu
tố còn lại như thì động từ, trạng ngữ chỉ thời gian, nơi chốn, và chỉ định chỉ thay đổi khi các động từ
tường thuật “say/ tell” chia ở quá khứ. Nếu các động từ này được chia ở các thì hiện tại (tất cả các thì hiện
tại) thì không có yếu tố nào được thay đổi trừ ngôi.
Ví dụ: My mother always says: “You have to clean up your table before you go back to your room”
→ My mother always says I have to clean up my table before I go back to my room, (câu gián tiếp chỉ
thay đổi ngôi từ you → I vì động từ “say” chia ở hiện tại đơn).

Trang 15
Question 23. Đáp án B
Dịch câu đề bài: Mọi người đã từng tin rằng Jane nghỉ hưu vì sức khỏe yếu.
A. Jane vẫn còn được tin là đã nghỉ hưu vì sức khỏe yếu.
B. Jane đã từng được tin là nghỉ hưu vì sức khỏe yếu.
C. Người ta vẫn tin rằng Jane đã nghỉ hưu vì sức khỏe yếu.
D. Không dịch do không tồn tại cấu trúc này. Với các động từ tường thuật (tt), để chuyển câu chủ động
“S1 + V (tt) + that + S2 + V2 + ...” về bị động, ta có hai cách:
(1) It + be + P2(tt) + that + S2 + V2 + ....
(2) S2 + be + P2 (tt) + to V/ to have P2 + ...
Nếu xét về cấu trúc, thì ba đáp án A, B, và C đều đúng. Tuy nhiên cần phải lưu ý thêm: “be + P2” ở cả hai
mẫu câu bị động trên đều phải chia theo thì của V (tt) ở câu chủ động. Trong trường hợp này, ở câu chủ
động có “believed” chia ở quá khứ đơn, nghĩa là “be + P2” phải là “was believed”. Vì vậy, chỉ có đáp án
B đúng.
Question 24. Đáp án B
Dịch câu đề bài: Charles đáng lẽ ra đã thắng trong cuộc thi viết luận nếu nó đánh máy lại bài viết của nó.
(ngụ ý rằng trong thực tế Charles không thắng cuộc thi viết luận vì không đánh máy bài viết của mình).
A. Charles đã thắng cuộc thi viết luận mặc dù không đánh máy bài viết của mình → sai nghĩa.
B. Charles không giành được giải trong cuộc thi viết luận vì nó không đánh máy lại bài thi của mình. →
đúng
C. Việc đánh máy lại bài viết của mình khiến cho Charles thắng cuộc thi viết luận → sai nghĩa
D. Charles không giành được giải trong cuộc thi viết luận mặc dù đã đánh máy lại bài viết của mình → sai
nghĩa một vế.
Đáp án B đúng vì phù hợp với nghĩa của đề bài. Câu điều kiện loại 3 giả sử những khả năng không có thật
trong quá khứ, vì vậy khi ý nghĩa của câu điều kiện được diễn tả lại bằng câu trần thuật (câu diễn tả điều
có thật) thì cần phải viết ngược lại cho đúng sự thật. Do đó ba đáp án còn lại đều sai về nghĩa.
Question 25. Đáp án C
Dịch đề bài: Chính phủ biết về mức độ nghiêm trọng của vấn đề. Chính phủ cần phải hành động sớm.
(ngụ ý: khi biết về tầm nghiêm trọng thì cần hành động ngay.)
A. Không dịch vì sai ngữ pháp. “Whereas” (trong khi đó) phải có dấu phẩy “,” theo trước nhưng ở đáp án
A không có dấu phẩy.
B. Không dịch vì sai ngữ pháp. “So that” có hai cách dùng. Khi có dấu phẩy “,” theo trước, nó mang
nghĩa “thế cho nên”. Khi không có dấu phẩy theo trước, nó mang nghĩa “để mà” và mệnh đề theo sau nó
buộc phải có một động từ khuyết thiếu. Đáp án B dùng “so that” không có dấu “,” nhưng cũng không có
động từ khuyết thiếu.

Trang 16
C. Khi biết mức độ nghiêm trọng của vấn đề, chính phủ cần phải hành động sớm. → đúng.
D. Chính phủ biết mức độ nghiêm trọng của vấn đề. Nếu không thì, nó cần hành động sớm. → sai nghĩa
Đáp án C đúng vì dùng phân từ 1 và cụ thể là hiện tại phân từ (V-ing) nối hai mệnh đề cùng chủ ngữ (the
government), mệnh đề V-ing chỉ điều kiện cho mệnh đề sau và có nghĩa phù hợp.
Các đáp án còn lại sai ngữ pháp hoặc sai nghĩa.
Question 26. Đáp án B
Dịch câu đề bài: Chất này rất độc. Quần áo bảo hộ phải luôn được mặc.
A. Không dịch vì sai ngữ pháp. Để nối hai mệnh đề, chỉ cân dùng một liên từ. Đáp án A có hai liên từ là
“since” và “so”.
B. Chất này độc đến nỗi mà quần áo bảo hộ phải luôn được mặc. → đúng
C. Không dịch vì sai ngữ pháp. Theo ngữ pháp, “such” + danh từ. Ở đây theo sau “such” chỉ là “toxic”
(tính từ) nên C sai.
D. Không dịch vì sai cách dùng cấu trúc. Có tồn tại cấu trúc S + V + Too + adj/adv + to V: quá....nên
không thể... Tuy nhiên, nếu dùng cấu trúc này thì “S” cũng là người gây ra hành động “to V”. Ở đáp án
này, “S” là “the substance” (chất này) mà “to V” lại là “to wear” (mặc quần áo). Chất thì không thể gây ra
hành động mặc quần áo → sai.
Đáp án B đúng vì phù hợp nghĩa. Ở trong đáp án này, cấu trúc đảo ngữ với “so” được sử dụng: So + adj/
adv + trợ động từ + S + V + ... (nếu động từ chính là “be” thì đảo “be” lên trước chủ ngữ).
Question 27. Đáp án A
“Thanks for the nice gift you brought to me!” - Cám ơn vì món quà đẹp mà bạn tặng mình. Đây là lời cám
ơn.
Đối với lời cám ơn, các câu trả lời thường là: (you are) welcome/ my pleasure/ don’t mention it/ (that’s)
the least I can do... + một lời khách sáo.
Xét 4 đáp án. Chọn A là phù hợp nhất.
A. Không có chi! Thật vui vì cậu thích nó.
B. Không, không một tí nào hết.
C. Thực ra thì bản thân mình chả thích nó.
D. Được rồi! Thế cậu có biết nó giá bao nhiêu không?
Question 28. Đáp án A
“Never mind.” - không sao đâu/ đừng bận tâm. Đây là câu trả lời cho những lời xin lỗi.
Xét 4 đáp án, đáp án A phù hợp vì là lời xin lỗi.
A. Xin lỗi vì đã làm bẩn tấm thảm của cậu. Để tớ đem nó đi giặt.

Trang 17
B. Cậu có phiền khi đi ăn tối vào chủ nhật tới không? → “Would you mind/ do you mind” là mẫu câu
dùng để mời/ nhờ vả/ hỏi xin phép. Với mẫu câu này, lời đáp lại thường là “No/ Not at all” để thể hiện sự
đồng tình; Khi không đồng tình, vẫn cũng “No/ Not at all” nhưng kèm theo “But....”.
C. Cám ơn vì đã thành thật với tớ. → Lời cám ơn. Nhìn lại câu 27 để biết cách trả lời câu cám ơn.
D. Chúc mừng! Thật tuyệt vời! lời khen, chúc mừng. Với loại câu này, đáp lại bằng cách cám ơn và kèm
theo một câu khách sáo.
Question 29. Đáp án D
“Walk on eggshells” (thành ngữ): cẩn thận trong hành xử, lời ăn, tiếng nói. Chọn đáp án D - had to be
very cautious: phải vô cùng cẩn trọng - là gần nghĩa nhất với thành ngữ bài cho.
A. nói chuyện một cách lo sợ
B. chịu nhiều áp lực
C. được cho nhiều trứng
D. phải vô cùng cẩn trọng
Tạm dịch: Ở lần đầu tiên gặp bố mẹ của bạn, tôi đã phải rất cẩn trọng lời ăn tiếng nói vì tôi biết quan
điểm chính trị của họ khác với của tôi.
Question 30. Đáp án A
“Lauded” (v): phân từ 2 của “khen ngợi”. Chọn đáp án A - acclaimed: phân từ 2 của “tán dương” - là gần
nghĩa nhất với từ đề bài cho.
A. acclaim (v): tán dương
B. dictate (v): đọc chính tả/ ra lệnh
C. describe (v): miêu tả
D. ordain (v): sắc phong (trong tôn giáo).
Tạm dịch: Albert Einstein được khen ngợi như một trong những nhà vật lý lý thuyết vĩ đại nhất mọi thời
đại.
Question 31. Đáp án A
“Acquitted” (v): tuyên trắng án. chọn đáp án A - found guilty: nhận thấy có tội - là từ trái nghĩa với từ đề
bài cho.
A. found guilty: nhận thấy có tội
B. declared innocent: tuyên bố vô tội
C. charged of being faulty: bị kết tội vì có lỗi.
D. advised of appealing: được báo cho việc biện hộ
Tạm dịch: Luật sư của Jack cho rằng anh ấy có nhiều cơ hội để được tuyên trắng án nếu như không có
thêm bằng chứng nào được tìm thấy.
* Mở rộng: Advise sb of sth: thông báo cho ai về chuyện gì đó

Trang 18
Question 32. Đáp án D
“Sizeable” (a): khá lớn/ đáng kể. Đáp án D - trivial (a): không đáng kể - là đáp án trái nghĩa với từ đề bài
cho.
A. minimum (a): tối thiểu
B. considerable (a): đáng kể
C. plentiful (a): nhiều/ dư dả
D. trivial (a): không đáng kể
Tạm dịch: Việc tái chế đồ phế thải cần một ngân quỹ khá đáng kể, ở những trường hợp như vậy, các công
ty công nghiệp thường thích xuất khẩu phế thải sang các nước khác.
Question 33. Đáp án B
A. known for sth: nổi tiếng vì cái gì
B. known as sb/sth: được biết đến với tư cách là
C. such as (prep): ví dụ như là - dùng để liệt kê
D. like (v)/ (prep): thích (v)/ giống như là (prep)
Câu hỏi về giới từ đi sau “be known”. Chọn đáp án B - as vì “known as sth/sb: được biết đến với tư cách
là...” phù hợp với nghĩa của câu trong bài.
Trích bài: The United Nations children's Fund (UNICEF), originally known as the United Nations’
International Children's Emergency Fund, ...
Tạm dịch: Quỹ trẻ em Liên hiệp Quốc (UNICEF), ban đầu được biết đến với tư cách là Quỹ Hỗ trợ Khẩn
cấp Trẻ em Quốc tế...
* Mở rộng: “originally known as...” là mệnh đề quan hệ rút gọn.
Question 34. Đáp án D
A. ancestor (n): tổ tiên
B. descendant (n): hậu duệ/ con cháu
C. pioneer (n): người tiên phong
D. founder (n): người sáng lập
Câu hỏi về từ vựng, căn cứ vào nghĩa để chọn đáp án D - founder.
Trích bài: The Polish physician Ludwik Rajchman is widely regarded as the founder of UNICEF and
served as its first chairman from 1946 to 1950.
Tạm dịch: Nhà vật lý học người Ba Lan Ludwik Rajchman được biết đến rộng rãi với tư cách là người
sáng lập UNICEF và cũng đóng vai trò chủ tịch đầu tiên của tổ chức này từ năm 1946 đến năm 1950.
Question 35. Đáp án
Câu hỏi về thì động từ. Trong bài cho trạng ngữ chỉ thời gian “in 1953” - thời gian ở quá khứ nên động từ
cũng cần chia ở quá khứ. Vậy nên, đáp án A - became là đáp án đúng.

Trang 19
Trích bài: In 1953 it became a permanent part of the United Nations System.
Tạm dịch: Vào năm 1953, tổ chức này trở thành một phần thường trực của hệ thống Liên hiệp Quốc.
Question 36. Đáp án D
A. focus on sth: tập trung vào sth
B. stand on sth: đứng lên trên sth
C. concentrate on sth: tập trung vào sth
D. rely on sb/sth: phụ thuộc vào sb/ sth
Câu hỏi về từ vựng, căn cứ vào nghĩa để chọn đáp án D - relies on
Trích bài: UNICEF relies on contributions from governments and private donors.
Tạm dịch: UNICEF phụ thuộc vào sự đóng góp từ các chính phủ và những nhà hảo tâm tư nhân.
* Mở rộng: “focus on sth” và “concentrate on sth” đồng nghĩa nhưng cũng có một chút khác biệt.
“Focus on sth”: tập trung sự chú ý vào sth
“Concentrate on sth”: tập trung để suy nghĩ thật kĩ về sth
Question 37. Đáp án A
A. promote (v): thúc đẩy, đẩy mạnh
B. provide (v): cung cấp
C. widen (v): mở rộng
D. increase (v): tăng lên/ làm tăng lên.
Câu hỏi về từ vựng. Căn cứ vào nghĩa để chọn đáp án A - promote
Trích bài: “UNICEF's programs emphasize developing community-level services to promote the health
and well-being of children.”
Tạm dịch: Các chương trình của UNICEF tập trung vào phát triển các dịch vụ ở cấp độ cộng đồng để đẩy
mạnh sức khỏe và đời sống trẻ em.
Question 38. Đáp án C
Dịch đề bài: Ý chính của đoạn văn này là gì?
A. Cánh là đặc điểm thể chất quan trọng nhất của các loài chim.
B. Các loại cánh chim khác nhau tiến hóa cho các kiểu bay khác nhau.
C. Các loài chim có nhiều đặc điểm chuyên biệt để hỗ trợ sự sinh tồn của mình.
D. Các loài chim bay bởi nhiều lý do.
* Hướng dẫn làm bài:
Đối với loại câu hỏi tìm ý chính/ tiêu đề của đoạn văn, việc cần làm là:
1/ Đọc 2-3 câu đâu tiên của bài vì những câu đó là câu thể hiện chủ đề của bài viết.
2/ Đọc lướt qua nội dung của bài để chắc chắn mình xác định ý chính đúng.

Trang 20
Câu đầu tiên của bài này là: “Birds have evolved many physical attributes that contribute to their flying
ability.” Và câu cuối của bài này là: “Birds fly to find a prey, escape predators, and attract mates-in other
words, to survive.”
Tạm dịch: “Các loài chim đã tiến hóa rất nhiều đặc điểm thể chất đóng góp quan trọng cho khả năng bay
của mình.” Và “Các loài chim bay để săn mồi, trốn thoát khỏi kẻ săn mình, và thu hút bạn tình, nói cách
khác, là để sinh tồn”.
Như vậy, câu đầu tiên của bài nêu ra ý chính là về các đặc điểm thể chất được tiến hóa để phù hợp với
việc bay. Các đoạn sau của bài nói sâu hơn về các đặc điểm đó. Câu cuối cùng của bài kết luận rằng đối
với chim, bay là để tồn tại. Nghĩa là, các loài chim đã tiến hóa các đặc điểm thể chất để phù hợp với việc
bay, và để tồn tại.
Vậy nên, đáp án C đúng.
Các đáp án còn lại chỉ nói về nội dung của một phần trong bài chứ không bao quát hết bài.
Question 39. Đáp án B
Dịch đề bài: Theo như bài đọc, điều gì giúp các loài chim nâng được độ cao khi bắt đầu bay?
A. Cánh dài với những chiếc lông rỗng ruột
B. Áp suất không khí dưới cánh cao hơn ở trên cánh
C. Trải hết đuôi của chúng ra
D. Sức mạnh cơ bắp thượng đẳng
* Hướng dẫn làm bài:
Đối với loại bài tìm thông tin, cần chọn ra một vài từ khóa ở đáp án cũng như ở câu hỏi rồi tìm những từ
khóa đó ở trên bài đọc. Khi đã tìm được đoạn chứa từ khóa trên bài đọc, cần đọc kĩ và chọn lọc những
thông tin nào ủng hộ đáp án. Sẽ chỉ có một đáp án được thông tin trong bài ủng hộ, và đó là đáp án đúng.
Trích thông tin trong bài: “This creates higher pressure under the wings, called lift, which pushes the bird
up”
Tạm dịch: Điều này tạo ra áp suất cao hơn ở dưới cánh, được gọi là lực đẩy, chính lực đẩy này đẩy cho
chim nâng độ cao.
Như vậy, áp suất phía dưới cánh cao hơn trên cánh là thứ đẩy cho chim tăng độ cao.
Chọn đáp án B là đúng.
Question 40. Đáp án D
Dịch đề bài: Cụm từ “finches and sparrows” nói đến...
A. cánh
B. vận động/ diễn tập
C. cách bay
D. các loài chim

Trang 21
Câu hỏi về từ vựng. “Finches and sparrows”: Các loài chim họ sẻ Thông và chim sẻ. Chọn đáp án D -
birds: các loại chim - là gần nghĩa nhất với cụm từ bài cho.
* Mở rộng: Đối với loại bài chọn từ gần nghĩa/ trái nghĩa, nếu không biết nghĩa của từ, có thể dựa vào dữ
liệu xung quanh để đoán nghĩa của từ/ cụm từ bài cho.
Ví dụ như câu hỏi này, bài đọc cho: “Finches and sparrows have short, broad wings”: Finches and
sparrows có cánh ngắn và rộng. Vì bài đọc đang nói về cánh chim, nên có thể suy luận được “Finches and
sparrows” là tên của hai loài chim (vì có cánh).
Question 41. Đáp án B
Dịch đề bài: Theo như bài đọc, lợi ích của việc có sẵn các cấu cản dòng là gì? (spoiler - cấu cản dòng -
cấu cản không khí để phanh máy bay)
A. khả năng bay nhanh hơn
B. góc chúi xuống săn mồi đứng hơn
C. các chuyến bay dài hơn
D. tính cơ động cao hơn khi tăng độ cao
Câu hỏi tìm thông tin. Đọc hướng dẫn làm bài ở câu 39.
Từ khóa của câu hỏi là “spoiler”.
Trích thông tin trong bài: “Faster birds, like hawks have built - in spoilers that reduce turbulence while
flying. This allows a steeper angle of attack without stalling.”
Tạm dịch: Các loài chim bay nhanh hơn, ví dụ như diều hâu, có sẵn những cấu cản dòng để giảm nhiễu
động khi đang bay. Điều này cho phép có góc tấn công đứng hơn mà không cần phải dừng lại đột ngột khi
bay.
Như vậy, lợi ích của việc có sẵn cấu cản dòng là một góc đứng hơn khi tấn công con mồi.
Vậy, chọn B là hợp lý.
Question 42. Đáp án C
Dịch đề bài: Tác giả ngụ ý điều vì về việc những người tập thể hình nếu họ có cánh?
A. Nếu họ vỗ cánh, họ có thể bay một chút.
B. Nếu họ có cánh, cơ bắp của họ sẽ đủ khỏe để bay.
C. Nếu họ có cánh, tim họ vẫn không đủ lớn để bay được.
D. Cánh của họ sẽ chiếm tổng cộng 15 phần trăm trọng lượng cơ thể.
* Hướng dẫn làm bài:
Đối với loại câu hỏi suy luận, đọc kĩ 4 đáp án và tìm thông tin hỗ trợ trong bài. Chắc chắn chỉ có một đáp
án được thông tin trong bài hỗ trợ, các đáp án còn lại đều sai hoặc không có thông tin trong bài. Lưu ý
phải tìm thông tin dựa vào từ khóa, và không dùng kiến thức bản thân để trả lời loại câu hỏi này. Trích
thông tin trong bài: “If body builders had wings, they still could not flap hard enough to leave the ground.

Trang 22
Birds have large, specialized hearts that beat much faster than the human heart and provide the necessary
oxygen to the muscles.”
Tạm dịch: Nếu những người tập thể hình có cánh, họ cũng không thể đập cánh đủ mạnh để rời bay lên
khỏi mặt đất. Chim có trái tim lớn và chuyên biệt, có thể đập nhanh hơn tim người rất nhiều và cung cấp
đủ lượng ô-xi cần thiết cho các cơ bắp”.
Vậy, dù những người tập thể hình có cánh, trái tim của họ vẫn không đủ lớn và bơm đủ máu đến các cơ
bắp như tim của các loài chim.
Vì vậy, đáp án C là đáp án đúng.
Question 43. Đáp án B
Dịch đề bài: Mục đích chính của đoạn văn này là gì?
A. để miêu tả sự phân biệt kì thị với người khuyết tật.
B. để giải thích các quy định trong Đạo luật Người Mỹ với Người Khuyết tật.
C. để đưa ra gợi ý thuê những người khuyết tật.
D. để bàn bạc về các thiết bị viễn thông cho người khiếm thính.
* Hướng dẫn làm bài:
Đối với loại câu hỏi tìm ý chính/ tiêu đề của đoạn văn, việc cần làm là:
1/ Đọc 2-3 câu đầu tiên của bài vì những câu đó là câu thể hiện chủ đề của bài viết
2/ Đọc lướt qua nội dung của bài để chắc chắn mình xác định ý chính đúng.
Câu đầu tiên của bài này là: “The Americans i with Disabilities Act (ADA) was signed into law in 1990”
Tạm dịch: “Đạo luật Người Mỹ với Người Khuyết tật được thông qua thành luật vào năm 1990.”
Như vậy, có thể thấy bài đọc này có chủ đề 1 là Đạo luật Người Mỹ với Người Khuyết tật. Thêm nữa, ở
các đoạn văn dưới, các cụm “The ADA States that” (Đạo luật này quy 3 định) hoặc “The ADA also
stipulats that” (Đạo luật này cũng quy định thêm), nghĩa là, bài này viết ra với mục đích chính là giải
thích các quy định của đạo luật này.
Vậy, đáp án B là đúng.
Question 44. Đáp án C
Dịch đề bài: Theo như bài đọc, tất cả những đối tượng dưới đây đều bị ảnh hưởng bởi Đạo luật Người Mỹ
với Người Khuyết tật, trừ……
A. một người gặp khó khăn trong việc đi lại
B. một cơ quan vận tải công cộng
C. một chủ thuê lao động có ít hơn 15 nhân viên
D. một người với tiền sử bị khuyết tật
* Hướng dẫn làm bài:

Trang 23
Đối với loại câu hỏi tìm ý sai hoặc tìm ý đúng (hoặc tất cả đều đúng/ sai ngoại trừ...), cần phải đọc hết cả
4 đáp án và tìm thông tin trong bài ủng hộ các đáp án đó. Nếu đáp án nào không có thông tin ủng hộ hoặc
ngược lại với thông tin trong bài, thì đó là đáp án sai. Việc tìm thông tin ủng hộ đáp án phải dựa vào từ
khóa của đáp án và trên bài đọc. Tuyệt đối không được dùng suy luận từ kiến thức của bản thân để trả lời
loại câu hỏi này mà phải dựa hết vào bài đọc.
Trích thông tin trong bài:
1/ “A person with disability is defined as someone with a mental or physical impairment that substantially
limits him or her in a major life activity, such as walking, talking, working, or self-care”: Một người
khuyết tật được định nghĩa là một người có một khiếm khuyết về trí tuệ cũng như thể chất có thể giới hạn
họ một cách đáng kể trong những hoạt động sống chính ví dụ như đi lại, nói năng, làm việc, hay tự chăm
sóc bản thân. → người gặp khó khăn trong việc đi đứng được coi là người khuyết tật → thông tin ủng hộ
đáp án A.
2/ “All new vehicles purchased by public transit authorities must be accessible to people with
disabilities”: Tất cả các phương tiện mới được mua về bởi các công ty vận tải công cộng phải tạo điều
kiện lên xuống dễ dàng cho người khuyết tật. → các cơ quan vận tải công cộng cũng bị điều chỉnh bởi
luật. → thông tin ủng hộ đáp án B.
3/ “The ADA States that employers with fifteen or more employees may not refuse to hire or promote a
person because of a disability if that person is qualified to perform the job.”: Đạo luật cũng quy định rằng
những người chủ thuê lao động có từ 15 nhân viên trở lên sẽ không được phép từ chối thuê hoặc đề bạt
thăng chức một người vì người đó bị khiếm khuyết nếu như họ có đủ khả năng để thực hiện công việc. →
chủ thuê lao động dưới 15 nhân viên không bị điều chỉnh bởi luật → thông tin không ủng hộ đáp án C.
4/ “A person with a disability may also be someone with a past record of such an impairment.”: Một
người khuyết tật cũng có thể là một người có tiền sử bị các loại khiếm khuyết nêu trên. → người có tiền
sử khuyết tật cũng được coi là người khuyết tật → thông tin ủng hộ đáp án D.
Vậy chọn đáp án C là đối tượng không bị luật tác động.
Question 45. Đáp án A
Dịch đề bài: Từ “impairment” in đậm trong đoạn một gần nghĩa nhất với...
A. khuyết tật
B. sự vi phạm
C. khả năng/ năng khiếu
D. mối hiểm nguy
Câu hỏi từ vựng, “impairment” (n): khuyết tật/ khiếm khuyết, chọn đáp án A - disability (n): khuyết tật -
là gần nghĩa nhất đối với từ đề bài cho.
Question 46. Đáp án D

Trang 24
Dịch đề bài: Tác giả nhắc đến cửa hàng tạp hóa như một ví dụ của...
A. vận chuyển công cộng
B. các rào cản
C. công việc ở lĩnh vực tư nhân
D. các tiện ích công cộng
* Hướng dẫn làm bài:
Đối với loại bài tìm thông tin, cần chọn ra một vài từ khóa ở đáp án cũng như ở câu hỏi rồi tìm những từ
khóa đó ở trên bài đọc. Khi đã tìm được đoạn chứa từ khóa trên bài đọc, cần đọc kĩ và chọn lọc những
thông tin nào ủng hộ đáp án. Sẽ chỉ có một đáp án được thông tin trong bài ủng hộ, và đó là đáp án đúng.
Trích thông tin trong bài: “Public accom-modations are businesses and services such as restaurants,
hotels, grocery stores, and parks.”
Tạm dịch: Các tiện ích công cộng là các cơ sở kinh doanh và dịch vụ như nhà hàng, khách sạn, cửa hàng
tạp hóa, và công viên.
Như vậy, cửa hàng tạp hóa là một loại tiện ích công cộng.
Vậy, đáp án D đúng
Question 47. Đáp án B
Dịch đề bài: Từ “facilities” in đậm ở đoạn 3 nói đến....
A. các rào cản
B. các tòa nhà
C. các ga tàu
D. các khuyết tật
* Hướng dẫn làm bài: Đối với loại câu hỏi này, đọc ngược lại 1-2 câu trước nó để tìm từ mà nó đang
thay thế.
Trích thông tin trong bài: “All new buildings must be made accessible, and existing facilities must
remove barriers if the removal can be accomplished without much difficulty or expense.”
Tạm dịch: Tất cả các tòa nhà mới xây đều phải dễ vào ra với người khuyết tật, và những phần cơ sở vật
chất có sẵn phải tháo dỡ hàng rào cản trở nếu như việc tháo dỡ có thể được thực hiện không quá khó khăn
và tốn kém.
Vậy, từ “facilities” (n) - phần cơ sở vật chất có sẵn - là từ dùng thay thế cho “buildings” (n): các tòa nhà.
Chọn đáp án B là đúng
Question 48. Đáp án A
Dịch đề bài: Tác giả ngụ ý tất cả những điều dưới đây trừ....
A. Đạo luật yêu cầu người khuyết tật phải trả tiền cho những tiện ích đặc biệt.
B. Đạo luật được thiết kế để bảo vệ quyền dân sự của nhiều người.

Trang 25
C. Giao thông công cộng phải đáp ứng được nhu cầu của người khuyết tật.
D. Đạo luật bảo vệ quyền của người có khiếm khuyết về mặt trí tuệ.
Câu hỏi loại trừ. Đọc hướng dẫn làm bài ở câu 44.
Trích thông tin trong bài:
1/ “This law extends civil rights protection to persons with disabilities in private sector employment, all
public services, and in public accommodations, transportation, and telecommunications.”: Luật này mở
rộng sự bảo vệ quyền dân sự của những người khuyết tật trong vấn đề việc làm ở lĩnh vực tư nhân, tất cả
các dịch vụ công, và các tiện ích công cộng, giao thông vận tải, và viễn thông.” → luật này bảo vệ quyền
dân sự của nhiều người ở nhiều lĩnh vực → thông tin ủng hộ đáp án B.
2/ “All new vehicles purchased by public transit authorities must be accessible to people with
disabilities”: Tất cả các phương tiện mới được mua về bởi các công ty vận tải công cộng phải tạo điều
kiện lên xuống dễ dàng cho người khuyết tật. → các phương tiện giao thông công cộng phải đáp ứng được
nhu cầu của người khuyết tật → thông tin ủng hộ đáp án C.
3/ “A person with disability is defined as someone with a mental or physical impairment that substantially
limits him or her in a major life activity, such as walking, talking, working, or self-care”: Một người
khuyết tật được định nghĩa là một người có một khiếm khuyết về trí tuệ cũng như thể chất có thể giới hạn
họ một cách đáng kể trong những hoạt động sống chính ví dụ như đi lại, nói năng, làm việc, hay tự chăm
sóc bản thân. → người có khiếm khuyết về trí tuệ cũng được coi là người khuyết tật và được luật bảo vệ
→ thông tin ủng hộ đáp án D.
Đáp án A không được nhắc đến trong bài → không được thông tin trong bài ủng hộ.
Chọn đáp án A là đúng.
Question 49. Đáp án C
Dịch đề bài: Từ “stipulates” in đậm ở đoạn 5 gần nghĩa nhất với từ...
A. cho rằng
B. thú nhận
C. quy định/ khẳng định
D. đề nghị
Câu hỏi từ vựng, “stipulates” (v): quy định. Chọn đáp án C - States (v): quy định - là gần nghĩa nhất
với từ đề bài cho.
Question 50. Đáp án D
Dịch đề bài: Có thể suy ra từ bài đọc rằng...
A. Các nhà hàng có thể từ chối phục vụ người khuyết tật.
B. Tất cả các toa của một đoàn tàu phải dễ lên xuống đối với người khuyết tật.
C. Đạo luật không được ủng hộ bởi những người chủ thuê công nhân.

Trang 26
D. Các công ty lớn không được phép phân biệt đối xử những nhân viên khuyết tật
* Hướng dẫn làm bài:
Đối với loại câu hỏi suy luận, đọc kĩ 4 đáp án và tìm thông tin hỗ trợ trong bài. Chắc chắn chỉ có một đáp
án được thông tin trong bài hỗ trợ, các đáp án còn lại đều sai hoặc không có thông tin trong bài. Lưu ý
phải tìm thông tin dựa vào từ khóa, và không dùng kiến thức bản thân để trả lời loại câu hỏi này. Trích
thông tin trong bài:
1/ “It is illegal for public accommodations to exclude or refuse persons with disabilities. Public
accommodations are businesses and services such as restaurants, hotels, grocery stores, and parks”: Việc
các cơ sở tiện ích công cộng loại trừ hoặc từ chối người khuyết tật là bất hợp pháp. Các tiện ích công
cộng là các cơ sở kinh doanh và dịch vụ như nhà hàng, khách sạn, cửa hàng tạp hóa, và công viên.” →
nhà hàng là cơ sở tiện ích công cộng, và việc từ chối phục vụ người khuyết tật là phạm pháp → thông tin
không ủng hộ đáp án A.
2/ “at least one car per train in existing rail systems must be made accessible”: Trên mỗi đoàn tàu hiện có
phải có ít nhất một toa dễ lên xuống đối với người khuyết tật. → chỉ cần có ít nhất một toa tàu dễ lên
xuống với người khuyết tật thông tin không ủng hộ đáp án B.
3/ “The ADA States that employers with fifteen or more employees may not refuse to hire or promote a
person because of a disability if that person is qualified to perform the job.”: Đạo luật cũng quy định rằng
những người chủ thuê lao động có từ 15 nhân viên trở lên sẽ không được phép từ chối thuê hoặc đề bạt
thăng chức một người vì người đó bị khiếm khuyết nếu như họ có đủ khả năng để thực hiện công việc. →
các công ty lớn chắc chắn không được phân biệt đối xử với nhân viên khuyết tật → thông tin ủng hộ đáp
án D.
Đáp án C không có thông tin trong bài. Vậy, chọn đáp án D là đúng

Trang 27
ĐỀ SỐ 04 ĐỀ THI THỬ TỐT NGHIỆP THPT
NĂM HỌC: 2020 – 2021
MÔN: TIẾNG ANH
Thời gian làm bài: 60 phút; không kể thời gian phát đề
Mark the letter A, B, C, or D on your answer sheet to indicate the word whose underlined part
differs from the other three in pronunciation in each of the following questions.
Question 1. A. divisible B. design C. disease D. excursion
Question 2. A. borrowed B. conserved C. approached D. complained
Mark the letter A, B, C or D on your answer sheet to indicate the word that differs from the rest in
the position of the main stress in each of the following questions.
Question 3. A. garbage B. dissolve C. bottle D. fishing
Question 4. A. beautiful B. happiness C. unhealthy D. neigh borhood
Mark the letter A, B, C or D on your answer sheet to indicate the correct answer to each of the
following questions.
Question 5. She looked active in___________.
A. a jean B. a jeans C. jean D. jeans
Question 6. We___________by a loud noise during the night.
A. woke up B. are woken up C. were woken up D. were waking up
Question 7. why___________she___________to be studying abroad?
A. does/ consider B. is/ considered C. is/ considering D. has/ considered
Question 8. Tom has the___________stamps in the collectors club.
A. most B. less C. more D. least
Question 9. Our boss would rather___________during the working hours.
A. us not chatting B. we didn’t chat C. us not chat D. we don’t chat
Question 10. ___________my car broke down, I came home late yesterday.
A. Since B. while C. For D. With
Question 11. ___________I first met my girlfriend.
A. It was in London that B. It was in London where
C. It was London that D. It was London which
Question 12. Last weekend, my family went to the cinema together. We chose a(an) ___________to
see.
A. new interesting film B. film interesting new
C. interesting film new D. interesting new film
Question 13. He has been very interested in doing research on___________since he was at high school.
A. biology B. biological C. biologist D. biologically
Question 14. The match will be televised___________ on VTV 3 tonight.

Trang 1
A. live B. lived C. lively D. living
Question 15. The students are conducting a major___________.
A. exam B. experiment C. test D. experience
Question 16. “The suitcase isn’t too heavy, is it?” “No, it’s as light as___________.”
A. dust B. lightning C. a feather D. a fish
Question 17. “Have you heard about the Welshman, the Irishman and a pig?” “Yes, we have. That
joke’s as old as___________.”
A. Solomon B. the hills C. a dinosaur D. Jupiter
Question 18. I applied for the job but was___________.
A. taken away B. got over C. turned down D. turned off
Mark the letter A, B, C, or D on your answer sheet to indicate the underlined part that needs
correction in each of the following questions.
Question 19. If anyone drops by while I am away, please take a message from him
A. drops B. while C. take D. him
Question 20. The fire began in the tenth floor of the block of flat, but it soon spread to other floors.
A. in B. tenth C. of D. soon spread
Question 21. The strong progress in poverty reduction and sharing prosperity that took place over the
first decade of the 2000s is at risk because of the global slowdown in growth.
A. reduction B. sharing C. the first D. at risk
Mark the letter A, B, C or D on your answer sheet to indicate the sentence that is closest in
meaning to each of the following questions.
Question 22. “Shall I make you a cup of milk?” the mother said to the son.
A. The mother wanted to make a cup of milk for the son.
B. The mother was asked to make a cup of milk for the son.
C. The mother promised to make a cup of milk for the son.
D. The mother offered to make a cup of milk for the son.
Question 23. The only student who failed the exam was Finet.
A. Together with other students, Finet failed the exam.
B. Everyone passed the exam.
C. Everyone, apart from Finet, failed the exam.
D. With the exception of Finet, everyone passed the exam.
Question 24. She had only just put the phone down when her boss rang back.
A. Hardly she puts the phone down when her boss rang back.
B. Hardly had she put the phone down when her boss rang back.
C. Hardly did she put the phone down than her boss rang back.
D. Hardly she had put the phone down when her boss rang back.

Trang 2
Mark the letter A, B, Cor D on your answer sheet to indicate the sentence that best combines each
pair of sentences in the following questions.
Question 25. Her explanation was clear. I didn’t understand it.
A. Despite of her clear explanation, I didn’t understand it.
B. Despite her clear explanation, I didn’t understand it.
C. Despite the fact that I didn’t understand it, her explanation was clear.
D. In spite of the fact that I didn’t understand it, her explanation was clear
Question 26. Dan is able to make English instruction videos for Vietnamese people. It is because
his Vietnamese wife helps him.
A. If only Dan were able to make English instruction videos for Vietnamese people.
B. If it weren’t for his Vietnamese wife’s help, Dan couldn’t make English instruction videos for
Vietnamese people.
C. Without his Vietnamese wife’s help, Dan couldn’t have made English instruction videos for
Vietnamese people.
D. But for his Vietnamese wife’s help, Dan couldn’t have made English instruction videos for
Vietnamese people.
Mark the letter A, B, C or D on your answer sheet to indicate the most suitable response to
complete each of the following exchanges.
Question 27. Two students are discussing their previous English class.
- Student 1. “I think the teacher should give us more exercises.”
- Student 2. “___________”
A. Yes, let’s B. Ok
C. That’s rubbish D. That’s what I was thinking
Question 28. Eden and Edward are chatting after work. Eden suggests eating out.
- Eden. “Shall we eat out tonight?”
- Edward. ___________!”
A. That’s all right B. That’s a great idea
C. It is very kind of you. D. You are very welcome.
Mark the letter A, B, C, or D on your answer sheet to indicate the word or phrase that is
CLOSEST in meaning to the underlined part in each of the following questions.
Question 29. People are not aware of the problem of overpopulation.
A. incident B. issue C. trouble D. case
Question 30. He drives me to the edge because he never stops talking.
A. frightens me B. cheers me C. irritates me D. surprises me
Mark the letter A, B, C, or D on your answer sheet to indicate the word or phrase that is
OPPOSITE in meaning to the underlined part in each of the following questions.

Trang 3
Question 31. In this world, emotion has become suspect- the accepted style is smooth, antiseptic and
passionless.
A. soft B. agitated C. silky D. easy
Question 32. Sorry, I can’t come to your party. I am snowed under with work at the moment.
A. busy with B. free from C. relaxed about D. interested in
Read the following passage and mark the letter A, B, C or D on your answer sheet to indicate the
correct word for each of the blanks from 33 to 37.
Is it worth reading books, (33) ___________ nowadays there are so many other forms of entertainment?
Some people say that even paperback books are expensive, and not everyone can borrow books from a
library. They might add that television is more exciting and that viewers can relax as they watch their
favourite (34) ___________. All that may be true, but books are still very popular. They encourage the
reader to use his or her imagination for a start. You can read a chapter of a book, or just a few pages,
and then stop. Of course, it may be so (35) ___________that you can't stop! There are many different
kinds of books, so you can choose a crime novel or an autobiography, or a book (36)
___________gives you interesting information. If you find it hard to choose, you can read reviews, or
ask friends for ideas. Personally, I can't do without books, but I can (37) ___________up television
easily enough. You can't watch television at bus stops!
Question 33. A. in B. or C. why D. since
Question 34. A. ones B. programmes C. episodes D. cereals
Question 35. A. current B. imagined C. interest D. gripping
Question 36. A. whose B. which C. what D. when
Question 37. A. pick B. look C. give D. turn
Read the following passage and mark the letter A, B, C, or D on your answer sheet to indicate the
correct answer to each of the questions from 38 to 42.
Harvard University, today recognized as part of the top echelon of the worlds universities, came from
very inauspicious and humble beginning.
This oldest of American universities was founded in 1636, just sixteen years after the Pilgrims landed at
Plymouth. Included in the Puritan emigrants to the Massachusetts colony during this period were more
than 100 graduates of England's prestigious Oxford and Cambridge universities, and these universities
graduates in the New World were determined that their sons would have the same educational
opportunities that they themselves had had. Because of this support in the colony for an institution of
higher learning, the General Court of Massachusetts appropriated 400 pounds for a college in October
of 1636 and early the following year decided on a parcel of land for the school; this land was in an area
called Newetowne, which was later renamed Cambridge after its English cousin and is the site of the
present-day university.

Trang 4
When a young minister named John Harvard, who came from the neighboring town of Charlestown,
died from tuberculosis in 1638, he willed half of his estate of 1,700 pounds to the fledgling college. In
spite of the fact that only half of the bequest was actually paid, the General Court named the college
after the minister in appreciation for what he had done. The amount of the bequest may not have been
large, particularly by today's standard, but it was more than the General Court had found it necessary to
appropriate in order to open the college.
Henry Dunster was appointed the first president of Harvard in 1640, and it should be noted that in
addition to serving as president, he was also the entire faculty, with an entering freshmen class of four
students. Although the staff did expand somewhat, for the first century of its existence the entire
teaching staff consisted of the president and three or four tutors.
Question 38. What is the main idea of the passage?
A. Harvard is one of the world’s most prestigious universities
B. What is today a great university started out small
C. John Harvard was key to the development of a great university
D. Harvard University developed under the auspices of the General Court of Massachusetts
Question 39. The pronoun "they" in the second paragraph refers to___________
A. Oxford and Cambridge universities B. university graduates
C. the Puritan sons D. educational opportunities
Question 40. The word "somewhat" in the last paragraph could best be replaced by___________
A. to and fro B. back and forth C. side by side D. more or less
Question 41. Which of the following is NOT mentioned about John Harvard?
A. What he died of B. where he came from
C. Where he was buried D. How much he bequeathed to Harvard
Question 42. The passage implies that___________
A. Henry Dunster was an ineffective president
B. someone else really served as president of Harvard before Henry Dunster
C. Henry Dunster spent much of his time as president managing the Harvard faculty
D. the position of president of Harvard was not merely an administrative position in the early years
Read the following passage and mark the letter A, B, C, or D on your answer sheet to indicate the
correct answer to each of the questions from 43 to 50.
Psychologists who study information processing have identified and described several memory
structures that clarify how our memory works. They include the sensory register, short-term memory,
and long-term memory. Each structure varies as to how much information it can hold and for how long.
A description of how human process information typically begins with environmental stimuli. Our
sense receptors are constantly stimulated by visual, auditory, tactile, olfactory, and gustatory stimuli.
These experiences are initially recorded in the sensory register, so named because information is

Trang 5
thought to be encoded there in the same form in which it was perceived. The purpose of the sensory
register is to hold information one to three seconds. Information not recognized or otherwise selected
by us disappears from the system. The sensory register can hold about twelve items of information at a
time. Typists make extensive use of the sensory register in order to remember words just long enough
to get them typed. If no further processing takes place, a typist’s ability to recall that information later
is minimal. Similarly, most of us have had the experience of reading an entire page of text, only to
discover when we got to the bottom of the page, we couldn’t say anything about it except that we had
indeed “read” every word.
Once information has been recognized as meaningful, it is sent to short-term memory. In this case,
short-term is approximately 20 seconds, while this may seem surprising, it can be easily demonstrated.
If you were asked to dial an unfamiliar phone number, received a busy signal, and were then distracted
by something or someone else for 15 to 20 seconds, chances are you would have forgotten the number
at that point. Short-term memory is often referred to as “working” memory.
Most cognitive psychologists believe that the storage capacity of long-term memory is unlimited and
contains a permanent record of everything an individual has learned and experienced. Information is
encoded there to enhance its meaningfulness and organization so that it can be easily retrieved when
necessary.
Question 43. what is the purpose of the passage?
A. to explain how our memory processes information.
B. to describe the sensory register.
C. to explain why we sometimes forget information.
D. to compare short-term and long-term memory.
Question 44. The word “They” in paragraph 1 refers to___________.
A. psychologists B. information C. memory structures D. environmental stimuli
Question 45. The word “stimuli” in lines 4 and 5 is closest in meaning to___________.
A. objects or events that activate our memory
B. objects we consider attractive
C. things that help us to later recall what happened
D. situations in which we experience emotions
Question 46. According to the passage, typists are unable to recall information they type
if___________.
A. they are tired
B. they are distracted by something or someone
C. they have too much work to be able to process it all
D. they do not recognize it as meaningful enough to remember
Question 47. The word “minimal” in paragraph 2 is closet in meaning to___________.

Trang 6
A. very big B. very good C. very pretty D. very small
Question 48. According to the passage, which type of information is sent to short-term memory?
A. Information we need for three seconds or less.
B. Information that surprises us.
C. Information that is relevant to us.
D. Environmental stimuli we do not perceive.
Question 49. It can be inferred that short-term memory is called “working” memory
because___________.
A. we use it extensively when we are working
B. it holds information we are working on at a given moment
C. it is very difficult to use effectively
D. we must work hard to retrieve information from it.
Question 50. which of the following would we most easily retrieve from long-term memory?
A. A wrong telephone number we dialed.
B. The face of a stranger on the street.
C. The birth date of our children
D. Voices from the television in the background
Đáp án
1-D 2-C 3-B 4-C 5-D 6-C 7-B 8-A 9-B 10-A
11-A 12-D 13-A 14-A 15-B 16-C 17-B 18-C 19-D 20-A
21-B 22-D 23-D 24-B 25-B 26-B 27-D 28-B 29-B 30-C
31-B 32-B 33-D 34-B 35-D 36-B 37-C 38-B 39-B 40-D
41-C 42-D 43-A 44-C 45-A 46-D 47-D 48-C 49-B 50-C

LỜI GIẢI CHI TIẾT


Question 1: Đáp án D
Đáp án D đúng vì phần gạch chân của đáp án D đọc là âm /ʃ/ còn phần gạch chân của các đáp án khác
được đọc là âm /z/
A. divisible /dɪˈvɪzəbl/ (adj): chia hết cho...
B. design /dɪˈzaɪn/ (v): thiết kế
C. disease /dɪˈziːz/ (n): bệnh tật
D. excursion /ɪkˈskɜːʃn/ (n) chuyến tham quan
Question 2. Đáp án C
Đáp án C đúng vì phần gạch chân của đáp án C đọc là âm /t/ còn phần gạch chân của các đáp án còn lại
được đọc là âm /d/
A. borrowed /ˈbɒrəʊd/ (v): mượn, vay

Trang 7
B. conserved /kənˈsɜːvd/ (v): bảo tồn, giữ gìn
C. approached /əˈprəʊtʃt/ (v): tiếp cận, đến gần
D. complained /kəmˈpleɪnd/ (v): phàn nàn, than phiền
Question 3. Đáp án B
Đáp án B đúng vì đáp án B trọng âm rơi vào âm tiết thứ hai còn ba đáp án còn lại trọng âm rơi vào âm tiết
thứ nhất.
A. garbage /ˈɡɑːbɪdʒ/ (n): rác thải
B. dissolve /dɪˈzɒlv/ (v): phân hủy, tan
C. bottle /ˈbɒtl/ (n): chai lọ
D. fishing /ˈfɪʃɪŋ/ (n): việc câu cá
❖ For review
Hầu hết động từ có hai âm tiết thì trọng ầm rơi vào âm tiết thứ hai, hâu hết danh từ, tính từ có hai âm tiết
thì trọng âm rơi vào âm tiết nhất.
Question 4.
Đáp án C đúng vì đáp án C trọng âm rơi vào âm tiết thứ hai còn 3 đáp án còn lại trọng âm rơi vào âm tiết
thứ nhất.
A. beautiful /ˈbjuːtɪfl/ (adj): xinh đẹp
B. happiness /ˈhæpinəs/ (n): niềm hạnh phúc
C. unhealthy /ʌnˈhelθi/ (adj): không khỏe mạnh
D. neighborhood /ˈneɪbəhʊd/ (n): hàng xóm, vùng lân cận
❖ For review
Từ có ba âm tiết được biến đổi từ việc thêm tiền tố un- hoặc các hậu tố như -fill, -hood, -ness thì trọng âm
sẽ không thay đổi so với từ gốc.
Question 5. Đáp án D
Ta thấy jeans là danh từ luôn ở dạng thức số nhiều. Vì vậy ta chọn được đáp án D.
Tạm dịch. Trông cô ấy rất năng động khi mặc quần jeans.
Question 6. Đáp án C
Ta có cụm từ “by a loud noise”: bởi một tiếng động lớn → dùng bị động nên loại A và D (động từ đang
chia ở chủ động). Ta thấy trong câu có cụm từ chỉ thời gian “during the night”: trong đêm → hành động
đã xảy ra rồi → động từ chia thời quá khứ đơn. Vì vậy ta chọn được đáp án C.
Tạm dịch. Chúng tôi bị đánh thức bởi một tiếng động lớn trong đêm.
Question 7. Đáp án B
Chúng ta thấy consider là một ngoại động từ nhưng đằng sau lại không có tân ngữ → chia ở dạng bị động.
Vì vậy ta chọn được đáp án B. Đây là câu hỏi có sử dụng cấu trúc bị động của động từ nêu ý kiến.
Tạm dịch. Tại sao cô ấy lại được cân nhắc để đi du học?
Question 8. Đáp án A
Trang 8
Ta thấy “stamp” là danh từ đếm được → loại đáp án B và D (less và least là so sánh hơn và so sánh nhất
của little - little chỉ đi cùng với danh từ không đếm được). Trong câu ta thấy có sự xuất hiện của cụm từ
“in the club”. Thông thường với cấu trúc so sánh nhất, trong câu sẽ xuất hiện các cụm từ như in/ of +
danh từ tập hợp. Vì vậy đáp án phù hợp là đáp án A
Tạm dịch. Trong câu lạc bộ thì Tom có nhiều tem nhất.
Question 9. Đáp án B
Đây là câu giả định với cấu trúc của would rather. Ta có cấu trúc:
S1+ would rather + S2 + V (quá khứ đơn)/ were vì vậy đáp án B phù hợp.
❖For review
Câu giả định sử dụng would rather được dùng để diễn đạt điều gì đó mong muốn được thực hiện trong
một tình huống cụ thể.
- Bản thân mong muốn làm việc gì/ không muốn làm việc gì, ta có thể sử dụng cấu trúc: S + would rather
+ (not) V + (than V)
- Bản thân mong muốn ai đó làm gì/ không làm gì, ta có cấu trúc:
S1+ would rather+ S2 + V-ed (quá khứ đơn)/ didn’t V
- Bản thân mong muốn làm việc gì/ không muốn làm việc gì trong quá khứ, ta có cấu trúc: S + would
rather + (not) have + P2
- Bản thân mong muốn ai đó làm việc gì/ không làm việc gì trong quá khứ, ta có cấu trúc:
S1 + would rather + S2 + had (not) +P2
Trong câu hỏi, trường hợp câu giả định được sử dụng chính là trường hợp số 2, muốn ai đó làm việc gì.
Tạm dịch: Ông chủ của chúng tôi muốn chúng tôi không nói chuyện trong khi làm việc.
Question 10. Đáp án A
Trong câu trên ta thấy 2 vế câu diễn tả nguyên nhân kết quả - broke down- came home late. Như vậy chỉ
có phương án A phù hợp. Ta thấy đáp án C - For cũng dùng với mệnh đề chỉ lí do nhưng không đứng đầu
câu. Phương án B - While thì không phù hợp về nghĩa còn phương án D - with thì sẽ kết hợp cùng danh từ
chứ không đứng trước một mệnh đề do with là giới từ.
Tạm dịch. Vì xe của tôi bị hỏng nên hôm qua tôi về nhà muộn.
Question 11. Đáp án A
Đây là câu hỏi liên quan đến câu chẻ trong tiếng Anh.
Thực chất câu chẻ là một trong những cách chen thêm cấu trúc vào trong câu (chẻ câu ra) để nhấn mạnh
một đối tượng nào đó.
Khi chúng ta muốn nhấn mạnh trạng ngữ chỉ thời gian/ nơi chốn ta sử dụng cấu trúc sau:
It is/ was + cụm trạng ngữ (chỉ thời gian/ nơi chốn gồm giới từ + từ chỉ thời gian/ nơi chốn) that S + V
Chúng ta thấy khi muốn nhấn mạnh nơi chốn có giới từ đi cùng thì chúng ta sẽ đem nguyên cả cụm giới
từ + từ chỉ nơi chốn lên - đầy cũng là điểm khác biệt đối với mệnh đề quan hệ.

Trang 9
Trong trường hợp này ta thấy đáp án B sai vì phương án B đang sử dụng cấu trúc câu chẻ It + was + giới
từ + địa điểm tuy nhiên lại sử dụng where thay cho that vì vậy phương án B đang sử dụng sai cấu trúc
câu chẻ.
Phương án C + D không chính xác vì ta thấy nếu sau danh từ có một mệnh đề không là mệnh đề chính
của câu đó thì sẽ là mệnh đề quan hệ. Tuy nhiên 2 đáp án này lại sử dụng từ quan hệ không chính xác.
Với phương án C từ quan hệ that sử dụng không chính xác do that sẽ sử dụng trong mệnh đề quan hệ xác
định và đóng vai trò làm chủ ngữ hoặc tân ngữ của mệnh đề. Trong trường hợp này ta thấy mệnh đề I first
met my girlfriend đã có đầy đủ chủ ngữ và tân ngữ. Tương tự với phương án D, từ quan hệ which thay thế
cho danh từ chỉ vật làm chủ ngữ hoặc tân ngữ trong mệnh đề quan hệ. Căn cứ vào các giải thích trên ta
chọn được đáp án A.
Tạm dịch. Đó chính là London nơi lần đầu tiên tôi gặp bạn gái mình.
Question 12. Đáp án D
Câu hỏi này liên quan đến trật tự tính từ
❖ For review
Để sắp xếp các tính từ theo trật tự đúng, ta có quy tắc OpSAShCOMP
Trong đó:
Op- OPINION -Ý kiến, quan điểm beautiful, lovely, good, comfortable, interesting
small, short, tall (lưu ý: tính từ chỉ kích thước và chiều
dài (tall, long, big, small) sẽ đi trước tính từ chỉ hình
S - SIZE - Kích thước:
dáng và chiều rộng (round, narrow, wide).
Ví dụ: a long narrow road a small round handbag
A - AGE -
old, young, ancient, new
Tuổi tác:
Sh - SHAPE -
round, triangular, square, oval, flat
Hình dáng
C- COLOUR - Màu sắc yellow, white, orange
O - ORIGIN - Nguồn gốc, xuất xứ Turkish, Italian, English
plastic, wood, gold, silver, cotton, leather, wool, silk,
M- MATERIAL - Chất liệu
velvet, nylon
P - PURPOSE - Mục đích cooking, cleaning, sleeping
Như vậy, đối chiếu với bảng trật tự tính từ ở trên ta chọn được đáp án D.
Tạm dịch. Tuần trước gia đình chúng tôi đã đi xem phim cùng nhau. Chúng tôi đã chọn một bộ phim mới
thú vị để xem.
Question 13. Đáp án A
Sau giới từ on ta cần một danh từ → loại B và D vì B là tính từ, và D là trạng từ. Phương án C không phù
hợp về nghĩa do đây là danh từ chỉ người. Cụm từ do research on...: nghiên cứu về... Như vậy ta chọn
được đáp án A.
A. biology (n): môn sinh vật học
B. biological (adj): liên quan đến sinh học

Trang 10
C. biologist (n): nhà sinh vật học
D. biologically (adv): trên phương diện sinh học
Tạm dịch. Cậu ấy rất hứng thú nghiên cứu sinh vật học kể từ khi còn học ở trường THPT.
Question 14. Đáp án A
Trong câu trên ta thấy có sự xuất hiện của các từ televised (P2): được truyền hình như vậy ta cần một
trạng từ để bổ sung ý nghĩa cho động từ → giữ lại đáp án A và C. Khi muốn diễn tả được truyền hình trực
tiếp ta sẽ dùng từ live
A. live (adv): tại chỗ, nóng hổi
B. lived (v): sống
C. lively (adv): sinh động, sống động
D. living (adj): sống, tồn tại
Tạm dịch. Trận đấu sẽ được truyền hình trực tiếp trên VTV3 tối nay.
❖ Note
Với một số động từ mang nghĩa trình chiếu như show, televise hoặc broadcast khi muốn thể hiện chiếu/
phát trực tiếp sẽ sử dụng cùng trạng từ live
Question 15. Đáp án B
Ta có cụm từ conduct an experiment: tiến hành/ thực hiện thí nghiệm
A. exam (n): kì thi
B. experiment (n): thực nghiệm/ thí nghiệm
C. test (n): bài thi
D. experience (n): kinh nghiệm
Tạm dịch. Sinh viên đang tiến hành một thí nghiệm nhỏ.
Question 16. Đáp án C
Ta có thành ngữ as light as a feather: rất nhẹ
A. dust (n): bụi
B. lightning (n): tia chớp
C. a feather (n): sợi lông (chim)
D. a fish (n): con cá
Tạm dịch. “Chiếc vali này không quá nặng phải không” “Không, nó nhẹ lắm”
Question 17. Đáp án B
Ta có thành ngữ as old as the hills: rất cũ, cổ xưa
A. Solomon (P.n): tên riêng của một vị vua Israel cổ đại
B. the hills (n): vùng đồi núi
C. a dinosaur (n): con khủng long
D. Jupiter (P.n): sao Mộc

Trang 11
Tạm dịch. “Đã bao giờ nghe kể về Welshman chưa - một câu chuyện về người Ai-len và con lợn?”
“Chúng tôi nghe rồi. Chuyện đó xưa như trái đất vậy”
Question 18. Đáp án C
Ta loại được A, B, D vì không hợp nghĩa.
A. taken away (ph.v): lấy đi, đem đi
B. got over (ph.v): vượt qua
C. turned down (ph.v): bác bỏ, khước từ
D. turned off (ph.v): tắt (điện)
Tạm dịch. Tôi nộp đơn xin việc nhưng đã bị từ chối.
Question 19.
Câu này liên quan đến cách sử dụng đại từ thay thế.
Trong câu này ta thấy đại từ bất định anyone đòi hỏi động từ theo sau chia số ít tuy nhiên đại từ thay thế
lại là số nhiều (they, them, their, theirs). Như vậy him sửa thành them.
Tạm dịch. Nếu ai đó ghé văn phòng khi tôi không ở đây thì hãy ghi lại lời nhắn.
Question 20.
Chúng ta có giới từ đứng trước danh từ floor là on vì vậy sửa in thành on
Tạm dịch: Ngọn lửa bắt đầu từ tầng 10 của tòa nhà nhưng sớm lan sang các tầng khác.
Question 21.
Câu trên liên quan đến kiến thức của hiện tại phân từ và quá khứ phân từ - thường được sử dụng trong
mệnh đề quan hệ tính ngữ rút gọn hoặc được sử dụng như một tính từ, đứng trước danh từ để bổ nghĩa
cho danh từ. Trong đó, hiện tại phân từ thường được dùng trong các câu/ mệnh đề chủ động; hoặc dùng
như một tính từ mang nghĩa chủ động còn quá khứ phân từ dùng trong các câu/mệnh đề bị động hoặc
dùng với nghĩa bị động.
Để xác định xem trong câu sử dụng hiện tại phân từ hay quá khứ phân từ thì ta làm 3 bước.
Bước 1: Xác định tính từ đó bổ sung ý nghĩa cho danh từ/ đại từ nào
Bước 2: Xác định xem tính từ đó được thành lập từ động từ nào
Bước 3: Trả lời câu hỏi xem danh từ/ đại từ mà nó bổ nghĩa có thực hiện hành động đó hay không, nếu
câu trả lời là có → dùng hiện tại phân từ còn nếu câu trả lời là không → dùng quá khứ phân từ.
Quay trở lại câu hỏi, ta thấy trong câu có sử dụng cấu trúc song song với từ nối and, trước and là một
danh từ → sau and cũng cần một danh từ → sharing đóng vai trò là tính từ, bổ sung ý nghĩa cho
prosperity (n): sự thịnh vượng. Sharing được thành lập từ động từ share, mà prosperity thì không thể thực
hiện hành động share được → không thể ở dạng hiện tại phân từ được mà sẽ ở dạng quá khứ phân từ. Vì
vậy sharing prosperity cần sửa lại thành shared prosperity (= prosperity that/ which was shared)
Tạm dịch. Sự tiến bộ mạnh mẽ về xóa đói giảm nghèo trong 10 năm đẩu của thập niên 2000 đang bị đe
dọa vì sự tăng trưởng chậm lại toàn cầu.
Question 22. Đáp án D

Trang 12
Dịch câu đề. Người mẹ nói với con trai “Mẹ pha cho con cốc sữa nhé?”
Dịch đáp án.
A. Người mẹ muốn pha một cốc sữa cho con trai.
B. Người mẹ được yêu cầu pha một cốc sữa cho con trai.
C. Người mẹ hứa pha một cốc sữa cho con trai.
D. Người mẹ đề nghị để pha một cốc sữa cho con trai.’
Trong câu đề ta thấy có cụm từ shall I - được sử dụng khi muốn đề nghị được làm một việc gì đó → Chọn
đáp án D vì ta có cấu trúc offer to V: đề nghị được làm gì, còn want to V: muốn làm việc gì, promise to V:
hứa làm việc gì và was asked to V: được yêu cầu làm việc gì.
Question 23. Đáp án D
Dịch câu đề. Học sinh duy nhất thi trượt là Finet.
Dịch đáp án.
A. Cùng với các bạn khác, Finet đã thi trượt.
B. Ai cũng thi đỗ
C. Ai cũng thi trượt trừ Finet.
D. Ngoại trừ Finet, mọi người ai cũng thi đỗ. Ta loại đáp án A, B và C vì không hợp nghĩa so với câu đề.
Chỉ có đáp án D là phù hợp.
Question 24.
Dịch câu đề. Cô ấy vừa đặt điện thoại xuống thì sếp cô ấy gọi lại
Dịch đáp án.
A. Không dịch vì sai ngữ pháp: khi hardly đứng đầu câu → đảo ngữ
B. Ngay khi cô ấy cúp máy thì sếp cô ấy gọi lại
C. Không dịch vì sai ngữ pháp: đảo ngữ với cấu trúc hardly sẽ là hardly... when chứ không phải than
D. Không dịch vì sai ngữ pháp: khi hardly đứng đầu câu → đảo ngữ
Ta thấy đây là câu hỏi liên quan đến cấu trúc đảo ngữ. Cấu trúc đảo ngữ với Hardly:
- Hardly had + S+ V (P2) when S+ V (quá khứ đơn): Ngay khi.. .thì....
Question 25. Đáp án B
Dịch câu đề. Sự giải thích của cô ấy rõ ràng. Tôi không hiểu được
Dịch đáp án.
A. Không dịch vì sai ngữ pháp ở chỗ despite không đi với of.
B. Mặc dù sự giải thích của cô ấy rõ ràng nhưng tôi vẫn không hiểu được.
C. Dù thực tế tôi không hiểu nhưng sự giải thích của cô ấy rõ ràng
D. Dù thực tế tôi không hiểu nhưng sự giải thích của cô ấy rõ ràng
Ở đây ta dùng cấu trúc nhượng bộ để thể hiện tương phản hai vế câu, đáp án đúng là B. A sai vì despite
không đi với of C và D sai vì diễn đạt sai ý.
Question 26. Đáp án B

Trang 13
Dịch câu đề. Dan có thể làm những video hướng dẫn học tiếng Anh cho người Việt. Đó là bởi vì người vợ
Việt giúp anh ấy.
Dịch đáp án.
A. Giá mà Dan có thể làm các video dạy tiếng Anh cho người Việt cho người Việt.
B. Nếu không nhờ sự giúp đỡ của vợ thì Dan không thể làm những video dạy tiếng Anh cho người Việt
C. Nếu không có sự giúp đỡ của vợ thì Dan không thể làm những video dạy tiếng Anh cho người Việt
D. Nếu không có sự giúp đỡ của vợ thì Dan không thể làm những video dạy tiếng Anh cho người Việt
Chúng ta thấy câu đã cho diễn tả hành động ở hiện tại vì vậy khi nói về một điều trái ngược với hiện tại,
không có thực ở hiện tại ta sẽ sử dụng câu điều kiện loại 2.
Đáp án A không chính xác vì If only = wish - thể hiện mong ước có thể làm một việc không có thật ở
hiện tại, tuy nhiên sự việc được nêu trong phương án A là sự việc đang xảy ra ở hiện tại rồi → chưa chính
xác
Đáp án C sai vì ta có without + N có thể thay thế cho mệnh đề if, tuy nhiên mệnh đề chính không thay
đổi. Trong trường hợp này, ta cần câu điều kiện loại 2 nhưng mệnh đề chính của phương án C đang sử
dụng câu điều kiện loại 3.
Đáp án D không chính xác vì ta dùng cấu trúc but for sb/ st đi cùng với mệnh đề chính ở điều kiện loại 3
hoặc but for that S + V để diễn tả câu điều kiện.
Như vậy đáp án đúng sẽ là phương án B, câu điều kiện loại 2 bắt đầu bằng If it weren’t for...
❖ For review
Ta có 3 loại câu điều kiện quen thuộc 1,2,3 sau: Câu điều kiện loại 1: diễn tả điều luôn đúng ở hiện tại
hoặc tương lai
- If S + V(s/es), S + will/ can/ may + do
Câu điều kiện loại 2: diễn tả điều không có thực ở hiện tại
- If S + V (ed)/ were ..., S would/ could/ might + do
Ngoài ra trong câu điều kiện loại 2, ta còn có cấu trúc khác: Nếu không có.. .thì....
- If it weren’t for sb/ st, S + would/ could/ might + V
Câu điều kiện loại 3: diễn tả điều không có thực ở quá khứ:
- If S + had P2, S + would/ could/ might + have P2
Bên cạnh đó chúng ta còn có các cấu trúc khác diễn tả điều kiện:
- Without + N, S + V: nếu không có..., thì....
+ Không có thực ở hiện tại: Without N, S would/ could/ might do
+ Không có thực ở quá khứ: Without N, S would/could/ might have P2
- But for that S + V: Nếu không thì:
+ Không có thực ở hiện tại: S + V (s/es) but for that S + would/ could/ might+ do
+ Không có thực ở quá khứ: S + v(ed)/ were... but for that S + would/ could/ might have P2
Question 27. Đáp án D

Trang 14
Hai bạn học sinh đang thảo luận về lớp học Tiếng Anh trước đó
Học sinh 1: Tớ nghĩ cô giáo nên giao thêm bài tập về nhà
Học sinh 2:__________
A. Ừ, làm thôi
B. Được
C. Thật vớ vẩn
D. Đó cũng là điều tớ đã nghĩ
Dựa vào tình huống chúng ta thấy đây đang bày tỏ quan điểm về việc giáo viên nên giao thêm bài tập và
đáp án D thể hiện sự đồng tình với quan điểm đó.
Question 28. Đáp án B
Eden và Edward đang tán gẫu sau khi làm xong việc, Eden gợi ý đi ăn nhà hàng.
Eden: Tối nay chúng ta đi ăn nhé.
Edward: __________
Ta thấy trong tình huống này Eden đang gợi ý đi ăn nhà hàng vì vậy Edward thể hiện sự đồng ý → đáp án
B là phù hợp
A. Ổn thôi
B. Ý kiến hay đấy.
C. Bạn thật tốt
D. Lúc nào bạn cũng được chào đón.
Question 29. Đáp án B
Đáp án B đúng vì ta có problem (n) = issue (n): vấn đề. Đây là câu hỏi tìm tù đồng nghĩa nên chọn đáp án
B
A. incident (n): sự việc
C. trouble (n): rắc rối
D. case (n): vụ việc
Tạm dịch. Mọi người chưa ý thức được vấn đề bùng nổ dân số
Question 30. Đáp án C
Đáp án C đúng vì ta có drive (sb) to the edge = irritate (v): làm ai đó bực bội. Đây là câu hỏi tìm từ đồng
nghĩa nên ta chọn phương án C.
A. làm tôi sợ
B. làm tôi vui
D. làm tôi ngạc nhiên
Tạm dịch. Anh ta làm tôi phát bực vì cứ luôn mồm lải nhải.
Question 31. Đáp án B
Đáp án B đúng vì ta có smooth (adj): suôn sẻ, nhịp nhàng, uyển chuyển >< agitated (adj): lo lắng bồn
chồn, dễ kích động. Đây là câu hỏi tìm từ trái nghĩa nên ta chọn phương án B

Trang 15
A. soft (adj): mềm, dẻo
C. silky (adj): mượt
D. easy (adj): dễ dàng
Tạm dịch. Trong thế giới này, xúc cảm đã trở nên không thể tin được- nét đặc trưng được thừa nhận là
hòa nhã, chỉn chu và điềm tĩnh
Question 32. Đáp án B
Đáp án B đúng vì ta có (be) snowed under with: có quá nhiêu việc >< free from: tự do, thoát khỏi cái gì.
Đây là câu hỏi tìm từ trái nghĩa nên ta chọn phương án B
A. busy with: bận rộn
C. relaxed about: thư giãn
D. interested in: thích thú
Tạm dịch. Xin lỗi, mình không thể đến bữa tiệc của bạn được. Hiện tại mình có quá nhiều việc phải làm.
Question 33.
Về mặt ngữ pháp ta thấy chỗ trống cần điền không thể điền in hay why vì in sẽ không đứng trước
“nowadays” còn why nếu không đóng vai trò làm từ để hỏi thì cần đứng trong mệnh đề danh từ hoặc
mệnh đề quan hệ tuy nhiên trường hợp này không phải vậy, còn or cũng không phù hợp vì hai vế câu
không có mối quan hệ tương đương. Chỉ có since là phù hợp nhất vì dùng để chỉ nguyên nhân.
Trích: Is it worth reading books, since nowadays there are so many other forms of entertainment?
Tạm dịch. Vì ngày nay đã có quá nhiều hình thức giải trí rỗi, liệu việc đọc sách có còn có ích không?
Question 34. Đáp án B
Ta thấy trong câu xuất hiện cụm từ television và động từ watch → Suy luận có thể chọn được phương án
B. Ta có cụm từ television programmes: chương trình tivi
A. ones: những người nào đó, những cái gì đó (đại từ thay thế)
B. programmes (n): chương trình
C. episodes (n): các tập (của một bộ phim)
D. cereals (n): ngũ cốc
Trích: They might add that television is more exciting and that viewers can relax as they watch their
favourite programmes.
Tạm dịch. Thêm vào đó họ cho rằng xem ti vi thì thú vị hơn và người xem có thể thư giãn vì họ có thể
xem chương trình mà họ thích.
Question 35. Đáp án D
Dựa vào nghĩa của từ ta chọn được D là đáp án đúng.
A. current (adj): ngay lúc này, hiện tại
B. imagined (v): tưởng tượng
C. interest (v): gây thích thú
D. gripping (adj): thú vị, thu hút

Trang 16
Trích: Of course, it may be so gripping that you can't stop!
Tạm dịch. Tất nhiên những cuốn sách rất cuốn hút khiến bạn không thể ngừng việc đọc lại.
Question 36. Đáp án B
Ở đây ta cần một đại từ quan hệ thay thế cho danh từ chỉ vật book và làm chủ ngữ của mệnh đề quan hệ -
→ chỉ có phương án B là phù hợp.
Trích: There are many different kinds of books, so you can choose a crime novel or an autobiography, or
a book which gives you interesting information.
Tạm dịch. Có rất nhiều loại sách vì thế bạn có thể chọn tiểu thuyết trinh thám tội phạm hay sách tự truyện
hoặc sách đem lại những thông tin thú vị.
Question 37. Đáp án C
Ta có cụm từ give up: từ bỏ, không làm gì.
Các đáp án khác không chính xác.
A. pick up: trả lời điện thoại
B. look up: tra cứu
C. give up: từ bỏ
D. turn up: tìm cái gì đó, cho to lên, khâu gấu quần/ áo cho ngắn lại
Trích: Personally, I can't do without books, but I can give up television easily enough.
Tạm dịch. Cá nhân tôi thì tôi không thể sống thiếu sách, nhưng lại đủ dễ cho tôi để từ bỏ ti vi.
Question 38. Đáp án B
Câu hỏi. Ý chính của đoạn văn là gì?
A. Harvard là một trong những trường đại học danh tiếng nhất thế giới
B. Một trường đại học nổi tiếng nhất hiện nay đã có một khởi đầu khiêm tốn
C. John Harvard là chìa khóa để phát triển một trường đại học lớn
D. Đại học Harvard được phát triển dưới sự bảo trợ của Tòa án Tối cao Massachusetts
Đây là câu hỏi hỏi ý chính của văn bản. Trong đoạn một dòng 1-2, ta tìm thấy thông tin: Trích Harvard
University, today recognized as part of the top echelon of the world's universities, came from very
inauspicious and humble beginning.
Tạm dịch. Đại học Harvard, ngày nay được công nhận là một trong số những trường đại học có xếp hạng
cao nhất thế giới, có một sự khởi đầu rất không may và khiêm tốn.
Từ đó suy ra ta chọn được đáp án B.
Question 39. Đáp án B
Câu hỏi. Đại từ “they” trong đoạn 2 đề cập đến điều nào sau đây:
A. đại học Oxford và Đại học Cambridge
B. những sinh viên đã tốt nghiệp
C. con của những người theo Thanh giáo
D. những cơ hội giáo dục

Trang 17
Đây là một câu hỏi suy luận. Ta tìm được thông tin trong đoạn 2, dòng 2-4:
Included in the Puritan emigrants to the Massachusetts colony during this period were more than 100
graduates of England's prestigious Oxford and Cambridge universities, and these universities graduates in
the New Word were determined that their sons would have the same educational opportunities that they
themselves had had.
Chủ ngữ của câu là cụm từ university graduates nên đại từ they là để thay thế cho chủ ngữ này. Vì vậy ta
chọn được đáp án B.
Tạm dịch. Đi cùng những người Thanh giáo đến Massachusetts trong giai đoạn này có hơn 100 cử nhân
của trường đại học Oxford và Cambridge danh tiếng, và những vị cử nhân này khi đến Thế giới Mới đã
quyết tâm rằng con trai của họ cũng phải có cơ hội giáo dục như họ đã từng.
Question 40. Đáp án D
Câu hỏi. Từ “somewhat” ở đoạn cuối có thể được thay thế bằng___________
A. lắc lư
B. đi xuôi ngược, tới lui
C. cạnh nhau
D. ít nhiều
Chúng ta tìm thấy thông tin trong đoạn 4 dòng 3-4.
Trích: Although the staff did expand somewhat, for the first century of its existence the entire teaching
staff consisted of the president and three or four tutors.
Tạm dịch. Mặc dù đội ngũ giảng dạy đã phần nào được tăng cường, trong thế kỉ đầu tiên tồn tại toàn bộ
cán bộ giảng dạy chỉ bao gồm hiệu trưởng và ba hoặc bốn trợ giảng.
Như vậy “somewhat” tương đương nghĩa với đáp án D.
Question 41. Đáp án C
Câu hỏi. Câu nào sau đây không được đề cập đến về John Harvard?
A. Ông ấy chết vì nguyên nhân gì
B. Ông ấy đến từ đâu
C. Ông ấy được an táng ở đâu
D. Ông ấy hiến tặng bao nhiêu cho Harvard
Chúng ta tìm thấy thông tin trong đoạn 3, dòng 1-2:
Trích: When a young minister named John Harvard, who came from the neighboring town of
Charlestown, died from tuberculosis in 1638, he willed half of his estate of 1,700 pounds to the fledgling
college.
Tạm dịch: Khi một vị cha xứ trẻ tuổi tên là John Harvard, người đến từ thị trấn lân cận của Charlestown,
chết vì bệnh lao năm 1636, ông để lại di chúc hiến một nửa tài sản của mình, tương đương 1.700 bảng
cho trường đại học non trẻ này
Dùng phép loại trừ ta chọn được đáp án C

Trang 18
Question 42. Đáp án D
Câu hỏi. Đoạn văn ngụ ý rằng
A. Henry Dunster là một hiệu trưởng vô tích sự
B. có một số người đã làm hiệu trưởng Harvard trước Henry Dunster
C. Henry Dunster dành phần lớn thời gian để quản lý khoa chuyên môn ở đại học Harvard
D. trong những năm đầu, hiệu trưởng Harvard không chỉ giữ vai trò quản lý
Ta tìm thông tin trong đoạn văn cuối của văn bản dòng 1-2:
Trích: Henry Dunster was appointed the first president of Harvard in 1640, and it should be noted that in
addition to serving as president, he was also the entire faculty, with an entering freshmen class of four
students.
Tạm dịch. Henry Dunster được bổ nhiệm làm hiệu trưởng đầu tiên của Harvard năm 1640 và cần lưu ý
rằng ngoài chức vụ hiệu trưởng, ông còn là giảng viên cho cả một khoa, với một lớp có 4 sinh viên nhập
học.
Từ dữ liệu này ta có thể loại được đáp án A, B, C vì không chính xác.
Question 43. Đáp án A
Câu hỏi: Mục đích của văn bản là gì?
A. Giải thích quy trình trí nhớ của chúng ta xử lý thông tin
B. Miêu tả bộ phận tiếp nhận thông tin
C. Giải thích tại sao đôi khi chúng ta quên thông tin
D. So sánh trí nhớ ngắn hạn và trí nhớ dài hạn
Chúng ta thấy ngay trong câu đầu tiên, văn bản đã đề cập đến mục đích:
Psychologists who study information processing have identified and described several memory structures
that clarify how our memory works.
Tạm dịch: Các nhà tâm lý học nghiên cứu về quá trình xử lý thông tin đã xác định và mô tả nhiều cấu trúc
ghi nhớ cái mà giúp làm rõ trí nhớ của chúng ta hoạt động như thế nào. Như vậy ta có thể suy ra đáp án
A. Các phương án còn lại chỉ là một ý nhỏ trong văn bản.
Question 44. Đáp án C
Câu hỏi. Từ “they” ở đoạn 1 liên quan tới__________
A. các nhà tâm lý học
B. thông tin
C. những cấu trúc của bộ nhớ
D. những kích thích của môi trường
Ta tìm thấy thông tin câu trả lời ở đoạn một dòng 1-3:
Trích: Psychologists who study information processing have identified and described several memory
structures that clarify how our memory works. They include the sensory register, short-term memory, and
long-term memory. Each structure varies as to how much information it can hold and for how long.

Trang 19
Tạm dịch: Các nhà tâm lý học nghiên cứu về quá trình xử lý thông tin đã xác định và mô tả nhiều cấu trúc
ghi nhớ cái mà giúp làm rõ trí nhớ của chúng ta hoạt động như thế nào. Các cấu trúc ghi nhớ này bao gồm
vùng giác quan, trí nhớ ngắn hạn và trí nhớ dài hạn.
Đại từ they dùng để thay thế cho danh từ xuất hiện trước đó. Trong câu hỏi này ta thấy sau they có động
từ include → liệt kê. Câu trước đó ta thấy có động từ identify và describe- làm rõ và miêu tả → they và cái
được làm rõ và được miêu tả ở đây là một đối tượng → đáp án C memory structures là đáp án đúng.
Question 45.
Câu hỏi. Từ “stimuli” ở dòng 4 và 5 gần nghĩa nhất với___________
A. sự vật, sự việc mà kích hoạt trí nhớ của chúng ta
B. sự vật mà chúng ta cho rằng nó hấp dẫn
C. thứ mà sau này giúp chúng ta gợi lại những điều đã xảy ra
D. tình huống mà ở đó chúng ta trải nghiệm cảm xúc
Ta tìm thấy thông tin ở đoạn 2 dòng 1-2: A description ofhowhumanprocess information typically begins
with environmental stimuli. Our sense receptors are constantly stimulated by visual, auditory, tactile,
olfactory, and gustatory stimuli.
Từ stimuli là danh từ số nhiều của stimulus có nghĩa là sự kích thích, tác nhân kích thích
Tạm dịch. Một miêu tả về cách con người xử lí thông tin thường bắt đầu với các kích thích của môi
trường. Bộ phận tiếp nhận cảm giác liên tục được kích thích bởi thị giác, thính giác, xúc giác, khứu giác,
các thần kinh vị giác
Question 46. Đáp án D
Câu hỏi. Theo bài, người đánh máy không thể gợi lại thông tin họ đã gõ nếu___________ .
A. Họ mệt
B. Họ bị phân tâm bởi ai hoặc cái gì đó
C. Họ phải làm quá nhiều việc để có thể xử lí hết được nó
D. Họ không nhận thức được nó đủ quan trọng để ghi nhớ
Ta tìm thấy thông tin trong đoạn 3 dòng 7 đến hết đoạn 3
Trích: Typists make extensive use of the sensory register in order to remember words just long enough to
get them typed. If no further processing takes place, a typist’s ability to recall that information later is
minimal. Similarly, most of us have had the experience of reading an entire page of text, only to discover
when we got to the bottom of the page, we couldn’t say anything about it except that we had indeed
“read” every word.
Tạm dịch: Người đánh máy tận dụng tối đa vùng giác quan để nhớ các từ đủ dài để giúp họ đánh máy.
Nếu không có quá trình nào khác diễn ra, khả năng để họ nhớ lại các thông tin sau đó là tối thiểu. Tương
tự như vậy, hầu hết chúng ta khi đọc cả trang sách, chỉ để xem đến cuối trang sách có gì, chúng ta không
thể kể lại bất cứ điều gì về nó trừ khi chúng ta đọc từng chữ một.

Trang 20
Như vậy có thể thấy để có thể gợi lại thông tin thì các thông tin đó phải đủ quan trọng để ghi nhớ, nếu
không sẽ quên → đáp án đúng là đáp án D
Question 47. Đáp án D
Câu hỏi. Từ “minimal” ở đoạn 2 gần nghĩa nhất với ___________
A. rất to B. rất tốt
C. rất đẹp D. rất nhỏ
Ta tìm thấy thông tin trong đoạn 2 dòng 7.
Trích: If no further processing takes place, a typist’s ability to recall that information later is minimal.
Từ minimal có nghĩa là rất nhỏ, tối thiểu
Tạm dịch: Nếu không có quá trình nào tiếp tục xảy ra, khả năng khơi gợi lại thông tin của người đánh
máy sẽ rất nhỏ.
Question 48. Đáp án C
Câu hỏi. Dựa vào văn bản, loại thông tin nào được gửi đến trí nhớ ngắn hạn?
A. Thông tin chúng ta cần trong 3 giây hoặc ngắn hơn
B. Thông tin mà làm chúng ta ngạc nhiên
C. Thông tin liên quan đến chúng ta
D. Kích thích từ môi trường mà chúng ta không tiếp thu được
Ta tìm thấy thông tin trong đoạn 3, dòng 1: Once information has been recognized as meaningful, it is
sent to short-term memory.
Tạm dịch: Một khi thông tin được xác nhận là có ý nghĩa, nó sẽ được chuyển tới trí nhớ ngắn hạn
Thông tin được cho là có ích, có ý nghĩa nên ta loại được đáp án A, B, D
Question 49. Đáp án B
Câu hỏi. Có thể suy ra được rằng trí nhớ ngắn hạn được gọi là trí nhớ “làm việc” bởi vì __________
A. Chúng ta dùng nó một cách rộng rãi khi làm việc
B. Nó lưu giữ thông tin mà chúng ta làm việc tại một thời điểm nhất định
C. Rất khó để sử dụng hiệu quả
D. Chúng ta phải mất nhiều công sức để gợi lại thông tin từ nó
Chúng ta tìm thấy thông tin ở đoạn 3, dòng 3-5:
Trích: If you were asked to dial an unfamiliar phone number, received a busy signal, and were then
distracted by something or someone else for 15 to 20 seconds, chances are you would have forgotten the
number at that point.
Tạm dịch: Nếu chúng ta được yêu cầu quay một dãy số điện thoại lạ, nhận được tín hiệu bận và rồi phân
tâm bởi một điều gì đó hay ai đó trong vòng 15 đến 20 giây, khả năng là bạn sẽ quên dãy số đấy tại thời
điểm đó.

Trang 21
Đây là thông tin được dùng để làm ví dụ chứng minh cho việc vì sao trí nhớ ngắn hạn được gọi là trí nhớ
“làm việc”. Dựa vào thông tin này ta thấy đây chính là thông tin mà chúng ta có được trong khi làm việc
ở một thời điểm nhất định → đáp án B đúng
Question 50. Đáp án C
Câu hỏi. Trường hợp nào dưới đây mà chúng ta hầu hết có thể dễ dàng gợi lại được từ trí nhớ dài hạn?
A. một số điện thoại chúng ta quay số sai
B. gương mặt một người lạ trên đường
C. ngày sinh nhật của con chúng ta
D. giọng nói trên truyền hình
Chúng ta tìm thấy thông tin trong đoạn văn cuối cùng:
Trích: Most cognitive psychologists believe that the storage capacity of long-term memory is unlimited
and contains a permanent record of everything an individual has learned and experienced. Information is
encoded there to enhance its meaningfulness and organization so that it can be easily retrieved when
necessary
Tạm dịch: Hầu hết các nhà tâm lý kinh nghiệm đều tin rằng sức chứa của trí nhớ dài hạn là vô tận và có
thể chứa dài hạn bản ghi lại của tất cả mọi thứ mà một cá nhân được học hoặc trải nghiệm. Thông tin
được mã hóa để tăng thêm ý nghĩa và liên kết của chúng để nó có thể dễ dàng gợi lại khi cần thiết
Từ dữ liệu trên ta có thể suy luận thông tin được mã hóa ở đây là những thông tin có ý nghĩa → đáp án
đúng là phương án C.

Trang 22
ĐỀ SỐ 05 ĐỀ THI THỬ TỐT NGHIỆP THPT
NĂM HỌC: 2020 – 2021
MÔN: TIẾNG ANH
Thời gian làm bài: 60 phút; không kể thời gian phát đề

Mark the letter A, B, C, or D on your answer sheet to indicate the word whose underlined part
differs from the other three in pronunciation in each of the following questions.
Question 1. A. grows B. tends C. roars D. sweeps
Question 2. A. performed B. finished C. interviewed D. delivered
Mark the letter A, B, C, or D on your answer sheet to indicate the word that differs from the other
three in the position of primary stress in each of the following questions.
Question 3. A. justice B. struggle C. neglect D. wildlife
Question 4. A. eradicate B. unexpected C. accompany D. commitment
Mark the letter A, B, C, or D on your answer sheet to indicate the correct answer to each of the
following questions.
Question 5. We___________in silence when he suddenly___________me to help him.
A. walked - was asking B. were walking - asked
C. were walking - was asking D. walked - asked
Question 6. I don’t know why John always refuses___________me in person.
A. to meet B. meeting C. met D. to meeting
Question 7. It___________Sue that you saw last night. She’s abroad with her family.
A. should have been B. must have been C. can’t have been D. needn’t have been
Question 8. At one time the entire world___________by dinosaurs.
A. has ruled B. was ruled C. will be ruled D. was ruling
Question 9. I asked Martha___________to enter law school.
A. are you planning B. is she planning C. was she planning D. if she was planning
Question 10. Extinction means a situation___________a plan, an animal or a way of life etc ...stops
existing.
A. to which B. in which C. on which D. for which
Question 11. ___________he arrived at the bus stop when the bus came.
A. Hardly had B. No sooner had C. No longer has D. Not until had
Question 12. Let’s begin our discussion now, ___________?
A. shall we B. will we C. don’t we D. won’t we
Question 13. The more___________and positive you look, the better you will feel.
A. confidence B. confident C. confide D. confidently
Question 14. Mr. Pike is certainly a___________writer; he has written quite a few books this year.

Trang 1
A. prolific B. fruitful C. fertile D. successful
Question 15. The 1st week of classes at university is a little___________because so many students get
lost, change classes or go to the wrong place.
A. uncontrolled B. arranged C. chaotic D. notorious
Question 16. Due to the computer malfunction, all our data was lost. So unhappily, we had to begin all
the calculations from___________.
A. onset B. source C. original D. scratch
Question 17. The players’ protests___________no difference to the referees decision at all.
A. did B. made C. caused D. created
Question 18. Although he is my friend, I___________find it hard to his selfishness.
A. put up with B. catch up with C. keep up with D. come down with
Mark A, B, C, orD on your answer sheet to indicate the underlined part that needs correction in each
of the following questions.
Question 19. A number of wild horses on Assateague is increasing, resulting in overgrazed marsh and
dune grasses.
A. A B. is C. resulting D. grasses
Question 20. Upon reaching the destination, a number of personnel are expected changing their
reservations and proceed to Hawaii.
A. reaching B. are C. changing D. proceed
Question 21. Although the Red cross accepts blood from most donors, the nurse will not let you to give
blood if you have just had cold.
A. accepts B. let C. to give D. have just had cold.
Mark the letter A, B, C or D on your answer sheet to indicate the sentence that is closest in
meaning to each of the following questions.
Question 22. “I’ll drop you from the team if you don’t train harder,” said the captain to John.
A. John was reminded to train harder so as not to be dropped from the team.
B. The captain threatened to drop John from the team unless he trained harder.
C. The captain urged that John should train harder in order not to be dropped from the team.
D. The captain promised to drop John from the team in case he trained harder.
Question 23. It’s possible that Joanna didn’t receive my message.
A. Joanna shouldn’t have received my message.
B. Joanna needn’t have received my message.
C. Joanna mightn’t have received my message.
D. Joanna can’t have received my message.
Question 24. The noise next door did not stop until after midnight.
A. The noise next door stopped at midnight.

Trang 2
B. It was midnight and the noise next door stopped.
C. It was not until after midnight that the noise next door stopped.
D. Not until after midnight the noise next door stopped.
Mark the letter A, B, C or D on your answer sheet to indicate the sentence that best combines
each pair of sentences in the following questions.
Question 25. Julian dropped out of college after his first year. Now he regrets it.
A. Julian regretted having dropped out of college after his first year.
B. Julian wishes he didn’t drop out of college after his first year.
C. Julian regrets having dropped out of college after his first year.
D. Only if Julian had dropped out of college after his first year.
Question 26. Seth informed us of his retirement from the company. He did it when arriving at the
meeting.
A. Only after his retiring from the company did Seth tell us about his arrival at the meeting.
B. Not until Seth said to us that he would leave the company did he turn up at the meeting
C. Hardly had Seth notified us of his retiring from the company when he arrived at the
meeting.
D. No sooner had Seth arrived at the meeting than we were told about his leaving the company.
Mark the letter A, B, C or D on your answer sheet to indicate the most suitable response to
complete each of the following exchanges.
Question 27. A mother is complaining to her son that his room gets dusty.
- Mother: “Haven’t you tidied up your room yet?”
- Son: “___________”
A. I will, after I do all the exercises in my textbook.
B. I am tired with my tidying.
C. The more I tidy, the worse it gets.
D. Why don’t you give me a hand with cleaning?
Question 28. Linda and Janet are talking about family living under one roof.
Linda: “I think it’s a good idea to have three or four generations living under one roof. They can help
one another.”
Janet: “___________. Many old-aged parents like to lead independent life in a nursing home.”
A. It’s nice to hear that B. Me, too.
C. I agree with you completely D. That’s not true
Mark the letter A, B, C or D on your answer sheet to indicate the word or phrase that is
CLOSEST in meaaningto the underlined part in each of the following questions.
Question 29. We spent the entire day looking for a new apartment.
A. all long day B. day after day C. all day long D. the long day

Trang 3
Question 30.1 just want to stay at home and watch TV and take it easy.
A. relax B. sit down C. sleep D. eat
Mark the letter A, B, C or D on your answer sheet to indicate the word(s) OPPOSITE in meaning
to the underlined word(s) in each of the following questions.
Question 31. In a study, more Asian students than American students hold a belief that a husband is
obliged to tell his wife his whereabouts if he comes home late.
A. urged B. free C. required D. suggested
Question 32. Though I persuaded my boss to solve a very serious problem in the new management
system, he just made light of it.
A. completely ignored B. treated as important
C. disagreed with D. discovered by chance
Read the following passage and mark the letter A, B, C or Don your answer sheet to indicate the
correct word or phrase that best fits each of the numbered blanks from 33 to 37.
CYBERFASHION
Most of us own modern gadgets such as mobile phones, or digital cameras. We may carry them round
in our pockets, or attach them to our bodies. But not for much longer! So far designers have succeeded
in (33) ___________tiny bits of technology directly into our clothing.
In actual fact “cyberfashion” is the latest trend! One example, the Musical Jacket, is already in the
shops and stores. This jacket is silk. It is (34) ___________by a keyboard, also manufactured from
fabric, which is connected to a tiny device (35) ___________plays music.
At present, you have to touch a shoulder pad to hear the music. But in the future, you will be able to
operate the device just by turning your wrist or walking! For many athletes, scientists have invented a
smart shirt which measures your heart rate, body temperature and respiration rate! (36) ___________,
the most romantic piece of cyberfashion must be the Heartthrob Brooch.
This item of jewellery, made from diamonds and rubies, has two miniature transmitters. They make the
brooch glow in time to the beating of its wearers heart. If you meet someone (37) ___________, your
heart will beat faster - and your brooch will let everyone know how you feel!
Question 33. A. dividing B. associate C. combining D. integrating
Question 34. A. monitored B. examined C. controlled D. managed
Question 35. A. that B. what C. who D. when
Question 36. A. Although B. However C. Moreover D. While
Question 37. A. gorge B. gorgeously C. gorgeousness D. gorgeous
Read the following passage and mark the letter A, B, C or Don your answer sheet to indicate the
correct answer to each of the questions
Imagine the busy streets of New York City, an enormous place with millions of people. Everyday, the
streets are congested with people going about their daily lives. Now imagine a small robot in the

Trang 4
middle of all of those people rolling down a busy sidewalk. Most people would not even notice the ten-
inch smiling robot, called a Tweenbot, rolling along the street. This strange machine may interest some
people, while others would ignore it completely. A researcher interested in studying how helpful people
really are uses such robots in her experiments that take place on the streets of New York.
The Tweenbots experiment is the idea and creation of Kacie Kinzer. Kinzer s idea was to make a robot
that could navigate the city and reach its destination only if it was aided by pedestrians. Tweenbots rely
on the kindnessof warm-hearted strangers. Made simply of cardboard, wheels, and a device to turn the
wheels, the Tweenbots face many dangers on the city streets. They could be run over by cars or
smashed by careless kids. Kinzer thought her little robots might even be seen as some kind of terrorist
device. The only real protection a Tweenbot has is its friendly smile. In addition to that, each of
Kinzer’s robots is fitted with a flag that displays instructions for the robot’s destination. The only way
these robots will reach their final point is if someone lends them a hand. Tweenbots are essentially a
social experiment aimed at providing people a chance to show how caring they are.
On a daily basis, people in New York City are often in a hurry to get around. However, the Tweenbots,
through their inability to look after themselves, took people out of their normal routines. The people
who noticed the helpless little robots were actually interested in helping the Tweenbots find their way
home. Tweenbots move at a constant speed and can only go in a straight line. If one was to get stuck, or
was going in the wrong direction, it would be up to strangers to free it or turn it in the right direction.
Surprisingly, no Tweenbot was lost or damaged, and each one arrived at its target in good condition. In
fact, most people treated the robot in a gentle manner, and some even treated it as though it were a
small living being. Even if you were in a rush to go somewhere, would you stop and help a Tweenbot
successfully reach its destination?
Question 38. what is this reading about?
A. A place to buy robots B. A new kind of toy
C. An experiment D. An interesting idea for the future
Question 39. what is a Tweenbot?
A. A person from New York City B. A ten-inch smiling robot
C. A pedestrian D. A terrorist device
Question 40. The word “congested” in paragraph 1 is closest in meaning to___________
A. flocked B. secluded C. stuffed D. crowded
Question 41. How did a Tweenbot get to its final destination?
A. With the help of other Tweenbots.
B. With the help of kind pedestrians on the street
C. With the help of Kacie Kinzer.
D. With the help of other robots in New York City.
Question 42. which of the following statements is NOT correct?

Trang 5
A. Most Tweenbots arrived at their destination damaged or broken.
B. Most people treated the Tweenbots in a gentle manner.
C. Tweenbots could not navigate the city on their own.
D. Tweenbots move at a constant speed and can only go in a straight line.
Read the following passage and mark the letter A, B, C or D on your answer sheet to indicate the
correct answer to each of the questions from 43 to 50
Being aware of one's own emotions - recognizing and acknowledging feelings as they happen - is at the
very heart of Emotional Intelligence. And this awareness encompasses not only moods but also
thoughts about those moods. People who are able to monitor their feelings as they arise are less likely
to be ruled by them and are thus better able to manage their emotions. Managing emotions does not
mean suppressing them; nor does it mean giving free rein to every feeling. Psychologist Daniel
Goleman, one of several authors who have popularized the notion of Emotional Intelligence, insisted
that the goal is balance and that every feeling has value and significance. As Goleman said, "A life
without passion would be a dull wasteland of neutrality, cut off and isolated from the richness of life
itself." Thus, we manage our emotions by expressing them in an appropriate manner. Emotions can also
be managed by engaging in activities that cheer us up, soothe our hurts, or reassure us when we feel
anxious.
Clearly, awareness and management of emotions are not independent. For instance, you might think
that individuals who seem to experience their feelings more intensely than others would be less able to
manage them. However, a critical component of awareness of emotions is the ability to assign meaning
to them - to know why we are experiencing a particular feeling or mood. Psychologists have found that,
among individuals who experience intense emotions, individual differences in the ability to assign
meaning to those feelings predict differences in the ability to manage them. In other words, if two
individuals are intensely angry, the one who is better able to understand why he or she is angry will
also be better able to manage the anger. Self-motivation refers to strong emotional self-control, which
enables a person to get moving and pursue worthy goals, persist at tasks even when frustrated, and
resist the temptation to act on impulse. Resisting impulsive behavior is, according to Goleman, "the
root of all emotional self-control."
Of all the attributes of Emotional Intelligence, the ability to postpone immediate gratification and to
persist in working toward some greater future gain is most closely related to success - whether one is
trying to build a business, get a college degree, or even stay on a diet. One researcher examined
whether this trait can predict a child's success in school. The study showed that 4-year-old children who
can delay instant gratification in order to advance toward some future goal will be "far superior as
students" when they graduate from high school than will 4-year-old who are not able to resist the
impulse to satisfy their immediate wishes.
Question 43. which of the following can we infer from paragraph 1?

Trang 6
A. Some people can understand their feelings better than others.
B. People who can manage their emotions will be controlled by them.
C. If people pay attention to their feelings, they will not be able to manage them.
D. If people pay attention to their feelings, they can control their emotions better.
Question 44. The word "encompasses" in paragraph 1 could be best replaced by ___________
A. maintains B. includes C. entertains D. divides
Question 45. The word "soothe" in paragraph 2 could be best replaced by___________.
A. reduce B. weaken C. worsen D. relieve
Question 46. From paragraph 2, we can see that Daniel Goleman___________
A. trained people to increase their Emotional Intelligence
B. treated patients who had emotional problems
C. wrote about Emotional Intelligence
D. studied how people manage their emotions
Question 47. All of the following are mentioned in paragraph 2 about our emotions
EXCEPT___________.
A. every feeling is important B. we can manage our emotions
C. emotions are part of a satisfying life D. we should ignore some feelings
Question 48. The word "them" in paragraph 3 refers to___________.
A. psychologists B. individuals
C. individual differences D. intense emotions
Question 49. In paragraph 3, the author explains the concept of awareness and management of
emotions by___________.
A. describing how people learn to control their emotions
B. giving an example of why people get angry
C. comparing how two people might respond to an intense emotion
D. explaining why some people are not aware of their emotions
Question 50. According to paragraph 5, children might be more successful in school if they can resist
impulses because they can ___________.
A. easily understand new information
B. be more popular with their teachers
C. have more friends at school
D. focus on their work and not get distracted
Đáp án
1-D 2-B 3-C 4-B 5-B 6-A 7-C 8-B 9-D 10-B
11-A 12-A 13-B 14-A 15-C 16-D 17-B 18-A 19-A 20-C
21-C 22-B 23-C 24-C 25-C 26-D 27-A 28-D 29-C 30-A

Trang 7
31-B 32-B 33-D 34-C 35-A 36-B 37-D 38-C 39-B 40-D
41-B 42-A 43-D 44-B 45D 46-C 47-D 48-D 49-C 50-D

LỜI GIẢI CHI TIẾT


Question 1: Đáp án D
Đáp án D đúng vì phân gạch chân của phương án D phát âm là /s/ vì kết thúc bằng phụ âm vô thanh /p/.
Các phương án còn lại phát âm là /z/ vì kết thúc bằng các phụ âm hữu thanh.
Lưu ý:
s phát âm là /s/ khi đứng sau t, th, k, f, gh, ph, p và thêm te, ke, pe (do e đứng sau các phụ âm này bị câm).
s phát âm là /iz/ khi đứng sau /s/-ss,ce,x, /z/- zz, /ʃ/-sh, /tʃ/-ch , /dʒ/-ge
s phát âm là /z/ khi đứng sau các trường hợp còn lại.
A. grows / /ɡrəʊz/ (v): sinh sôi, phát triển
B. tends /tendz/ (v): có dự định làm gì (tend to do sth)
C. roars /rɔː(r)z/ (v/n): gầm gừ/ tiếng gầm gừ (của sư tử)
D. sweeps /swi:ps/(v): quét dọn, lau
Question 2. Đáp án B
Đáp án B đúng vì phần gạch chận của đáp án B phát âm là /t/, các phương án còn lại có phần gạch chân
phát âm là /d/.
ed phát âm là /t/ khi đứng sau: k, p, gh, ph, ss, ce, x, sh, ch
ed phát âm là /id/ khi đứng sau: t, d
ed phát âm là /d/ với các trường hợp còn lại. Dịch đáp án.
A. performed /pəˈfɔːmd/ (v): biểu diễn, trình bày
B. finished /ˈfɪnɪʃt/ (v): hoàn thành, kết thúc
C. interviewed /ˈɪntəvjuːd/ (v): phỏng vấn
D. delivered /dɪˈlɪvə(r)d/ (v): vận chuyển, chuyển phát
Question 3. Đáp án C
Phương án C đúng vì có trọng âm rơi vào âm tiết số hai, các phương án còn lại có trọng âm rơi vào âm
tiết thứ nhất.
A. justice /ˈdʒʌstɪs/ (n): công lý
B. struggle /ˈstrʌɡl/ (v): đấu tranh
C. neglect /nɪˈɡlekt/ (v): bất cẩn, lơ là
D. wildlife /ˈwaɪldlaɪf/ (n): cuộc sống hoang dã
Question 4. Đáp án B
Phương án B đúng vì có trọng âm rơi vào âm tiết số ba, các phương án còn lại có trọng âm rơi vào âm tiết
số hai.
A. eradicate /ɪˈrædɪkeɪt/ (v): xóa sổ

Trang 8
B. unexpected /ˌʌnɪkˈspektɪd/ (adj): không mong đợi
C. accompany /əˈkʌmpəni/ (v): đi cùng, đi theo; đệm đàn cho ai hát
D. commitment /kəˈmɪtmənt/ (n): sự cam kết, bản cam kết
* Note: Hầu hết tiền tố và hậu tố khi được thêm vào từ sẽ không làm biến đổi trọng âm của từ gốc.
(ngoại trừ đuôi “tion”)
Ví dụ khi tìm hiểu trọng âm của B -unexpected, ta hãy bỏ ra tiền tố “un” và hậu tố “ed”, từ gốc expect
nhận trọng âm rơi vào "pect” và vẫn giữ nguyên trọng âm khi có cả tiền tố và hậu tố đi kèm.
Tương tự D-commitment, hậu tố “ment” khi thêm vào không làm thay đổi trọng âm gốc của từ commit.
Question 5. Đáp án B
Kiến thức được hỏi - Kết hợp thì.
Đây là dạng câu phối hợp thì để diễn tả một hành động đang xảy ra thì có một hành động khác xen vào.
Hành động đang xảy ra chia thì quá khứ tiếp diễn “were walking” , hành động xen vào chia thì quá khứ
đơn “asked”. Vậy chọn đáp án B.
Tạm dịch: chúng tôi đang đi trong im lặng thì anh ta đột nhiên nhờ tôi giúp đỡ.
Question 6. Đáp án A
Kiến thức được hỏi: Động từ theo sau là To V.
Ta có: refuse to V: từ chối làm gì đó
Do đó chọn đáp án A
Tạm dịch: Tôi chẳng hiểu tại sao mà John luôn từ chối gặp mặt tôi trực tiếp.
Question 7.
Kiến thức được hỏi - Động từ khuyết thiếu với dạng hoàn thành.
Với các tình huống nói lại về một chuyện/ phỏng đoán chuyện đã xảy ra trong quá khứ, chúng ta dùng cấu
trúc chung là Modal Verb+ have+ P2.
Đề bài có “last night” với ý phỏng đoán về quá khứ nên ta dùng cấu trúc trên.
Ta có: must/can't/couldn't + have + P2 thể hiện suy đoán chắc chắn trong quá khứ
must have P2: chắc hẳn đã làm gì
can’t/ couldn’t have P2: không thể nào đã làm điều này được
(nhớ rằng ta không có mustn’t have P2)
should have P2: nên làm nhưng đã không làm
needn’t have P2: đáng lẽ ra không cần thiết phải làm nhưng đã làm
Với ngữ nghĩa của câu cho trước cần một suy đoán là không thể nào trong quá khứ ta chọn C.
Tạm dịch: Người bạn nhìn thấy tối qua không thể là Sue được. Cô ấy đang ở nước ngoài cùng với gia
đình.
Question 8. Đáp án B
Kiến thức được hỏi - Câu bị động

Trang 9
Giải thích: Với dấu hiệu “by dinosaurs” và ngoại động từ “rule": thống trị cũng đang thiếu tân ngữ đi sau,
ta nhận ra đây là câu bị động với cấu trúc. S+ be + P2 (by O).
Và chỉ có câu B và C là đúng quy tắc bị động; A và D đang là chủ động nên bị loại.
Vì dấu hiệu thời gian “At one time": xưa kia, đã có lúc ở thì quá khứ nên chọn câu B, loại C do C thể hiện
ý tương lai.
Tạm dịch: Xưa kia, toàn bộ thế giới bị thống trị bởi loài khủng long.
Question 9. Đáp án D
Kiến thức được hỏi - Câu gián tiếp.
Đây là dạng câu gián tiếp (câu tường thuật) dạng Yes/No Question.
Theo đó, ta dùng cấu trúc. S+ asked sh If/ whether+ S+ V (lùi thì), (với whether có thể dùng “or not” ở
cuối hoặc là không)
Vậy chỉ có câu D là đúng cấu trúc nên đáp án là D. Lưu ý rằng trong câu dẫn gián tiếp ta sẽ không còn
hình thức đảo ngữ của câu hỏi nữa.
Tạm dịch: Tôi hỏi Martha rằng liệu có phải cô ấy đang có dự định vào trường luật không.
Question 10. Đáp án B
Kiến thức được hỏi - Mệnh đề quan hệ (MĐQH)
Câu hỏi này hỏi về MĐQH có đảo giới từ lên trước.
Ta có: giới từ + whom/ which (whom khi đối tượng là người và which khi đối tượng là vật)
Cả 4 đáp án đã dùng đúng which để thay thế cho danh từ “situation”, điều cần lưu tâm là dùng giới từ nào
đi với danh từ này, rõ ràng ta phải dùng “in a situation”- trong một tình huống. Vậy B được chọn.
Tạm dịch: Tuyệt chủng nghĩa là một tình huống khi mà một loài thực vật, một loài động vật hoặc một lối
sống không còn tồn tại nữa.
Question 11. Đáp án A
Kiến thức được hỏi - Đảo ngữ.
Liếc nhìn các phương án bắt đầu bằng Hardly, no sooner, no longer, not until ta nhận ra rằng câu được
hỏi đang dùng dạng đảo ngữ.
Có thể loại trước phương án C vì không hài hòa về thì, trong câu này gắn kết giữa hiện tại hoàn thành và
quá khứ đơn bị vô lí, cùng với đó ngữ nghĩa không ổn.
Câu có chứa “when” ở vế sau nên cấu trúc được hỏi chính là.
Hardly+ had+ S+ P2 When S+ Ved: vừa mới... thì đã → Đáp án A
Các phương án còn lại đi cấu trúc tương ứng:
No sooner+ had+ S + P2 Than S + Ved: vừa mới... thì đã
No longer + aux + S + V: không làm gì đó nữa
Not until+ time/ S + Ved + aux + S + V: phải đến tận khi... thì
Tạm dịch: Anh ấy vừa tới trạm xe buýt thì xe buýt cũng đến nơi.
Question 12. Đáp án A

Trang 10
Kiến thức - Câu hỏi đuôi
Câu hỏi đuôi của “ Let’s ...” chính là “ “ shall we?”. Đây là một trường hợp đặc biệt chúng ta cần học
thuộc.
→ Chọn đáp án A
Dạng câu hỏi đuôi thông thường là. S + V, aux+ S? (Chú ý vế trước và vế sau đối lập nhau, vế trước là
khẳng định thì vế sau là phủ định và ngược lại).
You don’t like icecream, do you?
She is Song Hye Kyo, isn’t she?
Tạm dịch: Chúng ta bắt đầu thảo luận bây giờ chứ nhỉ?”
Question 13. Đáp án B
Kiến thức được hỏi - Loại từ.
Xác định vị trí chỗ trống cần một tính từ do look+ adj - trông như thế nào, cũng như liên từ đẳng lập
“and” liên kết chỗ trống và tính từ “positive” nên khẳng định được chỗ trống cũng phải là tính từ. Ta có:
A. confidence (n): sự tự tin.
B. confident (adj): tự tin.
C. confide (v): giao phó, kể bí mật.
D. confidently (adv): một cách tự tin.
→ Như vậy ta chọn đáp án B
Tạm dịch: Bạn càng trông tự tin và tích cực thì bạn càng cảm thấy tốt hơn.
Question 14. Đáp án A
Kiến thức được hỏi - Lựa chọn từ
A. prolific (adj): viết nhiều, ra nhiều tác phẩm
B. fruitful (adj): sai quả, có kết quả, thành công
C. fertile (adj): màu mỡ, phì nhiêu
D. successful (adj): thành công
Căn cứ vào nghĩa ta chọn đáp án A.
Tạm dịch: Ông Pike chắc chắn là một nhà văn viết nhiều; ông đã viết rất nhiều cuốn sách trong năm nay.
Question 15. Đáp án C
Kiến thức được hỏi - Lựa chọn từ
A. uncontrolled (adj): không kiểm soát
B. arranged (adj): được sắp xếp
C. chaotic (adj): hỗn độn
D. notorious (adj): mang tiếng xấu
Căn cứ vào nghĩa ta chọn được đáp án C.
Tạm dịch: Tuần học đầu tiên ở trường đại học có một chút hỗn độn vì rất nhiều sinh viên bị lạc, đổi lớp
hoặc tới sai địa điểm.

Trang 11
Question 16. Đáp án D
Kiến thức được hỏi - Thành ngữ/ Cụm từ
Ta có cụm cố định from scratch : từ đầu
Thường kết hợp Do/ Begin/ Start things from scratch - làm/ bắt đầu cái gì từ đầu.
→ Ta chọn đáp án D.
Các phương án khác.
A. onset (n): khởi đầu của điều gì, đặc biệt là điều gì không dễ chịu.
B. source (n): nguồn gốc, nơi cung cấp, người cung cấp
C. original (n): bản chính, sản phẩm gốc
Tạm dịch : Do sự cố máy tính, tất cả các dữ liệu của chúng tôi đã bị mất. Vì vậy thật đáng buồn, chúng tôi
phải bắt đầu tất cả các tính toán từ đầu.
Question 17. Đáp án B
Kiến thức được hỏi - Kết hợp từ
Make (no) difference - (không) tạo ra sự khác biệt → Chọn đáp án B
A. Do (v): làm
C. Cause (v): gây ra
D. Create (v): tạo ra
Tạm dịch. Sự phản kháng của các cầu thủ không tạo ra sự khác biệt gì đối với quyết định của trọng tài.
Question 18. Đáp án A
Kiến thức được hỏi - Phrasal Verb
Nghĩa của các phương án:
A. (to) put up with: chịu đựng
B. (to) catch up with: đuổi kịp
C. (to) keep up with: theo kịp
D. (to) come down with: mắc bệnh
Căn cứ vào nghĩa ta chọn đáp án A
Tạm dịch: Mặc dù anh ấy là bạn của tôi, nhưng tôi không thể chịu được tính ích kỷ của anh ta
Question 19. Đáp án A
Lỗi sai ở cách dùng mạo từ đi với cụm A number of Ns với the number of Ns.
Ta biết: A number of Ns+ V số nhiều: Nhiều/ số lượng lớn cái gì đó
The number of Ns + V số ít: Số lượng của cái gì
Trong câu này ta cần dùng: the number of wild horses... is increasing: số lượng ngựa hoang... đang tăng
lên. (Chứ không phải a number of wild horses... is increasing: nhiều ngựa hoang đang tăng lên)
→ Chọn đáp án A
Sửa lại: A number of → the number of

Trang 12
Tạm dịch: Số lượng ngựa hoang ở Assateague đang gia tăng nhanh chóng, dẫn đến những bãi cỏ đầm lầy
và cỏ dốc trở nên quá tải.
Question 20. Đáp án C
Lỗi sai về dạng động từ to V/ Ving.
Ta có: expect + O + to V: kì vọng, mong chờ ai làm gì → C sai.
Sửa lại: changing → to change
Tạm dịch: Khi tới nơi, một số nhân sự được dự kiến sẽ thay đổi đặt chỗ của họ và tiến tới Hawaii.
Question 21. Đáp án C
Lỗi sai về cấu trúc
Ta có: Let + O + Vo (Vo động từ nguyên thể không to): để cho ai làm gì.
→ Ta chọn được C
Sửa lại: To give → give
Tạm dịch: Mặc dù hội chữ thập đỏ chấp nhận máu từ hầu hết những người hiến tặng, nhưng y tá sẽ không
cho bạn hiến máu nếu như bạn vừa mới bị cảm.
Question 22. Đáp án B
Câu chuyển từ trực tiếp sang gián tiếp.
Câu gốc: “Tôi sẽ loại cậu ra khỏi đội nếu cậu không luyện tập chăm chỉ hơn.” Đội trưởng nói với John
Dịch đáp án.
A. John được nhắc nhở phải luyện tập chăm chỉ hơn để mà không bị loại khỏi đội.
B. Đội trưởng dọa sẽ loại John ra khỏi đội nếu anh ta không luyện tập chăm chỉ hơn.
C. Đội trưởng thúc giục rằng John nên luyện tập chăm chỉ hơn để mà không bị loại ra khỏi đội.
D. Đội trưởng hứa sẽ loại John ra khỏi đội trong trường hợp anh ta luyện tập chăm chỉ.
Ta có các cấu trúc.
A. remind sb to do sth: nhắc ai nhớ làm gì
B. threaten to do sth: doạ sẽ làm gì
C. urge that.... giục ai làm gì
D. promise to do sth: hứa làm gì
B phù hợp với ý của câu gốc nhất nên ta chọn B.
Question 23. Đáp án C
Dịch câu gốc: Có thể là Joanna đã không nhận được tin nhắn của tôi
Dịch các phương án.
A. Joanna đáng lẽ không nên nhận được tin nhắn của tôi
B. Joanna đáng lẽ ra không cẩn nhận được tin nhắn của tôi
C. Joanna có lẽ đã không nhận được tin nhắn của tôi.
D. Joanna đã không thể nhận được tin nhắn của tôi
shouldn’t have P2: không nên làm gì trong quá khứ (nhưng đã làm)

Trang 13
needn’t have P2: không cần làm gì trong quá khứ (nhưng đã làm)
mightn’t have P2: có thể có lẽ đã không
can’t have P2: không thể nào đã làm
Phù hợp nhất với nghĩa gốc là đáp án C
Question 24. Đáp án C
Dịch câu hỏi: Tiếng ồn ở phòng bên cạnh đã không dừng lại cho tới tận nửa đêm.
Dịch đáp án.
A. Tiếng ồn ở phòng bên cạnh dừng lại lúc nửa đêm.
B. Đã là nửa đêm và tiếng ồn ở phòng bên cạnh đã dừng lại.
C. Phải cho đến sau nửa đêm thì tiếng ồn ở phòng bên cạnh mới dừng lại.
D. Sai ngữ pháp: the noise next door stopped → did the noise next door stop
D sai ngữ pháp, A và B sai ngữ nghĩa chỉ có C là đáp án đúng.
Ta có cách viết:
S + did + not + V + until + mốc thời gian: S đã không… cho tới tận...
= Not until + mốc thờigian+ did + S + V = It was not until + mốc thời gian+ that + S + Ved.
Question 25. Đáp án C
Dịch câu gốc: Julian bỏ học cao đẳng sau năm nhất. Bây giờ anh ta hối tiếc về điều đó.
Dịch các phương án.
A. Julian đã hối hận vì trước đó đã bỏ học cao đẳng sau năm nhất (Sai do sử dụng động từ “regretted” ở
dạng quá khứ đơn).
B. Julian ước anh ta đã không bỏ học cao đẳng sau năm nhất (Câu đang viết ở dạng ước muốn ở hiện tại S
+ wish(es) + S + Ved. Như vậy B sai do hành động “dropped out” diễn ra ở thì quá khứ đơn, phải sử
dụng cấu trúc ước muốn ở quá khứ S + wish(es) + S + had P2.
C. Julian hối hận vì đã bỏ học cao đẳng sau năm nhất
D. Chỉ khi Julian đã bỏ học cao đẳng sau năm nhất. (D sai do câu viết lại không hợp nghĩa với câu gốc).
→ Chọn đáp án C
(to) regret + Ving / having P2: hối hận về điều gì đã làm trong quá khứ.
Question 26. Đáp án D
Câu gốc: Seth thông báo cho chúng tôi vê' việc ông ấy nghỉ hưu ở công ty. Ông ấy đã làm điều đó khi đến
cuộc họp.
Các phương án.
A. Chỉ sau khi ông ấy nghỉ hưu ở công ty, Seth mới nói với chúng tôi về việc ông ấy đến cuộc họp.
B. Mãi cho đến khi Seth nói với chúng tôi rằng ông ấy sẽ rời công ty, ông ấy mới đến cuộc họp.
C. Ngay khi Seth thông báo cho chúng tôi về việc ông nghỉ hưu từ công ty thì ông đến cuộc họp.
D. Ngay khi Seth đến cuộc họp thì chúng tôi mới được nghe ông ấy nói về việc rời công ty.
Câu A, B, C sai về nghĩa. Chọn D.

Trang 14
Only after + N/Ving/clause + auxiliary + S + V: Chỉ sau khi... thì
Not until + N/clause + auxiliary + S + V: Chỉ đến khi... thì
Hardly + had + S + P2 + when + S + Ved: Ngay khi ... thì/ Vừa mới... thì đã... = No sooner + had + S +
P2 + than + S + Ved
Question 27. Đáp án A
Người mẹ đang phàn nàn về cái phòng bừa bộn của con trai mình.
- Mẹ: Con vẫn chưa dọn phòng à con trai?
- Con....
A. Làm xong bài tập trong sách rồi con sẽ dọn mẹ ạ.
B. Con chán với việc dọn phòng.
C. Con càng dọn thì phòng càng bẩn mẹ ạ.
D. Sao mẹ không giúp con một tay dọn phòng nhỉ?
Phù hợp nhất với tình huống này là đáp án A.
Question 28. Đáp án D
Linda và Janet đang thảo luận về những gia đình sống chung một mái nhà.
Linda: Tớ nghĩ rằng thật là một ý tưởng hay khi có ba hoặc bốn thế hệ cùng sống chung dưới một mái
nhà. Mọi người có thể giúp đỡ lẫn nhau.
Janet:__________. Nhiều người già thích sống một cuộc sống độc lập ở trại dưỡng lão.
A. Thật tuyệt khi nghe điều đó
B. Tớ cũng vậy
C. Tớ hoàn toàn đồng ý với bạn
D. Điều này chưa thực sự đúng.
Đây là một tình huống thể hiện quan điểm nên câu phản hồi có thể là đồng ý hoặc bất đồng. Rõ ràng Janet
có ý kiến khác với Linda nên ta chọn câu D thể hiện ý không tán đồng.
Question 29. Đáp án C
Bài cần tìm từ đồng nghĩa với cụm “the entire day” - cả ngày.
Không có cách diễn đạt A.
B. day after day: ngày qua ngày
C. all day long: cả ngày
D. the long day: ngày dài
“the entire day” đồng nghĩa với all day long - cả ngày. Chọn C.
Tạm dịch: chúng tôi đã dành cả ngày tìm một căn hộ mới.
Question 30. Đáp án A
Dữ liệu: Tôi chỉ muốn ở nhà, xem TV và take it easy.
Có thể đoán nghĩa của cụm từ thông qua văn cảnh hoặc thế từng phương án lên câu gốc.
take it easy: bình tĩnh, thư giãn

Trang 15
A. relax (v): thư giãn
B. sit down (v): ngồi xuống
C. sleep (v): ngủ
D. eat (v): ăn
Đồng nghĩa với take it easy là relax, đáp án A.
Tạm dịch: Tôi chỉ muốn ở nhà, xem TV và thư giãn
Question 31. Đáp án B
Nhận ra tính từ obliged xuất phát từ oblige (v): bắt buộc, có nghĩa vụ. (Từ vựng và bối cảnh trong SGK
lớp 12)
A. urge (v): thúc giục
B. free (adj): tự do
C. require (v): yêu cầu
D. suggest (v): đề nghị
Vậy trái nghĩa của obliged là free, đáp án B.
Tạm dịch: Một nghiên cứu cho thấy rằng, nhiều sinh viên châu Á hơn sinh viên Mỹ tin rằng người chồng
có nghĩa vụ phải nói cho vợ biết anh ta đang ở đâu nếu anh ta về nhà muộn.
Question 32. Đáp án B
Dữ liệu bài: Mặc dù tôi đã thuyết phục sếp của mình giải quyết một vấn đề rất nghiêm trọng trong hệ
thống quản lý mới, ông ấy chỉ made light of it.
Để đoán nghĩa từ gạch chân trong câu này chú ý đặc biệt từ Although- liên từ thể hiện ý đối lập. Dù đã cố
thuyết phục nhưng sếp vẫn coi nhẹ điều đó.
make light of something: coi nhẹ điều gì, xem như không quan trọng
A. completely ignored: hoàn toàn lờ đi
B. treated as important: xem là quan trọng
C. disagreed with: không đồng ý với
D. discovered by chance: tình cờ phát hiện ra
Trái nghĩa với made light of là treated as important. Chọn B.
Tạm dịch: Mặc dù tôi đã thuyết phục sếp của mình giải quyết một vấn đề rất nghiêm trọng trong hệ thống
quản lý mới, ông ấy xem điều đó không quan trọng.
Question 33. Đáp án D
Đây là câu hỏi về từ vựng, ta chọn phương án có ngữ nghĩa hợp nhất với văn cảnh của bài. Ý của hai câu
trước là mỗi chúng ta đều sở hữu các thiết bị công nghệ và có thể để trong túi áo và mang theo mình.
Nhưng ngày nay đã có cách để tích hợp thiết bị công nghệ vào quần áo của chúng ta. Chọn D với nghĩa
phù hợp nhất.
A. dividing (v-ing): chia
B. associate (v); liên kết

Trang 16
C. combining (v-ing): kết hợp
D. integrating (v-ing): tích hợp, hợp nhất
So far designers have succeeded in integrating tiny bits of technology directly into our clothing.
Tạm dịch: Cho đến nay, các nhà thiết kế đã thành công trong việc tích hợp các mẫu nhỏ công nghệ trực
tiếp vào quần áo của chúng ta.
Question 34. Đáp án C
Câu hỏi về từ vựng.
Bài đưa ra ví dụ về “Musical Jaket”- Chiếc áo khoác biết hát, hiện đã có mặt ở các cửa hàng, chiếc áo này
được điều khiển bằng bàn phím.
A. monitor (v): giám sát
B. examine (v): kiểm tra
C. control (v): điều khiển /kiểm soát
D. manage (v): xoay sở/quản lí
→ Chọn C.
...It is controlled by a keyboard, also manufactured from fabric,...
Tạm dịch: Nó được điều khiển bằng bàn phím, cũng được sản xuất từ vải,...
Question 35. Đáp án A
Chọn đại từ quan hệ đứng sau danh từ chỉ vật “a tiny device” - một thiết bị nhỏ và đứng trước động từ
“plays music” làm chủ ngữ trong câu. Như vậy ta chỉ có phương án A-that là phù hợp.
Trong mệnh đề quan hệ:
That thay cho “which/ who/ whom” trong mệnh đề quan hệ xác định.
What điều mà, cái mà (không phải đại từ quan hệ, what = everything that)
Who thay cho danh từ chỉ người; đóng vai trò chủ ngữ trong mệnh đề quan hệ
When đóng vai trò cho trạng từ chỉ thời gian, thay thế cho các danh từ chỉ thời gian; time when + S + V
It is controlled by a keyboard, also manufactured from fabric, which is connected to a tiny device that
plays music.
Tạm dịch: Nó được điều khiển bằng bàn phím, cũng được sản xuất từ vải, được kết nối với một thiết bị
nhỏ phát nhạc.
Question 36. Đáp án B
Câu hỏi về từ nối, trong tình huống này sử dụng từ nối để liên kết hai câu mang ý đối lập. Câu trước đề
cập đến chiếc áo thông minh có thể đo được nhịp tim, nhiệt độ cơ thể và nhịp thở của các vận động viên,
tuy nhiên phần lãng mạn độc đáo nhất của kiểu thời trang mạng phải là Heartthrob Brooch. Đáp án B là
phù hợp nhất.
A. Although: mặc dù
B. However: tuy nhiên
C. Moreover: hơn nữa

Trang 17
D. While: trong khi
For many athletes, scientists have invented a smart shirt which measures your heart rate, body
temperature and respiration rate! However, the most romantic piece of cyberfashion must be the
Heartthrob Brooch.
Tạm dịch: Đối với nhiều vận động viên, các nhà khoa học đã phát minh ra một chiếc áo thông minh đo
nhịp tim, nhiệt độ cơ thể và nhịp thở của bạn! Tuy nhiên, tác phẩm lãng mạn nhất của thời trang mạng
phải là Trâm Heartthrob.
Question 37.
Câu hỏi về từ loại, xác định vị trí chỗ trống nằm sau đại từ bất định someone ta dùng tính từ với nghĩa ai
đó như thế nào. Ta chọn D.
A. gorge (n); hẻm núi, đèo
B. gorgeously (adv): một cách rất đẹp
C. gorgeousness (n): vẻ đẹp, sự hoa mỹ
D. gorgeous (adj): đẹp lộng lẫy, rực rỡ
If you meet someone gorgeous, your heart will beat faster - and your brooch will let everyone know how
you feel!
Tạm dịch: Nếu bạn gặp một người tuyệt đẹp, trái tim bạn sẽ đập nhanh hơn - và chiếc trâm cài của bạn sẽ
cho mọi người biết bạn cảm thấy thế nào!
READING 1
Nội dung của bài xoay quanh một cuộc thử nghiệm với sự tham gia của những chú robot Tweenbots nhỏ
bé cỡ 10 inch, với gương mặt mỉm cười lăn trên đường phố. Những chú robot này không thể tự tìm đường
cũng như không thể tới được đích đến nếu như không có sự giúp đỡ của những người đi bộ tốt bụng. Hầu
hết mọi người đều đối xử rất nhẹ nhàng với những chú robot này và thật ngạc nhiên là không con robot
nào bị hư hại khi trở về cả. Đây là một thí nghiệm nhằm dành cho mọi người cơ hội được thể hiện sự
quan tâm đối với những điểu nhỏ bé trong cuộc sống và những người xung quanh.
Question 38. Đáp án C
Dịch đề: Bài đọc này nói về cái gì?
A. Nơi mua robot
B. Một loại đồ chơi mới
C. Một thí nghiệm
D. Một ý tưởng thú vị cho tương lai
Bài đọc xoay quanh về cuộc thí nghiệm thú vị về sự quan tâm của mọi người với nhau và với những thứ
trong cuộc sống.
Từ “experiment”- thí nghiệm xuất hiện ở hầu hết các đoạn trong bài.
Dòng cuối đoạn 1: A researcher interested in studying how helpful people really are uses such robots in
her experiments that take place on the streets of New York.

Trang 18
Một nhà nghiên cứu rất thích thú tìm hiểu xem mọi người có sẵn sàng giúp đỡ người khác hay không đã
sử dụng những con robot trong thí nghiệm của cô ấy diễn ra trên đường phố New York.
Đầu đoạn 2: The Tweenbots experiment is the idea and creation of Kacie Kinzer.
Cuộc thí nghiệm Tweenbots là ý tưởng và sản phẩm của Kacie Kinzer
Cuối đoạn 2: Tweenbots are essentially a social experiment aimed at providing people a chance to show
how caring they are.
Tweenbots thực sự là một thí nghiệm xã hội nhằm đem lại cho con người cơ hội được thể hiện sự quan
tâm với người khác.
→ Chọn C
Question 39. Đáp án B
Dịch đề: Tweenbot là gì?
A. Một người đến từ thành phố New York
B. Một robot biết cười cao 10 inch
C. Một người đi bộ
D. Một thiết bị khủng bố
Đây là câu hỏi thông tin chi tiết, ta xác định từ khóa Tweenbot và tìm câu trả lời ở đoạn 1 dòng 4: Most
people would not even notice the ten-inch smiling robot, called a Tweenbot, rolling along the street.
Tạm dịch: Hầu hết mọi người thậm chí sẽ không chú ý đến một chú robot mỉm cười cao 10 inch, được gọi
là Tweenbot, lăn dọc trên đường.
→ Đáp án B.
Question 40. Đáp án D
Từ “congested” trong đoạn 1 có nghĩa gần nhất với__________.
A. đổ xô B. hẻo lánh
C. nhồi bông D. đông đúc
Câu hỏi về từ vựng, ta có thể chọn được đáp án nếu biết nghĩa hoặc dự đoán nghĩa dựa vào thông tin ở
câu chứa từ congested
Đoạn 1, dòng 1-2: Imagine the busy streets of New York City, an enormous place with millions of people.
Everyday, the streets are congested with people going about their daily lives.
Tạm dịch: Hãy tưởng tượng những con phố sầm uất của thành phố New York, một nơi rộng lớn với hàng
triệu người. Hàng ngày, đường phố tắc nghẽn với những người đang hối hả trong cuộc sống hàng ngày
của họ.
→ Chọn D, congested đồng nghĩa với crowded
Question 41. Đáp án B
Làm thế nào một Tweenbot đến được đích cuối cùng của nó?
A. Với sự giúp đỡ của Tweenbots khác
B. Với sự giúp đỡ của những người đi bộ tốt bụng trên đường phố

Trang 19
C. Với sự giúp đỡ của Kacie Kinzer
D. Với sự giúp đỡ của các robot khác ở thành phố New York
Câu hỏi tìm thông tin chi tiết, ta thấy đáp án nằm ở đâu đoạn 2 dòng 1-2. Kinzer’s idea was to make a
robot that could navigate the city and reach its destination only if it was aided by pedestrians.
Tạm dịch: Ý tưởng của Kinzer là chế tạo một robot có thể tìm đường trong thành phố và đến đích chỉ khi
nó được hỗ trợ bởi người đi bộ. → Chọn đáp án B
Question 42. Đáp án A
Phát biểu nào sau đây KHÔNG đúng?
A. Hầu hết Tweenbots đến đích của họ bị hư hại hoặc bị hỏng.
B. Hầu hết mọi người đối xử với Tweenbots một cách nhẹ nhàng.
C. Tweenbots không thể tự tìm đường trong thành phố.
D. Tweenbots di chuyển với tốc độ không đổi và chỉ có thể đi trên một đường thẳng.
Câu hỏi chứa từ phủ định NOT, đáp án là câu trần thuật có sai khác về ngữ nghĩa so với thông tin trong
bài.
A là đáp án do sai khác so với thông tin tìm ở câu gần cuối đoạn 3: Surprisingly, no Tweenbot was lost or
damaged, and each one arrived at its target in good condition. - Thật ngạc nhiên, không có Tweenbot nào
bị mất hoặc bị hư hại, và mỗi người đều đến đích của mình trong trạng thái tốt.
B có xuất hiện trong câu: Most people treated the robot in a gentle manner, and some even treated it as
though it were a small living being – Hầu hết mọi người đối xử với Tweenbots một cách nhẹ nhàng, và
thậm chí có người còn đối xử như thế nó là một sinh vật sống nhỏ bé.
C có trong câu: Kinzer’s idea was to make a robot that could navigate the city and reach its destination
only if it was aided by pedestrians - Ý tưởng của Kinzer là tạo ra một chú robot có thể tìm đường trong
thành phố và tới được đích đến chỉ khi có sự hỗ trợ của những người đi bộ. Như vậy có nghĩa robot này
không thể tự mình tìm đường được.
D tìm thấy trong câu: Tweenbots move at a constant speed and can only go in a straight line - Tweenbots
di chuyển với tốc độ không đổi và chỉ có thể đi trên một đường thẳng.
Như vậy B, C, D đều đã xuất hiện trong bài nên bị loại, ta lựa chọn A là chứa thông tin sai khác với bài.
READING 2
Đoạn 1: Giới thiệu về Trí Ihông Minh Cảm Xúc. Đó là việc hiểu được cảm xúc của bản thân. Những
người có khả năng quan sát cảm xúc của mình thì sẽ ít bị cảm xúc chi phối và cũng dễ dàng kiểm soát
được chúng hơn.
Đoạn 2: Quản lý cảm xúc không có nghĩa phải đàn áp chúng nhưng cũng không có nghĩa cho phép chúng
được thể hiện tùy tiện. Mỗi cảm xúc đểu có giá trị và tầm quan trọng riêng của nó. Quản lý cảm xúc là
việc chúng ta thể hiện nó ở một chừng mực hợp lý.
Đoạn 3: Một yếu tố quan trọng của việc nhận thức về cảm xúc là khả năng đưa ra lý giải về chúng. Chúng
ta càng lý giải được vì sao mình lại cảm thấy như vậy, chúng ta càng dễ kiểm soát cảm xúc.

Trang 20
Đoạn 4: Người có thể tự tạo động lực cho bản thân thường có khả năng kiểm soát cảm xúc tốt hơn, điều
đó giúp họ có thể tiến về phía trước và theo đuổi những mục tiêu có giá trị.
Đoạn 5: Khả năng trì hoãn sự hứng thú tạm thời để tiếp tục làm việc vì một mục tiêu lớn trong tương lai
dường như có liên hệ mật thiết nhất với thành công
Question 43. Đáp án D
Dịch câu hỏi: Nội dung nào dưới đây có thể suy ra từ đoạn 1?
Dịch đáp án.
A. Một vài người có thể hiểu về cảm xúc của mình tốt hơn người khác.
B. Người kiểm soát được cảm xúc của mình sẽ bị kiểm soát lại bởi chúng.
C. Nếu con người dành sự chú ý cho cảm xúc của mình, họ sẽ không thể quản lý được chúng.
D. Nếu con người dành sự chú ý cho cảm xúc, họ có thể kiểm soát chúng tốt hơn.
Đọc kĩ đoạn 1, ta sẽ thấy được thông tin đúng là câu D. Cụ thể hơn khi đọc kỹ ở hai câu cuối: People who
are able to monitor their feelings as they arise are less likely to be ruled by them and are thus better able
to manage their emotions.”
Tạm dịch: Người có thể quan sát cảm xúc của mình khi chúng xuất hiện thì sẽ ít bị chúng chi phối và do
đó sẽ dễ dàng để quản lý cảm xúc hơn.
Các phương án khác ý nghĩa không đúng. → Chọn D.
Question 44. Đáp án B
Dịch câu hỏi. Từ "encompasses" ở đoạn 1 có thể thay thế bằng từ __________
Dịch đáp án.
A. Duy trì B. Bao gồm
C. Giải trí D. Chia, phân chia
Giải thích. Đây là câu hỏi từ vựng nên có thể dựa vào vốn từ để giải quyết hoặc dựa vào những câu xung
quanh để đoán nghĩa “And this awareness encompasses not only moods but also thoughts about those
moods.”
Tạm dịch: Và sự nhận thức này bao gồm không chỉ tâm trạng mà còn cả những suy nghĩ về những tâm
trạng đó.
Vì sau từ encompasses, tác giả có nêu ra “moods” và “thoughts about those moods” nên ta có thể suy ra
nghĩa của nó là bao gồm, gồm có và chọn đáp án B-includes
Question 45. Đáp án D
Dịch câu hỏi. Từ "soothe" ở đoạn 2 có thể thay thể bằng từ ___________
Dịch đáp án.
A. Giảm thiểu
B. Làm yếu đi
C. Làm tệ hơn
D. Làm giảm đi/ giải tỏa

Trang 21
Câu hỏi từ vựng có thể dùng vốn từ để giải quyết hoặc dựa vào văn cảnh.
Emotions can also be managed by engaging in activities that cheer us up, soothe our hurts, or reassure us
when we feel anxious.
Ta đoán nghĩa của từ dựa vào cụm “soothe our hurts”. Do phía trước là cụm “cheer us up”-làm chúng ta
vui lên, mang nghĩa tích cực nên ta có thể suy đoán “soothe our hurts” cũng là cụm từ mang nghĩa tích
cực. Ở đây nghĩa là làm giảm đau buồn và chọn D.
Lưu ý: reduce cũng là giảm nhưng hay dùng trong tình huống như reduce price - giảm giá, reduce speed
- giảm vận tốc, không phù hợp với văn cảnh.
Question 46: Đáp án C
Dịch câu hỏi: Từ đoạn 2, chúng ta có thể thấy rằng Daniel Goleman___________
Dịch đáp án.
A. Đào tạo con người để làm tăng Trí Thông Minh Cảm Xúc của họ
B. Chăm sóc bệnh nhân có vấn đề về cảm xúc
C. Viết về Trí Thông Minh Cảm Xúc
D. Nghiên cứu về cách làm thế nào để con người quản lý cảm xúc
Thông tin nằm ở dòng 2, đoạn 2 “Psychologist Daniel Goleman, one of several authors who have
popularized the notion of Emotional Intelligence...”
Tạm dịch: Nhà tâm lý học Daniel Goleman, một trong bảy tác giả đã đưa ra định nghĩa về Trí Thông
Minh Cảm Xúc.
Vì Daniel Goleman là một trong bảy tác giả nên ông sẽ viết về Trí Thông Minh Cảm Xúc, ta chọn C.
Question 47. Đáp án D
Dịch câu hỏi: Tất cả các ý dưới đây đểu được nêu ở đoạn hai NGOẠI TRỪ
Dịch đáp án.
A. Mọi cảm xúc đều quan trọng
B. Chúng ta có thể quản lý cảm xúc của mình
C. Cảm xúc là một phần của cuộc sống thỏa mãn
D. Chúng ta nên bỏ qua một vài cảm xúc
Giải thích. Đây là câu hỏi dạng thông tin không xuất hiện. Do đó chỉ cần tìm thông tin của 3 phương án
xuất hiện trong bài Phương án A (có trong bài): Psychologist Daniel Goleman, one of several authors
who have popularized the notion of Emotional Intelligence, insisted that the goal is balance and that
every feeling has value and significance, - ... mỗi cảm xúc đều có giá trị và tầm quan trọng riêng.
Phương án B (có trong bài): Có nhiều thông tin trong đoạn văn hỗ trợ cho luận điểm rằng chúng ta có thể
kiểm soát cảm xúc của mình. Có thể ví dụ ở câu ở đoạn 2:“Thus, we manage our emotions by expressing
them in an appropriate manner” - do đó, chúng ta có thể quản lý cảm xúc của mình bằng cách thể hiện
chúng một cách hợp lý.

Trang 22
Phương án C (có trong bài): A life without passion would be a dull wasteland of neutrality, cut off and
isolated from the richness of life itself - Cuộc sống không đam mê sẽ rất đỗi phí phạm của sự trung lập, bị
tách rời, bị cô lập khỏi sự giàu đẹp của cuộc sống.
Đam mê cũng là một cảm xúc, do đó nếu không có cảm xúc thì cuộc sống sẽ rất nhàm chán và vô vị hay
cảm xúc là một phần của cuộc sống thỏa mãn.
A, B, C đều đã xuất hiện trong bài nên D là đáp án cần chọn.
Question 48. Đáp án D
Dịch câu hỏi. Từ "them" ở đoạn 3 chỉ____________
Dịch đáp án.
A. Những nhà tâm lý học
B. Những cá nhân
C. Những cá nhân khác biệt
D. Những cảm xúc mãnh liệt
Câu hỏi tìm đại từ thay thế cho danh từ xuất hiện ở đằng trước, ta cần đọc các câu xung quanh để đưa ra
câu trả lời chính xác. “Psychologists have found that, among individuals who experience intense
emotions, individual differences in the ability to assign meaning to those feelings predict differences in
the ability to manage them.” - Các nhà tâm lý học tìm ra rằng, giữa những cá nhân với những cảm xúc
mãnh liệt, những điểm khác nhau trong khả năng lý giải những cảm xúc đó cho thấy sự khác nhau trong
việc quản lý chúng.”
Đọc qua câu dịch ta sẽ thấy từ “them” thay thế cho “intense emotions” → Đáp án D.
Question 49. Đáp án C
Dịch câu hỏi: Ở đoạn 3, tác giả giải thích vấn đề nhận thức và quản lý cảm xúc bằng cách ___________
Dịch đáp án.
A. Miêu tả cách con người học để kiểm soát cảm xúc
B. Đưa ra ví dụ về việc tại sao con người lại nổi giận
C. So sánh cách mà hai người phản hồi lại những cảm xúc mãnh liệt
D. Giải thích vì sao con người lại không nhận thức được cảm xúc của họ
Ở đoạn 3, tác giả nêu ra luận điểm của mình về “awareness and management of emotions”- nhận thức và
quản lý cảm xúc bằng cách so sánh hai người phản hồi lại những cảm xúc mãnh liệt “In other words, if
two individuals are intensely angry, the one who is better able to understand why he or she is angry will
also be better able to manage the anger”- Nói cách khác, nếu hai cá nhân đang rất giận dữ, người hiểu
rõ lý do tại sao mình tức giận sẽ dễ kiểm soát cảm xúc hơn.
→ Chọn đáp án C
Question 50. Đáp án D
Dịch câu hỏi: Theo như đoạn 5, những đứa trẻ có thể sẽ thành công hơn ở trường nếu chúng có thể chống
cự lại mong muốn tạm thời vì chúng có thể___________

Trang 23
Dịch đáp án.
A. Dễ dàng hiểu được thông tin mới
B. Trở nên nổi bật hơn với giáo viên
C. Có nhiều bạn hơn khi ở trường
D. Tập trung vào công việc và không bị làm xao lãng
Chúng ta có thể tìm thấy đáp án ở đoạn cuối bài. One researcher examined whether this trait can predict a
child's success in school. The study showed that 4-year-old children who can delay instant gratification in
order to advance toward some future goal will be "far superior as students" when they graduate from high
school than will 4-year-old who are not able to resist the impulse to satisfy their immediate wishes.
Một nhà nghiên cứu đã làm thử nghiệm xem rằng đặc điểm này có dự đoán được thành công của những
đứa trẻ ở trường hay không. Nghiên cứu chỉ ra rằng những đứa trẻ 4 tuổi mà có thể trì hoãn được những
ham muốn ngay tức thì để tập trung cho những mục tiêu ở phía trước thì sau này khi tốt nghiệp sẽ thành
những sinh viên xuất sắc hơn là những đứa trẻ 4 tuổi mà không có khả năng kiềm chế những bốc đồng
làm thỏa mãn cho những nhu cầu ngay lập tức.
Đọc phần dịch dẫn chứng ta có thể thấy chúng tương đồng với đáp án D - những đứa trẻ biết cách tập
trung và không bị xao lãng.
Chú ý cách sử dụng từ đồng nghĩa trong câu hỏi và thông tin trong bài:
be more successful in school= success in school not get distracted = delay instant gratification focus on
their work = ...in order to advance toward some future goal.

Trang 24
ĐỀ SỐ 06 ĐỀ THI THỬ TỐT NGHIỆP THPT
NĂM HỌC: 2020 – 2021
MÔN: TIẾNG ANH
Thời gian làm bài: 60 phút; không kể thời gian phát đề

Mark the letter A, B, C or D on you answer sheet to indicate the word whose underlined part
differs from the other three in pronunciation in each of the following questions.
Question 1. A. heritage B. package C. passage D. teenage
Question 2. A. clothes B. bosses C. boxes D. couches
Mark the letter A, B, C, or D on your answer sheet to indicate the word that differs from the other
three in the position of the primary stress in each of the following questions.
Question 3. A. desert B. dessert C. centre D. circle
Question 4. A. medieval B. managerial C. mediocre D. magnificent
Mark the letter A, B, C, or D on your answer sheet to indicate the correct answer to each of the
following questions.
Question 5. Due to the economic unrest, people___________happy lately.
A. hadn’t been B. aren’t C. weren’t D. haven’t been
Question 6. Television can make things memorable for the reason that it presents
information___________an effective way.
A. over B. with C. in D. on
Question 7. ___________how confident you are, it is almost impossible not to be a little nervous
before an important exam.
A. No matter B. Eventhough C. Not only D. whereas
Question 8. The Principal usually has his pupils___________waste paper for their mini-project.
A. collect B. having collected C. collected D. to collect
Question 9. Neither of the boys came to school yesterday, ___________?
A. didn’t they B. does they C. did they D. doesn’t they
Question 10. Anna is holding her shopping bag with one hand and turning the door handle with .
A. another B. the other C. other D. others
Question 11. ___________that Columbus discovered America.
A. It was in 1492 B. There was in 1492
C. In 1492 D. That was in 1492
Question 12. ___________be needed, the river basin would need to be dammed.
A. Hydroelectric power should B. When hydroelectric power
C. Hydroelectric power D. Should hydroelectric power
Question 13. Most office furniture is bought more on the basis of___________than comfort.

Trang 1
A. afford B. affording C. afforded D. affordability
Question 14. Although our opinions on many things___________, we still maintain a good relationship
with each other.
A. differ B. receive C. maintain D. separate
Question 15. The pointless war between the two countries left thousands of people dead and
seriously___________.
A. injured B. wounded C. spoilt D. damaged
Question 16. He enjoyed the dessert so much that he accepted the second___________when it was
offered.
A. sharing B. helping C. pile D. load
Question 17. I’ve just been offered a new job! Things are___________.
A. clearing up B. making up C. looking up D. turning up
Question 18. Ihe children had to___________in the principal’s office after they took part in a fight.
A. hit the right notes B. beat around the bush
C. play second fiddle D. face the music
Mark the letter A, B, C, or D on your answer sheet to indicate the underlined part that needs
correction in each of the following questions.
Question 19. Mobility is one of the characteristics often demanded of executives, and they must
accustom themselves to move quite regularly.
A. Mobility B. characteristics C. of D. to move
Question 20. Every city in the United States has traffic problems because the amount of cars on
American streets and highways is increasing every year.
A. in the United States B. traffic
C. amount D. on American streets
Question 21. Parents’ choices for their children’s names is based on names of their relatives or
ancestors.
A. choice B. their C. is based D. relatives
Mark the letter A, B, C or D on your answer sheet to indicate the sentence that is closest in
meaning to each of the following questions.
Question 22. More petrol is consumed nowadays than ten years ago.
A. Not so much petrol was consumed ten years ago as nowadays.
B. Petrol consumption is going down nowadays.
C. We had more petrol ten years ago than we do nowadays.
D. We should consume as much petrol as possible.
Question 23. The problem was so complicated that the students couldn’t understand it.
A. It was such complicated problem that the students couldn’t understand.

Trang 2
B. The problem wasn’t simple enough for the students to understand it.
C. The problem was too complicated for the students to understand.
D. It was so complicated problems that the students couldn’t understand
Question 24. My American friend finds it difficult to pick up food with chopsticks.
A. My American friend doesn’t feel like picking up food with chopsticks.
B. My American friend can’t find chopsticks to pick up food.
C. My American friend didn’t used to pick up food with chopsticks.
D. My American friend is not used to picking up food with chopsticks.
Mark the letter A, B, C, or D on your answer sheet to indicate the sentence that a best combines
each pair of sentences in the following questions.
Question 25. It was an interesting novel. I stayed up all night to finish it.
A. I stayed up all night to finish the novel so it was interesting.
B. Unless it were an interesting novel, I would not stay up all night to finish it.
C. Though it was an interesting novel, I stayed up all night to finish it.
D. So interesting was the novel that I stayed up all night to finish it.
Question 26. She fell over in the last minutes. She didn't win the race.
A. If she didn't fall over in the last minutes, she would win the race.
B. She didn't win the race even though she fell over in the last minutes
C. Not having won the race, she fell over in the last minutes
D. She could have won the race if she hadn't fallen over in the last minutes.
Mark the letter A, B, C, or D on your answer sheet to indicate the most suitable response to
complete each of the following questions.
Question 27. Jenny: “Wow! what a nice coat you are wearing!”
Peter: “___________”
A. Thanks. My mother bought it for me. B. Certainly. Do you like it, too?
C. I like you to say that. D. Yes, of course. It’s expensive.
Question 28. Stranger. “Excuse me! Can you show me the way to Main Street, please?”
Man. “___________”
A. Continue. B. It’s easy to do it.
C. Um, I am sorry I have no idea. D. Am I going right?
Mark the letter A, B, C, or D on your answer sheet to indicate the word(s) CLOSEST in meaning
to the underlined word(s) in each of the following questions.
Question 29. By being thrifty and shopping wisely, housewives in the city can feed an entire family on
as little as 500.000 VND a week.
A. luxurious B. economical C. sensible D. miserable
Question 30. Gerry didn’t go on the expedition - he made up that part of the story.

Trang 3
A. invented B. narrated C. unfolded D. recounted
Mark the letter A, B, C, or D on your answer sheet to indicate the word(s) OPPOSITE in meaning
to the underlined word(s) in each of the following questions.
Question 31. About 95 percent of all animals are invertebrates which can live anywhere, but most,
like the starfish and crabs, live in the ocean.
A. with backbones B. with ribs C. without ribs D. without backbones
Question 32. If you say bad things about the person who give a job you bite the hand that feeds you.
A. be unfriendly B. be ungrateful C. be thankful D. be devoted
Read the following passage and mark the letter A, B, C, or D on your answer sheet to indicate the
correct word or phrase that best fits each of the numbered blanks.
(33) ___________in technology have made a lot of changes to our everyday lifestyles, but one of the
biggest has got to be how we read books. Since the invention of the e-book, there has been a significant
change to our reading habits. Given the choice between taking a couple of heavy paperbacks on holiday
or an e-book device like a Kindle, most of us, including our parents and grandparents, would
unsurprisingly opt (34) ___________the Kindle.
But what would our lives be like with no books at all? It’s a (35) ___________question. Some
educational specialists are making predictions that in the future we wont even see books in classrooms -
everything will be done online! (36) ___________of the idea of getting rid of books say that there will
always be a need for paper-based versions of materials. However, to be realistic, we have to accept that
there is a (37) ___________chance that in a decade’s time schools and classrooms will be book-free!
what do you think of that?
Question 33. A. Progression B. Successes C. Increases D. Advances
Question 34. A. of B. on C. for D. at
Question 35. A. special B. naughty C. funny D. tricky
Question 36. A. Alternatives B. Contestants C. Opponents D. Enemies
Question 37. A. remote B. far C. long D. distant
Read the following passage and mark the letter A, B, C, or D on your answer sheet to indicate the
correct answer to each of the questions.
Bitcoins are a form of virtual currency. In other words, they are a type of money that does not exist in
the actual world. However, they can be used to purchase actual products and services from real
companies.
The bitcoin system was created in 2009 by an enigmatic person named Satoshi Nakamoto. In fact, no
one is sure if Satoshi Nakamoto is an actual person or a group of people. Bitcoins are designed to serve
as an alternative to national currencies, such as dollars and euros. They can be used to pay for things
instead of cash or credit cards, when bitcoins are transferred from a buyer to a seller, the transaction is
recorded in a public database.

Trang 4
Governments are concerned that bitcoins can easily be stolen by hackers. It has dawned on them that
they might be used for illegal purposes. For example, stolen goods could be purchased without the
government’s knowledge. Although more and more companies are beginning to accept bitcoins, the
percentage of purchases made using bitcoins is minuscule compared to other online payment methods,
such as credit cards. Instead, many bitcoin owners simply keep them as an investment since more
valuable in the future. This may or may not be a wise approach. Currently, the value of bitcoins is
fluctuating wildly, especially when compared to highly stable national currencies, Bitcoin Investors are
gambling on the hope that as this high-tech money becomes more widely accepted, its value will soar.
(Adaptedfrom, https.//www.digitalcommerce360.com/)
Question 38. What is the passage mainly about?
A. A new kind of currency in the virtual world
B. A way of doing business in the virtual world
C. An alternative to bitcoins created by Nakamoto
D. The future of bitcoins in the real world.
Question 39. The word “they” in paragraph 1 refers to___________
A. dollars B. euros C. things D. bitcoins
Question 40. The word “minuscule” is closest in meaning to___________
A. considerable B. small C. minimal D. increasing
Question 41. why are bitcoins of great concern to governments?
A. Because the value of bitcoins is fluctuating wildly.
B. Because bitcoins will eventually replace national currencies.
C. Because bitcoins may be used in illegal transactions.
D. Because most of bitcoin owners are hackers.
Question 42. Which of the following is defined in the passage?
A. Bitcoins B. Transactions C. Credit cards D. Public Database
Read the following passage and mark the letter A, B, C or D on your answer sheet to indicate the
correct answer to each of the following questions.
WHY DON’T YOU GET A PROPER JOB?
She wants to be a singer; you think she should go for a long-term career with job security and
eventually retire with a good pension. But a new report suggests that in fact she’s the practical one.
Why do parents make terrible careers advisers?
Today’s 14 and 15-year-olds are ambitious. They are optimistic about their prospects, but their career
ideas are rather vague. Although 80% of them have no intention of following in their parents’ footsteps,
69% still turn to their parents for advice. They look at their working future in a different way to their
parents.

Trang 5
A job for life is not in their vocabulary; neither is a dead-end but secure job that is boring but pays the
bills. Almost half the boys surveyed expected that their hobbies would lead them into the right sort of
job, while most girls seemed determined to avoid traditionally female careers such as nursing.
In the past, this might have counted as bad news. Certainly when I was 15, my guidance counsellors
were horrified at my plans to become a writer. I’m glad I didn’t change my plans to suit them. Even so,
their faith in rigid career paths was well-founded. In those days, that was the way to get ahead.
But the world has changed. The global economy is not kind to yesterday’s diligent and dependable
worker. The future belongs to quick-thinking people who are resourceful, ambitious and can take the
initiative. This means that a 14-year-old who sees her working future as a kind of adventure, to be made
up as she goes along is not necessarily being unrealistic.
However, she has to have the training and guidance to help her develop the right skills for today’s
market; not the rigid preparation for a workplace that disappeared twenty years ago.
Many young people are very aware of the pitfalls of the flexible workplace; they understand that
redundancy, downsizing and freelancing are all part of modern working life, but no one is telling them
how they might be able to turn the new rules of the employment game to their advantage. This is what
they need to know if they are to make a life for themselves.
So what is to be done? A good first step would be to change the way in which schools prepare young
people for adult life. The education system is becoming less flexible and more obsessed with traditional
skills at just the time that the employment market is going in the opposite direction.
Accurate, up-to-date information on new jobs and qualifications can help guidance counsellors to help
their students. Young people need solid information on the sort of training they need to pursue the
career of their dreams. Also, a little bit of encouragement can go a long way. If nothing else, a bit of
optimism from an adult can serve as an antidote to the constant criticism of teenagers in the press.
What, then, can we as parents do to help them? The best thing is to forget all the advice that your
parents gave you, and step into your teenagers shoes. Once you’ve done that, it’s easier to see how
important it is that they learn how to be independent, resourceful and resilient. Give them the courage
to follow their dreams -however odd they might sound right now. In a world that offers economic
security to almost no one, imagination is a terrible thing to waste.
Question 43. What is the writer’s attitude to the changing job market?
A. It is a challenge that must be faced.
B. It had made too many people unemployed.
C. It is something that young people are afraid of.
D. It has had a negative effect on education.
Question 44. How does the writer think the global economy has affected the employment market?
A. Workers have to be willing to change jobs.
B. Workers are unlikely to receive a pension.

Trang 6
C. It has made workers less dependable.
D. It has made work more adventurous.
Question 45. The writer uses the phrase “aware of the pitfalls” to show that young
people___________
A. feel that modern jobs are too flexible B. know about the problems of modern jobs
C. don’t think they get enough training D. accept that they will be made redundant
Question 46. What kind of employment would teenagers like to have?
A. A job similar to their parents. B. A job that gives them fulfillment.
C. A job that can also be a hobby. D. A job with economic security.
Question 47. The writer feels that most parents___________
A. give their children good career advice
B. do not tend to be particularly ambitious
C. have very traditional views about work
D. have realistic goals for their children
Question 48. How can parents help their children?
A. By trying to think the way they do B. By learning to be courageous
C. By ignoring advice given by others D. By becoming more independent
Question 49. what does the writer believe about her guidance counsellors?
A. That they should have treated her better. B. That the advice they gave was wrong
C. That they were in some ways right. D. That they had tried to ruin her career.
Question 50. What does the writer feel will happen if the education system does not change?
A. Young people will be discouraged from working.
B. Young people will receive mover criticism in the press,
C. Young people will be unable to fulfill their potential.
D. Young people will not be optimistic about their future.
Đáp án
1-D 2-A 3-B 4-D 5-D 6-C 7-A 8-A 9-C 10-B
11-A 12-D 13-D 14-A 15-B 16-B 17-C 18-D 19-D 20-C
21-C 22-A 23-C 24-D 25-D 26-D 27-A 28-C 29-B 30-A
31-A 32-C 33-D 34-C 35-D 36-C 37-A 38-A 39-D 40-B
41-C 42-A 43-A 44-D 45-B 46-B 47-B 48-A 49-C 50-B

LỜI GIẢI CHI TIẾT


Question 1: Đáp án D
Đáp án D đúng vì phần gạch chân của đáp án D được đọc là /eɪdʒ/. Các phương án còn lại phần gạch chân
được đọc là /ɪdʒ/.

Trang 7
A. heritage /ˈherɪtɪdʒ/ (n): di sản, gia tài
B. package /ˈpækɪdʒ/ (n)/(v): bưu kiện, gói hàng (n) / đóng gói (v)
C. passage /ˈpæsɪdʒ/ (n): hành lang, đoạn văn, sự thông qua một đạo luật
D. teenage /ˈtiːneɪdʒ/ (adj): ở độ tuổi thanh thiếu niên
Question 2. Đáp án A
Đáp án A đúng vì phân gạch chân của đáp án A được đọc là âm /z/. Các phương án còn lại phần gạch
chân được đọc là /iz/.
A. clothes /kləʊðz/ (pl.n): quần áo
B. bosses /bɒsiz/ (n)/(v)/(adj): ông chủ (n) / chỉ huy (v) / giỏi, xuất sắc (adj)
C. boxes /bɒksiz/ (n)Z(v): cái hộp (n) / bỏ vào hộp (v)
D. couches /kaʊtʃiz/ (n)/(v): ghế dài (n) / nằm dài ra, diễn tả (v)
* Note:
Chúng ta cần nhớ lại cách phát âm đuôi -s/es:
-s/-es được phát âm là: Khi tận cùng của từ là: Ví dụ
/s/ Các phụ âm vô thanh: /k/, /e/, /θ/, stops, laughs, accepts, months
/p/, /t/
llll Các âm: /s/, /z/, /ʃ/, /ʒ/,/tʃ/,/ /dʒ/ kisses, washes, matches, changes,
buzzes, bosses, boxes, couches
(chính là 3 phương án trong đề
bài)
/z/ Các âm hữu thanh còn lại styles, intends, orphans

Question 3. Đáp án B
Đáp án B đúng vì đáp án B có trọng âm rơi vào âm thứ hai. Các phương án còn lại có trọng âm rơi vào
âm đầu tiên.
A. desert /ˈdezət/ (n)/(v): sa mạc (n) / bỏ đi(v)
B. dessert /dɪˈzɜːt/ (n): món tráng miệng
C. centre /ˈsentə(r)/ (n): trung tâm
D. circle /ˈsɜːkl/ (n)/(v): vòng tròn (n) / đi vòng quanh, bao vây (v)
Question 4. Đáp án D
Đáp án D đúng vì đáp án D có trọng âm rơi vào âm thứ hai. Các phương án còn lại có trọng âm rơi vào
âm thứ 3.
A. medieval /ˌmediˈiːvl/ (adj): thời trung cổ
B. managerial /ˌmænəˈdʒɪəriəl/ (adj): thuộc về quản lý
C. mediocre /ˌmiːdiˈəʊkə(r)/ (adj): tầm thường
D. magnificent /mæɡˈnɪfɪsnt/ (adj): tráng lệ, lộng lẫy
Question 5. Đáp án D
Nhìn vào 4 phương án ta thấy đây là câu hỏi về thì động từ.

Trang 8
Trong câu xuất hiện trạng từ “lately - vừa mới, gần đây” đây là trạng từ của thì hiện tại hoàn thành nên ta
dễ dàng chọn ngay được đáp án D.
Tạm dịch: Do tình trạng bất ổn về kinh tế nên mọi người gần đây không vui vẻ cho lắm.
* Mở rộng: Trong câu trên ta học được kiến thức sau:
Unrest (n): sự bất ổn
Due to/ Because of / Owing to/ On account of....+ noun/ pronoun/V-ing, S + V....: Bởi vì...
Due to/ Because of / Owing to/ On account of ....là các cụm giới từ chỉ nguyên nhân.
Question 6. Đáp án C
Đây là câu hỏi về giới từ.
Khi muốn nói theo một cách nào đó ta dùng cụm giới từ in ....way. Cụ thể trong câu này là theo một cách
hiệu quả. Các phương án còn lại không dùng với nghĩa này nên đáp án là C.
Tạm dịch: Truyền hình có thể khiến mọi thứ trở nên đáng nhớ vì nó trình bày thông tin theo một cách
hiệu quả.
Question 7.
Đây là câu hỏi về cấu trúc câu diễn đạt nghĩa tương phản.
Ta thấy mệnh đề phía sau có impossible not to be a little nervous - không thể không có chút lo lắng (ngụ ý
là lo lắng) và mệnh đề phía trước có confident - tự tin → hai mệnh đề mang nghĩa tương phản. Như vậy,
trong đầu phải hình dung ra tất cả các cấu trúc liên quan đến mệnh đề chỉ sự tương phản sau:
S + V… + in spite of/ despite + noun/pronoun/ V-ing
Although/ though/ even though + S + V..., S + V...
No matter + what/who/when/where/why + S + V
No matter + how + adj/adv + S + V
Whatever (+ noun)/ whoever/ wherever/ whenever + S + V
However + adj/adv + S + V
Nhìn lên câu đề bài ta thấy mệnh đề phía trước đã có: ___________how confident you are, đối chiếu với
các cấu trúc ở trên ta tìm ra được cấu trúc tương ứng là No matter + how + adj/adv + S + V → Chỗ trống
cần điền từ No matter.
Tạm dịch: Cho dù bạn có tự tin đến mấy thì cũng không thể không có chút lo lắng trước một kỳ thi quan
trọng như này.
Question 8. Đáp án A
Câu hỏi kiểm tra về thể sai khiến.
Nhìn vào câu hỏi ta nhận thấy có: has his pupils → nghĩ ngay đến thể sai khiến với động từ have. Ngoài
từ have ta còn dùng thể sai khiến với động từ get. Ta có các cấu trúc sau:
Chủ động: S + have + O(person) + V + O
S + get + O(person) + to V + O
Bị động: S + have/ get + O (thing) + pp (+ by + O)

Trang 9
Trong câu hỏi có has his pupils → have + O (person) → chỗ trống cần một động từ ở dạng nguyên thể nên
đáp án đúng là A.
Tạm dịch: Hiệu trưởng yêu cầu học sinh thu lượm giấy đã dùng rồi cho dự án nhỏ của họ.
Question 9.
Kiến thức được hỏi - câu hỏi đuôi.
Ta có công thức của câu hỏi đuôi:
S + V + O..., TRỢ ĐỘNG TỪ + ĐẠI TỪ?
Trong đó:
- Ở phần câu hỏi đuôi ta dùng trợ động từ tương ứng với thì của câu và đại từ phía sau.
- Lựa chọn đại từ cho đúng và phù hợp với chủ ngữ ở trước.
- Mệnh đề đã cho và câu hỏi đuôi luôn lệch nhau về khẳng định- phủ định.
Đại từ - Lấy chủ từ của mệnh đề chính (mệnh đề phía trước) đổi thành đại từ.
Nam → he
Nữ → she
Vật (số ít) → it
Số nhiều (người và vật) → they
There → there
This/ That → it
These/ Those → they
Chủ ngữ của mệnh đề chính là “the boys” nên đại từ ở câu hỏi đuôi phải chuyển thành “they”
Động từ “came” được chia ở thì quá khứ đơn thì trợ động từ ở câu hỏi đuôi là “did” → loại B và D
Tuy nhiên trong mệnh đề chính không xuất hiện từ not mang nghĩa phủ định nhưng có một từ mang nghĩa
phủ định đó là Neither (không cái nào/người nào trong hai) → trợ động từ câu hỏi đuôi phải ở dạng
khẳng định → loại A và chọn được C.
* Note:
Với những câu hỏi về câu hỏi đuôi nếu mệnh đề chính không có NOT nhưng xuất hiện các từ mang nghĩa
phủ định như never, little, seldom, neither, hardly, scarely,... thì trợ động từ câu hỏi đuôi ở dạng khẳng
định.
Tạm dịch: Không một ai trong hai cậu bé này đến trường vào ngày hôm qua có phải không?
Question 10.
Kiến thức được hỏi - câu hỏi về danh từ số ít/ nhiều và mạo từ
Sau giới từ with cần danh từ/đại từ → Xét 4 phương án thì có other là danh từ ; mặt khác ta thấy ngữ cảnh
của câu là Anna cầm túi mua sắm bằng một tay mà người bình thường thì có hai tay, suy ra tay còn lại
được xác định rồi → the other.
Tạm dịch: Anna đàng cầm túi mua sắm bằng một tay và tay còn lại thì xoay nắm cửa.
* Mở rộng:

Trang 10
Chúng ta phân biệt 4 từ another/ the other/ other/ others. Trong đó:
- Another vừa là định từ vừa là đại từ, đi với danh từ đếm được số ít hoặc chỉ đứng một mình, mang nghĩa
“một cái khác”. Ngoài ra another còn đi với danh từ đếm được số nhiều đi cùng với một con số.
Ex: I’ll need another three days to finish the work
She’s borrowed another $20
- Other thì đằng sau là danh từ đếm được số nhiều, mang nghĩa “những cái khác”
- Others đứng một mình, nghĩa là “những người/cái gì khác”
- The other, cái còn lại trong hai
Question 11. Đáp án A
Thực chất câu chẻ là một trong những cách chen thêm cấu trúc vào trong câu (chẻ câu ra) để nhấn mạnh
một đối tượng nào đó.
Khi chúng ta muốn nhấn mạnh trạng ngữ chỉ thời gian/ nơi chốn ta sử dụng cấu trúc sau:
It is/ was + cụm trạng ngữ (chỉ thời gian/ nơi chốn gồm giới từ + từ chỉ thời gian/ nơi trốn) that S+V
Chúng ta thấy khi muốn nhấn mạnh thời gian có giới từ đi cùng thì chúng ta sẽ đem nguyên cả cụm giới
từ + từ chỉ thời gian lên - đây cũng là điểm khác biệt đối với mệnh đề quan hệ.
Trong trường hợp này ta thấy đáp án B và D sai vì phương án B và D dùng sai cấu trúc câu chẻ ở chỗ
There và That.
Phương án C sai ngữ pháp, không thể dùng trạng từ chỉ thời gian đứng trước mệnh đề That như này được.
Như vậy ta chọn được đáp án A.
Tạm dịch: Chính là vào năm 1492 Columbus đã khám phá ra Mỹ.
Question 12.
Kiến thức được hỏi - Đảo ngữ của câu điều kiện loại I
Phương án A sai vì nếu ghép phương án A vào chỗ trống thì xuất hiện hai câu (đã có đầy đủ S + V), mà
hai câu không thể được nối với nhau bằng dấu phẩy → sai về quy định ngữ pháp Tiếng Anh.
Phương án B dùng từ nối when nhưng vẫn sai vì nếu ghép phương án B vào chỗ trống thì tự nhiên xuất
hiện be needed (be không được chia theo thì) sau chủ ngữ hydroelectric power → sai ngữ pháp
Phương án C cũng sai vì nêu ghép C vào chỗ trống thì be không được chia theo thì và nếu nối 2 mệnh đề
thì phải cần từ nối → sai ngữ pháp
Đáp án đúng là D, đảo ngữ của câu điều kiện loại I ta đưa Should lên đầu
Tạm dịch: Nếu thủy điện là cần thiết thì lưu vực sông cần phải xả nước ra.
* Mở rộng:
Đảo ngữ với câu điều kiện loại I, II, III như sau:
Loại I: Should + S + V +
Loại II: Were + S + to V +
Loại III: Had + S + P2
Question 13. Đáp án D

Trang 11
Kiến thức được hỏi - loại từ
Ta nhận thấy sau giới từ of cần Noun / V-ing → loại A và C là động từ
Phương án B là động từ dạng V-ing thường dịch là “việc...” cụ thể trong câu này là “việc có đủ khả
năng”. Và thường sau động từ afford + to V: có khả năng để làm gì. Mặt khác, nếu chọn affording thì sẽ
bị tối nghĩa, không biết khả năng để làm gì → loại B
Vậy chỉ còn D là đáp án đúng vì affordability là danh từ mang nghĩa “khả năng chi trả”
Tạm dịch: Hầu hết các đồ nội thất trong văn phòng được mua vì khả năng chi trả hơn là sự thoải mái.
* Mở rộng: Câu trên ta học được cụm từ: on the basis of sth - bởi vì (tương đương nghĩa với because of
owing to, due to, on account of)
Question 14. Đáp án A
Kiến thức được hỏi - lựa chọn từ
Ta thấy receive, maintain và separate là ngoại động từ, nghĩa là phải có tân ngữ ở phía sau. Nếu không có
tân ngữ phía sau thì câu đó phải ở dạng bị động. Nhưng câu này đằng sau chỗ trống không có tân ngữ và
receive, maintain và separate cũng không được chia ở dạng bị động nên loại B, C và D → Đáp án đúng là
A - nội động từ.
Differ (v): khác, bất đồng
Receive (v): nhận
Maintain (v): duy trì
Separate (v): tách ra
Tạm dịch: Mặc dù quan điểm của chúng tôi về mọi thứ không giống nhau nhưng chúng tôi vẫn duy trì
một mối quan hệ tốt.
* Note: separate còn là nội động từ nhưng nếu là nội động từ thì cũng không hợp nghĩa trong câu hỏi
này.
Question 15.
Kiến thức được hỏi - lựa chọn từ
Ta nhận thấy trong câu hỏi có cấu trúc leave + O + adj/pp (để cho ai đó trong tình trạng như nào). Như
vậy sau tân ngữ phải là một : tính từ hoặc phân từ hai. Cả 4 phương án đều có dạng thức tính từ/phân từ
hai nhưng có sự khác biệt về cách sử dụng như sau:
Injured (adj): bị thương (về mặt thể chất thường là do tai nạn, đánh nhau, bị tấn công, thường dùng cho
người).
Wounded (adj): bị thương (thường là vết thương bị cắt, rạch, thủng trên da thịt do các loại vũ khí như
súng, dao, gươm, thủy tinh; thường dùng cho người).
Spoilt (adj): hư hỏng (thường dùng cho một đứa trẻ hư hỏng)
Damaged (pp): làm bị thương, cái gì bị hỏng hóc (thường dùng cho vật). Ngoài ra damage còn mang
nghĩa tổn hại những thứ trừu tượng như danh tiếng, uy tín, nền kinh tế.

Trang 12
Như vậy đáp án đúng là B vì trong câu đề cập đến chiến tranh mà chiến tranh thì thường có các loại vũ
khí như súng, dao.
Tạm dịch: Cuộc chiến tranh vô nghĩa giữa 2 đất nước này để lại cái chết của hàng nghìn người và nhiều
người bị thương nghiêm trọng.
Question 16. Đáp án B
Kiến thức được hỏi - lựa chọn từ
Trong câu hỏi có đề cập đến dessert - món tráng miệng → liên quan đến việc ăn uống.
Trong 4 phương án chỉ có helping – phần ăn thêm liên quan đến ăn uống → chọn B
Sharing (V-ing): việc chia sẻ
Pile (n): thường đi với cụm a pile of sth/piles of sth - một/nhiều chồng gì đó
Load (n)/(v): vật mang nặng (n) / chở, tải (v)
Tạm dịch: Anh ấy thích món tráng miệng này đến nỗi phải gọi thêm phần nữa.
Question 17. Đáp án C
Kiến thức được hỏi - Phrasal verb
Clear up (ph.v): dọn dẹp, làm sáng tỏ / trở nên tốt hơn (thời tiết, ốm đau)
Make up (ph.v): trang điểm, bịa chuyện, chiếm
Look up (ph.v): điều gì đó trở nên tốt hơn
Turn up (ph.v): đến, xuất hiện
Dựa vào nghĩa ta chọn được C.
Tạm dịch: Tôi vừa mới được nhận một công việc mới rồi! Mọi thứ đang trở nên tốt đẹp hơn
Question 18. Đáp án D
Kiến thức được hỏi - Thành ngữ
Hit the right note: đi đúng hướng
Beat around the bush: vòng vo tam quốc, không nói trực tiếp chuyện muốn nói đến.
Play second fiddle: đóng vai phụ, ở thế yếu hơn
Face the music: chấp nhận sự trừng phạt từ việc làm của mình
Dựa vào nghĩa chọn được đáp án D
Tạm dịch: Những đứa trẻ bị chịu phạt trong văn phòng của thầy hiệu trưởng sau khi đã tham gia vào một
vụ đánh nhau.
Question 19.
Ta có phrasal verb with accustom: accustom yourself/sb to sth/V-ing: tự làm quen vôi việc gì
Đối chiếu với phrasal verb trên thì sau themselves động từ phải ở dạng V-ing
Sửa: to move → moving
Tạm dịch: Di chuyển là một trong những đặc điểm được yêu cầu với các giám đốc điều hành, và họ phải
tự mình làm quen với việc di chuyển khá thường xuyên.
Question 20.

Trang 13
Danh từ cars là danh từ đếm được số nhiều nên không thể dùng lượng từ amount. Amount được sử dụng
với danh từ không đếm được. Ta phải sử dụng một lượng từ đi với danh từ đếm được số nhiều. Sửa:
amount → number
Tạm dịch: Mỗi thành phố ở Mỹ đều gặp vấn đề về giao thông bởi vì số lượng xe hơi trên đường phố và
trên cao tốc tăng dần theo năm.
* Mở rộng:
Ta có một số cấu trúc phổ biến cần lưu ý liên quan đến lượng từ:
A number of N(s) + V(plu): một vài # The number ofN(s) + v(sing): số lượng những cái gì
An amount of N(uncount) + V(sing): một vài
The amount of N(uncount) + V(sing): số lượng những cái gì
Question 21.
Ta thấy danh từ choices mới làm chủ ngữ chính trong câu hỏi. Mà choices ở số nhiều nên động từ
phải hòa họp với chủ ngữ, nghĩa là động từ được chia ở số nhiều. Động từ trong câu là is based cần được
sửa thành are based.
Tạm dịch: Sự lựa chọn tên của cha mẹ cho con cái được dựa trên tên của tổ tiên và họ hàng.
Question 22. Đáp án A
Dịch đề bài: Nhiều xăng hơn được tiêu thụ hiện nay hơn là 10 năm về trước, (ngụ ý: việc tiêu thụ xăng
của 10 năm về trước không nhiều bằng việc tiêu thụ xăng hiện nay).
A. Không nhiều xăng được tiêu thụ vào 10 năm trước bằng hiện nay. → đúng
B. Hiện nay việc tiêu thụ xăng đang giảm dần. → sai nghĩa so với câu gốc
C. Chúng ta có nhiều xăng vào 10 nám trước hơn hiện nay. → sai nghĩa so với câu gốc, không phải việc
có nhiều xăng mà là việc tiêu thụ xăng.
D. Chúng ta nên tiêu thụ nhiều xăng nhất có thể → sai nghĩa so với câu gốc.
→ Chọn đáp án A
Question 23. Đáp án C
Dịch đề bài: Vấn đề này phức tạp đến nỗi học sinh không thể hiểu được.
Trong câu đề bài sử dụng mệnh đề trạng ngữ chỉ kết quả: S + V + so + adj/adv + that + S + V: quá...đến
nỗi. Ngoài cấu trúc này ta còn có một số cấu trúc tương đương sau:
S + V... + so + adj + a/an + singular count, noun + that + S + V
S + V... + such (+ a/an) (+ adjective) + noun + that + S + V
S + V... + too + adj/adv + (for O) to-infinitive S + V... + adj/adv + enough +(for O) to- infinitive
A. Không dịch vì sai ngữ pháp: problem là danh từ đếm được số ít dùng cấu trúc với such thì phải có mạo
từ a → loại
B. Vấn đề này không đủ đơn giản cho học sinh để hiểu. →sai nghĩa so với câu gốc
C. Vấn đề này rất phức tạp cho học sinh để không thể hiểu. → đúng

Trang 14
D. Không dịch nghĩa vì sai ngữ pháp: Nếu có danh từ problem thì phải sử dụng such, không được dùng
so. Mặt khác cấu trúc đặc biệt với so chỉ đi với danh từ đếm được số ít, ở đây problem ở số nhiều → loại
→ Chọn đáp án C
Question 24. Đáp án D
Dịch đề bài: Người bạn Mỹ của tôi thấy nó khó để gắp thức ăn bằng đũa (ngụ ý: không quen với việc gắp
thức ăn bằng đũa)
A. Người bạn Mỹ của tôi không muốn gắp thức ăn bằng đũa sai nghĩa so với câu gốc
B. Người bạn Mỹ của tôi không thể tìm thấy đũa để gắp thức ăn → sai nghĩa so với câu gốc
C. Người bạn Mỹ của tôi không từng gắp thức ăn bằng đũa → sai nghĩa so với câu gốc
D. Người bạn Mỹ của tôi không quen với việc gắp thức ăn bằng đũa → đúng
→ Chọn đáp án D
* Mở rộng: Ta có một số cụm từ diễn đạt quen (không quen) với việc gì như sau:
Be/get used to V-ing/sth
Be/get accustomed to V-ing/sth
Accustom yourself/sb to sth/V-ing
Question 25. Đáp án D
Dịch đề bài: Đó là một cuốn tiểu thuyết thú vị. Tôi đã thức suốt đêm để đọc hết nó. (ngụ ý: vì hay nên
thức suốt đêm đọc hết → mệnh đề trạng ngữ chỉ kết quả. Nếu nhìn nhanh ta có thể chọn ngay được đáp án
D - sử dụng cấu trúc đảo ngữ với so)
A. Tôi đã thức suốt đêm để đọc hết cuốn tiểu thuyết vì vậy nó thú vị. → sai nghĩa so với câu gốc.
B. Nếu nó không phải là một cuốn tiểu thuyết thú vị thì tôi sẽ không thức suốt đêm để đọc nó. → sai nghĩa
so với câu gốc.
C. Mặc dù nó là một cuốn tiểu thuyết thú vị nhưng tôi đã thức suốt đêm để đọc hết nó. → sai nghĩa so với
câu gốc
D. Nó là cuốn tiểu thuyết thú vị đến nỗi tôi đã thức suốt đêm để đọc hết nó → đúng
→ Chọn đáp án D
* Mở rộng: Trong mệnh đề trạng ngữ chỉ kết quả, ta có thể dùng cấu trúc đảo với so... that/ such... that:
So + adj/adv + tobe/Auxiliary + S + that + clause
Such + tobe + (a/an) + adj + N + that + clause
Such + (a/an) + adj + N + tobe + that + clause
Question 26. Đáp án D
Dịch đề bài: Cô ấy đã ngã xuống ở những phút cuối. Cô ấy đã không chiến thắng cuộc đua.
A. Không dịch vì sai ngữ pháp: đề bài là sự thật ở trong quá khứ thì khi viết lại bằng câu điều kiện thì
phải viết bằng câu loại III loại
B. Cô ấy đã không giành chiến thắng cuộc đua mặc dù cô ấy ngã xuống ở những phút cuối cùng → sai
nghĩa, so với câu gốc thì đây không phải là mệnh đề mang nghĩa tương phản

Trang 15
C. Bởi vì không chiến thắng cuộc đua nên cô ấy đã ngã xuống ở những phút cuối cùng → sai nghĩa so với
câu gốc.
D. Cô ấy đã có thể chiến thắng cuộc đua nếu cô ấy không ngã xuống ở những phút cuối cùng → đúng vì
viết ở câu điều kiện loại III - giả sử một điều không có thật trong quá khứ.
→ Chọn đáp án D
Question 27. Đáp án A
“Wow! What a nice coat you are wearing!” “Ồ! Bạn đang mặc một chiếc áo khoác đẹp quá!” - đây là một
lời khen. Để đáp lại một lời khen thì ta phải cảm ơn lại, đáp án phù hợp nhất là đáp án A.
A. Cảm ơn. Mẹ tôi đã mua nó cho tôi
B. Chắc chắn rồi. Bạn cũng thích nó chứ?
C. Tôi thích bạn nói như vậy
D. Vâng, dĩ nhiên rồi. Nó đắt đó.
Question 28. Đáp án C
“Excuse me! Can you show me the way to Main Street, please?” - “Xin lỗi! Làm ơn chỉ cho tôi đường
đến phố Main được không?” - đây là một câu hỏi chỉ đường.
A. Tiếp tục
B. Nó thì dễ để làm
C. uhm, tôi xin lỗi tôi không biết tôi không biết lối đi.
D. Tôi đi đúng phải không?
→ Đáp án C là phù hợp nhất.
Question 29.
Thrifty (adj) : tiết kiệm. Chọn đáp án B là từ gần nghĩa nhất với từ đề bài cho.
Luxurious (adj): sang trọng
Economical (adj): tiết kiệm
Sensible (adj): hợp lý
Miserable (adj): khổ sở, phiền muộn
Tạm dịch: Bằng việc tiết kiệm và mua bán thông minh, các bà nội trợ ở thành phố có thể nuôi cả gia đình
chỉ với 500.000 đồng mỗi tuần
Question 30.
Made up (ph.v): bịa chuyện, trang điểm, làm hòa, chiếm (%). Chọn đáp án A gần nghĩa nhất với từ đề bài
cho.
Invented (v): phát minh, bịa chuyện
Narrated (v): kể lại, tóm tắt câu chuyện
Unfolded (v): mở ra, lộ ra
Recounted (v): thuật lại
Tạm dịch: Gerry không đi thám hiểm - anh ấy đã bịa ra một phần của câu chuyện.

Trang 16
Question 31. Đáp án A
Invertebrate (n): động vật không có xương sống. Chọn đáp án A là từ trái nghĩa với từ đề bài cho.
With backbone: có xương sống
With ribs : có xương sườn
Without ribs: không có xương sườn
Without backbone: không có xương sống
Tạm dịch: Khoảng 95% số động vật là động vật không có xương sống, chúng có thể sống ở bất kỳ nơi
nào, nhưng hầu hết sống ở dưới đại dương giống như sao biển và cua.
Question 32. Đáp án C
Bite the hand that feeds you (idiom): vô ơn (là thành ngữ chỉ hành động ăn cháo đá bát, gây tổn thương
hoặc làm hại những người đã từng giúp đỡ mình), chọn đáp án C là từ trái nghĩa với từ đề bài cho.
Be unfriendly: không thân thiện
Be ungrateful: vô ơn
Be thankful: biết ơn
Be devoted: tận tụy
Tạm dịch: Nếu bạn nói những điều không hay về một người đã cho bạn một công việc thì bạn là người vô
ơn.
Question 33. Đáp án D
Progression (n): sự tiến bộ
Successes (n): sự thành công
Increases (n): sự gia tăng
Advances (n): sự phát triển
Ta thấy động từ have được để ở dạng nguyên thể nghĩa là chủ ngữ phải ở dạng số nhiều → loại A
Dựa vào nghĩa và cách sử dụng ta chọn được đáp án D.
Advances in sth (collocation): Những bước tiến trong...
Trích bài: Advances in technology have made a lot of changes to our everyday lifestyles, but one of the
biggest has got to be how we read books.
Tạm dịch: Những bước tiến trong ngành công nghệ đã tạo ra nhiều thay đổi trong lối sống hàng ngày của
chúng ta, nhưng một trong những thay đổi to lớn nhất là cách chúng ta đọc sách.
Question 34.
Ta có cụm: opt for sth: lựa chọn
Trích bài: Since the invention of the e-book, there has been a significant change to our reading habits.
Given the choice between taking a couple of heavy paperbacks on holiday or an e-book device like a
Kindle, most of us, including our parents and grandparents, would unsurprisingly opt for the Kindle.
Tạm dịch: Kể từ khi phát minh ra sách điện tử đã có một sự thay đổi đáng kể đối với thói quen đọc sách
của chúng ta. Được cho lựa chọn giữa việc mang một vài cuốn sách bìa cứng nặng vào ngày nghỉ hoặc

Trang 17
một thiết bị sách điện tử như Kindle thì hầu hết chúng ta bao gồm cả ông bà bố mẹ đều không ngạc nhiên
khi chọn Kindle.
Question 35. Đáp án D
Special (adj): đặc biệt
Naughty (adj): tinh nghịch
Funny (adj): hài hước
Tricky (adj): khó, khôn lanh
Căn cứ vào nghĩa ta chọn được đáp án D.
Trích bài: But what would our lives be like with no books at all? It’s a tricky question
Tạm dịch: Những cuộc sống của chúng ta sẽ thế nào nếu không có sách? Nó là một câu hỏi khó.
Question 36. Đáp án C
Alternatives (n): sự lựa chọn, sự thay thế
Contestants (n): thí sinh
Opponents (n): đối thủ, người phản đối
Enemies (n): kẻ thù
Câu phía trước có đề cập đến ý kiến trong tương lai sẽ không còn thấy sách - tất cả sẽ được làm trực
tuyến: “Some educational specialists are making predictions that in the future we won’t even see books in
classrooms - everything will be done online!”. Và câu phía sau có đề cập “. ...of the idea of getting rid of
books say that...- quan điểm về việc loại bỏ sách cho rằng..Hai câu này có mối quan hệ tương phản, dựa
vào nghĩa ta chọn được đáp án C.
Trích bài: Some educational specialists are making predictions that in the future we wont even see books
in classrooms - everything will be done online! Opponents of the idea of getting rid of books say that
there will always be a need for paper-based versions of materials.
Tạm dịch: Một số chuyên gia giáo dục dự đoán trong tương lai chúng ta sẽ không còn thấy sách trong các
lớp học nữa - tất cả mọi thứ sẽ được làm trực tuyến! Những người phản đối quan điểm loại bỏ sách cho
rằng sẽ luôn có nhu cầu về các phiên bản tài liệu bằng giấy.
Question 37. Đáp án A
Remote (adj): nhỏ/một chút, xa xôi
Far (adj): (khoảng cách) xa
Long (adj): dài
Distant (adj): xa (về khoảng cách/ thời gian)
Dựa vào nghĩa ta chọn đáp án A.
Trích bài: However, to be realistic, we have to accept that there is a remote chance that in a decades time
schools and classrooms will be book-free! what do you think of that?
Tạm dịch: Tuy nhiên, trên thực tế, chúng ta phải chấp nhận rằng có một khả năng dù không lớn cho các
trường học và lớp học trong vòng một thập kỷ tới sẽ không còn sách nữa. Bạn nghĩ gì về điều đó?

Trang 18
Question 38.
Dịch đề bài: Ý chính của đoạn văn là gì?
A. Một loại tiền tệ mới trong thế giới ảo
B. Một cách kinh doanh trong thế giới ảo
C. Một sự thay thế cho bitcoin được tạo ra bởi Nakamoto
D. Tương lai của bitcoin trong thế giới thực
Câu hỏi về ý chính của đoạn văn là câu hỏi về quan điểm chính, nội dung chính xuyên suốt toàn đoạn
văn, không phải là một ý nằm trong từng đoạn văn nhỏ. Đối với loại câu hỏi này chúng ta nên:
1. Đọc kỹ vài câu đầu của đoạn văn để nắm ý chính vì thường những câu đầu chứa ý chính của toàn bài,
có thể đọc lướt qua phần sau để chắc chắn liệu rằng mình đã chọn đúng hay chưa.
2. Nếu như bài đọc có nhiều đoạn văn, các em nên đọc một vài dòng đầu tiên của mỗi đoạn văn nhỏ, tìm
chủ đề của mỗi đoạn nhỏ đó. Đọc lướt qua các phần sau xem mình phán đoán đã đúng hay chưa. Sau đó
phân tích tổng hợp ra ý chính của toàn bài.
3. Loại ra các phương án chắc chắn sai và loại bỏ những phương án quá cụ thể.
Đoạn 1: Giới thiệu bitcoin là một loại tiền ảo, có thế được sử dụng để mua các sản phẩm và dịch vụ.
Đoạn 2: Nguồn gốc và tính năng khác của bitcoin.
Đoạn 3: Những rủi ro có thể gặp phải được chính phủ cảnh báo khi dùng bitcoin.
Như vậy ta có thể tổng hợp lại ý chính xuyên suốt toàn bài đơn giản chỉ là đáp án A - Một loại tiền tệ mới
trong thế giới ảo. Phương án B chỉ là một thông tin nhỏ trong bài, phương án C và D không được đề cập.
Question 39.
Dịch đề bài: Từ “they” trong đoạn 1 thay thế cho___________
A. dollar B. euro
C. điều, việc D. bitcoin
Đối với câu hỏi về đại từ thay thế câu trả lời thường ở câu phía trước hoặc trong chính câu mà đại từ đó
xuất hiện. Để tìm được danh từ mà đại từ ngụ ý, ta làm như sau:
1. Tìm đại từ trong đoạn văn
2. Đọc phần đoạn văn phía trước đại từ cẩn thận
3. Tìm danh từ phù hợp ở phía trước đại từ
Trích bài: Bitcoins are a form of virtual currency. In other words, they are a type of money that does not
exist in the actual world. However, they can be used to purchase actual products and services from real
companies.
Tạm dịch: Bitcoin là một hình thức tiền tệ ảo. Hay nói cách khác, chúng là một loại tiền mà không tồn tại
trong thế giới thực. Tuy nhiên, chúng có thể được sử dụng để mua các sản phẩm và dịch vụ thật từ các
công ty thực. Như vậy they được thay thế cho bitcoins.
Question 40. Đáp án B
Dịch đề bài: Từ “minuscule” gần nghĩa nhất với____________

Trang 19
A. đáng kể B. nhỏ
C. tối thiểu D. tăng
Minuscule (adj): nhỏ - small (adj): nhỏ
Chọn đáp án B.
Question 41.
Dịch đề bài: Tại sao bitcoin là mối quan tâm lớn với chính phủ?
A. Bởi vì giá trị của bitcoin đang dao động mạnh.
B. Bởi vì bitcoin cuối cùng sẽ thay thế tiền tệ quốc gia.
C. Bởi vì bitcoin có thể được sử dụng trong các giao dịch bất hợp pháp khác.
D. Bởi vì hầu hết chủ sở hữu bitcoin là tin tặc.
Trích bài: Governments are concerned that bitcoins can easily be stolen by hackers. It has dawned on
them that they might be used for illegal purposes. For example, stolen goods could be purchased without
the government s knowledge.
Tạm dịch: Chính phủ lo ngại rằng bitcoin có thể dễ dàng bị đánh cắp bởi tin tặc. Họ nhận ra rằng chúng
có thể được sử dụng cho các mục đích bất hợp pháp. Ví dụ, hàng hóa bị đánh cắp có thể được mua mà
chính phủ không biết.
→ Chọn đáp án C
Question 42. Đáp án A
Dịch đề bài: Từ nào sau đây được định nghĩa trong đoạn văn?
A. Bitcoin
B. Giao dịch
C. Thẻ tín dụng
D. Cơ sở dữ liệu công cộng
Ngay từ câu mở đầu ta đã thấy tác giả định nghĩa về bitcoin: “Bitcoins are a form of virtual currency. In
other words, they are a type of money that does not exist in the actual world.” - Bitcoin là một hình thức
tiền tệ ảo. Hay nói cách khác, chúng là một loại tiền mà không tồn tại trong thế giới thực
→ Chọn đáp án A
Question 43. Đáp án A
Dịch đề bài: Thái độ của tác giả đối với thị trường công việc đang thay đổi là gì?
A. Nó là một thách thức phải đối mặt
B. Nó đã làm cho quá nhiều người thất nghiệp
C. Nó là điều các bạn trẻ sợ
D. Nó có tác động tiêu cực đến giáo dục.
Trích bài: But the world has changed. The global economy is not kind to yesterday’s diligent and
dependable worker. The future belongs to quick-thinking people who are resourceful, ambitious and can
take the initiative…

Trang 20
Many young people are very aware of the pitfalls of the flexible workplace; they understand that
redundancy, downsizing and freelancing are all part of modern working life, but no one is telling them
how they might be able to turn the new rules of the employment game to their advantage.
Tạm dịch: Nhưng thế giới đã thay đổi. Nền kinh tế toàn cầu không còn tốt bụng với những người siêng
năng và đáng tin cậy trước đầy nữa. Tương lai sẽ thuộc về những người có tư duy nhanh chóng, tháo vát,
tham vọng và chủ động.
Nhiều người trẻ rất ý thức về những cạm bẫy ở nơi làm việc; họ hiểu rằng sự dư thừa, cắt giảm nhân sự và
làm việc tự do đều là một phần của cuộc sống làm việc hiện đại, nhưng không ai nói cho họ biết làm thế
nào họ có thể biến các quy tắc mới của trò chơi tìm việc làm thành lợi thế của họ.
Như vậy, thế giới thay đổi xuất hiện nhiẽu cạm bẫy ở nơi làm việc và không ai nói cho họ biết cách để
làm thế nào → đây là một thách thức mà những người trẻ phải đối mặt. → Chọn đáp án A.
Question 44. Đáp án D
Dịch đề bài: Tác giả nghĩ nền kinh tế toàn cầu đã ảnh hưởng đến thị trường việc làm như thế nào?
A. Người lao động phải sẵn sàng thay đổi công việc
B. Người lao động có thể không được nhận lương hưu.
C. Nó đã làm cho người lao động ít có thể tin cậy được
D. Nó làm cho công việc mạo hiểm hơn
Trích bài: Many young people are very aware of the pitfalls of the flexible workplace; they understand
that redundancy, downsizing and freelancing are all part of modern working life, but no one is telling
them how they might be able to turn the new rules of the employment game to their advantage.
Tạm dịch: Nhiều người trẻ rất ý thức về những cạm bẫy ở nơi làm việc; họ hiểu rằng sự dư thừa, cắt giảm
nhân sự và làm việc tự do đều là một phần của cuộc sống làm việc hiện đại, nhưng không ai nói cho họ
biết làm thế nào họ có thể biến các quy tắc mới của trò chơi tìm việc làm thành lợi thế của họ.
Thế giới hiện đại xuất hiện nhiều cạm bẫy khiến người trẻ (người lao động) không biết phải làm thế nào
biến những quy tắc thành lợi thế cho họ → Điều này làm cho công việc của họ mạo hiểm hơn, khó khăn
hơn → chọn đáp án D, các phương án còn lại không được đề cập trong bài.
Question 45. Đáp án B
Dịch đề bài: Tác giả sử dụng cụm “aware of the pitfalls” để chỉ ra rằng những người trẻ____________
A. Cảm thấy rằng công việc hiện đại quá linh hoạt
B. Hiểu các vấn đề của công việc hiện đại
C. Không nghĩ rằng họ được đào tạo đầy đủ
D. Chấp nhận rằng họ sẽ bị dư thừa
Trích bài: Many young people are very aware of the pitfalls of the flexible workplace; they understand
that redundancy, downsizing and freelancing are all part of modern working life, but no one is telling
them how they might be able to turn the new rules of the employment game to their advantage.

Trang 21
Tạm dịch: Nhiều người trẻ rất ý thức về những cạm bẫy ở nơi làm việc; họ hiểu rằng sự dư thừa, cắt giảm
nhân sự và làm việc tự do đều là một phần của cuộc sống làm việc hiện đại, nhưng không ai nói cho họ
biết làm thế nào họ có thể biến các quy tắc mới của trò chơi tìm việc làm thành lợi thế của họ.
aware of the pitfalls: ý thức về những cạm bẫy ở nơi làm việc → ngụ ý là hiểu vể những vấn đề của công
việc hiện đại → chọn đáp án B.
Question 46. Đáp án B
Dịch đề bài: Thanh thiếu niên muốn làm loại công việc nào?
A. Công việc tương tự như cha mẹ của chúng
B. Công việc mang lại sự hài lòng cho chúng
C. Công việc đồng thời có thể là một sở thích
D. Công việc có sự ổn định kinh tế
1. Although 80% of them have no intention of following in their parents’ footsteps, 69% still turn to their
parents for advice. They look at their working future in a different way to their parents-Khoảng 80%
không có ý định theo bước chân của cha mẹ chúng, 69% vẫn cẩn cha mẹ tư vấn. Chúng nhìn thấy tương
lai công việc theo một cách khác với cha mẹ của chúng. → Thông tin không ủng hộ phương án A → loại
A.
2. A job for life is not in their vocabulary; neither is a dead-end but secure job that is boring but pays the
bills. Almost half the boys surveyed expected that their hobbies would lead them into the right sort of job,
while most girls seemed determined to avoid traditionally female careers such as nursing: Một công việc
cho cuộc sống không nằm trong từ vựng của họ; đó không phải là một công việc cuối cùng nhưng ổn
định, mà ổn định thì buồn chán, nhưng mà đủ để trả được các hóa đơn. Gần nửa các cậu bé được khảo sát
mong rằng sở thích của họ sẽ giúp họ chọn được loại công việc phù hợp, trong khi hầu hết các cô gái
quyết tâm tránh các nghề nghiệp truyền thống của nữ giới như điều dưỡng.
→ Sở thích có thể giúp họ lựa chọn được công việc phù hợp # công việc đồng thời là một sở thích, nghĩa
khác nhau hoàn toàn → Loại C
Trong đoạn trên có đề cập đến ổn định kinh tế nhưng công việc ổn định thì nhàm chán → công việc ổn
định kinh tế chưa phải là loại công việc mà hầu hết thanh thiếu niên muốn → loại D
→ Chọn phương án B vì sở thích có thể giúp họ lựa chọn được công việc phù hợp → ngụ ý là công việc
đó phải mang lại sự hài lòng và niềm vui cho họ.
Question 47. Đáp án B
Dịch đề bài: Tác giả cảm thấy rằng hầu hết các bậc cha mẹ
A. Cho con cái họ lời khuyên tốt về công việc
B. Không có xu hướng tham vọng
C. Có quan điểm truyền thống về công việc
D. Có mục tiêu thực tế cho con cái họ

Trang 22
Trích bài: why do parents make terrible careers advisers?....Today’s 14 and 15-year- olds are ambitious.
They are optimistic about their prospects, but their career ideas are rather vague. Although 80% of them
have no intention of following in their parents’ footsteps, 69% still turn to their parents for advice. They
look at their working future in a different way to their parents.
Tạm dịch: Tại sao cha mẹ lại là những người tư vấn nghề nghiệp tồi tệ?....Các cô cậu 14- 15 tuổi rất tham
vọng. Họ lạc quan về tương lai của họ nhưng quan điểm nghề nghiệp thì khá mơ hồ. Khoảng 80% không
có ý định theo bước chân của cha mẹ chúng, 69% vẫn cần cha mẹ tư vấn. Chúng nhìn thấy tương lai công
việc theo một cách khác với cha mẹ của chúng → Câu hỏi tại sao cha mẹ lại là người tư vấn nghề nghiệp
tồi tệ và tiếp theo nói đến tham vọng của các cô cậu 14-15 tuổi; tuổi này chúng khá là lạc quan và có cách
nhìn về tương lai công việc khác với cha mẹ → điều này ngụ ý rằng hầu hết các bậc cha mẹ không tham
vọng bằng bọn trẻ. Như vậy ta chọn được đáp án B.
Question 48. Đáp án A
Dịch đề bài: Các bậc cha mẹ có thể giúp con cái của họ như thế nào?
A. Bằng cách cố gắng suy nghĩ theo cách của chúng
B. Bằng cách học trở nên can đảm hơn
C. Bằng cách phớt lờ lời khuyên của những người khác
D. Bằng cách trở nên thoải mái hơn.
Trích bài: what, then, can we as parents do to help them? The best thing is to forget all the advice that
your parents gave you, and step into your teenagers shoe.
Tạm dịch: Vậy thì những người làm cha mẹ như chúng ta làm gì để giúp đỡ chúng? Điều tốt nhất là quên
đi tất cả những lời khuyên mà cha mẹ của bạn đã dành cho bạn và đứng vào vị trí của con bạn.
→ Đứng vào vị trí của con cái ngụ ý là hãy cố gắng thử suy nghĩ theo cách của chúng → chọn đáp án A
Question 49. Đáp án C
Dịch đề bài: Tác giả tin vào điều gì về những cố vấn hướng dẫn của cô ấy?
A. Rằng họ đáng lẽ nên đối xử với cô ấy tốt hơn
B. Rằng lời khuyên họ đưa ra là sai
C. Rằng họ cũng đúng theo cách nào đó
D. Rằng họ đã cố hủy hoại sự nghiệp của cô ấy
Trích bài: Certainly when I was 15, my guidance counsellors were horrified at my plans to become a
writer. I’m glad I didn’t change my plans to suit them. Even so, their faith in rigid career paths was well-
founded. In those days, that was the way to get ahead.
Tạm dịch: Khi tôi 15 tuổi, những cố vấn hướng dẫn của tôi ngạc nhiên với kế hoạch trở thành một nhà
văn của tôi. Tôi thì rất vui khi tôi đã không thay đổi kế hoạch để phù hợp với họ. Mặc dù vậy, niềm tin
của họ vào con đường sự nghiệp cứng nhắc là hoàn toàn có cơ sở. Ngày đó, đây là cách để tiến bộ.

Trang 23
→ Tác giả vẫn giữ nguyên kế hoạch của mình mặc dù những cố vấn có những quan điểm cứng nhắc về sự
nghiệp; xét về một khía cạnh nào đó điểu cứng nhắc này vẫn có cơ sở → Thông tin ủng hộ đáp án C. Các
phương án còn lại không được đề cập.
Question 50. Đáp án B
Dịch đề bài: Tác giả cảm thấy điều gì sẽ xảy ra nếu hệ thống giáo dục không thay đổi?
A. Giới trẻ sẽ cảm thấy không hứng thú khi làm việc
B. Giới trẻ sẽ nhận những lời chỉ trích trên báo chí
C. Giới trẻ không thể phát huy hết tiềm năng của họ
D. Giới trẻ sẽ không lạc quan về tương lai của họ
Trích bài: If nothing else, a bit of optimism from an adult can serve as an antidote to the constant criticism
of teenagers in the press.
Tạm dịch: Nếu không còn gì khác, một chút lạc quan từ một người trưởng thành có thể đóng vai trò là
liều thuốc giải độc cho những lời chỉ trích liên tục của thanh thiếu niên trên báo chí.
→ Đoạn văn có đề cập đến việc thay đổi hệ thống giáo dục vì nó đang trở nên kém linh hoạt và bị ám ảnh
bởi các kỹ năng truyền thống. Trong khi đó thị trường việc làm cần những sự linh hoạt, yêu cầu nhà
trường có sự cập nhật thông tin về việc làm thì mới giúp các cố vấn giúp đỡ được học sinh của mình. Và
nếu không có các thông tin cập nhật thì họ sẽ nhận những lời chỉ trích trên báo chí → chọn đáp án B.

Trang 24
ĐỀ SỐ 07 ĐỀ THI THỬ TỐT NGHIỆP THPT
NĂM HỌC: 2020 – 2021
MÔN: TIẾNG ANH
Thời gian làm bài: 60 phút; không kể thời gian phát đề

Mark the letter A, B, C, or D on your answer sheet to indicate the word whose underlined part
differs from the other three in pronunciation in each of the following questions.
Question 1. A. scholarship B. architect C. cherish D. chorus
Question 2. A. honest B. honor C. vehicle D. height
Mark the letter A, B, C or D on your answer sheet to indicate the word that differs from the rest in
the position of the main stress in each of the following questions.
Question 3. A. private B. protect C. reform D. regard
Question 4. A. policeman B. eyewitness C. copyright D. dressmaker
Mark the letter A, B, C or D on your answer sheet to indicate the correct answer to each of the
following questions.
Question 5. Our line of designer clothing is more expensive than____________ of our competitor.
A. this B. those C. that D. their
Question 6. The boat was sailing north when a terrible storm____________.
A. was breaking B. would break C. had broken D. broke
Question 7. It____________that neither the passengers nor the driver was injured in the crash
A. was announced B. is announcing C. is announced D. was announcing
Question 8. After____________goodbye to her friends, she got onto the bus.
A. to be said B. being said C. having said D. said
Question 9. It’s high time we____________these mice.
A. are getting rid of B. are going to get rid of
C. get rid of D. got rid of
Question 10. The lady____________design had been chosen stepped to the platform to receive
the award.
A. whom B. whose C. that D. which
Question 11. Professor Smith explained the lecture slowly____________.
A. as soon as his students understood it clearly
B. so that his students can understand it clearly
C. unless his students fail to understand it clearly
D. so that his students could understand it clearly
Question 12. Put all the toys away____________someone slips and falls on them.
A. provided that B. unless C. in case D. so long as

Trang 1
Question 13. The interviews with parents showed that the vast majority were____________of teachers.
A. support B. supportive C. supporter D. supporting
Question 14. Some____________animals become tame if they get used to people.
A. wild B. domestic C. endangered D. rare
Question 15. She made a big____________about not having a window seat on the plane.
A. fuss B. complaint C. interest D. excitement
Question 16. After the robbery of the bank, the police____________of the robbers and arrested
the whole band.
A. kept pace B. took notice C. made use D. kept track
Question 17. After a good nights sleep he woke up feeling as fresh as____________and eager to
start work again.
A. fruit B. a daisy C. a kitten D. a maiden
Question 18. Bill seems unhappy about his job because he doesn’t get____________ his boss.
A. up to B. on for C. on well with D. in with
Mark the letter A, B, C, or D on your answer sheet to indicate the underlined part that needs
correction in each of the following questions.
Question 19. If I wasn’t been wearing the seatbelt, I’d have been seriously injured.
A. wasn’t B. wearing C. I’d D. seriously injured
Question 20. The receptionist said I must fill in that form before I attended the interview.
A. The B. I must C. that form D. attended
Question 21. They had a discussion about training not only the new employees but also giving them
some challenges.
A. about B. training not only C. giving D. some changes
Mark the letter A, B, C or D on your answer sheet to indicate the sentence that is closest in
meaning to each of the following questions.
Question 22. Mary exclaimed that the singer’s voice was so sweet.
A. “How sweet is the singer’s voice?” said Mary.
B. “What a sweet voice the singer has” said Mary.
C. “How sweet voice the singer is” said Mary.
D. “What a sweet voice the singer is” said Mary.
Question 23. Without skillful surgery he would not have survived the operation.
A. But for skillful surgery he would have survived the operation.
B. With skillful surgery he would have survived the operation.
C. Had it not been for skillful surgery he would have survived the operation.
D. He wouldn’t have survived the operation if it hadn’t been for skillful surgery.
Question 24. The situation was so embarrassing that she did not know what to do.

Trang 2
A. It was such an embarrassing situation; however, she did not know what to do.
B. So embarrassing was the situation that she did not know what to do.
C. So embarrassing the situation was that she did not know what to do.
D. She did not know what to do, though it was not an embarrassing situation.
Mark the letter A, B, C or D on your answer sheet to indicate the sentence that best combines
each pair of sentences in the following questions.
Question 25. Laura practiced playing the instrument a lot. She could hardly improve her
performance
A. However much Laura practiced playing the instrument, she could hardly perform any better.
B. As soon as Laura practiced playing the instrument a lot, she could perform much better
C . Had Laura practiced playing the instrument a lot, she could have performed much better.
D. Hardly had Laura practiced playing the instrument a lot when she could improve her performance.
Question 26. The boy lost several of his fingers because of firecrackers. The doctors are operating
on him.
A. The boy who lost several of his fingers because of firecrackers is being operating on.
B. The boy whom the doctors are operating on him lost several of his fingers because of firecrackers.
C. The boy was whom the doctors are operating on him lost several of his fingers because of
firecrackers.
D. The doctors are operating on the boy who lost several of his fingers because of firecrackers.
Mark the letter A, B, C or D on your answer sheet to indicate the most suitable response to
complete each of the following exchanges.
Question 27. Bill and Jerry are in a coffee shop.
- Bill: “Can I get you another drink.”
- Jerry: “____________”
A. No, it isn’t B. Not just now C. No, I’ll think it over D. Forget it
Question 28. Mary and her son are in the kitchen.
- Mary: “Watch out or you’ll hurt yourself!”
- Son: “____________”
A. Yes, I won’t do that again B. That’s a good idea
C. Thank you! D. I totally agree with you.
Mark the letter A, B, C, or D on your answer sheet to indicate the word or phrase that is
CLOSEST in meaning to the underlined part in each of the following questions.
Question 29. The game of life is a lot like football. You have to tackle your problems, block your fears,
and score your points when you get the opportunity.
A. have B. take C. gain D. fulfill

Trang 3
Question 30. Because of a piece of paper in the soup bowl, the old lady raised a ballyhoo about it at
the restaurant.
A. made a fuss B. appeased C. applauded D. complained
Mark the letter A, B, C, or D on your answer sheet to indicate the word or phrase that is
OPPOSITE in meaning to the underlined part in each of the following questions.
Question 31. His physical condition was not an impediment to his career as a violinist. He has won a
lot of prizes.
A. advantage B. obstacle C. barrier D. disadvantage
Question 32. She told him she’d spent all her savings but he didn’t bat an eyelid.
A. didn’t tell anything B. showed surprise
C. didn’t care D. refused to listen
Read the following passage and mark the letter A, B, C or D on your answer sheet to indicate the
correct word for each of the blanks from 33 to 37.
Tattooing is an old art. Tattooing is an old art. In ancient Greece, people who had tattoos were (33)
____________as members of the upper classes. On the other hand, tattooing was banned in Europe by
the early Christians, (34) thought that it was a sinful thing to do. It was not (35) ____________ the late
18th century, when Captain Cook saw South Sea Islander decorating their bodies with tattoos that
attitudes began to change. Sailors came back from these islands with pictures of Christ on their backs
and from then on, tattooing gained (36) ____________popularity. A survey by the French army in 1881
showed that among the 387 men questioned there were 1,333 designs.
Nowadays, not everybody finds tattoos acceptable. Some people thing that getting one is silly because
tattoos are more or less permanent. There is also some (37) ____________about catching a blood
disease from unsterilized needles. Even for those who do want a tattoo, the process of getting one is not
painless, but the final result, in their eyes, is worth the pain.
Question 33. A. supposed B. realised C. held D. regarded
Question 34. A. who B. that C. they D. whose
Question 35. A. by far B. since C. because D. until
Question 36. A. at B. on C. in D. of
Question 37. A. danger B. trouble C. concern D. threat
Read the following passage and mark the letter A, B, C, or D on your answer sheet to indicate the
correct answer to each of the questions from 38 to 42.
Jupiter is the largest and most massive planet and is fifth in order of distance from the sun. It is well
placed for observation for several months in every year and on average is the brightest of the planets
apart from Venus, though for relatively brief periods Mars may outshine it. Jupiter’s less than 10-hour
rotation period gives it the shortest day in the solar system in so far as the principal planets are

Trang 4
concerned. There are no true seasons on Jupiter because the axial inclination to the perpendicular of the
orbital plane is only just over 3°-less than that for any other planet.
The most famous mark on Jupiter is the Great Red spot. It has shown variations in both intensity and
color, and at times it has been invisible, but it always returns after a few years. At its greatest extent it
may be 40,000 kilometers long and 14,000 kilometers wide, so its surface area is greater than that of
Earth. Though the latitude of the Red Spot varies little, it drifts about in longitude. Over the past
century the total longitudinal drift has amounted to approximately 1200°. The latitude is generally very
close to -22°. It was once thought that the Red Spot might be a solid or semisolid body floating in
Jupiter’s outer gas. However, the Pioneer and Voyager results have refuted that idea and proven the
Red Spot to be a phenomenon of Jovian meteorology. Its longevity may well due to its exceptional
size, but there are signs that it is decreasing in size, and it may not be permanent. Several smaller red
spots have been seen occasionally but have not lasted.
Question 38. According to the passage, which planet typically shines the most brightly?
A. Earth B. Jupiter C. Venus D. Mars
Question 39. The word "it" in paragraph 2 refers to ____________
A. Jupiter B. The Great Red Spot C. intensity D. color
Question 40. The word "exceptional" in paragraph 2 mostly means____________
A. extreme B. sustainable C. temporary D. infrequent
Question 41. According to the passage, which of the following is NOT true?
A. Jupiter is bigger than all other planet in the solar system
B. A day in Jupiter is nearly 10 hours long
C. The Red Great Spot moves more vertically than horizontally
D. Scientists have proof showing that smaller red spots are increasing their size to become other Great
Red Spots.
Question 42. The passage was probably taken from____________
A. an art journal B. a geology magazine
C. a high school textbook D. an archaeology book
Read the following passage and mark the letter A, B, C, or D on your answer sheet to indicate the
correct answer to each of the questions from 43 to 50.
Mickey Mantle was one of the greatest baseball players of all time. He played for the New York
Yankees in their years of glory. From the time Mantle began to play professionally in 1951 to his last
year in 1968, baseball was the most popular game in the United States. For many people, Mantle
symbolized the hope, prosperity, and confidence of America at that time.
Mantle was a fast and powerful player, a “switch-hitter” who could bat both right-handed and left-
handed. He won game after game, one World Series championship after another, for his team. He was a
wonderful athlete, but this alone cannot explain Americas fascination with him. Perhaps it was because

Trang 5
he was a handsome, red-haired country boy, the son of a poor miner from Oklahoma. His career, from
the lead mines of the West to the heights of success and fame, was a fairy-tale version of the American
dream. Or perhaps it was because America always loves a “natural”: a person who wins without
seeming to try, whose talent appears to come from an inner grace. That was Mickey Mantle.
But like many celebrities, Mickey Mantle had a private life that was full of problems. He played
without complaint despite constant pain from injuries. He lived to fulfill his father’s dreams and drank
to forget his father’s early death.
It was a terrible addiction that finally destroyed his body. It gave him cirrhosis of the liver and
accelerated the advance of liver cancer. Even when Mickey Mantle had turned away from his old life
and warned young people not to follow his example, the destructive process could not be stopped.
Despite a liver transplant operation that had all those who loved and admired him hoping for a
recovery, Mickey Mantle died of cancer at the age of 63.
Question 43. What is the main idea of the passage?
A. Mickey Mantle’s success and his private life full of problems
B. Mickey Mantle as the greatest baseball player of all time
C. Mickey Mantle and the history of baseball
D. Mickey Mantle and his career as a baseball player
Question 44. It can be inferred from paragraph 1 that Mantle____________
A. introduced baseball into the US
B. earned a lot of money from baseball
C. had to try hard to be a professional player
D. played for New York Yankees all his life
Question 45. The word “this” in paragraph 2 refers to____________.
A. Mantles being fascinated by many people
B. Mantle’s being a wonderful athlete
C. Mantles being a “switch-hitter”
D. Mantle’s being a fast and powerful player
Question 46. The word “fulfill” in paragraph 4 is closest in meaning to____________.
A. achieve what is hoped for, wished for, or expected
B. do something in the way that you have been told
C. do what you have promised or agreed to do
D. get closer to something that you are chasing
Question 47. which of the following is mentioned as the main cause of the destruction of Mantle’s
body?
A. His way of life B. His loneliness
C. His own dream D. His liver transplant operation

Trang 6
Question 48. The word “cirrhosis” in paragraph 5 probably means____________
A. a danger B. an operation C. the destruction D. a serious disease
Question 49. All of the following are true about Mantle EXCEPT____________ .
A. He was born into a poor family but later achieved fame and success
B. He was so severely addicted to alcohol that he couldn’t give it up to the last days of his life
C. He could hit with the bat on either side of his body
D. His father’s early death exerted a negative influence on him.
Question 50. It can be inferred from the passage that for most Americans____________.
A. success in Mantle’s career was difficult to believe
B. Mantle had a lot of difficulty achieving fame and success
C. success in Mantle’s career was unnatural
D. Mantle had to be trained hard to become a good player.

Trang 7
Đáp án
1-C 2-D 3-A 4-A 5-C 6-D 7-A 8-C 9-D 10-B
11-D 12-C 13-B 14-A 15-A 16-D 17-B 18-C 19-A 20-B
21-B 22-B 23-D 24-B 25-A 26-D 27-B 28-A 29-C 30-A
31-A 32-B 33-D 34-A 35-D 36-C 37-C 38-C 39-B 40-D
41-D 42-B 43-A 44-B 45-B 46-A 47-A 48-D 49-B 50-A

LỜI GIẢI CHI TIẾT


Question 1: Đáp án C
Đáp án C đúng vì phần gạch chân của đáp án C đọc là âm /tʃ/ còn phần gạch chân của các đáp án khác
được đọc là âm /k/
A. scholarship /ˈskɒləʃɪp/ (n): học bổng
B. architect /ˈɑːkɪtekt/ (n): kiến trúc sư
C. cherish /ˈtʃerɪʃ/ (v): yêu dấu
D. chorus /ˈkɔːrəs/ (n): hợp xướng
Question 2. Đáp án D
Đáp án D đúng vì phần gạch chân của đáp án D đọc là âm /h/ còn phần gạch chân của các đáp án còn lại
không phát âm (âm câm)
A. honest /ˈɒnɪst/ (adj): trung thực
B. honor /ˈɒnə(r)/ (n): niềm vinh dự
C. vehicle /ˈviːəkl/ (n): xe cộ
D. height /haɪt/ (n): chiều cao
Question 3. Đáp án A
Đáp án A đúng vì đáp án A trọng âm rơi vào âm tiết thứ nhất còn ba đáp án còn lại trọng âm rơi vào âm
tiết thứ hai
A. private /ˈpraɪvət/ (adj): riêng tư
B. protect /prəˈtekt/ (v): bảo vệ
C. reform /rɪˈfɔːm/ (v): cải cách
D. regard /rɪˈɡɑːd/ (n): sự quan tâm
Question 4. Đáp án A
Đáp án A đúng vì đáp án A trọng âm rơi vào âm tiết thứ hai còn ba đáp án còn lại trọng âm rơi vào âm
tiết thứ nhất
A. policeman /pəˈliːsmən/ (n): cảnh sát
B. eyewitness /ˈaɪwɪtnəs/ (n): nhân chứng
C. copyright /ˈkɒpiraɪt/ (n): bản quyền
D. dressmaker /ˈdresmeɪkə(r)/ (n): thợ may

Trang 8
Question 5. Đáp án C
Đây là câu hỏi liên quan đến đại từ. Ta thấy trong câu này, ta đang so sánh “line of designer clothing” của
chúng ta với cái tương tự của đối thủ “our competitors”. Ta thấy động từ to be đang được chia ở số ít chủ
ngữ của câu ở số ít → đại từ thay thế sẽ dùng that nên C là đáp án đúng.
Tạm dịch. Dòng quần áo thiết kế của chúng tôi đắt hơn của đối thủ.
Question 6. Đáp án D
Ta có cấu trúc S + was/were + V-ing when S+ V-ed (chia quá khứ đơn) dùng để diễn tả một hành động
đang xảy ra trong quá khứ thì có một hành động khác xen vào. Và hành động xen vào sẽ chia ở thì quá
khứ đơn. Nên đáp án đúng là D
Tạm dịch. Chiếc thuyền đang đi về phía Bắc thì có một cơn bão khủng khiếp đến phá vỡ.
Question 7. Đáp án A
Chúng ta thấy announce là một ngoại động từ nhưng không có tân ngữ đứng sau → ở dạng bị động → giữ
lại đáp án A và C. Ta thấy hành động được thông báo trong câu (was injured) không phải là một hành
động có tính lâu bền lặp đi lặp lạị (dựa vào từ khóa in the crash- vụ va chạm đã được xác định bởi mạo từ
the) vì vậy announce sẽ không thể chia ở thời hiện tại đơn do hiện tại đơn dùng để thể hiện hành động có
tính lâu bền. Câu văn này đơn thuần diễn tả lại hành động đã xảy ra nên ta chọn đáp án A- đã được thông
báo
Đây là câu hỏi có sử dụng cấu trúc bị động của động từ nêu ý kiến
Tạm dịch. Người ta đã thông báo rằng cả hành khách lẫn người lái xe không ai bị thương trong vụ va
chạm cả.
Question 8.
Đây là mệnh đề trạng ngữ chỉ thời gian đã được rút gọn do chung chủ ngữ. Ta thấy có after → sau after sẽ
là hành động xảy ra trước hành động còn lại → rút gọn thành having + P2
Mệnh đề chưa được rút gọn sẽ là: After she had said goodbye to her friends, she got onto the bus
Tạm dịch. Sau khi nói tạm biệt bạn bè, cô ấy đã lên xe bus.
Question 9.
Đây là câu giả định với cấu trúc của Its high time. Ta có cấu trúc
Its high/ about time + S + V-ed (quá khứ đơn)
Tạm dịch: Đã đến lúc chúng ta phải loại bỏ những con chuột này.
Question 10.
Trong câu trên ta thấy chủ ngữ của câu là the lady và vị ngữ của câu là stepped → cụm chủ vị nằm sau
danh từ the lady chính là mệnh đề quan hệ bổ sung ý nghĩa cho danh từ the lady ( Tips: nếu sau danh từ
có động từ làm vị ngữ nhưng không phải là vị ngữ của câu thì vị ngữ đó nằm trong mệnh đề quan hệ, bố
sung ý nghĩa cho danh từ đứng phía trước đó). Như vậy chỗ trống cần điền sẽ là từ quan hệ. Từ câu trên ta
có thể phân tích câu trên thành cấu trúc:
S (Noun) + Từ quan hệ + Noun+ V (của mệnh đề quan hệ, ở dạng bị động) + V (làm vị ngữ).

Trang 9
Có thể thấy từ quan hệ trong trường hợp này không làm chủ ngữ do mệnh đề quan hệ đã có chủ ngữ là
design → bỏ đáp án C và D. Từ quan hệ không làm tân ngữ do động từ của mệnh đề quan hệ được chia bị
động (tân ngữ được làm chủ ngữ) → bỏ đáp án A. Vậy đáp án của câu là phương án B.
Trong trường hợp này design là danh từ thuộc sở hữu của “the lady” nên ta cần một đại từ quan hệ chỉ sở
hữu chính là whose
Tạm dịch. Người phụ nữ có thiết kế được chọn đã bước lên bục nhận giải thưởng.
Question 11. Đáp án D
Chúng ta dựa vào từ khóa của 2 vế câu: explain slowly và understand clearly. Nếu ta chọn phương án A.
Liên từ as soon as sử dụng không phù hợp vì liên từ này dùng để thể hiện một hành động xảy ra liền ngay
sau một hành động khác (ngay khi mà...). Hành động explain slowly không phải là hành động liền ngay
sau understand clearly (không phù hợp về mặt logic và nghĩa của câu).
Nếu ta chọn đáp án C ta thấy động từ trong mệnh đề unless đang dùng sai thời với động từ của mệnh đề
chính (hiện tại đơn - quá khứ đơn) → đáp án B không chính xác.
Chỉ còn lại 2 phương án B và D. Ta thấy hai phương án đang thể hiện mục đích của hành động explain
slowly là để understand clearly. Khi thể hiện mục đích ta có thể sử dụng so that + mệnh đề. Động từ của
câu đang chia ở quá khứ (explained) → động từ trong mệnh đề cũng sẽ chia quá khứ → đáp án D đúng
Tạm dịch. Giáo sư Smith giảng bài một cách chậm rãi để học sinh của ông thật sự hiểu bài
❖ For review
Khi muốn thể hiện mục đích, chúng ta có thể sử dụng các cấu trúc sau:
- S + V + (not) to V/ in order (not) to V/ so as (not) to V: ai đó làm việc gì để làm gì/ không làm gì
- S1+ V1+ so that/ in order that S2+ V2 trong đó:
+ Nếu V1 ở hiện tại/ tương lai thì V2 = will/ can (not) V
+ Nếu V1 ở quá khứ thì V2 = would/ could (not) V
- With a view to (not) V-ing, S + V: để (không) làm gì, thì ai đó làm gì
Question 12. Đáp án C
Chúng ta thấy ba đáp án A, B và D đều không hợp nghĩa. A: với điều kiện..., B: Nếu... không..., C: phòng
khi/ phòng trường hợp, D: miễn là → chỉ có đáp án C là chính xác
Tạm dịch. Hãy cất đồ chơi đi phòng khi có ai đó trượt chân và ngã lên chúng.
Question 13.
Ta thấy sau chỗ trống cần điền là giới từ of → chọn đáp án B ta có (be) supportive of: ủng hộ ai. Ta có
support (n) đi với giới từ for còn support (v) thì không đi cùng giới từ. Tính từ supporting thường đứng
liền trước danh từ khác để bổ sung ý nghĩa cho danh từ đó.
A. support (v/n): ủng hộ/ sự ủng hộ, sự hỗ trộ
B. supportive (adj): thông cảm, ủng hộ
C. supporter (n): người ủng hộ
D. supporting (adj): phụ, thứ yếu

Trang 10
Tạm dịch. Các cuộc phỏng vấn với cha mẹ học sinh cho thấy đa số đều ủng hộ giáo viên.
Question 14. Đáp án A
Trong câu trên ta thấy có sự xuất hiện của các từ tame (adj): thuần hóa. Xét về nghĩa thì chỉ có đáp án A
là phù hợp
A. wild (adj): hoang dã
B. domestic (adj) trong nhà, trong nước
C. endangered (adj): bị nguy hiểm
D. rare (adj): quý hiếm
Tạm dịch. Một số loài động vật hoang dã đã thuần nếu chúng quen với con người.
Question 15.
Ta có cụm từ make a fuss over/ about: làm ầm lên
A. fuss (n): sự ồn ào, sự om sòm
B. complaint (n): lời than phiền
C. interest (n): sự hứng thú
D. excitement (n): sự sôi nổi, sự náo động
Tạm dịch. Cô ấy làm ầm ĩ lên vì không mua được vé ngồi cạnh cửa sổ máy bay
Question 16.
Ta có thành ngữ keep track of: theo dõi
A. kept pace with: theo kịp
B. took notice of: để ý đến
C. made use of: tận dụng
D. kept track of: theo dõi
Tạm dịch. Sau vụ cướp ngân hàng, cảnh sát đã theo dõi bọn cướp và bắt cả băng nhóm.
Question 17.
Ta có thành ngữ as fresh as a daisy: tươi như hoa
A. fruit (n): quả
B. a daisy (n): hoa cúc
C. a kitten (n): mèo con
D. a maiden (n): thiếu nữ
Tạm dịch. Sau một giấc ngủ ngon, anh ấy tỉnh dậy cảm thấy rất sảng khoái và hăm hở làm việc trở lại.
Question 18.
Ta loại được A, B, D vì không hợp nghĩa
A. get up to: lên đến, lên tới
B. get on for: gần, xấp xỉ
C. get on well with: hòa thuận, ăn ý với ai
D. get in with sb: trở nên thân thiện với ai

Trang 11
Tạm dịch: Bill có vẻ không vui với công việc bởi cậu ấy làm việc không ăn ý với sếp của mình.
Question 19. Đáp án A
Câu này liên quan đến câu điều kiện.
Trong câu này ta thấy xuất hiện would have been → đây là câu điều kiện loại 3 → đáp án A sửa thành
hadn’t
Tạm dịch. Nếu như lúc đó tôi không thắt dây an toàn thì tôi đã bị thương nặng rồi.
Question 20.
Câu này là một câu trần thuật có sự xuất hiện của động từ khuyết thiếu must. Trong trường hợp này ta
thấy must = have to: phải làm gì đó, nên khi chuyển sang câu gián tiếp ta sẽ lùi thời thành had to → B sửa
thành I had to
Tạm dịch. Lễ tân nói tôi phải điền vào đơn này trước khi tham gia phỏng vấn.
❖ Note
Một số trường hợp dùng câu trần thuật có sự xuất hiện của động từ khuyết thiếu must. Trong những văn
cảnh khác nhau khi động từ must diễn tả những ý nghĩa khác nhau thì cách biến đổi trong câu trần thuật
cũng khác nhau:
- Must = have to: phải làm gì (bắt buộc) thì khi chuyển sang câu trần thuật ta dùng had to để thay thế
Ví dụ:
He said, ‘I must do the duty with care”
=> He said (that) he had to do the duty with care
- Must: thể hiện sự suy đoán thì khi chuyển câu trần thuật ta giữ nguyên
Ví dụ:
She said, “You must be very satisfied”
=> She said (that) I must be very satisfied.
- Must: dùng để đưa ra lời khuyên thì khi chuyển câu trần thuật ta cũng giữ nguyên.
Ví dụ:
“The book is useful. You must read it”, Tom said to me
=> Tom said to me (that) the book was useful and I must read it.
- Must not: thể hiện sự cấm đoán thì khi chuyển câu trân thuật ta cũng giữ nguyên
Ví dụ:
She said, “You mustn’t drive fast”
=> She said (that) I mustn’t drive fast.
Question 21.
Câu trên sử dụng sai vị trí của liên từ tương quan. Liên từ tương quan thường sử dụng theo cặp để liên kết
các cụm từ hoặc mệnh đề có chức năng tương đương nhau về mặt ngữ pháp. Trong câu này cặp liên từ
tương quan not only... but also... dùng để nối hai danh động từ training và giving (đều nằm sau giới từ
about). Vị trí chính xác phải là not only training

Trang 12
Tạm dịch, chúng tôi có một cuộc thảo luận không những về việc tập huấn nhân viên mới mà còn về việc
tạo cho họ các thử thách.
❖ For review
Một số cặp liên từ tương quan hay gặp:
- Either...or: hoặc...hoặc
Ví dụ: I want either the pizza or the sandwich: Tôi ăn pizza hoặc sandwich
- Neither...nor...:không....(cũng) không...
Ví dụ: I want neither the pizza nor sandwich: Tôi không ăn pizza cũng không ăn sandwich
- Both...and...: vừa...vừa
Ví dụ: He is both tired and hungry: Anh ấy vừa mệt vừa đói
- Not only.. .but also...: không những.. mà còn...
Ví dụ: She can speak not only English but also Japanese: Cô ấy có thể nói không những tiếng Anh mà còn
cả tiếng Nhật nữa
Question 22.
Dịch câu đề. Mary thốt lên rằng giọng của ca sĩ đó rất ngọt ngào
Dịch đáp án.
A. Giọng của cô ca sĩ đó ngọt như thế nào?- Mary nói
B. Cô ca sĩ ấy có giọng hát thật ngọt!- Mary nói.
C. Không dịch vì sai cấu trúc
D. Cô ca sỹ ấy là giọng hát thật ngọt
Trong câu đề ta thấy có động từ exclaim (v): thốt lên → đây là câu tường thuật lại cấu trúc câu cảm thán.
Trong tiếng Anh khi thể hiện cảm thán, ta có thể sử dụng cấu trúc:
- What + (a/an) + adj+ Noun+ (S+V)!
- How + adj/ adv+ S+V!
Ta thấy đáp án A sai vì đây là cấu trúc câu hỏi, không phải câu cảm thán
Đáp án C dùng câu cảm thán với How nhưng lại dùng sai cấu trúc, sau how sẽ là tính từ/ trạng từ không
phải cụm danh từ. Sweet voice là cụm danh từ (tính từ + danh từ)
Đáp án D sai vì động từ to be is làm sai nghĩa của câu. Trong trường hợp này khi dùng V là to be is →
chủ ngữ the singer và danh từ sweet voice là một đối tượng, điều này không chính xác. Động từ phù hợp
trong trường hợp này là các động từ chỉ sở hữu như has hoặc possesses... vì sweet voice thuộc sở hữu của
the singer
Question 23.
Dịch câu đề. Nếu không có ca phẫu thuật đặc biệt này thì anh ta đã không sống được
Dịch đáp án.
A. Nếu không nhờ ca phẫu thuật đặc biệt, thì anh ta đã sống sót
B. Nhờ ca phẫu thuật đặc biệt này, đáng lẽ anh ta đã sống sót

Trang 13
C. Nếu không nhờ ca phẫu thuật đặc biệt, thì anh ta đã sống sót
D. Anh ta đã không sống được nếu không nhờ có ca phẫu thuật đặc biệt này.
Ta thấy trong câu đề xuất hiện cấu trúc without + N, S + would not have P2. Đây là cấu trúc diễn tả điều
kiện loại 3.
Khi diễn tả điều kiện loại 3, ta có thể sử dụng các cấu trúc sau:
- If S had P2, S would/ could/ might have P2
- Had S (not) P2, Swould/ could/ might have P2
- If it hadn’t been for sb/ st, S+ would/ could/ might have P2
= Had it not been for sb/ st, S + would/ could/ might have P2
= But for sb/ st, S+ would/ could/ might have P2 = nếu không vì/ nếu không nhờ...
Như vậy ta thấy đáp án A đúng cấu trúc nhưng diễn đạt sai ý câu gốc
Đáp án B sai vì ta thấy câu gốc dùng câu điều kiện loại 3 → diễn tả sự việc không có thật trong quá khứ,
trong quá khứ anh ta sống sót nhờ ca phẫu thuật đặc biệt này. Vì vậy khi dùng với cấu trúc with + N, S+
V: nhờ có... mà...- diễn tả điều có thật, với trường hợp này là điều có thật trong quá khứ thì động từ ở
mệnh đề chính sẽ chia ở quá khứ đơn
Đáp án C sai vì đúng cấu trúc nhưng diễn đạt sai ý câu gốc
Question 24.
Dịch câu đề. Tình huống quá bối rối đến nỗi cô ấy chẳng biết phải làm gì
Dịch đáp án.
A. Nó là một tình huống quá bối rối, tuy nhiên cô ấy đã không biết phải làm gì
B. Tình huống quá bối rối đến nỗi cô ấy không biết phải làm gì
C. Không dịch vì sai cấu trúc
D. Cô ấy không biết phải làm gì mặc dù đó không phải là một tình huống bối rối.
Ta thấy đây là câu hỏi liên quan đến mệnh đề chỉ nguyên nhân kết quả với so... that và such... that: quá...
đến nỗi...
* For review
Để diễn tả mối quan hệ nguyên nhân kết quả, ta có thể sử dụng các cấu trúc sau:
- Cấu trúc với so... that:
* S + v+ SO+ adj/ adv that S + V
* S + V + SO+ adj+ a/ an+ N that S + V
* Đảo ngữ với so: So+ adj+ be+ S that S+V hoặc So + adv+ do/does/did/can...+ S + V that S+ V
- Cấu trúc với such... that:
* S + V + such + (a/an) + adj+ N that S + V
Như vậy ta thấy đáp án A sai vì diễn đạt sai nghĩa của câu
Đáp án C sai vì dùng sai cấu trúc đảo ngữ với so
Đáp án D sai vì diễn đạt sai ý của câu gốc

Trang 14
Question 25. Đáp án A
Dịch câu đề. Laura tập chơi nhạc cụ rất chăm chỉ. Cô ấy không thể cải thiện màn trình diễn của mình
Dịch đáp án.
A. Dù Laura có luyện tập chơi nhạc cụ nhiều thế nào, cô ấy vẫn không thể biểu diễn tốt hơn.
B. Ngay khi Laura tập chơi nhạc cụ chăm chỉ, cô ấy đã có thể biểu diễn tốt hơn.
C. Nếu Laura luyện tập chơi nhạc cụ chăm chỉ, cô ấy đã có thể biểu diễn tốt hơn nhiều
D. Ngay khi Laura luyện tập chơi nhạc cụ chăm chỉ thì cô ấy có thể cải thiện màn trình diễn của mình
Ở đây ta thấy câu đề xuất hiện hai thông tin tương phản đối lập nhau: practiced a lot: luyện tập nhiều ><
hardly improve: không cải thiện mấy → ta sẽ dùng các cấu trúc của mệnh đề nhượng bộ để nối 2 vế câu →
đáp án A đúng
❖ For review
Khi diễn đạt mối quan hệ tương phản đối lập giữa
2 vế câu, ta có thể sử dụng các cấu trúc nhượng bộ:
- Although/ though/ even though+ S1 + V1, S2+ V2: Mặc dù...
- Despite/ In spite of + N/ (not) V-ing, S2+ V2
- Despite/ In spite of + the fact that S1 + V1, S2+ V2
- Adj/ Adv+ as/ though + S1 + V1, S2+ V2
- However adj/ adv = No matter how adj/ adv = It doesn’t/ didn’t matter how adj/ adv + S1 + V1, S2+ V2
Trong trường hợp này, ta thấy đáp án A đang sử dụng cấu trúc nhượng bộ bắt đầu bằng however.
Đáp án B không chính xác do chuyến tải không đúng ý nghĩa câu gốc và mệnh đề thời gian as soon as
chưa thể hiện được quan hệ tương phản giữa 2 vế câu
Đáp án C không đúng vì sử dụng cấu trúc đảo ngữ của câu điều kiện loại 3, tuy nhiên lại diễn đạt sai ý
câu gốc
Đáp án D không chính xác vì diễn đạt sai ý câu gốc
Question 26.
Dịch câu đề. Cậu bé bị mất một số ngón tay vì pháo hoa. Các bác sỹ đang phẫu thuật cho cậu ấy
Dịch đáp án.
A. Không dịch vì sai cấu trúc.
B. Không dịch vì sai cấu trúc
C. Không dịch vì sai cấu trúc.
D. Các bác sỹ đang phẫu thuật cho cậu bé bị mất một số ngón tay vì pháo hoa
Ta thấy câu trên đang được viết lại bằng cách sử dụng mệnh đề quan hệ.
Đáp án A sai vì operating phải ở dạng bị động là operated do lúc này tân ngữ him đã trở thành chủ ngữ
the boy ở câu mới
Đáp án B sai vì ta đã có từ quan hệ whom làm tân ngữ rồi nên không còn him nữa

Trang 15
Đáp án C sai vì thừa was và him, do câu đã có động từ chính làm vị ngữ là lost và từ quan hệ làm tân ngữ
là whom
Question 27.
Bill và Jerry đang ở một quán cà phê
Bill: Tớ lấy thêm ly nước nữa cho cậu nhé
Jerry:_____________
A. Không, không đúng
B. Không phải bây giờ
C. Không, tớ sẽ cân nhắc
D. Quên nó đi
Trong câu trên ta thấy bắt đầu bằng Can I- cấu trúc đề nghị làm việc gì đó. Trong 4 đáp án trên thì có đáp
án B là phù hợp để phản hồi lại cho lời đề nghị.
Question 28. Đáp án A
Mary và con trai đang ở trong bếp.
Mary: Cẩn thận không là con sẽ bị đau đấy.
Con trai:______________
A. Vâng con sẽ không làm thế nữa
B. Ý kiến hay đấy ạ.
C. Cảm ơn mẹ!
D. Con hoàn toàn đồng ý với mẹ.
Ta thấy câu trên bắt đầu bằng watch out- cẩn thận- đây chính là cấu trúc cảnh báo. Để đáp lại lời cảnh
báo, ta sẽ sử dụng cấu trúc có các từ khuyết thiếu như will hoặc should thể hiện việc tiếp thu ý kiến. Nên
đáp án đúng là A.
Đáp án B thường dùng để phản hồi cho lời gợi ý
Đáp án C có thể dùng để cảm ơn
Đáp án D dùng để tán thành ý kiến
Question 29. Đáp án C
Đáp án C đúng vì ta có score (v) = gain (v): ghi bàn, ghi điểm, giành thắng lợi. Đây là câu hỏi tìm từ
đồng nghĩa nên chọn đáp án C
A. have (v): có, sở hữu
B. take (v): lấy, chiếm
D. fulfill (v): hoàn thành
Tạm dịch. Trò chơi của cuộc đời cũng giống như một trận đấu bóng. Bạn phải xử lý vấn đề, ngăn nỗi sợ
hãi và ghi điểm khi có cơ hội
Question 30. Đáp án A

Trang 16
Đáp án A đúng vì ta có raise a ballyhoo about it = make a fuss: làm ầm ĩ, rùm beng. Đây là câu hỏi tìm từ
đồng nghĩa nên ta chọn phương án A
B. appeased (v): an ủi, dỗ dành
C. applauded (v): khen ngợi
D. complained (v): phàn nàn, than phiền
Tạm dịch. Vì có một mẩu giấy trong bát súp nên người phụ nữ lớn tuổi làm ầm lên trong nhà hàng
Question 31. Đáp án A
Đáp án A đúng vì ta có impediment (n): chướng ngại vật, trở ngại >< advantage (n): thuận lợi. Đây là câu
hỏi tìm từ trái nghĩa nên ta chọn phương án A
B. obstacle (n): vật cản
C. barrier (n): thanh chắn
D. disadvantage (n): bất lợi
Tạm dịch. Tình trạng thể chất của anh ấy không phải là một trở ngại cho sự nghiệp là một nghệ sỹ chơi vĩ
cầm của mình. Anh ấy đã dành được rất nhiều giải thưởng
Question 32. Đáp án B
Đáp án B đúng vì ta có didn’t bat an eyelid: không bất ngờ >< showed surprise: bất ngờ
Đây là câu hỏi tìm từ trái nghĩa nên ta chọn phương án B
A. không nói câu gì
C. không quan tâm
D. từ chối lắng nghe
Tạm dịch. Cô ấy nói với anh ta rằng cô ấy đã tiêu hết số tiền tiết kiệm của mình nhưng anh ta không ngạc
nhiên.
Question 33. Đáp án D
Ta loại đáp án B, C vì không hợp nghĩa. Loại A do supposed thường đi với that + mệnh đề
A. supposed (v): cho là, cho rằng
B. realized (v): nhận ra
C. held (v): cầm, tổ chức
D. regarded (v): coi như
In ancient Greece, people who had tattoos were regarded as members of the upper classes.
Tạm dịch. Ở Hy Lạp cổ đại, những người có hình xăm được coi là những người thuộc tầng lớp thượng
lưu
Question 34. Đáp án A
Ở đây ta cần một đại từ quan hệ thay thế cho the early Christians- danh từ chỉ người → giữ lại A và B.
Trong câu này ta thấy danh từ và mệnh đề quan hệ được ngăn cách bởi dấu phẩy → đây là mệnh đề quan
hệ không xác định → không thể dùng từ quan hệ that do từ quan hệ that dùng cho mệnh đề quan hệ xác
định → Chọn A

Trang 17
On the other hand, tattooing was banned in Europe by the early Christians, who thought that it was a
sinful thing to do.
Tạm dịch. Tuy nhiên ở châu Âu, xăm mình bị coi là điều cấm kị với người Cơ Đốc, những người cho
rằng đó là một điều tội lỗi
Question 35. Đáp án D
Ta thấy sau chỗ trống cần điền từ là một cụm từ chỉ thời gian..that + mệnh đề → chọn đáp án D. Ta có
cụm từ It was not until + từ chỉ thời gian + that + S + V mãi đến khi... thì....
It was not until the late 18th century, when Captain Cook saw South Sea Islander decorating their bodies
with tattoos that attitudes began to change.
Tạm dịch. Mãi đến cuối thế kỷ 18, khi thuyền trưởng Cook thấy người dân trên đảo Nam Hải trang trí cơ
thể của họ bằng hình xăm, thì quan niệm về xăm mình mới bắt đầu thay đổi
Question 36. Đáp án C
Ở đây ta có cụm từ gain in popularity: trở nên phổ biến → chọn đáp án C
Sailors came back from these islands with pictures of Christ on their backs and from then on, tattooing
gained in popularity.
Tạm dịch. Từ khi những thủy thủ trở về từ hòn đảo này với hình xăm của chúa Jesu trên lưng thì xăm
mình trở nên rất phổ biến
Question 37. Đáp án C
Ta thấy sau chỗ trống có giới từ about → chọn C. Ta loại A và D vì A và D đi cùng giới từ of. Loại B vì
trouble đi với giới từ for
Các đáp án khác không chính xác.
A. danger (n): nguy hiểm
B. trouble (n): rắc rối
C. concern (n): mối quan tâm, sự lo lắng
D. threat (n): mối đe dọa
There is also some concern about catching a blood disease from unsterilized needles.
Tạm dịch. Ngoài ra còn có một số lo ngại về việc bị nhiễm một số căn bệnh về máu do kim xăm chưa
được tiệt trùng
Question 38.
Câu hỏi. Theo đoạn văn, hành tinh nào tỏa sáng nhất
A. Trái Đất B. Sao Mộc
C. Sao Kim D. Sao Hỏa
Ta tìm thấy thông tin trong đoạn 1 dòng 2: It is well placed for observation for several months in every
year and on average is the brightest of the planets apart from Venus.
Tạm dịch. Nó được nhìn thấy rõ nét trong trong vài tháng mỗi năm và tính trung bình, đó là hành tinh
sáng nhất nếu không tính Sao Kim

Trang 18
Question 39. Đáp án B
Câu hỏi. Từ “it” trong đoạn 2 đề cập đến
A. Sao Mộc B. Điểm Đỏ
C. cường độ D. màu sắc
Đây là 1 câu hỏi suy luận. Ta tìm được thông tin trong đoạn 2, dòng 1-2:
The most famous mark on Jupiter is the Great Red Spot. It has shown variations in both intensity and
color, and at times it has been invisible, but it always returns after a few years.
Đại từ được sử dụng để thay thế cho danh từ đứng trước nó. Trong toàn bộ câu thứ 2 ta thấy đều có chung
một chủ ngữ it. Liên hệ với câu văn trước ta thấy it chính là Great Red Spot → đáp án B đúng
Tạm dịch. Đặc điểm nổi bật nhất trên sao Mộc là Điểm Đỏ khổng lồ. Nó đã cho thấy các biến thể về
cường độ và màu sắc, và đôi khi nó vô hình, nhưng nó luôn xuất hiện trở lại sau một vài năm.
Question 40. Đáp án D
Câu hỏi. Từ “exceptional” ở đoạn 2 có nghĩa____________
A. extreme (adj): cực đoan
B. sustainable (adj): bền vững
C. temporary (adj): tạm thời
D. infrequent (adj): không thường xuyên, ít xảy ra
Ta có từ exceptional (adj): hiếm có, không bình thường → gần nghĩa với đáp án D
Chúng ta tìm thấy thông tin trong đoạn 2, 2 dòng cuối
Its longevity may well due to its exceptional size, but there are signs that it is decreasing in size, and it
may not be permanent.
Tạm dịch. Tuổi thọ của nó cũng có thể do kích thước đặc biệt của nó, nhưng có những dấu hiệu cho thấy
nó đang giảm về kích thước, và nó có thể không vĩnh viễn.
Question 41. Đáp án D
Câu hỏi. Theo đoạn văn, điều nào sau đây không đúng
A. Sao Mộc lớn hơn tất cả các hành tinh khác trong hệ mặt trời
B. Một ngày ở sao Mộc kéo dài gần 10 giờ
C. Điểm Đỏ di chuyển nhiều theo chiều dọc hơn chiều ngang
D. Các nhà khoa học có bằng chứng cho thấy các đốm đỏ nhỏ hơn đang gia tăng kích thước của chúng để
trở thành các Điểm Đỏ khác.
Để trả lời câu hỏi này, ta có thể dùng phép loại trừ
Chúng ta tìm thấy thông tin của đáp án A ở đoạn 1 dòng 1:
Jupiter is the largest and most massive planet and is fifth in order of distance from the sun
Tạm dịch: Sao Mộc là hành tinh lớn nhất và đứng thứ 5 theo khoảng cách từ mặt trời
Đáp án B được tìm thấy trong đoạn 1 dòng 3-4: Jupiter’s less than 10- hour rotation period gives it the
shortest day in the solar system

Trang 19
Tạm dịch. Chu kỳ một vòng quay chưa đến 10 giờ của sao Mộc làm cho nó trở thành hành tinh có ngày
ngắn nhất trong hệ mặt trời
Đáp án C được tìm thấy trong đoạn 2, dòng 4-5: Over the past century the total longitudinal drift has
amounted to approximately 1200°.
Tạm dịch. Trong thế kỷ qua, nó trôi dạt theo chiều dọc tổng cộng đã lên tới xấp xỉ 1200°.
Hoặc ta tìm thấy thông tin đáp án D không chính xác trong đoạn 2 dòng cuối:
Several smaller red spots have been seen occasionally but have not lasted.
Tạm dịch. Một số đốm đỏ nhỏ hơn thi thoảng được nhìn thấy nhưng không kéo dài
Như vậy có nghĩa các đốm đỏ nhỏ không to lên để trở thành các điểm đỏ khác → chọn đáp án D
Question 42. Đáp án B
Câu hỏi. Đoạn văn có thể được lấy từ
A. một tạp chí nghệ thuật
B. một tạp chí địa chất
C. sách giáo khoa trung học
D. một cuốn sách khảo cổ học
Để trả lời câu hỏi này ta dựa vào các thông tin lặp đi lặp lại trong đoạn văn (redundancy) và các trường từ
vựng xuất hiện trong bài. Ta thấy bài nhắc đến nhiều lần tên các vì sao như Jupiter, Venus, Mars, Great
Spots trong đó nội dung chủ yếu xoay quanh sao Mộc và các Điểm Đỏ trên sao Mộc → đây là những nội
dung liên quan đến địa chất → đáp án B đúng
Question 43. Đáp án A
Câu hỏi: Ý chính của văn bản là gì?
A. Sự thành công của Mickey Mantle và cuộc sống riêng tư đầy những vấn đề
B. Mickey Mantle là cầu thủ bóng chày vĩ đại nhất mọi thời đại
C. Mickey Mantle và lịch sử bóng chày
D. Mickey Mantle và sự nghiệp chơi bóng chày của mình
Câu hỏi hỏi ý chính vì vậy chúng ta cần đọc toàn bộ bài và tập trung vào câu văn đầu tiên mỗi đoạn văn.
Ta thấy trong 3 đoạn văn đầu tiên, những thành công trong sự nghiệp bóng chày của Mantle đã được đề
cập đến và 2 đoạn văn cuối cùng nói về cuộc sống riêng tư đầy rắc rối của ông → đáp án A đúng
Question 44. Đáp án B
Câu hỏi. Từ đoạn 1 có thể suy ra rằng Mantle___________
A. đưa môn bóng chày tới nước Mỹ
B. kiếm được nhiều tiền từ bóng chày
C. đã phải cố gắng rất nhiều để trở thành cầu thủ chuyên nghiệp
D. chơi cho New York Yankees suốt cuộc đời của ông
Ta tìm thấy thông tin câu trả lời ở đoạn 1 dòng cuối:
For many people, Mantle symbolized the hope, prosperity, and confidence of America at that time.

Trang 20
Tạm dịch: Đối với nhiều người, Mantle tượng trưng cho niềm hy vọng, sự thịnh vượng và sự tự tin của
nước Mỹ vào thời đó.
Ta thấy đáp án A và C không được đề cập trong đoạn 1
Thông tin của phương án D thì không chính xác, trong đoạn văn ta thấy ông chỉ chơi cho Yankees trong
thời kỳ hoàng kim của đội bóng (in their years of glory) chứ không phải suốt cuộc đời ông.
Cuối đoạn 1 ta thấy xuất hiện từ khóa prosperity (n): sự thịnh vượng → có thể suy luận đáp án B chính
xác
Question 45.
Câu hỏi. Từ “this” trong đoạn 2 đề cập đến___________
A. Mantle được nhiều người hâm mộ
B. Mantle là một vận động viên tuyệt vời
C. Mantle là một “kẻ tấn công”
D. Mantle là một cầu thủ nhanh và mạnh
Ta tìm thấy trông tin ở đoạn 2 dòng 2-3:
He was a wonderful athlete, but this alone g; cannot explain Americas fascination with him
Đại từ this được sử dụng để thay thế cho thông tin đã xuất hiện phía trước nó. Trong câu này thông tin
được nối với nhau bằng liên từ but → có mối liên hệ giữa 2 thông tin này, mà thông tin đã xuất hiện ở
trước chính là đáp án B
Tạm dịch. Ông là một vận động viên tuyệt vời nhưng điều này không thể giải thích hết niềm dam mê của
người Mỹ đối với ông
Question 46. Đáp án A
Câu hỏi. Từ “fulfill” trong đoạn 4 gần nghĩa nhất với____________
A. đạt được những gì mong muốn, chờ đợi
B. làm gì đó theo cách bạn được bảo phải làm
C. làm những gì đã hứa hoặc đồng ý làm
D. đến gần hơn với những gì bạn đang theo
Ta tìm thấy thông tin trong đoạn 4 dòng 2
He lived to fulfill his father’s dreams.
Tạm dịch: Ông sống để hoàn thành giấc mơ của cha mình.
Ta thấy trong thông tin tìm được có từ khóa fulfill dreams- ta có thể dễ dàng suy luận nghĩa của fulfill
trong trường hợp này chính là hoàn thành, đạt được điều gì → phù hợp nhất là đáp án A
Question 47. Đáp án A
Câu hỏi. Trong đoạn văn đâu là nguyên nhân chính phá hủy cơ thể Mantle ?
A. Cách sống của ông
B. Sự cô đơn của ông
C. Giấc mơ riêng của ông

Trang 21
D. Cuộc phẫu thuật ghép gan
Ta tìm thấy thông tin trong đoạn 4 dòng 2 “He lived to fulfill his fathers dreams and drank to forget his
father s early death”.
Tạm dịch. Ông sống để hoàn thành giấc mơ của cha và ngập chìm trong rượu để quên đi cái chết của cha
mình.
Và thông tin trong đoạn 5 dòng 1 “It was a terrible addiction that finally destroyed his body”.
Tạm dịch. Nghiện rượu khủng khiếp cuối cùng đã phá hủy cơ thể của ông ta
It trong câu đầu đoạn văn chính là thay thế cho việc drank trong đoạn 4, và addiction- sự nghiện ngập đã
cho thấy đó là lối sống không lành mạnh và là lí do phá hủy cơ thể ông ấy → đáp án đúng là A
Question 48. Đáp án D
Câu hỏi. Từ “cirrhosis” trong đoạn 5 có thể có nghĩa là____________
A. mối nguy hiểm
B. cuộc phẫu thuật
C. sự phá hủy
D. cuộc phẫu thuật ghép gan
Ta tìm thấy thông tin trong đoạn 5 dòng 1-2
It was a terrible addiction that finally destroyed his body. It gave him cirrhosis of the liver and accelerated
the advance of liver cancer.
Tạm dịch: Nghiên rượu khủng khiếp đã phá hủy cơ thể ông ta. Nó khiến ông ta bị xơ gan và đẩy nhanh
quá trình ung thư gan.
Theo thông tin ta tìm thấy, it ở câu thứ 2 thay thế cho danh từ addiction ở câu đầu. Ta có thể suy luận việc
addiction (nghiện rượu) gây ra cirrhosis, và từ cirrhosis sau này thành ung thư → có thể suy luận
cirrhosis là một căn bệnh nguy hiểm → đáp án D đúng
Cirrhosis (n): bệnh xơ gan
Question 49.
Câu hỏi: Tất cả những điểu sau đều đúng trừ____________
A. ông sinh ra trong một gia đình nghèo nhưng sau đó đã thành công và nổi tiếng
B. ông bị nghiện rượu quá nặng đến mức không thể bỏ được cho đến ngày cuối đời
C. ông chơi thành thạo hai tay
D. cái chết của bố đã có ảnh hưởng xấu đến ông
Câu hỏi này ta có thể dùng phép loại trừ
Đáp án A được đề cập trong đoạn 3 dòng 1-3
Perhaps it was because he was a handsome, red-haired country boy, the son of a poor miner from
Oklahoma. His career, from the lead mines of the West to the heights of success and fame.
Tạm dịch. Có lẽ đó là bởi vì ông là một cậu bé vùng nông thôn tóc đỏ đẹp trai, con của một thợ mỏ nghèo
ở Oklahoma. Sự nghiệp của ông, từ mỏ dầu phía Tây bước lên đỉnh cao danh vọng..

Trang 22
Đáp án C được đề cập trong đoạn 2 dòng 1: Mantle was a fast and powerful player, a “switch-hitter” who
could bat both right- handed and left-handed.
Tạm dịch. Mantle là 1 cầu thủ nhanh và mạnh, “1 kẻ tấn công” có thể đánh cả tay phải lẫn trái
Đáp án D được đề cập trong đoạn 4 dòng 2-3: drank to forget his fathers early death- uống để quên cái
chết của cha. Và như ta phân tích ở câu hỏi 48, việc này khiến ông nghiện rượu và bị xơ gan
→ Đáp án B không đúng. Ta có thể tìm thấy thông tin trong đoạn 5 dòng 2-3: Even when Mickey Mantle
had turned away from his old life and warned young people not to follow his example, the destructive
process could not be stopped.
Tạm dịch. Ngay cả khi Mickey Mantle từ bỏ lối sống cũ và cảnh báo những người trẻ không theo gương
mình thì quá trình phá hủy của bệnh tật không thể dừng lại. Như vậy có nghĩa ông ấy bỏ rượu nhưng quá
muộn
Question 50. Đáp án A
Câu hỏi. Từ đoạn văn có thể suy ra đối với hầu hết người Mỹ____________
A. Những thành công trong sự nghiệp của Mantle thật khó tin
B. Mantle gặp nhiều khó khăn để nổi tiếng và thành công
C. thành công trong sự nghiệp của Mantle là khác tự nhiên
D. Mantle phải tập luyện rất vất vả đế trở thành một vận động viên giỏi
Chúng ta tìm thấy thông tin trong đoạn 3 dòng 1-3:
His career, from the lead mines of the West to the heights of success and fame, was a fairy-tale version of
the American dream.
Tạm dịch: Sự nghiệp của ông, từ mỏ dầu phía Tây bước lên đỉnh cao danh vọng, là một phiên bản cổ tích
của giấc mơ Mỹ.
Từ fairy-tale cho thấy thành công của ông là điều gì đó khó tin, hiếm có như truyện cổ tích → đáp án A
đúng.

Trang 23
ĐỀ SỐ 08 ĐỀ THI THỬ TỐT NGHIỆP THPT
NĂM HỌC: 2020 – 2021
MÔN: TIẾNG ANH
Thời gian làm bài: 60 phút; không kể thời gian phát đề

Mark the letter A, B, C or D on you answer sheet to indicate the word whose underlined part
differs from the other three in pronunciation in each of the following questions.
Question 1. A. whispered B. wandered C. sympathized D. sentenced
Question 2. A. compile B. facile C. facsimile D. textile
Mark the letter A, B, C, or D on your answer sheet to indicate the word that differs from the other
three in the position of the primary stress in each of the following questions.
Question 3. A. carpet B. country C. idea D. volume
Question 4. A. familiar B. uncertainty C. impatient D. arrogantly
Mark the letter A, B, C, or D on your answer sheet to indicate the correct answer to each of the
following questions.
Question 5. I demand to know how this vase_____________, and no one is leaving till I find out.
A. got broken B. was breaking C. has broken D. is broke
Question 6. The more you practice speaking in public, _____________.
A. the more you become confident B. the more you become confidently
C. the greater confidence you become D. the more confident you become
Question 7. It is said the Robinhood robbed_____________rich and gave the money to poor.
A. a/ a B. a/ the C. the/ the D. the/ a
Question 8. Not only_____________among the largest animals that ever lived, but they are also among
the most intelligent.
A. some whales B. they are whales C. whales D. are whales
Question 9. _____________, he couldn’t finish that test in 60 minutes.
A. Intelligent as was the boy B. As intelligent the boy was
C. As the boy was intelligent D. Intelligent as the boy was
Question 10. There is no excuse for your late submission! You_____________the report by last
Monday.
A. should have finished B. mightn’t have finished
C. needn’t have finished D. must have finished
Question 11. My parents lent me the money. _____________, I couldn’t have afforded the trip.
A. Therefore B. Otherwise C. Only if D. However
Question 12. The biologists have found more than one thousand types of butterflies in the forest,
_____________its special characteristics.

Trang 1
A. each one has B. which has C. each having D. having
Question 13. John is feeling_____________because he hasn’t had enough sleep recently.
A. irritate B. irritation C. irritably D. irritable
Question 14. Jenny has an_____________command of Japanese cuisine.
A. intensive B. utter C. impressive D. extreme
Question 15. why are you always so jealous_____________other people?
A. in B. of C. below D. on
Question 16. The head teacher has asked me to take the new boy under my_____________and look
after him.
A. sleeves B. arm C. wing D. cloak
Question 17. The online game “Dumb ways to die” quickly_____________with young people after
being released in 2013.
A. took on B. caught up C. caught on D. took up
Question 18. Just think! Next month you’ll be_____________and it seems like only yesterday you
were a baby.
A. in your teens B. in your teenage C. teenager D. at your teens
Mark the letter A, B, C, or D on your answer sheet to indicate the underlined part that needs
correction in each of the following questions.
Question 19. The children are extremely excited about the visit to the town where their grandparents
were born in.
A. about B. were C. where D. the town
Question 20. Chandler was shocked when his entire class seemed to come down with the same
imaginative disease.
A. was shocked B. entire class C. come down with D. imaginative disease
Question 21. Transgender people are denied the ability to join the armed forces due to discriminating
policies implemented by the government.
A. Transgender B. are denied C. armed forces D. discriminating
Mark the letter A, B, C or D on your answer sheet to indicate the sentence that is closest in
meaning to each of the following questions.
Question 22. “Please, let my child go!” she begged the kidnapper.
A. She begged the kidnapper to let her child to go.
B. She pleaded with the kidnapper to release her child.
C. She solemnly ordered the kidnapper to set her child free.
D. She pleaded the kidnapper to let her child go.
Question 23. Sometimes having professional assistance with your CV can increase your chance of
finding a job.

Trang 2
A. Having your CV professionally prepared determines whether you'll get the job or not.
B. The prospects for finding employment can possibly be enhanced by a professionally produced CV.
C. Having a professional help you with your CV is a sure way to find a job.
D. Without a professional CV, it is impossible to find a job
Question 24. It seems to me that we’ve taken the wrong train.
A. The trained turned out to be not the one we were supposed to have taken.
B. There is no chance that we’ll catch the train that we’re supposed to.
C. I have a feeling that this train is not the one we should be on.
D. I wish we had been more careful and taken the right train from the station.
Mark the letter A, B, C or D on your answer sheet to indicate the sentence that best combines
each pair of sentences in the following questions.
Question 25. She was the first woman in the Philippines. She was elected as the president of the
country.
A. She was the first woman who is elected as the president of the Philippines.
B. She was the first woman elected as the president of the Philippines.
C. She was the first woman being elected as the president of the Philippines.
D. She was the first woman to be elected as the president of the Philippines.
Question 26. They are my two sisters. They aren’t teachers like me.
A. Unlike me, neither of my two sisters aren’t teachers.
B. They are my two sisters, both of those are teachers like me.
C. They are my two sisters, neither of whom is teacher like me.
D. They are my two sisters who neither are teachers like me.
Mark the letter A, B, C or D on your answer sheet to indicate the most suitable response to
complete each of the following exchanges.
Question 27. Tony and Bob are talking in their classroom.
Tony: “We are buying Lily a graduation present.” - Bob: “_____________”
A. She’s out of my league. B. Can you all be more down-to-earth?
C. I’m raking in money now. D. Could I chip in?
Question 28. Mai and Joey are talking about their favorite pastimes.
Joey: “What sort of things do you like doing in your free time?” - Mai: “_____________”
A. I love checking out the shops for new clothes.
B. None. Been starved since 9 yesterday.
C. I hate shopping.
D. Nothing special. Just some photos I took on the trip to Nepal.
Mark the letter A, B, C, or D on your answer sheet to indicate the word or phrase that is
CLOSEST in meaning to the underlined part in each of the following questions.

Trang 3
Question 29. Some operations may have to be halted unless more blood donors come forward to help.
A. offer B. claim C. attempt D. refuse
Question 30. The natives were angry when foreigners came to their country and took over their land.
A. migrants B. tourists C. members D. locals
Mark the letter A, B, C, or D on your answer sheet to indicate the word or phrase that is
OPPOSITE in meaning to the underlined part in each of the following questions.
Question 31. Later a wine reception will be followed by a concert before guests tuck into a banquet.
A. a formal party B. a formal conference
C. an informal party D. an enormous breakfast
Question 32. The nominating committee always meet behind closed doors, lest its deliberations
become known prematurely.
A. dangerously B. safely C. privately D. publicly
Read the following passage and mark the letter A, B, C or D on your answer sheet to indicate the
correct word for each of the blanks from 33 to 37.
Left-handers are the odd ones out. Sure, lefties (33) _____________up about 10 percent of the
population - but, frankly, it seems like society has forgotten about them. Just consider all of the right-
handed gadgets, awkwardly designed desks, and cooking tools that fit comfortably only in your right
hand, what (34) _____________someone to become a lefthand? Scientists aren’t exactly sure, but
research points to a complex (35) _____________between genes and environment.
While no exact set of “leftie genes” have been discovered, people who dominantly use their left hands
do have more left-handed family members. And researchers have found different brain wirings in
righties vs. lefties. But no matter (36) _____________it is that drives someone to use their antipodal
paw, science has also uncovered a particular set of personality traits that lefthanded people tend to
have. So for all of you lefties, leftie-loving righties, and ambidextrous folks out there - it’s time to
brush up on your left-handed knowledge and help (37) _____________an end to leftie discrimination
once and for all.
Question 33. A. consist B. account C. hold D. make
Question 34. A. causes B. makes C. gets D. does
Question 35. A. collaborate B. collaboration C. collaborating D. collaborated
Question 36. A. which B. who C. what D. that
Question 37. A. put B. bring C. make D. take
Read the following passage and mark the letter A, B, C or D on your answer sheet to indicate the
correct answer to each of the questions.
Trees are useful to man in three very important ways: they provide him with wood and other products,
they give him shade, and they help to prevent drought and floods.

Trang 4
Unfortunately, in many parts of the world man has not realized that the third of these services is the most
important. In his eagerness to draw quick profit from the trees, he has cut them down in large numbers,
only to find that without them he has lost the best friends he had.
Two thousand years ago a rich and powerful country cut down its trees to build warships, with which to
gain itself an empire. It gained the empire but, without its trees, its soil became hard and poor. When the
empire fell to pieces, the country found itself faced by floods and starvation.
Even though a government realizes the importance of a plentiful supply of trees, it is difficult for it to
persuade the villager to see this. The villager wants wood to cook his food
with, and he can earn money by making charcoal or selling wood to the townsman. He is usually too lazy
or too careless to plant and look after trees. So unless the government has a good system of control, or can
educate the people, the forests will slowly disappear.
This does not only mean that there will be fewer trees. The results are even more serious. For where there
are trees their roots break the soil up, allowing the rain to sink in and also bind the soil, thus preventing it
being washed away easily, but where there are no trees, the soil becomes hard and poor. The rain falls on
hard ground and flows away on the surface, causing floods and carrying away with it the rich topsoil, in
which crops grow so well, when all the topsoil is gone, nothing remains but a worthless desert.
Question 38. In the writer's opinion, _____________, or the forests slowly disappear.
A. people shouldn't draw benefit from the tree
B. measures must be taken
C. government must realize the serious results
D. unless trees never be cut down
Question 39. The word “bind ” in the passage probably means___________
A. to make wet B. to wash away C. to make stay together D. to improve
Question 40. In the last two paragraphs the writer wanted to make it clear that____________
A. where there are no trees, the soil becomes hard and poor
B. where there are no trees, the land might become desert slowly
C. where there are many trees, there are fewer floods
D. floods will make the land become desert
Question 41. It’s a great pity that in many places____________
A. man is not eager to make profits from trees
B. man hasn’t found out that he has lost all trees
C. man hasn’t realised the importance of trees to him
D. man is only interested in building empire
Question 42. Trees are useful to man mainly in three ways, the most important of which is that they
can ____________
A. keep him from the hot sunshine B. make him draw quick profit from them

Trang 5
C. enable him to build warships D. protecthim from drought and floods
Read the following passage and mark the letter A, B, C or D on your answer sheet to indicate the
correct answer to each of the questions.
PERCEPTIONS OF ANIMALS ACROSS CULTURES
When living and working in another country, there are numerous things to consider apart from the more
obvious ones of climate, language, religion, currency, etc. Some important considerations are less
obvious. For example, do you have a pet or do you enjoy a hobby such as horse riding? Your animal or
hobby may be perceived in a completely different light in another culture so it’s important to consider the
significance given to specific animals in different parts of the world and general perceptions towards
them.
One example which is often mentioned in popular press is the case of dogs. In some cultures,
like the US or UK, dogs are loved and considered a great pet to have at home and with the family. In
other cultures, such as those where Islam is the majority religion, dogs may be perceived as dirty or
dangerous. Muslims treatment of dogs is still a matter of debate amongst Islamic scholars, while these
animals are widely considered by many Western cultures to be “mans best friend”, the Koran describes
them as “unhygienic”. Muslims will therefore avoid touching a dog unless he can wash his hands
immediately afterwards, and they will almost never keep a dog in their home.
In Iran, for instance, a cleric once denounced “the moral depravity” of dog owners and even demanded
their arrest. If you are an international assignee living and working in Saudi Arabia or another Arabic
country, you should remember this when inviting Arab counterparts to your house in case you have a dog
as a pet. This is just one example of how Islam and other cultural beliefs can impact on aspects of
everyday life that someone else may not even question. A Middle Eastern man might be very surprised
when going to Japan, for instance, and seeing dogs being dressed and pampered like humans and carried
around in baby prams!
Dogs are not the only animals which are perceived quite differently from one culture to another. In India,
for example, cows are sacred and are treated with the utmost respect. Conversely in Argentina, beef is a
symbol of national pride because of its tradition and the high quality of its cuts. An Indian working in
Argentina who has not done his research or participated in a cross cultural training programme such as
Doing Business in Argentina may be surprised at his first welcome dinner with his Argentinean
counterparts where a main dish of beef would be served.
It is therefore crucial to be aware of the specific values assigned to objects or animals in different cultures
to avoid faux-pas or cultural misunderstandings, particularly when living and working in another culture.
Learning how people value animals and other symbols around the world is one of the numerous cultural
examples discussed in Communicaid’s intercultural training courses. Understanding how your
international colleagues may perceive certain animals can help you ensure you aren’t insensitive and it
may even provide you with a good topic for conversation.

Trang 6
(Source: https. //www.communicaid.com)
Question 43. What does the author suggest in the last paragraph?
A. Talking about different perceptions with others will help you overcome insensitivity.
B. To avoid cultural shocks, people should not live or work in another culture.
C. It’s important to value the objects or animals in different countries before going there.
D. Understanding different perceptions of animals will help you avoid faux-pas in another nation.
Question 44. According to paragraph 2, which sentence is INCORRECT?
A. The dog is a typical example of different views in the world about animals.
B. Dogs are well-treated and loved in the US and UK.
C. Muslims are those considering dogs as their best pets at home.
D. People whose religion is Islam don’t like having dogs in their home.
Question 45. The word “unhygienic” in the second paragraph is closest in meaning to____________
A. unhealthy B. undependable C. unreliable D. unacceptable
Question 46. What does the word “this” in paragraph 3 refer to?
A. you are an international assignee
B. you are having a dog as pet
C. a cleric once denounced the moral depravity of dog owners and even demanded their arrest
D. you are living and working in Saudi Arabia or another Arabic country
Question 47. The author mentioned cows in paragraph 4 as an example of______________
A. the animals that are differently perceived in numerous cultures
B. sacred animals in Argentina
C. a symbol of a nation for its high quality of nutrients
D. which may cause surprise for Argentinian people at dinner
Question 48. which of the following could be the main idea of the passage?
A. Perceptions of animals across cultures
B. What should be learnt before going to another country
C. Dogs and different beliefs in the world
D. Muslims and their opinions about animals
Question 49. The word “pampered” in the third paragraph could be best replaced by____________
A. indulged B. taken care of C. made up D. respected
Question 50. It can be inferred from the passage that______________
A. people will change their perceptions of animals when living in another culture
B. you should not be surprised if other counterparts consider your sacred animals as food
C. there are many things to research before going to live and work in another country
D. respecting other cultures is a good way to have a successful life abroad

Trang 7
Đáp án
1-D 2-C 3-C 4-D 5-A 6-D 7-C 8-D 9-D 10-A
11-B 12-C 13-D 14-C 15-B 16-C 17-C 18-A 19-C 20-D
21-D 22-B 23-B 24-C 25-D 26-C 27-D 28-A 29-A 30-D
31-C 32-D 33-D 34-A 35-B 36-C 37-A 38-B 39-C 40-B
41-C 42-D 43-D 44-C 45-A 46-C 47-A 48-A 49-B 50-C

LỜI GIẢI CHI TIẾT


Question 1: Đáp án D
Đáp án D đúng vì phần gạch chân của đáp án D được đọc là /t/. Các phương án còn lại phần gạch chân
được đọc là /d/.
A. whispered /ˈwɪspə(r)d/ (v)/(n): nói thì thầm (v)/ tiếng xì xào (n)
B. wandered /ˈwɒndə(r)d/ (v): đi lang thang
C. sympathized /ˈsɪmpəθaɪzd/ (v): cảm thông
D. sentenced /ˈsentənst/ (v)/(n): kết án (v)/ câu văn (n)
* Note: Chúng ta cần ôn tập lại quy tắc phát âm đuôi “-ed”
-ed được phát âm là: Khi tận cùng của từ là: Ví dụ
/ɪd/ /t/ hoặc /d/ decided, accepted, visited
/t/ Các phụ âm vô thanh: /s/, /ʃ/, /tʃ/, missed, stopped, washed, ranked,
/k/, /p/, /f/, /θ/ approached, sentenced
/d/ Các âm hữu thanh còn lại involved, played, whispered,
wandered, sympathized

Question 2: Đáp án C
Đáp án C đúng vì phần gạch chân của đáp án C được đọc là /əli/. Các phương án còn lại có phần gạch
chân được đọc là /aɪl/.
A. compile /kəmˈpaɪl/ (v): biên soạn
B. facile /ˈfæsaɪl/ (adj): dễ dàng, đơn giản
C. facsimile /fækˈsɪməli/ (n): bản sao, bản chép
D. textile /ˈtekstaɪl/ (n): vải dệt
Question 3. Đáp án C
Đáp án C đúng vì đáp án D trọng âm rơi vào âm thứ hai. Các phương án còn lại trọng âm rơi vào âm đầu
tiên.
A. carpet /ˈkɑːpɪt/ (n)/(v): cái thảm (n)/ trải thảm, chỉ trích (v)
B. country /ˈkʌntri/ (n): đất nước
C. idea /aɪˈdɪə/ (n): ý tưởng, quan điểm
D. volume /ˈvɒljuːm/ (n): âm lượng, số lượng

Trang 8
Question 4. Đáp án D
Đáp án D đúng vì đáp án D trọng âm rơi vào âm thứ nhất. Các phương án còn lại trọng âm rơi vào âm thứ
hai.
A. familiar /fəˈmɪliə(r)/ (adj)/(n): thân mật, quen biết (adj)/ người thân cận (n)
B. uncertainty /ʌnˈsɜːtnti/ (n): tính không chắc chắn
C. impatient /ɪmˈpeɪʃnt/ (adj): sự nóng nảy, mất kiên nhẫn
D. arrogantly /ˈærəɡəntli/ (adv): một cách kiêu ngạo
Question 5.
Kiến thức được hỏi - Bị động với get
Tobe chỉ đi với phân từ 2 nếu câu đó mang nghĩa bị động hoặc đi với V-ing nếu câu đó chia ở thì tiếp diễn
→ loại D vì tobe không đi với động từ chia quá khứ như vậy.
Break là ngoại động từ nên đằng sau nó là một tân ngữ, nếu không có tân ngữ thì câu đó phải ở dạng bị
động. Hay có thể hiểu một cách khác, chủ ngữ trong câu là this vase không thể tự gây ra hành động break
được, nên nó phải ở câu bị động → loại B và C vì đang ở dạng chủ động. Như vậy ta chọn được đáp án A
- got broken - bị vỡ.
Tạm dịch: Tôi cần phải biết chiếc bình này bị vỡ như thế nào, và không một ai rời đi cho đến khi tôi phát
hiện ra.
* Mở rộng:
- Get có thể được theo sau một số tính từ để đưa ra ý kiến mang tính thay đổi
Ex: I stopped working because I got sleepy (Tôi dừng làm việc bởi vì đang dần trở nên buồn ngủ).
- Get còn có thể được theo sau một phân từ 2 mang nghĩa bị động. Phân từ 2 này đóng vai trò như một
tính từ miêu tả cho chủ thể: get dressed, get drunk, get involved, get married,...
Ex: The eggs got broken. (Trứng đã bị vỡ). [ = were broken]
Question 6. Đáp án D
Kiến thức được hỏi - so sánh đồng tiến
Ta có công thức so sánh đồng tiến như sau:
The + comparative + S + V, the + comparative + S + V
Become là một linking verb nên đằng sau phải là một tính từ → loại B và C
Ta đảo so sánh hơn của tính/trạng từ lên trước thì tính từ confident phải đứng sau more → loại A
→ Chọn đáp án D, đúng công thức đảo so sánh hơn của tính từ lên trước.
Tạm dịch: Bạn luyện nói trước công chúng càng nhiều thì bạn càng trở nên tự tin.
Question 7. Đáp án C
Kiến thức được hỏi - Mạo từ
Ta có cách sử dụng đặc biệt với mạo từ The-. The + adj = N(s) - chỉ một lớp người → Chọn đáp án C
Tạm dịch: Người ta nói Robinhood cướp của những người giàu và chia tiền cho những người nghèo.
Question 8. Đáp án D

Trang 9
Kiến thức được hỏi - Đảo ngữ
Thông thường những từ mang nghĩa phủ định đứng đầu câu sẽ xuất hiện đảo ngữ. Cụ thể trong câu này là
đảo ngữ với cụm Not only....but also...: Not only + Auxiliary + S + V...., but S also + V.....
Trong đó Not only, but also dùng để nối và nhấn mạnh hai thành phần ngữ pháp giống nhau về vị trí, chức
năng.
Ở đây trợ động từ chính là động từ tobe are vì sau là cụm giới từ among the largest.. .ever lived.
Loại A, B, C vì không có đảo ngữ. Chọn đáp án D
Tạm dịch: Cá voi không chỉ nằm trong số động vật lớn nhất đã từng tồn tại mà chúng còn nằm trong số
động vật thông minh nhất.
Question 9. Đáp án D
Kiến thức được hỏi - Cấu trúc đặc biệt với mệnh đề nhượng bộ
Ta có thể loại được B luôn vì không tồn tại cấu trúc As + adj + S + V
Phương án C là cấu trúc của mệnh đề chỉ nguyên nhân nhưng xét về nghĩa thì không phù hợp → Loại C
Ta có một số cấu trúc đặc biệt diễn đạt sự nhượng bộ:
1. No matter + what/who/when/where/why + S + V
2. No matter + how + adj/adv + S + V
3. Whatever (+ noun)/ whoever/ wherever/ whenever + S + V
4. However + adj/adv + S + V
5. Adj/ adv + as/though + S + V
Như vậy, loại được A vì không sử dụng đảo ngữ với cấu trúc (5). → Chọn đáp án D
Tạm dịch: Mặc dù cậu bé rất thông minh nhưng cũng không thể hoàn thành được bài kiểm tra trong vòng
60 phút.
Question 10. Đáp án A
Kiến thức được hỏi - động từ khuyết thiếu dạng hoàn thành (Modal perfect)
Ta có cách sử dụng của một số Modal perfect sau:
- Should have P2: được dùng để diễn đạt một điều gì đó có lẽ nên hoặc phải xảy ra nhưng đã không xảy ra
trong quá khứ (đáng lẽ nên làm gì mà lại không làm). Shouldn’t have + P2 được dùng để diễn đạt điều gì
đó lẽ ra không nên xảy ra nhưng đã xảy ra trong quá khứ.
- May/ might/ could have + P2 được dùng để diễn đạt điều gì đó có thể xảy ra hoặc có thể đúng trong quá
khứ.
- Can't/ couldn't have + P2 được dùng để diễn đạt điều gì đó chắc chắn không thể xảy ra trong quá khứ.
- Must have + P2 được dùng để suy luận về một điều trong quá khứ, thường được dịch là ắt hẳn là....
- Needn’t have + P2 diễn tả một điều gì đó đáng lẽ không cần làm gì nhưng lại làm trong quá khứ
Dựa vào nghĩa của câu ta chọn được đáp án A.
Tạm dịch: Không có một lời biện minh nào cho việc nộp muộn của em! Em đáng lẽ nên hoàn thành bản
báo cáo này từ thứ 2 tuần trước rồi.

Trang 10
Question 11. Đáp án B
Kiến thức được hỏi - Từ nối
Therefore: cho nên, vì vậy - trạng tù nối hai câu chỉ kết quả
Otherwise: nếu không thì - trạng từ nối hai câu, dùng để diễn đạt điều kiện
Only if: chỉ khi - liên từ, nối hai mệnh đề, dùng trong mệnh đề trạng ngữ chỉ điều kiện.
Nếu đứng đầu câu sẽ xuất hiện đảo ngữ. → loại C vì Only if nối hai mệnh đề, không nối hai câu.
However: tuy nhiên - trạng từ nối 2 câu chỉ sự tương phản.
Câu trước là lent me the money - cho tôi mượn tiền và câu sau là couldn’t have afford the trip - đã không
thể có được chuyến đi → hai câu mang nghĩa điều kiện → Chọn đáp án B
Tạm dịch: Bố mẹ cho tôi mượn tiền. Nếu không thì tôi đã không thể có được chuyến đi đó.
Question 12. Đáp án C
Kiến thức được hỏi - mệnh đề phân từ
Ta thấy câu đề bài đã có đầy đủ chủ vị: S - The biologists; V - have found; O- more than one thousand
types of butterflies in the forest
Như vậy loại A vì nếu ghép A vào thì sẽ thành hai câu độc lập được nối với nhau bằng dấu phẩy → sai
quy định về ngữ pháp Tiếng Anh.
Loại B vì nếu ghép B vào thì sẽ thành câu có mệnh đề quan hệ; nhưng which ở đây không hợp lý vì which
thay thế cho the forest hay cả một mệnh đề phía trước đều không hợp nghĩa.
Nếu muốn thêm động từ “have” phía sau thì phải nối bằng các liên từ hoặc để ở dạng rút gọn bằng mệnh
đề phân từ.
Trong câu không có liên từ nên trường hợp này loại. Xét đến trường hợp rút gọn bằng mệnh đề phân từ
thì phải cùng chủ ngũ nhưng trong câu này hai chủ thể của hai mệnh đề khác nhau (không cùng chủ ngữ)
thì liệu ta có rút gọn được không????
Đây là một kiến thức đặc biệt, ngoài việc có thể rút gọn hai câu cùng chủ ngữ thì ta vẫn có thể rút gọn
được hai câu không cùng chủ ngữ dùng phân từ/tính từ/cụm giới từ. Kiểu rút gọn bằng mệnh đề phân từ
này còn được gọi là cấu trúc tuyệt đối, thường được dùng trong văn chương và thơ. Nó có thể đứng trước,
sau hay chen vào giữa câu mà không làm ảnh hưởng đến cấu trúc tổng thể của câu. Nếu câu mang nghĩa
chủ động thì ta có cấu trúc: N/Pronoun + V-ing
Nếu câu mang nghĩa bị động thì ta có cấu trúc: N/Pronoun + P2
Như vậy ta chọn được đáp án C.
Tạm dịch: Các nhà sinh học tìm thấy hơn một nghìn loài bướm trong khu rừng này, mỗi một loài bướm
đều có những đặc điểm riêng biệt.
* Mở rộng: Dưới đây là một vài ví dụ để chúng ta hiểu hơn về kiến thức này
- Câu mang nghĩa chủ động
The girl stood all by herself. Her hair fluttered in the wind (The girl - her hair - 2 chủ ngữ khác nhau)

Trang 11
→ The girl stood all by herself, her hair FLUTTERING in the wind.(Cô gái đứng một mình, tóc dài bay
trong gió.)
It was a fine day yesterday. I took my son fishing. (It -1 - 2 chủ ngữ khác nhau)
→ It BEING a fine day yesterday, I took my son fishing. (Hôm qua là một ngày đẹp trời, tôi đã đưa con
trai đi câu cá)
- Câu mang nghĩa bị động
He sat all by by himself in the room. His back was turned to the window. (He - his back - khác chủ ngữ)
→ He sat all by by himself in the room, his back TURNED to the window.
- Ngoài ra, N/Pronoun+ cụm giới từ/tính từ
A girl came in, book in hand.
He was waiting, his eyes on her back.
Her determination stronger than ever, Nexisa resolved not to give up until she had achieved her dreams.
Lưu ý: Với cấu trúc này thường thì 2 vế phải khác chủ ngữ và có dấu phẩy để chúng ta nhận biết.
Question 13. Đáp án D
Kiến thức được hỏi - Loại từ
Ta thấy linking verb feel được theo sau bởi tính từ loại A - động từ, loại B - danh từ, loại C - trạng từ
Chọn đáp án D - tính từ
Irritate (v): làm cho tức giận
Irritation (n): sự tức giận
Irritably (adv): một cách tức giận
Irritable (adj): dễ tức giận
Tạm dịch: John đang cảm thấy rất dễ tức giận bởi vì gần đây anh ấy thiếu ngủ.
Question 14.
Kiến thức được hỏi - Collocation
Chỗ trống cần một từ phù hợp về nghĩa và sự kết hợp từ. Ta có cụm have an impressive command of sth -
có kiến thức chuyên sâu/ sâu sắc về cái gì.
Intensive (adj): chuyên sâu
Utter (adj)/(v): hoàn toàn, tuyệt đối (adj)/ nói ra, phát biểu (v)
Extreme (adj)/(n): cực đoan, cùng cực (adj)/ mức độ, thái cực (n)
Impressive (adj): ấn tượng, chuyên sâu
Tạm dịch: Jenny có kiến thức chuyên sâu về ẩm thực Nhật Bản
Question 15.
Kiến thức được hỏi - giới từ
Ta có jealous ofsb/sth - ghen tị về ai/cái gì
Các giới từ còn lại không đi với tính từ jealous.
Tạm dịch: Tại sao bạn lại luôn ghen tị về những người khác?

Trang 12
Question 16.
Kiến thức được hỏi - Thành ngữ
Ta có thành ngữ take sb under your wing - bảo vệ, che chở, bao bọc cho ai đó
Sleeves (n): tay áo
Arm (n)/(v): cánh tay (n)/ cấp vũ khí (v)
Cloak (n)/(v): áo choàng (n)/ che giấu (v)
Tạm dịch: Giáo viên chủ nhiệm yêu cầu tôi bảo vệ và chăm sóc cho cậu bé mới đến.
Question 17. Đáp án C
Kiến thức được hỏi - Phrasal verb
Ta có một số cụm động từ sau:
Catch up with sb: gây ra vấn đề cho ai đó, trừng phạt ai đó
Catch on with: được ưa chuộng, phổ biến
Take up with sb: trở nên thân thiện với ai đó
Không có cụm take on with như phương án A
Dựa vào ngữ nghĩa của câu ta chọn được đáp án C.
Tạm dịch: Game trực tuyến “Dumb ways to die” nhanh chóng được ưa chuộng với các bạn trẻ sau khi
được tung ra vào năm 2013.
Question 18. Đáp án A
Kiến thức được hỏi - Lựa chọn từ
Loại được B vì teenage là tính từ không thể đứng sau tính từ sở hữu your được.
Loại tiếp được C vì không phù hợp về nghĩa
Và khi muốn nói ai đó trong độ tuổi teen (từ 13-19) thì ta dùng giới từ in → chọn đáp án A
Tạm dịch: Hãy nghĩ xem! Tháng tới bạn sẽ bước vào giai đoạn tuổi teen rồi đấy và nó dường như mới
hôm qua thôi bạn vẫn còn là một đứa trẻ.
Question 19.
Lỗi sai - Trạng từ quan hệ
Ta thấy Where. (= in/at which): được dùng thay cho danh từ chỉ nơi chốn. Như vậy where thay thế cho
in/at which; nếu đã dùng trạng từ quan hệ where rồi thì không dùng in/at which. Trong trường hợp giới từ
không được chuyển lên trước thì phải dùng đại từ quan hệ which/That.
Sửa: where → which/that
Tạm dịch: Những đứa trẻ rất hào hứng với chuyến đi về quê nơi mà ông bà chúng được sinh ra.
Question 20. Đáp án D
Lỗi sai - Cách dùng từ
Trong câu này nếu chưa thể chọn ngay được đáp án thì có thể dùng phương pháp loại trừ, loại đi những
phương án đúng; rõ ràng A, B và C đều không có vấn đề gì về mặt ngữ pháp và từ vựng. Từ đó suy ra
chọn được đáp án D. Đây là lỗi sai về cách dùng từ không đúng với ngữ cảnh. Imaginative disease - căn

Trang 13
bệnh giàu trí tưởng tượng; chúng ta không thể nói như vậy mà phải chuyển sang một tính từ mang nghĩa
khác imaginary disease - căn bệnh hoang tưởng thì mới phù hợp với nghĩa của câu.
Tạm dịch: chandler sốc khi toàn bộ lớp học của anh ấy cùng mắc một căn bệnh hoang tưởng.
Question 21. Đáp án D
Lỗi sai - Cách dùng từ
Trong câu này nếu chưa thể chọn ngay được đáp án thì có thể dùng phương pháp loại trừ, loại đi những
phương án đúng; rõ ràng A, B và C đều không có vấn đề gì về mặt ngữ pháp và từ vựng. Từ đó suy ra
chọn được đáp án D. Đây là lỗi sai về cách dùng từ không đúng với ngữ cảnh.
Discrimanating (adj): biết suy xét, có nhận thức sáng suốt → nếu làm tính từ cho policies thì không phù
hợp với nghĩa của câu. Ta phải sửa sang một tính từ khác đó là discrimanatory - phân biệt đối xử để phù
hợp với nghĩa của câu.
Tạm dịch: Những người chuyển giới bị từ chối tham gia vào quân ngũ bởi vì những chính sách phân biệt
đối xử của chính phủ.
Question 22. Đáp án B
Dịch câu đề: “Làm ơn hãy để cho con tôi đi!” cô gái cầu xin kẻ bắt cóc.
A. Không dịch vì sai ngữ pháp chỗ “to go” vì let + O + V bare.inf
B. Cô gái cầu xin kẻ bắt cóc thả con của cô ấy ra. → đúng nghĩa, đúng ngữ pháp.
Ta có cấu trúc plead with sb to V- cầu xin ai đó làm gì
C. Cô gái trang nghiêm ra lệnh cho kẻ bắt cóc giải thoát cho con mình. → sai nghĩa so với câu gốc.
D. Không dịch vì sai ngữ pháp ở chỗ thiếu giới từ with trước the kidnapper (theo cấu trúc được đề cập ý
B)
→ Chọn đáp án B
Question 23. Đáp án B
Dịch câu đề: Đôi khi có sự tư vấn chuyên nghiệp về CV của bạn có thể gia tăng cơ hội tìm việc làm.
A. Có một CV được chuẩn bị một cách chuyên nghiệp quyết định việc bạn có tìm được việc hay không →
sai nghĩa so với câu gốc.
B. Khả năng tìm được việc làm có thể được gia tăng bằng một CV được làm một cách chuyên nghiệp. →
đúng.
C. Có một sự giúp đỡ chuyên nghiệp về CV của bạn là một cách chắc chắn để tìm được việc làm. → sai
nghĩa so với câu gốc.
D. Không có một CV chuyên nghiệp, bạn sẽ không thể tìm được việc làm. → sai nghĩa so với câu gốc.
→ Chọn đáp án B
Question 24. Đáp án C
Dịch câu đề: Tôi cảm thấy rằng dường như chúng ta đã đi sai chuyến tàu.
A. Người được đào tạo hóa ra không phải là người mà chúng tôi phải có. → sai nghĩa so với câu gốc.

Trang 14
B. Không có cơ hội nào để chúng tôi bắt được chuyến tàu mà chúng tôi phải đi. → sai nghĩa so với câu
gốc
C. Tôi có cảm giác rằng chuyến tàu này không phải là chuyến tàu chúng tôi nên đi. → đúng
D. Tôi ước chúng tôi đã cẩn thận hơn và đi đúng chuyến tàu từ nhà ga. → sai nghĩa so với câu gốc.
→ Chọn đáp án C
Question 25. Đáp án D
Dịch câu đề: Cô ấy là người phụ nữ đầu tiên ở Phi-lip-pin. Cô ấy được bầu làm tổng thống của đất nước
này.
A. Không dịch vì sai ngữ pháp ở chỗ động từ tobe “is”, không bảo toàn thì so với câu gốc.
B và C không dịch vì sai ngữ pháp: Mệnh đề quan hệ xác định có thể được rút gọn bằng cụm động từ
nguyên mẫu. To infinitive có thể được dùng sau các từ first, second,..., last, next, only và dạng so sánh
nhất . Danh từ được thay thế phía trước có chứa first nên không thể được rút gọn bằng P2 và being P2 như
B và C được.
Chọn đáp án D, đúng cấu trúc rút gọn mệnh đề quan hệ trong trường hợp danh từ được thay thế phía trước
có chứa first, second,...
Question 26. Đáp án C
Dịch câu đề: Họ là hai chị gái của tôi. Họ không phải là giáo viên giống tôi
A. Không giống tôi, không một ai trong hai chị gái của tôi không là giáo viên, (ngụ ý cả hai chị gái đều là
giáo viên) → sai nghĩa so với câu gốc.
B. Không dịch vì sai ngữ pháp: Trong mệnh đề quan hệ không xác định, các cụm từ chỉ số lượng all of,
most of, neither of, many of, both of...có thể được dùng với whom nếu danh từ được thay thế phía trước
chỉ người, dùng với which nếu danh từ được thay thế phía trước chỉ vật. B sử dụng those là sai ngữ pháp.
C. Họ là hai chị gái của tôi, không một ai trong hai người là giáo viên giống tôi. → đúng nghĩa, đúng ngữ
pháp
D. Không dịch vì sai ngữ pháp ở chỗ who neither
→ Chọn đáp án C
Question 27. Đáp án D
Dịch để: Tony và Bob đang nói chuyện trong lớp.
Tony: “Chúng tớ đang định mua cho Lily một món quà tốt nghiệp”
Bob: “____________”
A. Cô ấy không thuộc tẩm của tôi.
B. Các cậu có thể thực tế hơn không?
C. Mình đang kiếm nhiều tiền bây giờ
D. Mình có thể góp tiền với không?
Như vậy đáp án D là phù hợp với ngữ cảnh.
* Mở rộng:

Trang 15
Out of one’s league (idiom): quá tốt, quá đắt, ngoài khả năng, ngoài đẳng cấp của ai
Down-to-earth (adj): thực tế
Rake in sth (ph.v): kiếm nhiều tiền
Chip in (ph.v): góp chung tiền
Question 28. Đáp án A
Dịch đề: Mai và Joey đang nói chuyện về sở thích.
Joey: “ Bạn thích làm gì trong thời gian rảnh rỗi?”
Mai: “_____________”
A. Mình thích ghé qua các cửa hàng xem quần áo
B. Chưa ăn gì. Mình bị đói từ 9 giờ ngày hôm qua
C. Mình không thích mua sắm
D. Không có gì đặc biệt. Chỉ là mấy tấm ảnh mình chụp trong chuyên đi tới Nepal.
Như vậy chỉ có đáp án A là phù hợp ngữ cảnh
Question 29. Đáp án A
Ta có come forward – đề nghị giúp đỡ. Ta chọn được A là từ gần nghĩa nhất với từ đề bài cho
Offer (v): đề nghị
Claim (v): quả quyết, khẳng định, đòi quyền
Attempt (v) + To inf: cố gắng, nỗ lực làm gì
Refuse (v) + To inf: từ chối làm gì
Tạm dịch: Một số cuộc phẫu thuật có thể phải dừng lại nếu như không có nhiều người hiến máu đề nghị
giúp đỡ.
Question 30. Đáp án D
Native - người bản địa. Ta chọn ngay được D - locals là từ gần nghĩa nhất với từ đề bài cho.
Migrant (n): người di cư
Tourist (n): khách du lịch
Member (n): thành viên
Local (n): người địa phương, người bản địa
Tạm dịch: Những người bản địa rất giận dữ khi người nước ngoài đến đất nước của họ và châm chiếm
đất.
Question 31. Đáp án C
Banquet - bữa tiệc trang trọng. Ta chọn đáp án C - bữa tiệc thân mật là từ trái nghĩa với từ đề bài cho.
a formal party: bữa tiệc trang trọng
a formal conference: hội nghị trang trọng
an informal party: bữa tiệc thân mật
an enormous breakfast: bữa sáng thịnh soạn

Trang 16
Tạm dịch: Sau đó, việc mời rượu sẽ để sau khi bản hòa nhạc kết thúc nhưng trước khi khách mời bắt đầu
tham gia vào bữa tiệc trang trọng.
Question 32. Đáp án D
Behind closed doors (idiom): kín, không công khai. Ta chọn được đáp án D - công khai là từ trái nghĩa
với từ đề bài cho.
Dangerously (adv): một cách nguy hiểm
Safely (adv): một cách an toàn
Privately (adv): một cách riêng tư
Publicly (adv): một cách công khai.
Tạm dịch: ủy ban đề cử luôn họp kín kẻo để lộ ra thông tin sớm.
Question 33. Đáp án D
Ta thấy đằng sau chỗ trống là số phần trăm nên có thể đoán là chiếm bao nhiêu phần trăm. Để diễn đạt
điều này có một Phrasal verb đó là make up - chiếm. Phương án A và B không đi với giới từ up.
Hold up : vẫn giữ vững, làm trì trệ, làm tắc nghẽn → loại vì không phù hợp về nghĩa.
Consist thường đi với giới từ of: bao gồm
Account thường đi với giới từ for: chiếm
→ Chọn đáp án D
Trích bài: Left-handers are the odd ones out. Sure, lefties make up about 10 percent of the population -
but, frankly, it seems like society has forgotten about them.
Tạm dịch: Những người thuận tay trái là những người khác biệt. Chắc chắn rằng những người thuận tay
trái chiếm 10% dân số - nhưng thẳng thắn mà nói, có vẻ như xã hội đã quên về họ.
Question 34. Đáp án A
Ta thấy sau someone là to become (to V) → loại B vì make + O + Vbare inf và loại được cả D vì không có
cấu trúc này với do.
Ta có cấu trúc với get ở thể sai khiến get + O(person) + to V + O. Tuy nhiên xét về nghĩa thì không thể nói
nhờ ai đó trở thành người thuận tay trái được → loại C. Như vậy chọn đáp án A, ta có cấu trúc cause sb to
do sth - khiến ai đó làm việc gì.
Trích bài: Just consider all of the right- handed gadgets, awkwardly designed desks, and cooking tools
that fit comfortably only in your right hand, what causes someone to become a lefthand?
Tạm dịch: Chỉ cần xem xét tất cả các đổ dùng sử dụng tay phải, ví như những chiếc bàn được thiết kế một
cách vụng về và những dụng cụ nấu ăn mà chỉ phù hợp với tay phải. Vậy điều gì khiến cho một người trở
nên thuận tay trái?
Question 35. Đáp án B
Ta thấy đằng trước chỗ trống là mạo từ a và tính từ complex → Chỗ trống cần một danh từ → Do vậy
chọn được đáp án B.
Collaborate (v): cộng tác, kết hợp

Trang 17
Collaboration (n): sự cộng tác, sự kết hợp
Collaborated (V-ed)
Trích bài: Scientists aren’t exactly sure, but research points to a complex collaboration between genes
and environment.
Tạm dịch: Các nhà khoa học thì không chắc chắn về điều này nhưng nghiên cứu chỉ ra một sự kết hợp
giữa gen và môi trường.
Question 36. Đáp án C
Ta loại ngay được đáp án D vì that không đi với cấu trúc no matter
Ta thấy đằng sau có chủ ngữ it - ám chỉ vật, sự vật, sự việc nên không thể dùng who
Which dùng để hỏi có sự lựa chọn còn What được dùng đế hỏi khi không biết số lượng cụ thể hay nói
cách khác là ở một số lượng nhiều, không xác định. Căn cứ vào cách sử dụng và nghĩa ta chọn được đáp
án C. Mặt khác, no matter what là một thành ngữ mang nghĩa cho dù là gì
Trích bài: While no exact set of “leftie genes” have been discovered, people who dominantly use their left
hands do have more left-handed family members. And researchers have found different brain wirings in
righties vs. lefties. But no matter what it is that drives someone to use their antipodal paw, science has
also uncovered a particular set of personality traits that left-handed people tend to have.
Tạm dịch: Trong khi không có một bộ gen chính xác nào cho những người thuận tay trái thì những người
sử dụng tay trái lại có nhiều thành viên trong gia đình thuận tay trái hơn. Và các nhà nghiên cứu đã tìm ra
các dây thần kinh khác nhau ở tay phải và tay trái. Nhưng cho điều gì có thúc đẩy con người sử dụng bàn
tay đối diện thì khoa học cũng khám phá ra một đặc điểm tính cách đặc biệt mà những người thuận tay
trái có xu hướng có.
Question 37.
Ta có thành ngữ put an end to sth - chấm dứt, kết thúc cái gì
Do vậy chọn đáp án A
Trích bài: So for all of you lefties, leftie- loving righties, and ambidextrous folks out there - it’s time to
brush up on your lefthanded knowledge and help put an end to leftie discrimination once and for all.
Tạm dịch: Vì vậy, đối với tất cả những người thuận tay trái, những người thích dùng tay trái và những
người thuận cả hai tay - đây là lúc nâng cao kiến thức về thuận tay trái và chấm dứt mãi mãi sự phân biệt
đối xử với người thuận tay trái.
Question 38. Đáp án B
Dịch đề. Theo ý kiến của tác giả,______________, hoặc những khu rừng sẽ dần biến mất.
A. Con người không nên kiếm lợi từ cây cối
B. Các biện pháp phải được thực hiện
C. Chính phủ phải nhận ra những hậu quả nghiêm trọng.
D. Trừ phi cây cối không bao giờ bị chặt.
Ta thấy thông tin ở câu cuối cùng của đoạn 4.

Trang 18
Trích bài: So unless the government has a good system of control, or can educate the people, the forests
will slowly disappear.
Tạm dịch: Vì vậy nếu chính phủ không có một hệ thống kiểm soát tốt hoặc giáo dục người dân thì các
khu rừng sẽ dần biến mất
→ Xét 4 phương án ta chọn được đáp án B, ngụ ý cần phải có biện pháp được thực hiện, có thể có một hệ
thống kiểm soát hoặc giáo dục người dân.
Question 39. Đáp án C
Dịch đề. Từ “bind” trong đoạn văn có nghĩa_____________
A. Làm ẩm ướt
B. Rửa trôi
C. Làm cho liên kết với nhau
D. Cải thiện
Bind (v) = make stay together: liên kết lại làm cho mạnh mẽ, cứng cáp hơn. → Chọn đáp án C
Trích bài: For where there are trees their roots break the soil up, allowing the rain to sink in and also bind
the soil, thus preventing it being washed away easily, but where there are no trees, the soil becomes hard
and poor.
Tạm dịch: Đối với những nơi có cây cối, rễ của chúng xới đất, cho phép nước mưa chìm xuống và liên kết
đất; do đó ngăn chặn việc nó bị cuốn trôi một cách dễ dàng, nhưng những nơi không có cây, đất trở nên
khô cứng và bạc màu.
Question 40. Đáp án B
Dịch đề. Trong 2 đoạn văn cuối cùng tác giả muốn làm rõ điều______________
A. Những nơi không có cây, đất trở nên khô cứng và bạc màu.
B. Những nơi không có cây, đất dần biến thành sa mạc.
C. Những nơi có nhiều cây thì có ít lũ lụt.
D. Lũ lụt làm cho đất thành sa mạc.
Trích bài:
Đoạn 4: So unless the government has a good system of control, or can educate the people, the forests
will slowly disappear - Vì vậy nếu chính phủ không có một hệ thống kiểm soát tốt hoặc giáo dục người
dân thì các khu rừng sẽ dần biến mất
Đoạn 5: For where there are trees their roots break the soil up, allowing the rain to sink in and also bind
the soil, thus preventing it being washed away easily, but where there are no trees, the soil becomes hard
and poor. The rain falls on hard ground and flows away on the surface, causing floods and carrying away
with it the rich topsoil, in which crops grow so well, when all the topsoil is gone, nothing remains but a
worthless desert. – Đối với những nơi có cây cối, rễ của chúng xới đất, cho phép nước mưa chìm xuống
và liên kết đất; do đó ngăn chặn việc nó bị cuốn trôi một cách dễ dàng, nhưng những nơi không có cây,
đăt trở nên khô cứng và bạc màu. Mưa rơi trên mặt đất cứng và chảy trên bề mặt gây ra lũ lụt và mang

Trang 19
theo lớp đất màu mỡ - nơi mà cây phát triển tốt. Khi tất cả lớp đất mặt biến mất, không còn gì ngoài một
sa mạc vô giá trị.
Logic: Nếu tiếp tục chặt phá rừng → đất trở nên khô cằn, nước mưa không chìm xuống và giữ được đất -
→ lũ lụt cuốn đi lớp đất màu mỡ → rừng biến thành sa mạc.
Phương án C không được đề cập đến Phương án A và D chỉ là một ý rất nhỏ, không phải ý mà tác giả
muốn làm rõ ở cả 2 đoạn 4, 5.
→ Chọn đáp án B.
Question 41. Đáp án C
Dịch đề. Thật đáng tiếc khi ở nhiều nơi____________
A. Con người không thiết tha kiếm lợi nhuận từ cây xanh
B. Con người chưa thấy được rằng mình đã mất đi toàn bộ cây cối
C. Con người chưa nhận ra tầm quan trọng của cây đối với mình
D. Con người chỉ quan tâm đến việc xây dựng đế chế
Trích bài: Trees are useful to man in three very important ways: they provide him with wood and other
products, they give him shade, and they help to prevent drought and floods.
Unfortunately, in many parts of the world man has not realized that the third of these services is the most
important
Tạm dịch: Cây cối rất hữu ích với con người trong 3 khía cạnh quan trọng: chúng cung cấp gỗ và các sản
phẩm khác, chúng cho bóng mát, và chúng giúp ngăn chặn hạn hán và lũ lụt. Nhưng thật đáng tiếc, ở
nhiều nơi trên thế giới con người không nhận ra rằng khía cạnh thứ 3 là quan trọng nhất.
→ Xét 4 phương án thì chỉ có đáp án C là phù hợp.
Question 42. Đáp án D
Cây cối rất hữu ích cho con người chủ yếu theo ba cách, trong đó quan trọng nhất là chúng có
thể___________
A. Giữ con người tranh khỏi ánh nắng mặt trời nóng bức
B. Làm cho con người kiếm được lợi nhuận nhanh chóng từ chúng
C. Cho phép con người chế tạo tàu chiến
D. Bảo vệ con người khỏi hạn hán và lũ lụt
Trích bài: Trees are useful to man in three very important ways: they provide him with wood and other
products, they give him shade, and they help to prevent drought and floods.
Unfortunately, in many parts of the world man has not realized that the third of these services is the most
important.
Tạm dịch: Cây xanh rất hữu ích cho con người theo ba cách rất quan trọng: chúng cung cấp cho mọi
người gỗ và các sản phẩm khác, chúng cho con người bóng mát, và giúp ngăn chặn hạn hán và lũ lụt.
Thật không may, ở nhiều nơi trên thế giới, con người đã không nhận ra rằng lợi ích thứ ba trong số đó là
quan trọng nhất.

Trang 20
→ Ta chọn được đáp án D.
Question 43. Đáp án D
Dịch đề. Tác giả đề nghị điều gì ở đoạn văn cuối cùng?
A. Bàn luận về các nhận thức khác nhau với người khác sẽ giúp bạn vượt qua sự thiếu nhạy cảm.
B. Để tránh những cú sốc văn hóa, mọi người không nên sống hoặc làm việc ở một nền văn hóa khác.
C. Điều quan trọng là phải coi trọng các đồ vật hoặc loài động vật ở các quốc gia khác nhau trước khi đến
đó.
D. Hiểu được những nhận thức khác nhau về động vật sẽ giúp bạn tránh những hành động sai lầm ở một
quốc gia khác.
Trích bài: It is therefore crucial to be aware of the specific values assigned to objects or animals in
different cultures to avoid faux-pas or cultural misunderstandings, particularly when living and working
in another culture.
Tạm dịch: Do đó, điều quan trọng là phải nhận thức được các giá trị cụ thể được gán cho các đối tượng
hoặc loài vật ở các nền văn hóa khác nhau để tránh những hành động xấu hổ hoặc sự hiểu lầm về văn hóa,
đặc biệt là khi sống và làm việc ở một nền văn hóa khác.
→ Thông tin ủng hộ đáp án D
Question 44. Đáp án C
Dịch đề. Theo đoạn văn số 2, điều nào sau đây thì KHÔNG ĐÚNG?
A. Loài chó là một ví dụ điển hình cho những quan điểm khác nhau trên thế giới về động vật.
B. Loài chó được đối xử tốt và được yêu quý ở Mỹ và Anh.
C. Các tín đồ Hồi giáo là những người coi chó là con thú cưng tốt nhất của họ ở nhà.
D. Những người có tôn giáo là Hồi giáo không thích nuôi chó trong nhà của họ.
Trích bài:
1. In some cultures, like the US or UK, dogs are loved and considered a great pet to have at home and
with the family - Ở một số nền văn hóa như Anh và Mỹ, chó đượcyêu quý và được xem như thú cưng ở
nhà. → thông tin ủng hộ B → không chọn B
2. In other cultures, such as those where Islam is the majority religion, dogs may be perceived as dirty or
dangerous - Ở những nền văn hóa khác như những người theo đạo Hồi, chó được xem là con vật rất bẩn
và nguy hiểm → suy ra họ không thích nuôi ở nhà → thông tin ủng hộ D → không chọn D
3. Your animal or hobby may be perceived in a completely different light in another culture so its
important to consider the significance given to specific animals in different parts of the world and general
perceptions towards them- Loài vật và sở thích của bạn có thể được nhận thức ở một khía cạnh khác hoàn
toàn ở một nền văn hóa khác, vì vậy nó rất quan trọng phải xem xét tầm quan trọng của một số động vật
ở các quốc gia khác nhau trên thế giới và có nhận thức chung đối với chúng.
One example which is often mentioned in popular press is the case of dogs - Một ví dụ được đề cập trên
báo chí khá phổ biến là về loài chó.

Trang 21
→ Thông tin ủng hộ A → Không chọn A
4. Muslims treatment of dogs is still a matter of debate amongst Islamic scholars - Việc đối xử với chó của
những người theo Hồi giáo vẫn là vấn đề tranh luận giữa các tín đồ Hồi giáo → Thông tin không ủng hộ
C
→ Chọn đáp án C
Question 45.
Dịch đề. Từ “unhygienic” trong đoạn số 2 gần nghĩa nhất với_____________
A. Không khỏe mạnh
B. Không đáng tin cậy
C. Không đáng tin cậy
D. Không thể chấp nhận
Unhygienic (adj): không vệ sinh, có thể gây bệnh truyền nhiễm = unhealthy (adj)
Trích bài: while these animals are widely considered by many Western cultures to be “man’s best friend”,
the Koran describes them as “unhygienic”
Tạm dịch: Trong khi loài chó được coi trọng bởi người phương Tây là một “người bạn tốt nhất” thì người
Koran lại xem chúng rất “mất vệ sinh”
Question 46. Đáp án C
Từ “this”trong đoạn 3 chỉ điều gì?
A. Bạn là một người được phái đi làm việc ở nước ngoài
B. Bạn đang có một con chó làm thú cưng
C. Một giáo sĩ đã từng tố cáo sự đồi bại đạo đức của những người nuôi chó và thậm chí còn yêu cầu bắt
giữ họ
D. Bạn đang sống và làm việc tại Ả Rập Saudi hoặc một quốc gia Ả Rập khác
Đối với câu hỏi về đại từ thay thế câu trả lời thường ở câu phía trước hoặc trong chính câu mà đại từ đó
xuất hiện. Để tìm được danh từ mà đại từ ngụ ý, ta làm như sau:
1. Tìm đại từ trong đoạn văn
2. Đọc phần đoạn văn phía trước đại từ cẩn thận
3. Tìm danh từ phù hợp ở phía trước đại từ
Trích bài: In Iran, for instance, a cleric once denounced “the moral depravity” of dog owners and even
demanded their arrest. If you are an international assignee living and working in Saudi Arabia or another
Arabic country, you should remember this when inviting Arab counterparts to your house in case you
have a dog as a pet.
Tạm dịch: Ví dụ, ở Iran, một giáo sĩ đã từng tố cáo “sự đồi bại đạo đức” của những người nuôi chó và
thậm chí còn yêu cầu bắt giữ họ. Nếu bạn là một người được phái đi làm việc ở nước ngoài, sống và làm
việc tại Ả Rập Saudi hoặc một quốc gia Ả Rập khác, bạn nên nhớ điều này khi mời các đối tác Ả Rập
đến nhà bạn trong trường hợp bạn có nuôi một con chó làm thú cưng.

Trang 22
→ Điều đã được đề cập ở phía trước chính là đáp án C.
Question 47. Đáp án A
Dịch đề. Tác giả đã đề cập đến những con bò trong đoạn 4 như một ví dụ về____________.
A. Các động vật được nhận thức khác nhau trong nhiều nền văn hóa
B. Động vật linh thiêng ở Argentina
C. Một biểu tượng của một quốc gia về chất lượng dinh dưỡng cao
D. Có thể gây bất ngờ cho người Argentina trong bữa tối
Trích bài: Dogs are not the only animals which are perceived quite differently from one culture to
another. In India, for example, cows are sacred and are treated with the utmost respect. Conversely in
Argentina, beef is a symbol of national pride because of its tradition and the high quality of its cuts.
Tạm dịch: Loài chó không phải là động vật 1 duy nhất được nhận thức hoàn toàn khác 1 nhau từ nền văn
hóa này sang nền văn hóa khác. Ví dụ, ở Ấn Độ, loài bò là linh thiêng và ; được đối xử với sự tôn trọng
tối đa. Ngược lại ở Argentina, thịt bò là biểu tượng của niềm tự hào dân tộc vì truyền thống và chất lượng
cao của thịt.
→ Như vậy, ở mỗi nền văn hóa, bò có những nhận thức khác nhau → Chọn đáp án A. Các phương án còn
lại chỉ là một ý nhỏ.
Question 48. Đáp án A
Điều nào sau đây có thể là ý chính của đoạn văn?
A. Nhận thức về động vật giữa các nền văn hóa
B. Nên học gì trước khi đến một đất nước khác
C. Chó và các tín ngưỡng khác nhau trên thế giới
D. Người Hồi giáo và ý kiến của họ về động vật
Trích bài: Your animal or hobby may be perceived in a completely different light in another culture so it’s
important to consider the significance given to specific animals in different parts of the world and general
perceptions towards them.
Understanding how your international colleagues may perceive certain animals can help you ensure you
aren’t insensitive and it may even provide you with a good topic for conversation
Tạm dịch: Loài vật và sở thích của bạn có thể được nhận thức ở một khía cạnh khác hoàn toàn ở một nền
văn hóa khác, vì vậy nó rất quan trọng phải xem xét tầm quan trọng của một số động vật ở các quốc gia
khác nhau trên thế giới và có nhận thức chung đối với chúng.
Việc hiểu cách các đồng nghiệp quốc tế của bạn có thể nhận thức về một số loài vật nhất định có thể giúp
bạn đảm bảo bạn không vô tình xúc phạm và thậm chí nó có thể cho bạn một chủ đề hay để trò chuyện.
Ý chính của toàn bài là ý mà xuyên suốt từ đầu đến cuối, không phải là một thông tin nhỏ trong bài.
Trong các đoạn đều có nhắc đến việc nhận thức về loài vật ở các nền văn hóa khác nhau, nếu làm những
câu hỏi trên thì chúng ta có thể thấy được điều đó.
→ Chọn đáp án A. Các phương án còn lại chỉ là một ý nhỏ trong bài.

Trang 23
Question 49.
Dịch đề. Từ “pampered” ở đoạn 3 có thể được thay thế bằng____________
A. Thỏa niềm dam mê
B. Chăm sóc
C. Bịa chuyện, hình thành, đền bù
D. Tôn trọng
Pamper (v) = take care of: chăm sóc
Trích bài: A Middle Eastern man might be very surprised when going to Japan, for instance, and seeing
dogs being dressed and pampered like humans and carried around in baby prams!
Tạm dịch: Một người Trung Đông sẽ rất ngạc nhiên khi đến Nhật Bản và khi nhìn thấy những chú chó
được mặc quần áo và được chăm sóc như con người và đi xung quanh chiếc xe đẩy của trẻ em.
Question 50. Đáp án C
Có thể suy ra từ đoạn văn rằng_____________.
A. Mọi người sẽ thay đổi nhận thức về động vật khi sống ở một nền văn hóa khác
B. Bạn không nên ngạc nhiên nếu các đối tác khác coi động vật linh thiêng của bạn là thức ăn
C. Có nhiều điều cần nghiên cứu trước khi chuẩn bị sống và làm việc ở nước khác
D. Tôn trọng các nền văn hóa khác là một cách tốt để có một cuộc sống thành công ở nước ngoài
Trích bài: Learning how people value animals and other symbols around the world is one of the numerous
cultural examples discussed in Communicaid’s intercultural training courses. Understanding how your
international colleagues may perceive certain animals can help you ensure you aren’t insensitive and it
may even provide you with a good topic for conversation.
Tạm dịch: Học cách mọi người coi trọng động vật và các biểu tượng khác trên khắp thế giới là một trong
nhiều ví dụ văn hóa được thảo luận trong các khóa đào tạo liên văn hóa của Communicaid. Việc hiểu
cách các đồng nghiệp quốc tế của bạn có thể nhận thức về một số loài vật nhất định có thể giúp bạn đảm
bảo bạn không vô tình xúc phạm và thậm chí nó có thể cho bạn một chủ đề hay để trò chuyện.
→ Như vậy trước khi sống và làm việc ở đất nước khác, bạn nên nghiên cứu văn hóa của đất nước đó (có
thể về động vật như trong bài này) để tránh vô tình xúc phạm và có thể có một chủ đề hay để trò chuyện.
→ Đây là một câu hỏi lấy thông tin ở trong bài để suy luận một điều có logic → Xét 4 phương án chỉ có C
là hợp lý.

Trang 24
ĐỀ SỐ 09 ĐỀ THI THỬ TỐT NGHIỆP THPT
NĂM HỌC: 2020 – 2021
MÔN: TIẾNG ANH
Thời gian làm bài: 60 phút; không kể thời gian phát đề

Mark the letter A, B, C, or D on your answer sheet to indicate the word whose underlined part
differs from the other three in pronunciation in each of the following questions.
Question 1. A. chemistry B. Christmas C. machine D. headache
Question 2. A. honest B. honey C. hour D. heir
Mark the letter A, B, C, or D on your answer sheet to indicate the word that differs from the other
three in the position of primary stress in each of the following questions.
Question 3. A. digest B. cactus C. camel D. human
Question 4. A. compliment B. heartbroken C. labour-saving D. short-sighted
Mark the letter A, B, C, or D on your answer sheet to indicate the correct answer to each of the
following questions.
Question 5. I have bought a present for my brother, and now I need some____________.
A. paper wrapper B. wrap paper C. wrapped paper D. wrapping paper
Question 6. You____________by my secretary next week.
A. will notify B. would notify C. will be notified D. is going to be notified
Question 7. Her boy friend is said____________in the army two years ago.
A. to have served B. to serve C. serving D. have served
Question 8. Nothing is____________easy as it looks.
A. so B. such C. as D. that
Question 9. The local council recommended that John____________to the head of the apartment.
A. is appointed B. was appointed C. be appointed D. could be appointed.
Question 10. ____________they've already made their decision, there's nothing much we can do.
A. Seeing that B. On grounds that C. Assuming that D. For reason that
Question 11. It was under the tree____________he found the wallet he had lost before.
A. that B. whom C. who D. where
Question 12. Can I help you, sir?” - “I’m looking for a____________ table”
A. wooden round fashionable B. round wooden fashionable
C. wooden fashionable round D. fashionable round wooden
Question 13. The architects have made____________use of glass and transparent plastic.
A. imaginative B. imagine C. imagination D. imaginatively
Question 14. I only tell my secrets to my____________ friend as she never reveals them to anyone.
A. creative B. enthusiastic C. trustworthy D. unrealizable

Trang 1
Question 15. Paul is a very____________character, he is never relaxed with strangers.
A. self-conscious B. self-satisfied C. self-directed D. self-confident
Question 16. I haven’t read any medical books or articles on the subject for a long time, so I’m
____________with recent developments.
A. out of reach B. out of condition C. out of touch D. out of the question
Question 17. He was wearing very shabby, dirty clothes and looked very____________.
A. easy- going B. down to earth C. out of shape D. down at heel
Question 18. Attempts must be made to____________the barriers of fear and hostility which divide the
two communities.
A. break down B. set up C. get off D. pass over
Mark the letter A, B, C or D on your answer sheet to indicate the underlined part that needs
correction in each of the following questions.
Question 19. Tom’s jokes are inappropriate but we have to put up with it just because he’s the boss.
A. inappropriate B. it C. because D. the
Question 20. For its establishment, ASEAN Tourism Association has played an important role in
promoting and developing ASEAN Tourism services.
A. Tourism Association B. played
C. in promoting and developing D. For its
Question 21. Sleeping, resting and to drink fruit juice are the best ways to care for a cold.
A. juice B. best ways C. Sleeping D. to drink
Mark the letter A, B, C, or D on your answer sheet to indicate the sentence that is closest in
meaning to each of the following questions.
Question 22. “I agree that I am narrow-minded,” said the manager.
A. The manager denied being narrow-minded.
B. The manager admitted being narrow-minded.
C. The manager refused to be narrow-minded.
D. The manager promised to be narrow-minded.
Question 23. A supermarket is more convenient than a shopping centre.
A. A shopping centre is not as convenient as a supermarket.
B. A shopping centre is more convenient than a supermarket.
C. A supermarket is not as convenient as a shopping centre.
D. A supermarket is as inconvenient as a shopping centre.
Question 24. It was a mistake for Tony to buy that house.
A. Tony couldn’t have bought that house.
B. Tony can’t have bought that house.
C. Tony needn’t have bought that house.

Trang 2
D. Tony shouldn’t have bought that house.
Mark the letter A, B, C, or D on your answer sheet to indicate the sentence that best combines
each pair of sentences in the following questions
Question 25. Marie didn’t turn up at John’s birthday party. I feel so sorry for that.
A. If only Marie turn up at John’s birthday party.
B. I wish Marie had turned up at Johns birthday party.
C. I wished Marie wouldn’t turn up at John’s birthday party.
D. It’s a shame Marie had turned up at John’s birthday party.
Question 26. Their team performed excellently at the elimination tournament. They didn’t win
the trophy.
A. Subsequent to their performance at the elimination tournament, they were afraid to win the trophy
excellently.
B. Were it not for their excellent performance at the elimination tournament, they wouldn’t have won
the trophy.
C. Despite their excellent performance at the elimination tournament, they didn’t win the trophy.
D. Because they didn’t win the trophy, their performance at the elimination tournament was however
excellent.
Mark the letter A, B, C, or D on your answer sheet to indicate the most suitable response to
complete each of the following questions.
Question 27. Sue and Anne are talking about their future plans.
Sue: “I am not interested in the idea of taking a gap year and going backpacking in Nepal.” Anne: “
Well,_____________.”
A. I am B. help yourself C. neither do I D. that’s life
Question 28. Two students are chatting in the corridor after class.
Tim: “ We should make a slide show for our history presentation next week.”
Laura: “_____________”
A. Sorry, I have to check my diary. B. That’s exactly what I was going to say.
C. I’d love to but I just can’t now. D. That’s true. I understand how you feel.
Mark the letter A, B, C or D on your answer sheet to indicate the word or phrase that is
CLOSEST in meaaning to the underlined part in each of the following questions.
Question 29. Sports and festivals form an integral part of every human society.
A. essential B. informative C. invented D. exciting
Question 30. Please stop making that noise! It really gets on my nerves.
A. cheers me up B. wakes me up C. annoys me D. amuses me
Mark the letter A, B, C, or D on your answer sheet to indicate the word(s) OPPOSITE in meaning
to the underlined word(s) in each of the following questions.

Trang 3
Question 31. One is supposed to expend far more energy in marathon run than expected.
A. exhaust B. spend C. consume D. reserve
Question 32. The information you have got is actually off the record, so be careful if you intend to use
it for publication.
A. private B. official C. confidential D. important
Read the following passage and mark the letter A, B, C, or D on your answer sheet to indicate the
correct word or phrase that best fits each of the numbered blanks.
Amy Tan was born on February 19th , 1952 in Oakland, California. Tan grew up in Northern
California, (33)____________when her father and older brother both died from brain tumors in 1966,
she moved with her mother and younger brother to Europe, where she attended high school in
Montreux, Switzerland. She returned to the United States for college. After college, Tan worked as a
language development consultant and as a corporate freelance writer. In 1985, she wrote the story
"Rules of the Game" for a writing workshop, which laid the early (34) ____________for her first novel
The Joy Luck Club. Published in 1989, the book explored the (35) ____________between Chinese
women and their Chinese-American daughters, and became the longest-running New York Times
bestseller for that year. The Joy Luck Club received numerous awards, including the Los Angeles
Times Book Award. It has been translated into 25 languages, including Chinese, and was made into a
major motion picture for (36) ____________Tan co-wrote the screenplay. Tan's other works have also
been (37) ____________into several different forms of media.
Question 33. A. however B. moreover C. so D. but
Question 34. A. preparation B. base C. source D. foundation
Question 35. A. relate B. relative C. relationship D. relatively
Question 36. A. whom B. that C. what D. which
Question 37. A. adjoined B. adapted C. adjusted D. adopted
Read the following passage and mark the letter A, B, C, or D on your answer sheet to indicate the
correct answer to each of the questions.
The first thing to do when you have a trip abroad is to check that your passport is valid. Holders of out-
of-date passports are not allowed to travel overseas. Then you can prepare for your trip. If you don't
know the language, you can have all kinds of problems communicating with local people. Buying a
pocket dictionary can make a difference. You'll be able to order food, buy things in shops and ask for
directions. It's worth getting one. Also there's nothing worse than arriving at your destination to find
there are no hotels available. The obvious way to avoid this is to book in advance. This can save you
money too. Another frustrating thing that can happen is to go somewhere and not know about important
sightseeing places. Get a guide book before you leave and make the most of your trip. It's a must.

Trang 4
Then, when you are ready to pack your clothes, make sure they are the right kind. It's no good packing
sweaters and coats for a hot country or T-shirts and shorts for a cold one. Check the local climate before
you leave.
Also, be careful how much you pack in your bags. It's easy to take too many clothes and then not have
enough space for souvenirs. But make sure you pack essentials, what about money? Well, it's a good
idea to take some local currency with you but not too much. There are conveniently located cash
machines (ATMs) in most big cities, and it's usually cheaper to use them than change your cash in
banks. Then you'll have more money to spend. When you are at your destination, other travellers often
have great information they are happy to share. Find out what they have to say. It could enhance your
travelling experience.
(Adapted from Pearson Test of English General Skills)
Question 38. what is the passage mainly about?
A. Things to avoid when you go abroad B. Tips for Travellers Overseas
C. The benefits of travelling D. How to find ATMs in big cities
Question 39. According to the passage, you should do all of the following before leaving
EXCEPT____________.
A. taking money from an ATM B. making sure of the validity of your passport
C. preparing suitable clothes D. getting a guide book
Question 40. The word "This" in paragraph 2 refers to____________.
A. finding are no hotels to stay at B. saving money
C. booking in advance D. asking for directions
Question 41. According to the passage, holders of out-of-date passports____________.
A. have to show an ID instead when they travel
B. cannot travel to other countries
C. should ask for help from local people
D. may have their passports renewed in any country.
Question 42. The word “essentials” in paragraph 4 mostly means____________.
A. everything B. valuables C. necessities D. food
Read the following passage and mark the letter A, B, C or D on your answer sheet to indicate the
correct answer to each of the questions from 43-50.
Choosing a career may be one of the hardest jobs you ever have, and it must be done with care. View a
career as an opportunity to do something you love, not simply as a way to earn a living. Investing the
time and effort to thoroughly explore your options can mean the difference between finding a
stimulating and rewarding career and move from job to unsatisfying job in an attempt to find the right
one. Work influences virtually every aspect of your life, from your choice of friends to where you live.
Here are just a few of the factors to consider.

Trang 5
Deciding what matters most to you is essential to making the right decision. You may want to begin by
assessing your likes, dislikes, strengths, and weaknesses. Think about the classes, hobbies, and
surroundings that you find most appealing. Ask yourself questions, such as “Would you like to travel?
Do you want to work with children? Are you more suited to solitary or co-operative work?” There are
no right or wrong answers; only you know what is important to you. Determine which job features you
require, which ones you would prefer, and which ones you cannot accept. Then rank them in order of
importance to you. The setting of the job is one factor to take into account. You may not want to sit at a
desk all day. If not, there are diversity occupations - building inspector, supervisor, real estate agent -
that involve a great deal of time away from the office. Geographical location may be a concern, and
employment in some fields is concentrated in certain regions. Advertising job can generally be found
only in large cities. On the other hand, many industries such as hospitality, law education, and retail
sales are found in all regions of the country. If a high salary is important to you, do not judge a career
by its starting wages. Many jobs, such as insurance sales, offers relatively low starting salaries;
however, pay substantially increases along with your experience, additional training, promotions and
commission.
Don’t rule out any occupation without learning more about it. Some industries evoke positive or
negative associations. The traveling life of a flight attendant appears glamorous, while that of a plumber
does not. Remember that many jobs are not what they appear to be at first, and may have merits or
demerits that are less obvious. Flight attendants must work long, grueling hours without sleeps,
whereas plumbers can be as highly paid as some doctors. Another point to consider is that as you get
mature, you will likely to develop new interests and skills that may point the way to new opportunities.
The choice you make today need not be your final one.
Question 43. The author states that “There are no right or wrong answers” in order
to____________
A. emphasize that each persons answers will be different.
B. show that answering the questions is a long and difficult process.
C. indicate that the answers are not really important.
D. indicate that each person’s answers may change over time.
Question 44. The word “them” in paragraph 2 refers to____________.
A. questions B. answers C. features D. jobs
Question 45. The word “assessing” in paragraph 2 could best be replaced by____________.
A. discovering B. considering C. measuring D. disposing
Question 46. According to paragraph 3, which of the following fields is NOT suitable for a person
who does not want to live in a big city?
A. plumbing B. law C. retail sales D. advertising

Trang 6
Question 47. Those are all the factors you should take into account when choosing a job
EXCEPT____________
A.Your likes and your dislikes B. The atmosphere at work
C. Geographical location D. Your strengths and weaknesses
Question 48. In paragraph 5, the author suggests that ____________
A. you may want to change careers at some time in the future.
B. as you get older, your career will probably less fulfilling.
C. you will be at your job for a lifetime, so choose carefully.
D. you will probably jobless at some time in the future.
Question 49. The word “grueling” in paragraph 3 could best be replaced by____________.
A. tiring and hard B. relaxing C. painful D. enjoyable
Question 50. According to the passage, which of the following is true?
A. To make a lot of money, you should not take a job with a low starting salary.
B. To make lots of money, you should rule out all factory jobs.
C. If you want an easy and glamorous lifestyle, you should consider becoming flight attendant.
D. Your initial view of certain careers may not be accurate.

Trang 7
Đáp án
1-C 2-B 3-A 4-D 5-D 6-C 7-A 8-C 9-C 10-A
11-A 12-D 13-A 14-C 15-A 16-C 17-D 18-A 19-B 20-D
21-C 22-B 23-A 24-D 25-B 26-C 27-A 28-B 29-A 30-C
31-D 32-B 33-D 34-D 35-C 36-D 37-B 38-B 39-A 40-C
41-B 42-C 43-A 44-C 45-B 46-D 47-B 48-A 49-A 50-D

LỜI GIẢI CHI TIẾT


Question 1: Đáp án C
Phần gạch chân của đáp án c phát âm là /ʃ/, phần gạch chân của các phương án còn lại phát âm là /k/.
A. chemistry /ˈkemɪstri/ (n): môn hóa học
B. Christmas /ˈkrɪsməs/ (n): Lễ Giáng Sinh
C. machine /məˈʃiːn/ (n): máy móc
D. headache /ˈhedeɪk/ (n): cơn đau đầu
Question 2. Đáp án B
Phần gạch chân của đáp án B phát âm là /h/, phần gạch chân của các phương án còn lại là âm câm (không
được đọc lên).
A. honest /ˈɒnɪst/ (adj): thành thật, thật thà
B. honey /ˈhʌni/ (n): mật ong
C. hour /ˈaʊə(r)/ (n): giờ đồng hồ
D. heir /eə(r)/ (n): người thừa kế
Question 3. Đáp án A
Đáp án A có trọng âm rơi vào âm tiết số 2, các phương án còn lại có trọng âm rơi vào âm tiết đầu tiên.
A. digest /daɪˈdʒest/ (v): tiêu hóa
B. cactus /ˈkæktəs/ (n): cây xương rồng
C. camel /ˈkæml/ (n): con lạc đà
D. human /ˈhjuːmən/ (n): con người
Question 4. Đáp án D
Đáp án D có trọng âm rơi vào âm tiết số 2, các phương án còn lại có trọng tâm rơi âm tiết đầu tiên.
A. compliment /ˈkɒmplɪmənt/ (n): lời khen ngợi
B. heartbroken /ˈhɑːtbrəʊkən/ (adj): đau lòng, buồn đến tan nát cõi lòng
C. labour-saving /ˈleɪbə seɪvɪŋ/ (adj): tiết kiệm sức lao động
D. short-sighted /ˌʃɔːt ˈsaɪtɪd/ (adj): cận thị, tầm mắt ngắn
Lưu ý:
- Hầu hết các tính từ ghép (compound adjectives)
+ Tính từ ghép bắt đầu bằng danh từ, trọng âm rơi vào phần thứ nhất: heartbroken, labour-saving

Trang 8
+ Tính từ ghép bắt đầu bằng tính từ hoặc trạng từ thì trọng âm rơi vào phần thứ hai: short-sighted, well-
done
- Hầu hết các danh từ ghép (compound nouns) có 2 âm tiết đều có trọng âm chính rơi vào âm tiết thứ
nhất: schoolboy, cupboard, weight-lifting, boyfriend, filmmaker, hotdog, raincoat, bookshop...
Question 5. Đáp án D
Kiến thức được hỏi: Danh từ
Ta có: wrapping paper - giãy bọc (paper for wrapping purpose, such as wrapping presents)
Ta cần xác định danh từ chính/ danh từ gốc chỉ đối tượng (ai/ cái gì) là paper, nên vị trí của nó đứng phía
sau → loại A.
Wrapping đứng vai trò như một tính từ để xác định thêm ý nghĩa cho danh từ gốc. Danh từ đó làm sao,
như thế nào, trong câu này thì nghĩa là giấy làm gì - giấy để bọc quà. Các ví dụ tương tự như: dining
table, living room, swimming pool...
→ Chọn D.
Tạm dịch. Tôi mới mua một món quà cho anh trai và bây giờ tôi cần vài tờ giấy bọc quà.
Question 6. Đáp án C
Kiến thức được hỏi: Bị động
Nhận thấy trong câu có dấu hiệu “by my secretary” và ngoại động từ “notify” cần điền vào chỗ trống
không có tân ngữ đi kèm ta khẳng định được câu này cần dùng bị động: S + be + P2 (by O)
Loại bỏ A và B đang mang ý chủ động, loại D do “you” không đi với “is”
Thời điểm “next week” phù hợp với bị động mang ý tương lai của đáp án c.
Tạm dịch: Bạn sẽ được thư kí của tôi báo cho vào tuần tới.
Question 7. Đáp án A
Kiến thức được hỏi: Bị động với từ chỉ ý kiến. Vì đề bài có “two years ago”, với các hành động xảy ra
trong quá khứ ta dùng cấu trúc: S+ be+ said (/believed/ thought...)+ to have P2
Do vậy ta chọn đáp án A.
Tạm dịch: Người ta bảo rằng bạn trai của cô ấy đã phục vụ trong quân ngũ 2 năm trước.
* Mở rộng:
Với các hành động xảy ra ở hiện tại hoặc tương lai, ta dùng cấu trúc:
S+ be+ believed (/said/ thought...) + to V.
Question 8. Đáp án C
Kiến thức được hỏi: So sánh
Câu đang sử dụng so sánh ngang bằng (ta thấy được qua chữ as ở phía sau) với công thức: as... as
Không chọn A do so chỉ xuất hiện trong so sánh ngang bằng có chứa phủ định “not”.
→ Chọn đáp án C.
Tạm dịch: Không gì dễ như vẻ ngoài của nó.
Question 9. Đáp án C

Trang 9
Kiến thức được hỏi: Thức giả định kèm bị động Ta sử dụng thức giả định với các động từ thể hiện đề
xuất, yêu cầu với công thức như sau. S+ V+ that+ S (should) + Vo.
V: recommend, suggest, require, request, demand, ask, propose, insist, urge...
Trong các phương án, duy nhất C ở dạng động từ nguyên mẫu không to, câu đang thể hiện ý bị động nữa
nên phải là be appointed. → Chọn đáp án C
Tạm dịch: Hội đồng địa phương đề nghị rằng John nên được chỉ định là trưởng phòng.
Question 10. Đáp án A
Kiến thức được hỏi: Liên từ.
Không có cụm B-On grounds that và D-For reason that nên loại B và D. Ta có cụm On the grounds that
= For the reason that = Because: bởi vì.
A. Seeing that: bởi vì
C. Assuming that: cứ cho là
Xét theo ý nghĩa của câu ta chọn A.
Tạm dịch: Bởi vì họ đã đưa ra quyết định của mình, không còn gì nhiều để chúng ta có thể làm.
Question 11. Đáp án A
Kiến thức được hỏi: Câu chẻ/ Câu nhấn mạnh
Ta dùng công thức sau cho câu chẻ, khi muốn nhấn mạnh một yếu tố nào đó.
It is/ was .... (phần muốn nhấn mạnh)+ that + …….. : Đó chính là...
Chọn đáp án A - that
Tạm dịch: Đó chính là dưới cái gốc cây nơi mà anh ấy đã tìm thấy cái ví đã bị mất trước đó.
Question 12. Đáp án D
Kiến thức được hỏi: Trật tự tính từ
Ta có cách sắp xếp tính từ. OSASCOMP + N o - opinion: ý kiến
S- size: kích cỡ
A - Age: tuổi
S - shape: hình dáng
C - colour: màu sắc
O - origin: nguồn gốc
M - material: chất liệu
P -purpose: mục đích
→ fashionable round wooden → Ta chọn đáp án D
Tạm dịch: “Tôi có thể giúp gì thưa ngài?” - “Tôi đang tìm một chiếc bàn gỗ hình tròn hợp thời trang”.
Question 13. Đáp án A
Kiến thức được hỏi: Loại từ
Do phía sau chỗ trống là danh từ nên ta xác định được chỗ trống cần dùng một tính từ để tạo ra cụm Adj+
N

Trang 10
A. imaginative (adj): sáng tạo
B. imagine (v): tưởng tượng
C. imagination (n): trí tưởng tượng
D. imaginatively (adv): một cách sáng tạo
Như vậy, trong 4 phương án chọn được đáp án A là tính từ
Tạm dịch: Các kiến trúc sư đã sử dụng sáng tạo thủy tinh và nhựa trong suốt.
Question 14. Đáp án C
Kiến thức được hỏi: Lựa chọn từ
A. creative (adj): sáng tạo
B. enthusiastic (adj): đầy nhiệt tình
C. trustworthy (adj): đáng tin cậy
D. unrealizable (adj): không thể thực hiện, không làm được
Căn cứ vào nghĩa ta chọn được trusworthy → Chọn đáp án C
Tạm dịch: Tôi chỉ nói những bí mật của tôi với người bạn đáng tin cậy vì cô ấy không bao giờ tiết lộ cho
bất cứ ai.
Question 15. Đáp án A
Kiến thức được hỏi: Lựa chọn từ
A. self-consicious (adj): e ngại
B. self- satisfied (adj): tự mãn
C. self-directed (adj): tự quyết
D. self-confident (adj): tự tin
Căn cứ vào nghĩa ta chọn được self-consicious
→ Ta chọn đáp án A
Tạm dịch: Paul có tính e ngại, anh ấy không bao giờ thoải mái với người lạ.
Question 16. Đáp án C
Kiến thức được hỏi: Cụm từ
A. out of reach: ngoài tầm với
B. out of condition: không đủ sức khỏe
C. out of touch: mất liên lạc với, không có tin tức về
D. out of the question: không thể được, điều không thể
Căn cứ vào nghĩa của câu ta chọn được đáp án C - out of touch.
Tạm dịch: Tôi đã không đọc bất kỳ sách y khoa hoặc bài viết về chủ đề này trong một thời gian dài, vì
vậy tôi không có tin tức về những phát triển gần đây.
Question 17. Đáp án D
Kiến thức được hỏi: Thành ngữ/ Cụm từ
A. easy- going: dễ tính, thân thiện

Trang 11
B. down to earth: thực tế
C. out of shape: không khỏe mạnh
D. down at heel: tàn tạ
Căn cứ vào nghĩa ta chọn được D - down at heel
Tạm dịch: Anh ấy đang mặc quần áo cũ kĩ, bẩn thỉu và trông rất là tàn tạ.
Question 18. Đáp án A
Kiến thức được hỏi: Phrasal Verb - Ngữ động từ
A. break down (v): phá vỡ
B. set up (v): dựng nên, thiết lập
C. get off (v): xuống (tàu, xe...)
D. pass over (v): lờ, tránh đi
Căn cứ vào nghĩa ta chọn được đáp án A - break down.
Tạm dịch: Các nỗ lực phải được tạo ra để phá vỡ những rào cản sợ hãi và sự thù hận chia rẽ hai cộng
đồng.
Question 19. Đáp án B
Lỗi sai: Không phù hợp về đại từ tân ngữ “jokes” là danh từ số nhiều nên phải dùng tân ngữ “them”
để thay thế. → chọn đáp án B. Sửa lại. it → them
Tạm dịch: Những câu chuyện cười của Tom không phù hợp nhưng chúng tôi phải chịu đựng chúng chỉ vì
ông ấy là ông chủ.
Question 20. Đáp án D
Lỗi sai: Sử dụng giới từ chưa đúng, để chỉ một mốc thời gian ta phải dùng since chứ không phải for.
Sine + mốc thời gian/ mốc sự kiện: kể từ khi
For+ khoảng thời gian: được bao nhiêu lâu
→ Chọn đáp án D
Sửa lại. For → Since
Tạm dịch: Kể từ khi thành lập, Hiệp hội Du lịch ASEAN đóng vai trò quan trọng trong việc thúc đẩy và
phát triển dịch vụ Du lịch ASEAN.
Question 21. Đáp án C
Lỗi sai. Sử dụng cấu trúc song hành chưa đúng. Với các phần nắm giữ vị trí ngữ pháp giống nhau (cùng
làm chủ ngữ, cùng làm tân ngữ, cùng làm động từ chính...) thì có dạng thức giống nhau (cùng là danh từ,
cùng là tính từ, cùng là động từ...)
Trong câu này, đứng ở vị trí chủ ngữ giống như “sleeping” và “resting”, được liên kết với liên từ đẳng lập
“and” thì ta cũng phải dùng dạng V-ing- drinking → Ta chọn đáp án C.
Sửa lại: to drink → drinking
Tạm dịch: Ngủ đủ, nghỉ ngơi và uống nước ép hoa quả là những cách tốt nhất để chữa bệnh cảm lạnh.
Question 22. Đáp án B

Trang 12
Kiến thức: Câu chuyển trực tiếp sang gián tiếp
deny + V-ing: phủ nhận việc gì
admit + V-ing: thừa nhận việc gì
refuse + to V: từ chối làm việc gì
promise + to V: hứa làm việc gì
Dịch câu gốc. “Tôi đồng ý rằng tôi là người hẹp hòi.”, người quản lí nói.
Dịch các phương án.
A. Người quản lí phủ nhận là người hẹp hòi.
B. Người quản lý thừa nhận là người hẹp hòi.
C. Người quản lý từ chối là người hẹp hòi
D. Người quản lý hứa là người hẹp hòi.
Câu phù hợp với câu gốc nhất là B.
Question 23. Đáp án A
Kiến thức: Câu so sánh
Dịch câu gốc. Một siêu thị thì thuận tiện hơn một trung tâm mua sắm.
Dịch các phương án.
A. Một trung tâm mua sắm không thuận tiện như siêu thị.
B. Một trung tâm mua sắm thuận tiện hơn siêu thị.
C. Một siêu thị không thuận tiện như một trung tâm mua sắm.
D. Một siêu thị cũng bất tiện như một trung tâm mua sắm.
Các phương án B, C, D sai về nghĩa nên bị loại; ta chọn A phù hợp về nghĩa nhất.
Question 24. Đáp án D
Kiến thức. Động từ khuyết thiếu
Dịch câu gốc. Đó là lỗi của Tony khi mua căn nhà đó. / Tony đã sai lầm khi mua căn nhà đó.
cant/ couldn’t have P2: đã không thể nào làm gì trong quá khứ
needn’t have P2: đáng lẽ ra không cần làm gì trong quá khứ
shouldn’t have P2: đã không nên làm gì làm gì trong quá khứ
Dịch các phương án.
A. Tony đã không thể nào mua căn nhà đó được.
B. Tony đã không thể nào mua căn nhà đó được.
C. Tony đáng lẽ đã không cần mua căn nhà đó
D. Tony đã không nên mua căn nhà đó.
Câu gốc: điều đó là sai lầm khi mua căn nhà đó tương ứng với đã không nên mua → D đúng.
Question 25. Đáp án B
Dịch câu gốc: Marie đã không đến bữa tiệc sinh nhật của John. Tôi cảm thấy rất tiếc vì điều đó.
Tình huống xảy ra trong quá khứ vì vậy chúng ta sẽ sử dụng câu ước không có thật trong quá khứ.

Trang 13
S + wish(es/ed) + S + had (not) + P2.
Dịch các phương án.
A. Sai ngữ pháp: turn up, lẽ ra phải là had turned up → loại A
B. Tôi ước Marie đã đến bữa tiệc sinh nhật của John.
C. Tôi đã ước rằng Marie sẽ không đến bữa tiệc của John (Câu này đang viết theo câu ước mong muốn có
một sự thay đổi trong tương lai, vừa không đúng với tình huống ở quá khứ vừa không hợp ngữ nghĩa với
câu gốc) → loại C
D. Thật là đáng tiếc khi Marie xuất hiện tại bữa tiệc sinh nhật của John. → sai về nghĩa, loại D.
→ Chọn đáp án B
Question 26. Đáp án C
Dịch câu gốc: Đội của họ đã thể hiện xuất sắc tại vòng đấu loại. Họ không giành được chiếc cúp nào.
A. Sau màn trình diễn của họ ở vòng loại, họ e ngại giành được chiếc cúp một cách xuất sắc. → sai về
nghĩa
B. Sai ngữ pháp: were it not for (điều kiện loại 2) đi với wouldn’t have won (điều kiện loại 3) là bất hợp lí
C. Tuy có màn trình diễn xuất sắc tại vòng đấu loại, nhưng họ không giành được chiếp cúp nào cả.
D. Bởi vì họ không giành được cúp, tuy nhiên màn trình diễn của họ ở vòng loại là tuyệt vời. → sai về
nghĩa
→ Chọn đáp án C
Question 27. Đáp án A
Sue và Anne đang nói về kế hoạch tương lai của họ.
Sue: “Mình không hứng thú với ý tưởng nghỉ học tạm thời trong một năm và đi du lịch bụi ở Nepal.”
Anne: “À,____________.”
A. Mình thì có
B. Cứ tự nhiên
C. Sai ngữ pháp: phải là neither am I
D. Cuộc sống mà
Tình huống đang thể hiện ý kiến cá nhân, câu phản hồi có thể tán đồng hoặc là không.
Câu tình huống bắt đầu là “ I am not...” nên khi không tán đồng chỉ cần khẳng định “ I am” là được.
Các phản hồi B, D không phù hợp với ngữ cảnh → Chọn A
Question 28. Đáp án B
Hai học sinh đang tán gẫu ở hành lang sau giờ học.
Tim: “Chúng ta nên làm bài trình chiếu cho buổi thuyết trình lịch sử vào tuần sau.”
Laura: _____________.
A. Xin lỗi, tôi phải kiểm tra nhật kí.
B. Đó chính xác là điều tôi đã định nói.
C. Tôi rất thích nhưng giờ tôi không thể.

Trang 14
D. Đúng. Tôi hiểu bạn cảm thấy thế nào.
Tình huống đưa ra một lời đề nghị chúng ta chọn đáp án B, thể hiện sự tán đồng đề nghị đó. Các phản hồi
A, C, D không phù hợp với ngữ cảnh.
Question 29. Đáp án A
Dữ liệu đề bài. Thể thao và lễ hội làm nên một phần integral trong mỗi xã hội loài người.
Câu này có thể dùng kiến thức từ vựng của bản thân, hoặc suy đoán logic về ngữ nghĩa hoặc thế từng
phương án lên trên câu gốc để tìm ra đáp án đúng.
Ta có: (be) intergral = (be) essential (adj): quan trọng, thiết yếu
Như vậy ta chọn được đáp án A
Các đáp án khác.
B. informative (adj): giàu thông tin, cung cấp nhiều tin tức
C. invented (P2): được phát minh
D. exciting (adj): thú vị
Tạm dịch: Thể thao và lễ hội làm nên một phần không thể thiếu trong mọi xã hội loài người.
Question 30. Đáp án C
Dữ liệu đề bài. Xin hãy dừng cái tiếng động đó ngay lại! Nó thực sự gets on my nerves.
Dựa vào ý của câu ta có thể đoán được rằng phần gạch chân thể hiện ý tiêu cực, sự khó chịu vì tiếng ồn.
get on one's nerves = annoy sb: làm ai phát bực lên
→ Chọn đáp án C
Các đáp án khác.
A. cheers me up: khiến tôi phấn chấn lên
B. wakes me up: đánh thức tôi
D. amuses me: làm cho tôi cười
Tạm dịch: Xin hãy dừng cái tiếng động đó ngay lại! Nó thực sự khiến tôi bực mình.
Question 31. Đáp án D
Dữ liệu đề bài. Mỗi người được cho là đã expend nhiều năng lượng trong cuộc chạy đua marathon hơn
dự kiến.
Qua nghĩa của câu ta có thể đoán được chạy đua marathon nhất định phải tiêu tốn nhiều năng lượng.
Ta có: expend (v): tiêu, sử dụng
A. exhaust (v): cạn kiệt
B. spend (v): sử dụng
C. consume (v): tiêu thụ
D. reserve (v): dự trữ, giữ
Tìm từ trái nghĩa nên ta có expend >< reserve, chọn D
Tạm dịch: Mỗi người được cho là đã tiêu tốn nhiều năng lượng trong cuộc chạy đua marathon hơn dự
kiến.

Trang 15
Question 32. Đáp án B
Dữ liệu đề bài. Thông tin mà bạn có được là off the record, vậy nên hãy cẩn thận nếu bạn có ý định sử
dụng nó công khai
Tại sao phải cẩn thận nếu muốn sử dụng tin tức này công khai, thông tin này có thể gợi mở cho ta đoán
nghĩa của từ được gạch chân.
off the record: không chính thức
A. private (adj): riêng tư
B. official (adj): chính thức
C. confidential (adj): bí mật
D. important (adj): quan trọng
Tìm từ trái nghĩa nên ta có off the record >< official, chọn B.
Tạm dịch: Thông tin mà bạn có được không chính thức, vậy nên hãy cẩn thận nếu bạn có ý định sử dụng
nó công khai.
Question 33.
Câu hỏi về: Liên từ
Câu hỏi yêu cầu liên kết ý của hai mệnh đề có nghĩa đối lập trong câu, Tan lớn lên ở Bắc California,
nhưng sau đó lại chuyển tới Montreux, Thụy Sĩ.
Loại B - moreover - hơn nữa; C - so - vậy nên vì không theo logic của bài.
Giữa A - however-tuy nhiên và D- but-nhưng đều thể hiện ý đối lập, ta chọn D loại A, do từ however
không đứng sau dấu phảy như thế kia, however có thể đứng sau dấu chấm để kết nối hai câu, hoặc được
đặt sau dấu chấm phảy và dấu phảy như thế này however,” để kết nối hai mệnh đề.
Tan grew up in Northern California, (33) but when her father and older brother both died from brain
tumors in 1966, she moved with her mother and younger brother to Europe, where she attended high
school in Montreux, Switzerland.
Tạm dịch: Tan lớn lên ở Bắc California, nhưng khi cha và anh trai cô đều chết vì khối u não vào nàm
1966, cô cùng mẹ và em trai đến châu Âu, nơi cô học trung học ở Montreux, Thụy Sĩ
Question 34. Đáp án D
Đây là câu hỏi về lựa chọn từ
Ta có cụm từ: lay foundation / grounds for: đặt nền móng cho
→ Ta chọn được đáp án D.
A. preparation: sự chuẩn bị
B. base: nền tảng
C. source: nguồn
In 1985, she wrote the story "Rules of the Game" for a writing workshop, which laid the early (34)
foundation for her first novel The Joy Luck Club.

Trang 16
Tạm dịch: Năm 1985, cô đã viết câu chuyện "Luật chơi" cho một hội thảo viết văn, đặt nền móng ban đầu
cho cuốn tiểu thuyết đầu tiên của cô The Joy Luck Club.
Question 35. Đáp án C
Câu hỏi vê' từ loại
Chỗ trống trong bài đang đứng sau động từ “explored” vì vậy nó đảm nhận vị trí tân ngữ trong câu, hơn
nữa trước chỗ trống có “the” càng giúp ta khẳng định được chỗ trống cần dùng danh từ.
A. relate (v): liên quan
B. relative (n): họ hàng
C. relationship (n): mối quan hệ
D. relatively (adv): khá
to explore the relationship: khám phá mối quan hệ
→ Ta chọn đáp án C.
Published in 1989, the book explored the (35) relationship between Chinese women and their Chinese-
American daughters, and became the longest-running New York Times bestseller for that year.
Tạm dịch: Được xuất bản năm 1989, cuốn sách đã khám phá mối quan hệ giữa phụ nữ Trung Quốc và các
cô con gái người Mỹ gốc Hoa của họ, và trở thành cuốn sách bán chạy nhất của tờ New York Times trong
năm đó.
Question 36. Đáp án D
Câu hỏi yêu cầu chọn đại từ quan hệ.
Nhận thấy rằng trước chỗ trống cần điền đại từ
quan hệ có giới từ “for” mà ta luôn có: giới từ + which / whom
- which: thay thế cho danh từ chỉ vật
- whom: thay thế cho danh từ chỉ người
picture (n): bức tranh là danh từ chỉ vật, vì vậy ta chọn D - which
It has been translated into 25 languages, including Chinese, and was made into a major motion picture for
(36) which Tan co-wrote the screenplay.
Tạm dịch: Nó đã được dịch sang 25 ngôn ngữ, bao gồm cả tiếng Trung Quốc, và được dựng thành một
bức tranh chuyển động lớn mà Tan đồng sáng tác kịch bản.
Question 37. Đáp án B
Câu hỏi về lựa chọn từ. Câu chứa chỗ trống phù hợp với ý nghĩa rằng các tác phẩm khác của Tan được
chuyển thể thành nhiều hình thức khác nữa.
A. adjoin (v): tiếp giáp
B. adapt (v): chuyển thể, phỏng theo
C. adjust (v): điều chỉnh
D. adopt (v): chấp nhận sử dụng; nhận nuôi
→Ta chọn được đáp án B.

Trang 17
Tan's other works have also been (37) adapted into several different forms of media.
Tạm dịch: Các tác phẩm khác của Tan cũng đã được chuyển thể thành nhiều hình thức truyền thông khác
nhau.
READING 1
Bài đọc là những lời khuyên hữu ích dành cho những người chuẩn bị ra nước ngoài đi du lịch về.
- Hộ chiếu
- Ngôn ngữ
- Đặt trước phòng khách sạn
- Quần áo và các vật dụng cần thiết
- Tiền bạc...
Question 38. Đáp án B
Ý chính của bài là gì?
A. Những điều cần tránh khi bạn đi ra nước ngoài
B. Những lời khuyên cho khách du lịch ở nước ngoài
C. Những lợi ích của việc đi du lịch
D. Cách tìm máy ATM ở các thành phố lớn
Nội dung của bài xoay quanh những điều cần lưu ý cho những ai đang muốn đi du lịch ở nước ngoài,
những điều cần làm, những thứ cần chuẩn bị về hộ chiếu, ngôn ngữ, tiền, đặt trước khách sạn, mang theo
quần áo và vật dụng cần thiết.
“The first thing to do when you have a trip abroad is...”: điều đầu tiên bạn cần làm khi có một chuyến du
lịch nước ngoài là...
“Then you can prepare for your trip...”: Sau đó bạn có thể chuẩn bị cho chuyến du lịch của mình
“Also, be careful": Hãy cẩn thận...
Vì vậy ta chọn đáp án B.
Question 39. Đáp án A
Theo đoạn văn, bạn nên làm tất cả những điều sau đây trưác khi rời khỏi NGOẠI TRỪ___________
A. lấy tiền từ máy ATM
B. đảm bảo tính hợp lệ của hộ chiếu của bạn
C. chuẩn bị quần áo phù hợp
D. nhận sách hướng dẫn
Đây là câu hỏi chứa từ phủ định nên ta sẽ loại dần các phương án đã xuất hiện trong bài. Loại B do thông
tin có trong câu đầu tiên của bài.
The first thing to do when you have a trip abroad is to check that your passport is valid: Điều đầu tiên
cần làm khi đi du lịch nước ngoài là kiểm tra xem hộ chiếu của bạn có hợp lệ không.

Trang 18
Loại C do có thông tin này ở đầu đoạn 2. Then, when you are ready to pack your clothes, make sure they
are the right kind: Sau đó, khi bạn sẵn sàng sắp xếp quần áo, hãy nhớ đảm bảo rằng bạn chuẩn bị quần áo
phù hợp.
Loại D do thông tin này xuất hiện cuối đoạn 1
Get a guide book before you leave and make the most of your trip: Hãy lấy một cuốn sách hướng dẫn
trước khi bạn rời đi và tận hưởng hết mình chuyến du lịch của bạn.
Do vậy ta chọn A.
Question 40.
Từ “this” đoạn 2 ám chỉ___________.
A. tìm kiếm không có khách sạn để ở
B. tiết kiệm tiền
C. đặt phòng trước
D. hỏi đường
Câu hỏi về đại từ nên ta đặt this vào trong câu nó xuất hiện. Also there's nothing worse than arriving at
your destination to find there are no hotels available. The obvious way to avoid this is to book in advance.
This can save you money too. Chẳng còn gì tồi tệ hơn là khi đến nơi mà không có sẵn phòng khách sạn.
Cách rõ ràng để tránh điều này là đặt trước phòng. Điều này cũng có thể giúp bạn tiết kiệm tiền. Vậy This
là booking in advance - đặt phòng trước → đáp án C.
Question 41. Đáp án B
Theo đoạn văn, người mang hộ chiếu hết hạn___________
A. phải xuất trình ID (thẻ căn cước) thay thế khi họ đi du lịch
B. không thể đi du lịch đến các nước khác
C. nên yêu cầu sự giúp đỡ từ người dân địa phương
D. có thể gia hạn hộ chiếu ở bất kỳ quốc gia nào
Đây là câu hỏi thông tin chi tiết, đáp án tìm được ở câu đâu đoạn 1 với key words “holders of out-of-date
passports”. Holders of out-of-date passports are not allowed to travel overseas - Người mang hộ chiếu
quá hạn không được phép đi du lịch nước ngoài.
→ Chọn B
Question 42.
Từ “essentials” trong đoạn 4 gần nghĩa nhất với:
A. mọi thứ
B. những vật có giá trị
C. những vật cần thiết
D. thức ăn
Đây là câu hỏi về từ vựng, chúng ta có thể dùng kiến thức từ vựng của bản thân để tìm đáp án hoặc quay
trở lại câu chứa từ essentials trong bài đọc.

Trang 19
Also, be careful how much you pack in your bags. It's easy to take too many clothes and then not have
enough space for souvenirs. But make sure you pack essentials.
Hãy cẩn thận với số lượng bạn mang đi vào trong túi của mình. Rất dễ là bạn sẽ mang quá nhiều quần áo
và thế là không còn đủ chỗ cho các đồ lưu niệm. Nhưng bạn vẫn cần đảm bảo rằng bạn mang đủ những
vật dụng cần thiết.
Vậy essentials = necessities, chọn được đáp án C.
READING 2
Đoạn 1. Lựa chọn nghề nghiệp là việc bạn cần làm một cách thận trọng vì nó có thể ảnh hưởng đến mọi
khía cạnh trong cuộc sống của bạn.
Đoạn 2. Để đưa ra sự lựa chọn chính xác hãy ngẫm nghĩ về những điều quan trọng nhất với bản thân và
đánh giá ưu nhược điểm của mình.
Đoạn 3. Điều kiện làm việc là một yếu tố để xem xét.
Đoạn 4. Đừng bỏ qua bất kỳ công việc nào mà không xem xét kỹ lưỡng bởi chúng sẽ không hẳn như vẻ
bề ngoài.
Question 43. Đáp án A
Tác giả đề cập rằng “ chẳng hề có câu trả lời nào là đúng hoặc sai” để mà___________
A. để nhấn mạnh rằng câu trả lời của mỗi người sẽ khác nhau.
B. để cho thấy việc trả lời câu hỏi là một quá trình dài và khó khăn
C. để chỉ ra rằng câu trả lời không thực sự quan trọng
D. để chỉ ra rằng câu trả lời của mỗi người có thể thay đổi theo thời gian
Ta có thể suy ra đáp án từ câu “There are no right or wrong answers; only you know what is important to
you”
Tạm dịch. Chẳng hề có câu trả lời nào là đúng hoặc sai; chỉ có bạn mới biết điều gì là quan trọng với
bản thân mình.
Vì chỉ có bạn mới biết điểu gì là quan trọng với mình cũng chính vì thế mà mỗi người sẽ có một câu trả
lời khác nhau. Do đó đáp án A là hoàn toàn chính xác.
Question 44. Đáp án C
Từ “them” ở đoạn 2 đề cập tới___________
A. Các câu hỏi
B. Các câu trả lời
C. Các đặc trưng, đặc điểm.
D. Các công việc.
Đây là câu hỏi tìm từ thay thế. Ta cần đọc câu ở phía trước để tìm từ thay thế. “Determine which job
features you require, which ones you would prefer, and which ones you cannot accept. Then rank them in
order of importance to you.”

Trang 20
Tạm dịch: Quyết định xem đặc trưng công việc mà bạn muốn, bạn thích cái nào hơn, cái nào bạn không
thể chấp nhận được. Sau đó hãy xếp hạng chúng theo độ quan trọng với bạn.
Đọc xong câu dịch ta có thể suy ra từ “them” thay thế cho từ “features”.
→ Chọn C.
Question 45. Đáp án B
Tù “assessing” ở đoạn 2 có thể thay thế tốt nhất bằng từ____________
A. Khám phá
B. Xem xét, đánh giá
C. Đo lường
D. Bố trí, sắp xếp
assessing /əˈses/ (v): đánh giá, xem xét.
Câu hỏi từ vựng có thể dùng vốn từ để giải quyết hoặc dựa vào văn cảnh để đoán nghĩa. “You may want
to begin by assessing your likes, dislikes, strengths, and weaknesses”
Tạm dịch: Bạn có lẽ muốn bắt đầu bằng cách assessing những điều bạn thích, không thích, ưu điểm,
nhược điểm.
Xét nghĩa của câu ta có thể suy ra nghĩa của từ “assessing” là xem xét, đánh giá và tương ứng là từ
“considering”. → Chọn B.
Question 46. Đáp án D
Theo như đoạn 3, lĩnh vực nào dưới đây KHÔNG phù hợp với người không muốn ở thành phố?
A. Thợ sửa ống nước
B. Luật sư
C. Đại lý bán lẻ
D. Nghề quảng cáo
Thông tin nằm ở câu 5, đoạn 3: “Advertising job can generally be found only in large cities”
Tạm dịch. Công việc quảng cáo thường chỉ có thể tìm thấy ở những thành phố lớn.
Câu hỏi nói đến việc KHÔNG MUỐN ở thành phố mà việc quảng cáo chỉ có ở thành phố nên đáp án sẽ là
nghề quảng cáo “advertising”. → Đáp án D
Question 47. Đáp án B
Dưới đây là những yếu tố bạn nên xem xét khi chọn nghề TRỪ___________
A. Điều bạn thích và không thích
B. Không khí ở nơi làm việc
C. Vị trí địa lý
D. Ưu điểm và nhược điểm của bạn.
Đây là câu hỏi tìm thông tin không xuất hiện nên ta cần tìm những thông tin có trong bài và loại bỏ đi thì
sẽ suy ra được đáp án.

Trang 21
Loại A: “Deciding what matters most to you is essential to making the right decision. You may want to
begin by assessing your likes, dislikes, strengths, and weaknesses”- ....xem xét điều bạn thích và không
thích...
Loại C dòng 3 đoạn 3: “Geographical location may be a concern, and employment in some fields
is concentrated in certain regions” – Vị trí địa lý có lẽ là một điều để quan tâm và công việc ở một số lĩnh
vực chỉ có ở một vài vùng nhất định.
Loại D: Deciding what matters most to you is essential to making the right decision. You may want to
begin by assessing your likes, dislikes, strengths, and weaknesses.”- ... xem xét, điểm mạnh và điểm yếu
Các đáp án A, C, D đã xuất hiện trong bài do đó chọn phương án B.
Question 48. Đáp án A
Ở đoạn 5, tác giả đã gợi ý rằng
A. Bạn có lẽ sẽ thay đổi công việc vào một thời điểm nào đó trong tương lai
B. Khi bạn lớn lên, công việc của bạn có lẽ sẽ không còn thỏa mãn nữa
C. Bạn sẽ làm một công việc suốt đời nên hãy chọn cẩn thận
D. Bạn có lẽ sẽ thất nghiệp trong tương lai.
Ở dòng cuối đoạn 5 có câu: “Another point to consider is that as you get mature, you will likely to
develop new interests and skills that may point the way to new opportunities.”
Tạm dịch: Một điều khác nữa để xem xét là khi bạn trưởng thành, bạn sẽ có nhiều sở thích và kỹ năng
mới có thể mở ra cho bạn những cơ hội mới.
Khi có sở thích, kỹ năng và cơ hội mới mở ra thì rất có thể bạn sẽ thay đổi công việc của mình trong
tương lai. Do đó chọn câu A là hợp lý nhất.
Question 49. Đáp án A
Từ “grueling ” ở đoạn 3 có thể được thay thể bởi từ
A. Mệt mỏi và khó khăn
B. Thư giãn
C. Đau đớn
D. Đầy hứng thú, vui vẻ
grueling /ˈɡruːəlɪŋ/ (adj): mệt mỏi quá mức
Câu hỏi từ vựng ngoài dùng vốn từ để trả lời, ta còn có thể dùng văn cảnh của câu: “Flight attendants
must work long grueling hours without sleeps...”
Tạm dịch: Những người tiếp viên hàng không phải làm việc trong grueling nhiều giờ liền mà không được
ngủ.
Ở câu trên ta thấy ví dụ đang nêu ra sự khó nhọc, mệt mỏi của những người tiếp viên hàng không trong
điều kiện làm việc khắc nghiệt. Họ phải làm việc trong thời gian dài và không được ngủ nên có thể suy ra
từ grueling mang nghĩa tiêu cực. Mà theo suy luận thì khi làm việc dài và thiếu ngủ thì sẽ dẫn đến mệt
mỏi. Do đó đáp án A là phù hợp nhất.

Trang 22
Question 50. Đáp án D
Theo như đoạn văn, điều nào dưới đây là đúng?
A. Để kiếm được nhiều tiền, bạn không nên bắt đầu bằng công việc có mức lương thấp
B. Để kiếm được nhiều tiền, bạn nên bỏ qua hết những công việc trong nhà máy
C. Nếu bạn muốn có một cuộc sống sung sướng, huy hoàng, bạn nên xem xét công việc làm một tiếp viên
hàng không
D. Cái nhìn đầu tiên của bạn về một công việc có thể không chính xác.
Nhận định D xuất hiện ở đoạn 5 trong câu. Don’t rule out any occupation without learning more about it.
Đừng bỏ qua bất cứ công việc gì trước khi tìm hiểu kĩ về nó.
Và câu: Remember that many jobs are not what they appear to be at first, and may have merits or
demerits that are less obvious - Hãy nhớ rằng rất nhiều công việc không giống như vẻ bề ngoài lúc đầu
của nó, và có thể ẩn chứa những điểm tốt và không tốt mà sẽ không nhìn thấy rõ ngay được.
Các đáp án còn lại sai ở chỗ:
Đáp án A ở đoạn 3 có câu “If a high salary is important to you, do not judge a career by its starting
wages” Nếu lương cao là quan trọng với bạn thì đừng đánh giá một công việc từ mức lương khởi điểm.
Tác giả nói không nên đánh giá thấp những công việc bắt đầu bằng mức lương không cao nên A sai.
Đáp án B cũng tương tự đáp án A. Vì không nên đánh giá mức lương của công việc từ mức khởi đầu nên
việc bỏ qua các công việc ở nhà máy là sai.
Đáp án C tác giả đã đề cập đến sự vất vả của công việc tiếp viên hàng không (tham khảo câu 49) nên
không thể nói muốn sống sung sướng thì chọn nghề này. Do đó câu C sai

Trang 23
ĐỀ SỐ 10 ĐỀ THI THỬ TỐT NGHIỆP THPT
NĂM HỌC: 2020 – 2021
MÔN: TIẾNG ANH
Thời gian làm bài: 60 phút; không kể thời gian phát đề

Mark the letter A, B, C, or D on your answer sheet to indicate the word whose underlined part
differs from the other three in pronunciation in each of the following questions.
Question 1. A. wisdom B. minor C. lively D. wildness
Question 2. A. hatred B. naked C. knocked D. sacred
Mark the letter A, B, C or D on your answer sheet to indicate the word that differs from the rest in
the position of the main stress in each of the following questions.
Question 3. A. ignite B. igloo C. dogsled D. asset
Question 4. A. domestic B. dormitory C. dogmatic D. deliberate
Mark the letter A, B, C or D on your answer sheet to indicate the correct answer to each of the
following questions.
Question 5. “When is your math exam?” “Well, this time tomorrow I___________for it”.
A. will be sitting B. will sit C. will have been sitting D. sit
Question 6. The President mentioned pension reform and went on___________that he would reduce
unemployment figures.
A. adding B. add C. to be adding D. to add
Question 7. ___________you to be offered that job, would you have to move to another city?
A. Provided that B. Should C. Were D. Had
Question 8. I agree___________one point with Chris: it will be hard for us to walk 80km.
A. in B. of C. on D. for
Question 9. Up___________when it saw its master.
A. jumped the dog B. did the dog jump C. the dog jump D. does the dog jump
Question 10. ___________will be carried in the next space payload has not yet been announced to
the public.
A. It B. What C. When D. That
Question 11. ___________extremely bad weather in the mountains, we’re no longer considering
our skiing trip.
A. Due to B. Because C. Since D. Due to the fact that
Question 12. Having been selected to represent the Association of American Engineers at the
International Convention, ___________.
A. the members applauded him B. a speech had to be given by him
C. the members congratulated him D. he gave a short acceptance speech

Trang 1
Question 13. The authorities have shown no signs of ___________to the kidnappers’ demands.
A. standing up B. bringing about C. getting down D. giving in
Question 14. My main___________to the new motorway is that it will spoil the countryside.
A. objection B. object C. objective D. objecting
Question 15. Sue would suffer obesity if she couldn’t control her___________ overeating.
A. compulsive B. disastrous C. exhaustive D. unbearable
Question 16. After a six-year relation, Martha and Billy have decided to___________.
A. break the bank B. tie the knot C. turn the page D. make ends meet
Question 17. Most of the in this workshop do not work very seriously or productively.
A. tooth and nail B. rank and file C. eager beavers D. old hand
Question 18. A penalty shot is___________when a major foul is committed inside the 5-meter line.
A. prevented B. awarded C. committed D. ranged
Mark the letter A, B, C, or D on your answer sheet to indicate the underlined part that needs
correction in each of the following questions.
Question 19. The train to Ho chi Minh city left at precisely 7 o’clock as usually, but the train to Ha Noi
capital left at half past six o’clock, which was exactly 20 minutes late.
A. exactly B. as usually C. which D. precisely
Question 20. Sam found it hard to concentrate on his work because the noise.
A. it B. concentrate C. work D. because
Question 21. Peacocks are among the most exotic birds in nature; its long tail feathers fan out to reveal
a profusion of vivid colors.
A. fan out B. its C. most exotic D. among
Mark the letter A, B, C or D on your answer sheet to indicate the sentence that is closest in
meaning to each of the following questions.
Question 22. It is not until a Vietnamese girl getting 18 years old that she is allowed to get
married legally.
A. A Vietnamese girl is not allowed to get married legally only when she gets 18 years old.
B. A Vietnamese girl is allowed to get married legally only when she gets 18 years old.
C. The legal allowance for a Vietnamese girl to get married will be issued in 18 years.
D. They never allow a Vietnamese girl to get married legally when she is 18 years old.
Question 23. “I would be grateful if you could send me further information about the job.”, Lee
said to me.
A. Lee thanked me for sending him further information about the job.
B. Lee felt great because further information about the job had been sent to him.
C. Lee politely asked me to send him further information about the job.
D. Lee flattered me because I sent him further information about the job.

Trang 2
Question 24. The workers only complain because of their unfair treatment.
A. Were the workers fairly treated, they wouldn’t complain.
B. The workers complain because their employees are unfair.
C. If the workers are treated fairly, they will not complain.
D. Are the workers treated fairly, they will not complain.
Mark the letter A, B, C, or D on your answer sheet to indicate the sentence that a best combines
each pair of sentences in the following questions
Question 25. He was overconfident. Therefore, he ruined our plan completely.
A. It was because his overconfidence that ruined our plan completely.
B. He was overconfident, which ruined our plan completely.
C. That he was overconfidence ruined our plan completely.
D. It was his overconfidence ruined our plan completely.
Question 26. He felt very tired. However, he was determined to continue to climb up the
mountain.
A. Tired as he might feel, he was determined to continue to climb up the mountain.
B. As the result of his tiredness, he was determined to continue to climb up the mountain.
C. Feeling very tired, he was determined to continue to climb up the mountain
D. He felt so tired that he was determined to continue to climb up the mountain.
Mark the letter A, B, C, or D on your answer sheet to indicate the most suitable response to
complete each of the following questions.
Question 27. - “Excuse me. where's the parking lot?” - “___________”
A. You missed the turn. It's back that way.
B. Do you get lost? I do too.
C. You are going the wrong way. It's not here
D. Why do you ask me? I don't know.
Question 28. Hoa is asking Hai, who is sitting at a corner of the room, seeming too shy.
- Hoa: "Why aren't you taking part in our activities?” “__________”
- Hai: "Yes, I can. Certainly!"
A. Shall I take your hat off?
B. Can you help me with this decoration?
C. Can I help you?
D. Could you please show me how to get the nearest post office?
Mark the letter A, B, C, or D on your answer sheet to indicate the word(s) CLOSEST in meaning
to the underlined word(s) in each of the following questions.
Question 29. I think we should tell Peter that the location of the picnic has been changed. Let’s put
him in the picture.

Trang 3
A. show him B. take his photo C. explain to him D. inform him
Question 30. Mike pretended to be sick, but I saw through his deception at once.
A. was used to B. got angry with C. got bored with D. was aware of
Mark the letter A, B, C, or D on your answer sheet to indicate the word(s) OPPOSITE in meaning
to the underlined word(s) in each of the following questions.
Question 31. You’re 25 years old, but you still haven’t cut the apron strings.
A. become independent B. bought a new house
C. relied on others D. started doing well
Question 32. Though built almost five hundred years ago, the church remained practically intact.
A. in perfection B. in completion C. in chaos D. in ruins
Read the following passage and mark the letter A, B, C, or D on your answer sheet to indicate the
correct word or phrase that best fits each of the numbered blanks.
MODERN SCIENCE
It seems entirely natural to US that there are teams of scientists in universities and (33)
___________institutions around the world, attempting to discover the way the world works. (34)
___________, it hasn’t always been that way. Although the scientific method is now four or five
hundred years old, the ancient Greeks, for example, believed that they could (35) ___________the
cause of natural events just by the power of thought.
During the 17th century, more and more people began to realize that they could test their scientific
ideas by designing a relevant experiment and seeing what happened. A lot of (36) ___________was
made in this way by individual scientists. These men and women often worked alone, carrying out
research into many different areas of science, and they often received very little (37) ___________for
their hard work. At the start of the 20th century, though, it became clear that science was becoming
more complicated and more expensive. The individual scientists disappeared, to be replaced by highly
qualified teams of experts. Modern science was born.
Question 33. A. every B. whole C. other D. another
Question 34. A. However B. Accordingly C. Thus D. Besides
Question 35. A. solve out B. work out C. come out D. give out
Question 36. A. evolution B. progress C. movement D. development
Question 37. A. reward B. present C. gift D. prize
Read the following passage and mark the letter A, B, C, or D on your answer sheet to indicate the
correct answer to each of the questions.
Accidents do not occur at random. People eighty-five years of age and older are twenty-two times
likely to die accidentally than are children five to nine years old. The risk for native Americans is four
times that for Asian-Americans and twice that for white Americans or African-Americans. Males
suffer accidents at more than twice the rate of females, in part because they are more prone to risky

Trang 4
behavior. Alaskans are more than three times as likely as Rhode Islanders to die in an accident. Texans
are twenty- one times more likely than New Jerseyites to die in a natural disaster. Among the one
hundred most populous counties, Kern County, California (Bakersfield), has an accident fatality rate
three times greater than Summit County, Ohio (Akron).
Accidents happens more often to poor people. Those living in poverty receive inferior medical care,
are more apt to reside in houses with faulty heating and electrical systems, drive older cars with fewer
safety features, and are less likely to use safety belts. People in rural areas have more accidents than
city or suburban dwellers because farming is much riskier than working in a factory or office and
because emergency medical services are less readily available. These two factors - low income and
rural residence - may explain why the south has a higher accident rate than the north.
(Source: Proficiency Reading)
Question 38. which of the following is true according to the passage?
A. Children aged five to nine face the greatest accident risk.
B. All people face an equal risk of having an accident.
C. One in every 22 people aged 85 and over will die in an accident.
D. The risk of having an accident is greater among certain groups of people.
Question 39. The word “inferior” in the passage is closest in meaning to___________.
A. modern B. low-quality C. well-equipped D. unsafe
Question 40. According to the passage, which of the following groups of people in America face
the highest risk of having an accident?
A. Native Americans B. Asian-Americans
C. White Americans D. African-Americans
Question 41. What does the word “that” in the passage refer to___________?
A. males B. native Americans
C. the risk D. African-Americans
Question 42. which of the following is NOT mentioned as a reason for a higher accident rate
among the poor?
A. Little knowledge about safety.
B. Inadequate medical services.
C. Poor housing and working conditions.
D. Use of cars which incorporate fewer safety features.
Read the following passage and mark the letter A, B, C or D on your answer sheet to indicate the
correct answer to each of the following questions.
The concept of being environmentally conscious, or “green”, has become more prevalent in twenty
first-century U.S culture. It has begun to affect the manufacturing of everything from non- toxic
household cleaning products to motor vehicles powered by alternative sources of energy. However, one

Trang 5
way of being “green” that is perhaps not as apparent to the viewer but of equal importance in being
environmentally conscious, is the construction of buildings that are considered “sustainable”.
Sustainable buildings are those that do not impose on the environment or rely on the over-utilization of
energy or natural resources. There are four main principles of sustainability, which includes
consideration of the health and stability of all living things and their environmental diversity, as well as
the economic opportunities of humanity.
Sustainable architecture consists of environmentally conscious design techniques. In the past, the
demolition of an old building meant that all or most of the debris of the building would end up in a
landfill or a waste disposal site. Today, architects can plan and design a building that uses recycled
materials, such as wood, concrete, stone, or metal. These materials are salvaged from the demolition of
an older building and can be appropriately incorporated into a new construction. Architects and
construction supervisors may also choose to recycle more organic parts of demolished buildings, such
as wooden doors, windows and other glass, ceramics, paper, and textiles.
A problem that has often arisen has been with how a site crew-whether it is demolition or
construction crew determines and sorts what is “waste” and what is recyclable. Architects and
environmental scientists have to decide whether or not a material is appropriate for use in new
construction and how it will impact the environment. They must evaluate the materials from the
demolition and determine what those materials contain, and if they meet the standards set by the U.S,
government’s Environmental Protection Agency (the EPA). If the debris from the demolition contains
hazardous materials that are harmful to the environment or to the consumer, such as asbestos, then the
material is not salvageable. Use of the asbestos for insulation and as a form of fire retardation in
buildings and fabrics was common in the nineteenth century. Asbestos was once used in shingles on the
sides of buildings, as well as in the insulation in the interior walls of homes or other construction. In
new “green” construction, insulation that once asbestos- based can be replaced with recycled denim or
constructed with cellulose-a fibrous material found in paper products. The same-assessment applies to
wood or wallboard painted with toxic lead-based paints. In addition, gas-flow regulators and meters on
both water and gas heating systems constructed prior to 1961 must be carefully evaluated to determine
that they do not contain dangerous substances such as mercury. Mercury can be harmful to humans and
the environment if it is spilled during the removal of these devices.
Question 43. The word “prevalent” in paragraph 1 is closest in meaning to___________.
A. rare B. unusual C. widespread D. valuable
Question 44. In paragraph 1, the author implies that___________.
A. some companies manufacture vehicles that are not powered by gasoline
B. the concept of being green has not influenced manufacture
C. uses of alternative energy are apparent to the consumer
D. all buildings impose on the environment

Trang 6
Question 45. According to paragraph 1, which of the following is true?
A. All construction follows the concept of sustainable architecture.
B. Sustainable buildings do not overuse electricity, oil, or gas.
C. Todays cleaning products and appliances harm the environment.
D. Construction of “green” buildings is an old idea.
Question 46. In paragraph 2, the word “salvaged” is closest in meaning to___________.
A. lost B. destroyed C. buried D. saved
Question 47. According to paragraph 2, environmentally conscious design
incorporates___________.
A. safe, organic, recycled materials B. new wood, stone, or concrete
C. debris from a demolished building D. materials from a landfill
Question 48. In paragraph 3, the author mention both demolition and construction crews in order
to___________
A. give an example of how choosing materials is not easily determined on a site
B. illustrate the types of crews that sort waste and recyclables
C. demonstrate that choosing recyclable materials is challenging for both crews
D. contrast the work of the two types of crews on a site
Question 49. The word “they” in paragraph 3 refers to___________.
A. the EPA
B. the site crew
C. architects and environmental scientists
D. the materials from the demolition
Question 50. In paragraph 3, the author mentions all of the following hazardous materials found
in a debris from a demolition site EXCEPT___________.
A. cellulose B. mercury C. asbestos D. lead

Trang 7
Đáp án
1-A 2-C 3-A 4-B 5-A 6-D 7-C 8-C 9-A 10-B
11-A 12-D 13-D 14-A 15-A 16-B 17-D 18-B 19-B 20-D
21-B 22-B 23-C 24-A 25-B 26-A 27-A 28-B 29-D 30-D
31-C 32-D 33-C 34-A 35-B 36-B 37-A 38-D 39-B 40-A
41-C 42-A 43-C 44-A 45-B 46-D 47-A 48-C 49-D 50-A

LỜI GIẢI CHI TIẾT


Question 1: Đáp án A
Đáp án A đúng vì phần gạch chân của đáp án A được đọc là âm /ɪ/. Các phương án còn lại có phần gạch
chân được đọc là /aɪ/.
A. wisdom /ˈwɪzdəm/ (n): sự khôn ngoan
B. minor /ˈmaɪnə(r)/ (a)/(n)/(v): không quan trọng, nhỏ (a)/ vị thành niên (n)/ học cái gì như một đề tài
phụ (v)
C. lively /ˈlaɪvli/ (a): đây sức sống
D. wildness /ˈwaɪldnəs/ (n): sự hoang dã, sự bừa bãi.
Question 2. Đáp án C
Đáp án C đúng phần gạch chân của đáp án C được đọc là âm /t/. Các phương án còn lại có phần gạch
chân được đọc là âm /ɪd/
A. hatred /ˈheɪtrɪd/ (n): sự thù ghét
B. naked /ˈneɪkɪd/ (a): trần truồng
C. knocked /nɒkt/ (v): gõ cửa, đánh, gây ồn
D. sacred /ˈseɪkrɪd/ (a): linh thiêng
* Note: Chúng ta cần phải ôn lại quy tắc phát âm đuôi –ed
-ed được phát âm là: Khi tận cùng của từ là: Ví dụ
/id/ /t/ hoặc /d/ decided, accepted, visited
/t/ Các phụ âm vô thanh: /s/, /ʃ/, /tʃ/, missed, stopped, washed, ranked,
/k/, /p/, /f/, /θ/ approached, knocked
/d/ Các âm hữu thanh còn lại involved, planned, played
Tuy nhiên, đuôi “-ed” trong các động từ sau khi sử dụng như tính từ sẽ được phát âm là /ɪd/ bất luận “ed”
sau âm gì: aged, blessed, crooked, dogged, learned, naked, ragged, wicked, wretched. Nhưng khi sử dụng
như động từ, ta áp dụng quy tắc thông thường.
Từ hatred và sacred có đuôi “ed” được phát âm là /ɪd/ vì từ gốc của hai từ này là vậy rồi, không áp dụng
quy tắc phát âm “ed” cho hai từ này được.
Question 3. Đáp án A
Đáp án A đúng vì đáp án A có trọng âm rơi vào âm tiết thứ hai. Các phương án còn lại có trọng âm rơi
vào âm tiết thứ nhất.
Trang 8
A. ignite /ɪɡˈnaɪt/ (v): đốt cháy
B. igloo /ˈɪɡluː/ (n): lều tuyết
C. dogsled /ˈdɒɡsled/ (n): xe trượt tuyết do chó kéo
D. asset /ˈæset/ (n): tài sản
Question 4. Đáp án B
Đáp án B đúng vì đáp án B có trọng âm rơi vào âm tiết thứ nhất. Các phương án còn lại có trọng âm rơi
vào âm tiết thứ hai.
A. domestic /dəˈmestɪk/ (a)/(n): thuộc trong nưóc, thuộc gia đình (a)/ người ở, nội chiến (n)
B. dormitory /ˈdɔːmətri/ (n): ký túc xá
C. dogmatic /dɒɡˈmætɪk/ (a): độc đoán
D. deliberate /dɪˈlɪbərət/ (a)/(v): cố ý (a)/ đắn đo, suy xét (v)
Question 5. Đáp án A
Kiến thức được hỏi - Thì động từ
Ta thấy có trạng từ thời gian “this time tomorrow - vào thời điểm này ngày mai” chỉ thời điểm cụ thể
trong tương lai → loại D - chia thì hiện tại đơn, không phù hợp với thời gian trong tương lai.
Đối với trạng từ chỉ thời điểm cụ thể trong tương lai thì ta chia động từ ở thì tương lai tiếp diễn.
Do vậy, chọn đáp án A.
Tạm dịch: “Khi nào bạn thi toán?” - “Ổ, vào thời điểm này ngày mai tôi thi nhé”.
* Note: sit for : tham dự kỳ thi.
Question 6. Đáp án D
Kiến thức được hỏi - Cấu trúc với “go on”
Ta thấy “go on” là một phrasal verb nên đằng sau nó thường là danh từ hoặc động từ đuôi -ing. → Loại B
- đang ở động từ nguyên thể.
Ta có 2 cấu trúc của “go on” như sau:
Go on + V-ing : tiếp tục làm gì
Go on + to V: chuyển sang một chủ đề/giai đoạn khác
Căn cứ vào nghĩa của câu ta loại A và chọn được đáp án D. Loại C vì câu không cần phải mang nghĩa tiếp
diễn.
Tạm dịch: Tổng thống đề cập đến vấn đề cải cách lương hưu và chuyển sang nói thêm việc ông ấy sẽ làm
giảm con số thất nghiệp.
Question 7.
Kiến thức được hỏi - Đảo ngữ câu điều kiện loại II.
Nhìn vào 4 phương án và động từ của hai mệnh đề ta nghĩ ngay đến câu điều kiện. Ta thấy mệnh đề chính
có would you have to → câu điều kiện loại II. Suy ra về điều kiện động từ phải ở quá khứ đơn. Tuy nhiên
đây không phải là câu điều kiện bình thường với if mà là dạng đảo ngữ của câu điều kiện loại II vì về điều
kiện có you to be → Loại A vì Provided that - giả sử rằng - không được dùng cho đảo ngữ của câu điều

Trang 9
kiện. Loại B và D vì không dùng cho đảo ngữ của câu điều kiện loại II. Ta chọn C - were - đảo were lên
trước để biến thành đảo ngữ của câu điều kiện loại II.
Tạm dịch: Nếu bạn được yêu cầu công việc thì bạn sẽ phải di chuyển tới thành phố khác à?
* Mở rộng:
- Đối với câu điều kiện loại I ta dùng should lên trước chủ ngữ
Ex: If anyone calls, please take a message. (Nếu có ai đó gọi, làm ơn hãy để lại tin nhắn).
→ Should anyone call, please take a message.
- Đối với câu điều kiện loại II ta dùng were lên trước chủ ngữ
Ex: If I didn't read newspaper, I wouldn't know what was happening in the world. (Nếu tôi không đọc tờ
báo đó thì tôi đã không biết chuyện gì đang xảy ra trên thế giới)
→ Were I not to read newspaper, I wouldn't know what was happening in the world.
- Đối với câu điều kiện loại III ta dùng had lên trước chủ ngữ
If they had asked me, I could have given them some advice. (Giá như họ hỏi tôi thì tôi đã có thể cho họ
vài lời khuyên).
→ Had they asked me, I could have given them some advice
Question 8. Đáp án C
Kiến thức được hỏi - Giới từ
Ta có một số cấu trúc: agree (with sb) about/ on sth: đồng tình (với ai) về việc gì./ Agree to sth: chấp nhận
việc gì.
Trong 4 phương án có giới từ on nên chọn đáp án C. Các giới từ còn lại không đi với động từ agree
Tạm dịch: Tôi đồng ý với một quan điểm của Chris: Chúng ta sẽ khó đi bộ được 80km.
Question 9. Đáp án A
Kiến thức được hỏi - Đảo ngữ toàn phần
Ta có thể loại luôn được C và D vì không hòa hợp thì động từ. Câu đề bài là sự việc đã xảy ra rồi vì có từ
“saw” nên không thể chia động từ ở vế trước là hiện tại đơn được. Thêm nữa, động từ “jump” ở phương
án C không được chia nên loại.
Ta thấy với các trạng từ chỉ nơi chốn đứng đầu câu sẽ xuất hiện đảo ngữ toàn phần. Nếu như đảo ngữ với
các từ mang nghĩa phủ định đứng đầu câu thì ta đảo trợ động từ lên. Tuy nhiên, đảo ngữ với các trạng từ
chỉ nơi chốn đứng đầu câu thì các động từ chỉ sự di chuyển và vị trí được đảo lên trước chủ ngữ, không
cần mượn trợ động từ và chia thì như bình thường ( chỉ áp dụng khi chủ ngữ là danh từ). → Ta chọn được
đáp án A - động từ jump được đảo lên chia ở thì quá khứ đơn, phù hợp về thì và cấu trúc đảo toàn phần.
Tạm dịch: Con chó nhảy lên khi thấy chủ của nó.
Question 10.
Kiến thức được hỏi - Mệnh đề danh ngữ.
Ta thấy trong câu xuất hiện 2 động từ will be carried và has not yet been announced bị tách ra bởi các
thành phần khác, không được nối với nhau bằng các liên từ. Giả sử ghép It vào câu là hoàn toàn sai quy

Trang 10
định ngữ pháp trong Tiếng Anh - trong một câu không thể xuất hiện hai động từ khác thì mà không được
nối với nhau bằng liên từ hoặc các biện pháp nối khác → loại A.
Như vậy ta phải nghĩ ngay đến trường hợp có thể dùng mệnh đề quan hệ hoặc mệnh đề danh ngữ. Mệnh
đề quan hệ không thể đứng đầu câu mà không bổ sung thông tin cho danh từ nào phía trước nên loại
trường hợp dùng mệnh đề quan hệ. Mệnh đề danh ngữ thường bắt đầu bằng if/whether và các từ để hỏi
như what, why, when, where, .. .và từ that. Mệnh đề danh ngữ có chức năng như một danh từ, nó có thể
làm chủ ngữ, tân ngữ cho động từ, tân ngữ cho giới từ và làm bổ ngữ cho chủ ngữ.
Làm chủ ngữ: where/ when/ why/ what/ that...+ S+ V + V. → động từ chính luôn chia số ít.
Làm tân ngữ cho động từ: S + V + what/ where/ when/ why/ that….+ S + V
Làm tân ngữ cho giới từ: S + V/be adj + preposition + where/ what/ when/ why/ that....+ S + V.
Làm bổ ngữ cho chủ ngữ: S + to be + what/ where/ when/ why/ that....+ S+ V
Thậm chí các từ để hỏi có thể làm chủ ngữ luôn trong mệnh đề danh ngữ và câu này là một ví dụ.
Căn cứ vào cấu tạo thành phần câu và nghĩa ta chọn what.
Tạm dịch: Cái gì sẽ được mang trong tải trọng không gian sắp tới thì vẫn chưa được công bố tới công
chúng.
Question 11. Đáp án A
Kiến thức được hỏi - Cụm từ chỉ nguyên nhân
Ta thấy đằng sau chỗ trống là một cụm danh từ “bad weather” nên loại được B, C và D luôn. Chọn được
đáp án A.
Due to + N/V-ing: bởi vì
Because/ Since/ Due to the fact that + clause : bởi vì
Tạm dịch: Bởi vì thời tiết khá tệ trên núi nên chúng tôi không còn xem xét chuyến đi trượt tuyết nữa.
Question 12. Đáp án D
Kiến thức được hỏi - Mệnh đề phân từ
Ta thấy có “Having been selected” là dạng phân từ hoàn thành được sử dụng để nối hai mệnh đề cùng chủ
ngữ chỉ sự việc xảy ra có trước có sau. Như vậy chủ ngữ của mệnh đề phía sau chính là chủ ngữ của mệnh
đề phía trước. Ta thấy động từ của mệnh đề đầu được chia ở dạng bị động “Having been selected to
represent - được bầu là đại diện cho”. Xét 4 phương án chỉ có đáp án D là hợp nghĩa. Các phương án còn
lại không hợp nghĩa của câu nên loại A, B và C.
Tạm dịch: Sau khi được bầu là đại diện cho Hiệp hội Kỹ sư Mỹ ở Hội nghị quốc tế, anh ấy đã đưa ra một
bài phát biểu nhận chức.
Question 13. Đáp án D
Kiến thức được hỏi - Phrasal verb
Stand up (ph.v): đứng lên, ủng hộ
Bring about sth (ph.v): gây ra, mang lại cái gì
Get down to sth (ph.v): bắt tay vào việc gì

Trang 11
Give in to sth (ph.v): nhượng bộ, đầu hàng
Căn cứ vào nghĩa ta chọn đáp án D.
Tạm dịch: Chính quyền không có dấu hiệu nhượng bộ trước những yêu cầu của kẻ bắt cóc.
Question 14. Đáp án A
Kiến thức được hỏi - Loại từ
Ta thấy trước chỗ trống là định từ “my” + tính từ → Chỗ trống cân một danh từ → loại C
Thêm nữa, có giới từ to ở sau nên loại B vì object - vật cũng là một danh từ nhưng danh từ này không đi
với giới từ to. Ta chỉ có động từ object to V-ing/N - phản đối việc gì và khi chuyên sang danh từ sẽ thành
objection to V-ing/N - sự phản đối việc gì → Chọn A.
Không chọn D vì objecting - việc phản đối không phù hợp về nghĩa
Tạm dịch: Sự phản đối chủ yếu của tôi về đường cao tốc mới là nó sẽ làm hỏng khu vực nông thôn.
Question 15. Đáp án A
Kiến thức được hỏi - Lựa chọn từ
Compulsive /kəmˈpʌlsɪv/ (a): làm gì quá nhiều và không thể ngừng được
Disastrous /dɪˈzɑːstrəs/ (a): thảm khốc
Exhaustive /ɪɡˈzɔːstɪv/ (a): toàn diện
Unbearable /ʌnˈbeərəbl/ (a): không chịu nổi
Căn cứ vào nghĩa ta chọn đáp án A.
Tạm dịch: Sue sẽ chịu sự béo phì nếu cô ấy không thể kiểm soát được việc ăn không ngừng nghỉ như vậy.
Question 16. Đáp án B
Kiến thức được hỏi - Thành ngữ
Break the bank: tiêu nhiều tiền cho cái gì
Tie the knot: kết hôn
Turn the page: cuộc đời sang trang
Make ends meet: kiếm tiền trang trải cuộc sống
Căn cứ vào nghĩa chọn đáp án B.
Tạm dịch: Sau 6 năm gắn bó, Martha và Billy đã quyết định kết hôn
Question 17. Đáp án B
Kiến thức được hỏi - Thành ngữ
Tooth and nail: đánh nhau dữ dội, cấu xé nhau
Rank and file: các thành viên bình thường của một tổ chức
Eager beavers: người nhiệt tình
Old hand: người lão luyện trong nghề
Căn cứ vào nghĩa ta chọn đáp án B.
Tạm dịch: Hầu hết thành viên ở hội thảo này đều không làm việc nghiêm túc và hiệu quả.
Question 18.

Trang 12
Kiến thức được hỏi - Lựa chọn từ
Prevent (v): ngăn cản ai đó khỏi việc gì ( + O + from + V-ing)
Award (v): trao giải, hưởng một cái gì đó
Commit (v): cam kết việc gì đó (to + V-ing)
Range (v): trải dài
Căn cứ vào nghĩa ta chọn đáp án B
Tạm dịch: Một cú sút phạt được trao khi phạm lỗi lớn được thực hiện trong phạm vi 5 mét.
Question 19. Đáp án B
Lỗi sai về thành ngữ
Ta có thành ngữ as usual - như thường lệ nên B là đáp án đúng.
Sửa: as usually → as usual
Tạm dịch: Chuyến tàu đến thành phố Hồ Chí Minh đã rời chính xác vào lúc 7 giờ như thường lệ, nhưng
chuyến tàu về thủ đô Hà Nội rời đi lúc 6 giờ 30 phút, như vậy là muộn đúng 20 phút.
Question 20.
Lỗi sai về từ nối
Sau “because” là một mệnh đề (S + V +…..).
Tuy nhiên trong đề bài sau “because” là danh từ “the noise”. Ta cần một từ nối khác để nối danh từ này.
Sửa: because → because of
Tạm dịch: Sam thấy khó để tập trung vào công việc bởi vì tiếng ồn.
Question 21. Đáp án B
Lỗi sai về đại từ
Ta thấy đằng trước có danh từ Peacocks ở dạng số nhiều và mệnh đề phía sau để tránh lặp lại đã dùng
định từ its. Tuy nhiên Peacocks ở dạng số nhiều nên không thể dùng its (số ít) mà phải dùng định từ mang
nghĩa nhiều. → Chọn đáp án B
Sửa: its → their
Tạm dịch: Con công là một trong những loài chim kỳ lạ nhất trong tự nhiên; lông đuôi dài của chúng để
lộ ra màu sắc thật sống động.
Question 22. Đáp án B
Dịch câu đề: Cho đến tận 18 tuổi thì một cô gái Việt Nam mới được phép kết hôn hợp pháp.
A. Một cô gái Việt Nam không được phép kết hôn hợp pháp chỉ khi cô ấy 18 tuổi. → sai nghĩa so với câu
gốc.
B. Một cô gái Việt Nam được phép kết hôn hợp pháp chỉ khi cô ấy 18 tuổi. → đúng
C. Sự cho phép hợp pháp cho một cô gái Việt Nam kết hôn sẽ được ban hành trong 18 năm nữa. → sai
nghĩa so với câu gốc.
D. Họ không bao giờ cho phép một cô gái Việt Nam kết hôn hợp pháp khi cô ấy 18 tuổi → sai nghĩa so
với câu gốc.

Trang 13
→ Chọn đáp án B.
Question 23. Đáp án C
Dịch câu đề: “Tôi sẽ biết ơn nếu bạn có thể gửi cho tôi thêm thông tin về công việc”., Lee nói với tôi.
A. Lee cảm ơn tôi về việc đã gửi cho anh ấy thêm thông tin về công việc. → sai nghĩa so với câu gốc
B. Lee cảm thấy tuyệt vời bởi vì thông tin thêm vể công việc đã được gửi cho anh ấy. → sai nghĩa so với
câu gốc.
C. Lee lịch sự yêu cầu tôi gửi cho anh ấy thêm thông tin về công việc. → đúng.
D. Lee tâng bốc tôi bởi vì tôi đã gửi cho anh ấy thêm thông tin về công việc. → sai nghĩa so với câu gốc.
Câu đề bài cho là một cách nói lịch sự khi ai đó muốn có thêm thông tin từ người khác.
Như vậy ta chọn được đáp án C.
Question 24. Đáp án A
Dịch câu đề: Những người công nhân chỉ phàn nàn bởi vì sự đối xử không công bằng (ngụ ý nếu được đối
xử công bằng họ sẽ không phàn nàn).
A. Nếu những người công nhân được đối xử một cách công bằng, họ sẽ không phàn nàn → đúng
B. Nhũng người công nhàn phàn nàn bởi vì nhân viên của họ không công bằng → sai nghĩa so với câu
gốc.
C. Không dịch vì sai ngữ pháp: Câu đề bài thực tế là ở hiện tại đơn nên khi giả sử một sự việc không có
thật ở hiện tại thì ta phải viết lại ở câu điều kiện loại II.
D. Không dịch vì sai ngữ pháp: đây không phải là câu điểu kiện, nếu là câu điều kiện dạng đảo thì phải
dùng các từ khác, không dùng Are.
Như vậy chọn đáp án A, dùng câu điều kiện loại II để giả sử một điều không có thật ở hiện tại.
Question 25. Đáp án B
Dịch câu đề: Anh ấy quá tự tin. Vì vậy, anh ấy đã phá hỏng hoàn toàn kế hoạch của chúng tôi.
A. Không dịch vì sai ngữ pháp: sau because là cụm danh từ his overconfidence...là sai vì sau because phải
là một mệnh đề.
B. Anh ấy quá tự tin, điều này đã phá hỏng hoàn toàn kế hoạch của chúng tôi → đúng, dùng đại từ which
để thay thế cho cả sự việc phía trước.
C. Không dịch vì sai ngữ pháp: sau was là danh từ overconfidence là sai vì tobe phải đi với tính từ hoặc
nếu là danh từ thì phải có một từ xác định trước danh từ.
D. Không dịch vì sai ngữ pháp: Trong câu xuất hiện hai động từ không được nối với nhau bằng các liên từ
thì phải nghĩ ngay đến trường hợp mệnh đề quan hệ đầy đủ hoặc tỉnh lược. Giả sử câu này là mệnh đề
quan hệ được tỉnh lược thì động từ ruined có chủ ngữ là danh từ overconfidence phía trước → không hợp
lý vì mang nghĩa bị động, không thể nói sự quá tự tin bị phá hủy kế hoạch → sai nghĩa → loại D.
Như vậy ta chọn được đáp án B.
Question 26. Đáp án A
Dịch câu đề: Anh ấy cảm thấy rất mệt mỏi. Tuy nhiên, anh ẫy vẫn quyết tâm tiếp tục leo núi.

Trang 14
A. Mặc dù cảm thấy mệt mỏi, anh ấy vẫn quyết tâm tiếp tục leo núi. → đúng
B. Kết quả của sự mệt mỏi của anh ấy là anh ấy quyết tâm tiếp tục leo núi. → sai nghĩa so với câu gốc.
C. Cảm thấy rất mệt mỏi nên anh ấy quyết tâm tiếp tục leo núi. → sai nghĩa so vái câu gốc.
D. Anh ấy rất mệt mỏi đến nỗi anh ấy quyết tâm tiếp tục leo núi. → sai nghĩa so với câu gốc.
Như vậy ta chọn được đáp án A. (cấu trúc số 5)
* Mở rộng:
Hai câu mang nghĩa tương phản có thể được nối với nhau bằng các cấu trúc sau:
1. No matter + what/who/when/where/why + S + V
2. No matter + how + adj/adv + S+ V
3. Whatever (+ noun)/ whoever/ wherever/ whenever + S + V
4. However + adj/adv + S + V
5. Adj/ adv + as/though + S + V
Question 27. Đáp án A
Dịch đề: - “Xin lỗi. Bãi đỗ xe ở đâu vậy?” – ___________
A. Bạn vượt qua lối rẽ rồi. Bạn quay lại lối đó đi.
B. Bạn có bị lạc không? Tôi cũng vậy
C. Bạn đang đi sai đường. Nó không ở đây
D. Tại sao bạn hỏi tôi? Tôi không biết đâu.
Như vậy đáp án A là phù hợp để đáp lại câu hỏi địa điểm.
Question 28. Đáp án B
Dịch đề: Hoa đang hỏi Hải, người đang ngồi ở một góc phòng, dường như rất ngại ngùng.
- Hoa: “Tại sao bạn không tham gia vào các hoạt động của chúng tôi?” “___________”
- Hải: “Vâng, tôi có thể. Chắc chắn đó!”
A. Tôi bỏ mũ xuống được chứ?
B. Bạn có thể giúp tôi trang trí không?
C. Tôi có thể giúp gì cho bạn?
D. Bạn có thể vui lòng chỉ cho tôi cách đến bưu điện gần nhất được không?
Đoạn hội thoại đang đề cập đến việc tham gia các hoạt động nên đáp án B là phù hợp nhất - hoạt động
trang trí.
Question 29. Đáp án D
Ta có thành ngữ: put sb in the picture - nói cho ai biết. Trong câu này chọn đáp án D là gần nghĩa nhất với
từ đề bài cho.
Show him: thể hiện cho anh ấy
Take his photo: chụp ảnh của anh ấy
Explain to him: giải thích cho anh ấy
Inform him: thông báo cho anh ấy biết

Trang 15
Tạm dịch: Tôi nghĩ chúng ta nên nói cho Peter biết rằng địa điểm của chuyến đi dã ngoại đã được thay
đổi. Hãy thông báo cho anh ấy biết.
Question 30. Đáp án D
Ta có cụm động từ see through - nhìn thấu được điều gì
Was used to: quen với việc gì ( + V-ing)
Got angry with sb: giận dữ với ai
Got bored with sth: chán nản điều gì
Was aware of sth: nhận thức được về điều gì
Xét về nghĩa ta chọn được đáp án D là từ gần nghĩa nhất với từ đề bài cho.
Tạm dịch: Mike giả vờ ốm nhưng tôi đã nhìn thấu được sự lừa dối của anh ấy ngay lập tức.
Question 31. Đáp án C
Ta có thành ngữ: cut the apron strings - độc lập, tự lập
Become independent: trở nên tự lập
Bought a new house: mua một ngôi nhà mới
Relied on others: phụ thuộc vào những cái khác
Started doing well: bắt đầu làm tốt
Như vậy ta chọn đáp án C trái nghĩa với từ đề bài cho.
Tạm dịch: Bạn đã 25 tuổi rồi nhưng vẫn chưa tự lập gì hết.
Question 32. Đáp án D
Ta có: intact (adj) - còn nguyên vẹn
In perfection: hoàn hảo
In completion: toàn vẹn
In chaos: hỗn loạn
In ruins: bị phá hủy hoàn toàn
Như vậy ta chọn được đáp án D là từ trái nghĩa với từ đề bài cho.
Tạm dịch: Mặc dù được xây cách đây 500 năm nhưng nhà thờ vẫn còn nguyên vẹn.
Question 33. Đáp án C
Ta thấy có danh từ số nhiều institutions ở phía sau. Xét 4 phương án:
every: mỗi (đi với danh từ đếm được số ít)
whole: toàn bộ
other: khác (là một tính từ, đứng trước danh từ số nhiều)
another: một cái khác
Dựa vào nghĩa và cách sử dụng chọn được đáp án C.
Trích bài: It seems entirely natural to us that there are teams of scientists in universities and other
institutions around the world, attempting to discover the way the world works.

Trang 16
Tạm dịch: Dường như một cách hoàn toàn tự nhiên với chúng ta rằng có các nhóm nhà khoa học ở các
trường đại học và các tổ chức khác trên thế giới đang cố gắng khám phá cách thế giới hoạt động.
Question 34.
However : Tuy nhiên (nối hai câu hoặc mệnh để mang nghĩa tương phản)
Accordingly: Một cách phù hợp
Thus: Vì vậy, cho nên (nối hai câu mang nghĩa nguyên nhân - kết quả)
Besides: Bên cạnh đó, ngoài ra (nối hai câu mang nghĩa bổ sung)
Dựa vào nghĩa ta chọn được đáp án A
Trích bài: However , it hasn’t always been that way.
Tạm dịch: Tuy nhiên, không phải lúc nào cũng vậy.
Question 35. Đáp án B
Solve sth (v): giải quyết vấn đề (không đi với “out”)
Work out sth (ph.v): tìm ra đáp án, cách giải quyết
Come out (ph.v): xuất bản
Give out (ph.v): ban, phát
Dựa vào nghĩa ta chọn đáp án B.
Trích bài: Although the scientific method is now four or five hundred years old, the ancient Greeks, for
example, believed that mi they could work out the cause of natural events just by the power of thought.
Tạm dịch: Mặc dù phương pháp khoa học hiện nay đã bốn hoặc năm trăm năm tuổi nhưng người Hy Lạp
cổ đại chẳng hạn, tin rằng họ có thể tìm ra nguyên nhân của các sự kiện tự nhiên chỉ bằng năng lực tư
tưởng.
Question 36. Đáp án B
Evolution (n): sự tiến hóa, sự phát triển
Progress (n): sự tiến bộ
Movement (n): sự vận động, phong trào
Development (n): sự phát triển
Căn cứ vào nghĩa ta chọn đáp án B
Trích bài: During the 17th century, more and more people began to realize that they could test their
scientific ideas by designing a relevant experiment and seeing what happened. A lot of progress was
made in this way by individual scientists.
Tạm dịch: Trong suốt thế kỷ 17, ngày càng nhiều người bắt đầu nhận ra rằng họ có thể thử nghiệm các ý
tưởng khoa học bằng việc thiết kế một thí nghiệm có liên quan và quan sát những gì xảy ra. Rất nhiều tiến
bộ được thực hiện theo cách này bởi cá nhân các nhà khoa học.
Question 37. Đáp án A
Reward (n): phần thưởng vì đã làm việc tốt, làm việc chăm chỉ
Present = gift (n): món quà

Trang 17
Prize (n): giải thưởng
Căn cứ vào nghĩa ta chọn đáp án A.
Trích bài: These men and women often worked alone, carrying out research into many different areas of
science, and they often received very little reward for their hard work. At the start of the 20th century,
though, it became clear that science was becoming more complicated and more expensive. The individual
scientists disappeared, to be replaced by highly qualified teams of experts. Modern science was born.
Tạm dịch: Những người đàn ông và phụ nữ này thường làm việc một mình, thực hiện nghiên cứu trong
nhiều lĩnh vực khoa học khác nhau và họ thường nhận được rất ít phần thưởng cho công việc khó khăn
của họ. Mặc dù vậy, đầu thế kỷ 20, có một điểu rõ ràng rằng khoa học ngày càng trở nên phức tạp và mất
khá nhiều chi phí. Các nhà khoa học làm việc một mình này dần biến mất, được thay thế bằng các nhóm
chuyên gia chất lượng cao. Khoa học hiện đại được ra đời.
Question 38. Đáp án D
Dịch đề: Ý nào sau đây thì đúng theo đoạn văn?
A. Trẻ em từ năm đến chín tuổi đối mặt với tỷ lệ tai nạn cao nhất.
B. Tất cả mọi người đối mặt với tỷ lệ tai nạn như nhau.
C. Cứ 22 người thì có 1 người trong độ tuổi 85 sẽ chết vì tai nạn.
D. Nguy cơ gặp tai nạn cao hơn trong một số nhóm người nhất định.
Thông tin nằm ở ngay những dòng đầu:
Trích bài: People eighty-five years of age and older are twenty-two times likely to die accidentally than
are children five to nine years old - Những người ở độ tuổi 85 hoặc hơn có khả năng chết do tai nạn cao
gấp 22 lần so với trẻ em từ độ tuổi 5 đến 9.→ có nghĩa trẻ em ở độ tuổi 5-9 không thể có nguy cơ cao nhất
được
→ Thông tin không ủng hộ phương án A
→ Không ủng hộ cả phương án B - không phải tất cả mọi người có nguy cơ bị tai nạn như nhau
→ Không ủng hộ cả phương án C - cao gấp 22 lần chứ không phải 22 người thì có 1 người ở độ tuổi 85 bị
tai nạn.
Ta chọn được đáp án D - Tỷ lệ tai nạn cao hơn ở một số nhóm người nhất định: Nhóm người 5-9 tuổi -
nhóm người từ 85 trở lên. Nếu đọc tiếp thì còn có cả nhóm người da đỏ và người Mỹ gốc Á,….
Question 39. Đáp án B
Dịch đề: Từ “inferior” trong đoạn văn thì gẩn nghĩa nhất với____________
A. Hiện đại B. Kém chất lượng
C. Trang bị tốt D. Không an toàn
Ta chọn được đáp án B là từ gần nghĩa với từ đề bài cho
Trích bài: Those living in poverty receive inferior medical care, are more apt to reside in houses with
faulty heating and electrical systems, drive older cars with fewer safety features, and are less likely to use
safety belts

Trang 18
Tạm dịch: Những người sống nghèo khổ nhận được sự chăm sóc y tế kém và họ ở trong những ngôi nhà
có các hệ thống điện và sưởi bị lỗi, lái xe cũ với những tính năng ít an toàn và dường như không sử dụng
dây đeo an toàn.
Question 40. Đáp án A
Dịch đề: Theo đoạn văn, nhóm nào sau đây ở Mỹ phải đối mặt với nguy cơ tai nạn cao nhất?
A. Người da đỏ
B. Người Mỹ gốc Á
C. Người Mỹ trắng
D. Người Mỹ gốc Phi
Ta tìm thấy thông tin ở câu số 3.
Trích bài: The risk for native Americans is four times that for Asian-Americans and twice that for white
Americans or African- Americans.
Tạm dịch: Nguy cơ bị tai nạn cho người da đỏ thì cao gấp 4 lần người Mỹ gốc Á và cao gấp 2 lần người
Mỹ da trắng hoặc gốc Phi
Từ đó suy ra người Mỹ bản địa có nguy cơ tai nạn cao nhất → chọn đáp án A.
Question 41. Đáp án C
Dịch đề: Từ “that” trong đoạn văn đề cập tới điều gì?
A. Nam giới B. Người da đỏ
C. Nguy cơ D. Người Mỹ gốc Phi
Trích bài: The risk for native Americans is four times that for Asian-Americans and twice that for white
Americans or African- Americans.
Tạm dịch: Nguy cơ bị tai nạn cho người da đỏ thì cao gấp 4 lần người Mỹ gốc Á và cao gấp 2 lần người
Mỹ da trắng hoặc gốc Phi.
Để tránh lặp lại từ phía trước và đang so sánh nguy cơ tai nạn của người da đỏ với nguy cơ tai nạn của
người Mỹ gốc Á hoặc gốc Phi → chọn đáp án C.
Question 42. Đáp án A
Dịch đề: Điều nào KHÔNG được đề cập như một lý do cho tỷ lệ tại nạn cao hơn trong nhóm người
nghèo?
A. ít kiến thức về an toàn.
B. Thiếu dịch vụ y tế.
C. Nhà ở và điều kiện làm việc kém.
D. Sử dụng ô tô kết hợp ít tính năng an toàn hơn.
Ta tìm thấy thông tin ở đoạn: Those living in poverty receive inferior medical care, are more apt to reside
in houses with faulty heating and electrical systems, drive older cars with fewer safety features, and are
less likely to use safety belts

Trang 19
Tạm dịch: Những người sống nghèo khổ nhận được sự chăm sóc y tế kém và họ ở trong những ngôi nhà
có các hệ thống điện và sưởi bị lỗi, lái xe cũ với những tính năng ít an toàn và dường như không sử dụng
dây đeo an toàn.
Như vậy, B, C và D đều xuẫt hiện trong đoạn trên là lí do gặp tai nạn cao ở nhóm người nghèo. Duy nhất
A không được đề cập nên chọn đáp án A.
Question 43. Đáp án C
Dịch để: Từ “prevalent” trong đoạn 1 gần nghĩa với____________
A. Qúy hiếm/ hiếm khi
B. Khác thường
C. Rộng rãi/ phổ biến
D. Có giá trị
Prevalent (adj) = widespread (adj): rộng rãi, phổ biến
Trích bài: The concept of being environmen-tally conscious, or “green”, has become more prevalent in
twenty first-century U.S culture.
Tạm dịch: Quan niệm vẽ ý thức về bảo vệ môi trường hay còn gọi là “sống xanh” ngày càng trở nên phổ
biến hơn trong văn hóa Hoa Kỳ thế kỷ hai mốt.
→ Chọn đáp án C.
Question 44. Đáp án A
Dịch đề: Ở đoạn 1, tác giả ngụ ý___________
A. Một số công ty chế tạo phương tiện không chạy bằng xăng dầu
B. Khái niệm xanh chưa ảnh hưởng đến sản xuất
C. Sử dụng năng lượng thay thế phổ biến đối với người tiêu dùng
D. Tất cả các tòa nhà đều đặt áp lực lên môi trường
1. It has begun to affect the manufacturing of everything from non- toxic household cleaning products to
motor vehicles powered by alternative sources of energy. - Nó đã bắt đầu ảnh hưởng đến việc sản xuất
mọi thứ từ các sản phẩm tẩy rửa gia dụng không độc hại đến những chiếc xe cơ giới được chạy bằng các
nguồn năng lượng thay thế. → Thông tin không ủng hộ phương án B → Ngụ ý các công ty chế tạo phương
tiện không chạy bằng xăng dầu, thay vào đó chúng được chạy bằng các nguồn năng lượng thay thế
2. Sustainable buildings are those that do not impose on the environment or rely on the over-utilization of
energy or natural resources - Các tòa nhà bền vững là những tòa nhà không đặt áp lực lên môi trường
hoặc dựa vào việc sử dụng quá mức năng lượng hoặc tài nguyên thiên nhiên. → Thông tin không ủng hộ
D.
Phương án C không được đề cập trong đoạn văn nên loại.
→ Ta chọn đáp án A
Question 45. Đáp án B
Dịch đề: Theo đoạn 1, ý nào dưới đây thì đúng?

Trang 20
A. Tất cả các công trình xây dựng theo quan niệm về kiến trúc bền vững.
B. Các tòa nhà bền vững không lạm dụng điện, dầu hoặc khí đốt.
C. Ngày nay, các sản phẩm và thiết bị làm sạch làm hại môi trường.
D. Xây dựng các công trình xanh là một ý tưởng cũ.
1. The concept of being environmentally conscious, or “green”, has become more prevalent in twenty
first-century U.S culture - Quan niệm vể ý thức về bảo vệ môi trường hay còn gọi là “sống xanh” ngày
càng trở nên phổ biến hơn trong văn hóa Hoa Kỳ thế kỷ hai mốt. → Việc xây dựng các công trình xanh
nằm trong quan niệm về ý thức bảo vệ môi trường xanh sạch phổ biến ở thế kỷ 21, thời gian ở hiện tại
nên không thể nói nó là một ý tưởng cũ. → Thông tin không ủng hộ D
2. It has begun to affect the manufacturing of everything from non- toxic household cleaning products to
motor vehicles powered by alternative sources of energy. - Nó đã bắt đầu ảnh hưởng đến việc sản xuất
mọi thứ từ các sản phẩm tẩy rửa gia dụng không độc hại đến những chiếc xe cơ giới được chạy bằng các
nguồn năng lượng thay thế → Quan niệm này ảnh hưởng đến việc sản xuất mọi thứ từ các sản phẩm làm
sạch/ tẩy rửa gia dụng, không có nghĩa là các sản phẩm này làm hại môi trường → Thông tin không ủng
hộ C.
3. Sustainable buildings are those that do not impose on the environment or rely on the over-utilization of
energy or natural resources - Các tòa nhà bền vững là những tòa nhà không đặt áp lực lên môi trường
hoặc dựa vào việc sử dụng quá mức năng lượng hoặc tài nguyên thiên nhiên, → Ngụ ý rằng các tòa nhà
bền vững không lạm dụng điện, dầu hoặc khí đốt vì dầu, điện và khí đốt cũng là các nguồn năng lượng
hoặc tài nguyên thiên nhiên.
Phương án A không được nói đến trong bài là “Tất cả” các công trình xây dựng theo quan niệm về kiến
trúc bền vững. → Loại A
Do vậy ta chọn được đáp án B
Question 46.
Dịch đề: Trong đoạn 2, từ “salvaged” gần nghĩa nhất với___________
A. Mất
B. Phá hủy
C. Chôn cất
D. Cứu, để dành, giữ lại
Salvaged (v) = saved (v) : giữ lại, để dành
Trích bài: These materials are salvaged from the demolition of an older building and can be appropriately
incorporated into a new construction.
Tạm dịch: Những vật liệu này được tận dụng/ giữ lại từ việc phá hủy một tòa nhà cũ và có thể được kết
hợp một cách thích hợp trong một công trình mới.
Question 47. Đáp án A
Dịch đề: Theo đoạn 2, thiết kế mang ý thức bảo vệ môi trường kết hợp____________

Trang 21
A. Vật liệu tái chế hữu cơ an toàn
B. Gỗ, đá hoặc bê tông mới
C. Mảnh vỡ từ một tòa nhà bị phá hủy
D. Vật liệu từ bãi rác
Trích bài: Today, architects can plan and design a building that uses recycled materials, such as wood,
concrete, stone, or metal. These materials are salvaged from the demolition of an older building and can
be appropriately incorporated into a new construction. Architects and construction supervisors may also
choose to recycle more organic parts of demolished buildings, such as wooden doors, windows and other
glass, ceramics, paper, and textiles.
Tạm dịch: Ngày nay, các kiến trúc sư có thể lập kế hoạch và thiết kế một tòa nhà sử dụng vật liệu tái chế,
chẳng hạn như gỗ, bê tông, đá hoặc kim loại. Những vật liệu này được tận dụng từ việc phá hủy một tòa
nhà cũ và có thể được kết hợp một cách thích hợp một công trình mới. Kiến trúc sư và giám sát thi công
cũng có thể chọn tái chế hữu cơ nhiều phần của tòa nhà bị phá hủy, như cửa gỗ, cửa sổ và các loại kính
khác, gốm sứ, giấy và vải vóc.
→ Kết hợp tất cả các ý lại thì đó là những vật liệu tái chế hữu cơ an toàn → chọn đáp án A. Các phương
án B, C chỉ là một ý nhỏ; phương án D không được nhắc đến.
Question 48. Đáp án C
Dịch đề: Theo đoạn 3, tác giả đề cập đến các đội phá hủy và xây dựng để___________.
A. Đưa ra một ví dụ về cách chọn vật liệu không dễ dàng xác định ở một địa điểm
B. Minh họa các đội phân loại rác thải và rác tái chế
C. Chứng minh rằng việc lựa chọn vật liệu có thể tái chế là thách thức đối với cả hai đội
D. Đối chiếu công việc của hai đội trên ở một địa điểm
Trích bài: A problem that has often arisen has been with how a site crew-whether it is demolition or
construction crew determines and sorts what is “waste” and what is recyclable.
Tạm dịch: Một vấn đề thường xảy ra là làm thế nào một đội ngũ - cho dù là đội phá hủy hay đội xây dựng
xác định và phân loại những gì là rác thải và những gì có thể tái chế. → Ta chọn ngay được đáp án C
Question 49.
Dịch đề: Từ “they” ở đoạn 3 chỉ____________
A. EPA (Cơ quan Bảo vệ Môi trường của chính phủ Hoa Kỳ)
B. Đội xây dựng
C. Các kiến trúc sư và nhà khoa học môi trường
D. Các vật liệu từ việc phá hủy
Trích bài: They must evaluate the materials from the demolition and determine what those materials
contain, and if they meet the standards set by the U.S, governments Environmental Protection Agency
(the EPA).

Trang 22
Tạm dịch: Họ phải đánh giá các vật liệu từ việc phá hủy và xác định những vật liệu đó chứa gì, và liệu
chúng có đáp ứng các tiêu chuẩn của Cơ quan Bảo vệ Môi trường của chính phủ Hoa Kỳ (EPA).
Như vậy “they” ở đây chính là các vật liệu từ việc phá hủy
Question 50. Đáp án A
Dịch đề: Trong đoạn 3, tác giả đã đề cập đến tất cả các vật liệu nguy hiểm sau đây được tìm thấy trong
một mảnh vỡ từ một địa điểm phá hủy NGOẠI TRỪ_____________.
A. Xenluloza B. Thủy ngân
C. Amiăng D. chì
Trích bài: If the debris from the demolition contains hazardous materials that are harmful to the
environment or to the consumer, such as asbestos, then the material is not salvageable. In new “green”
construction, insulation that once asbestos- based can be replaced with recycled denim or constructed
with cellulose-a fibrous material found in paper products .The same- assessment applies to wood or
wallboard painted with toxic lead-based paints. In addition, gas-flow regulators and meters on both water
and gas heating systems constructed prior to 1961 must be carefully evaluated to determine that they do
not contain dangerous substances such as mercury.
Tạm dịch: Nếu các mảnh vụn từ việc phá hủy có chứa các vật liệu nguy hiểm có hại cho môi trường hoặc
người tiêu dùng, chẳng hạn như amiăng, thì vật liệu này không thể tận dụng được. Trong xây dựng xanh
mới, cách nhiệt đã từng dựa vào amiăng có thể được thay thế bằng denim tái chế hoặc được chế tạo bằng
cellulose - một vật liệu sợi có trong các sản phẩm giấy. Đánh giá tương tự áp dụng cho gỗ hoặc tấm tường
được sơn bằng sơn có chứa chì độc hại. Ngoài ra, các bộ điều chỉnh lưu lượng khí và đồng hồ đo trên cả
hệ thống làm nóng nước và không khí được xây dựng trước năm 1961 phải được đánh giá cẩn thận để xác
định rằng chúng không chứa các chất nguy hiểm như thủy ngân.
Như vậy amiang, thủy ngân và chì là các chất độc hại ngoài trừ Xenluloza - không được đề cập
Ta chọn đáp án A.

Trang 23
ĐỀ SỐ 11 ĐỀ THI THỬ TỐT NGHIỆP THPT
NĂM HỌC: 2020 – 2021
MÔN: TIẾNG ANH
Thời gian làm bài: 60 phút; không kể thời gian phát đề

Mark the letter A, B, C, or D on your answer sheet to indicate the word whose underlined part
differs from the other three in pronunciation in each of the following questions.
Question 1. A. climate B. examine C. invitation D. interview
Question 2. A. society B. geography C. dry D. sandy
Mark the letter A, B, C or D on your answer sheet to indicate the word that differs from the rest in
the position of the main stress in each of the following questions.
Question 3. A. dismiss B. destroy C. display D. district
Question 4. A. entertain B. candidate C. referee D. afternoon
Mark the letter A, B, C or D on your answer sheet to indicate the correct answer to each of the
following questions.
Question 5. When Bill got home, his children_____________in the yard.
A. will be playing B. were playing C. are playing D. played
Question 6. Did you manage_____________the book you were looking for?
A. to find B. found C. finding D. find
Question 7. That cannot be a true story. He_____________it up.
A. must have made B. should have made
C. would have made D. can have made
Question 8. Beethovens Fifth Symphony_____________next weekend.
A. is going to be performed B. has been performed
C. will be performing D. will have performed
Question 9. She said that when she_____________to school, she saw an accident
A. had been walking B. has walked C. was walking D. has been walking
Question 10. She wanted Tom, _____________she liked, as a partner, but she got Jack, She
didn’t like.
A. whom/ whose B. whose/ whom C. whose/ whose D. whom/ whom
Question 11. Scarcely_____________out of bed when_____________
A. had I got/ did the doorbell ring B. had I got/ the doorbell rang
C. I had got/ did the doorbell ring D. I had got/ the doorbell rang
Question 12. I think he will join us, _____________?
A. won’t he B. don’t I C. will not he D. do not I
Question 13. For more than 10 years, we have seen the significant_____________in the economy

Trang 1
of our country.
A. develop B. developments C. developers D. developed
Question 14. The_____________load for this car is one ton.
A. top B. max C. peak D. height
Question 15. Most modern office_____________includes facsimile machines, photocopiers and
telephone systems.
A. device B. tool C. equipment D. instrument
Question 16. I just took it_____________that he’d always be on time.
A. into consideration B. easy C. into account D. for granted
Question 17. That’s exactly what I mean, Tom. You’ve_____________!
A. put your foot in it B. killed two birds with one stone
C. put two and two together D. hit the nail on the head
Question 18. My car is getting unreliable. I think I’ll trade it_____________for a new one.
A. off B. away C. in D. up
Mark the letter A, B, C, orD on your answer sheet to indicate the underlined part that needs
correction in each of the following questions.
Question 19. I visited Mexico and United States last year.
A. visited B. Mexico C. United States D. last year
Question 20. Yesterday he said he regretted not to have gone to the exhibition last week.
A. yesterday B. said C. regretted D. not to have gone
Question 21. Minh’s mother would sooner not to meet her friend in person than phone her.
A. sooner B. not to meet C. in person D. than
Mark the letter A, B, C or D on your answer sheet to indicate the sentence that is closest in meaning
to each of the following questions.
Question 22. Friendly though he may seem, he’s not to he trusted.
A. He’s too friendly to be trusted.
B. However he seems friendly, he’s not to be trusted.
C. He may have friends, but he’s not to be trusted.
D. However friendly he seems, he’s not to be trusted.
Question 23. The last time I saw her was three years ago.
A. I have not seen her for three years.
B. About three years ago, I used to meet her.
C. I have often seen her for the last three years.
D. I saw her three years ago and will never meet her
Question 24. Calling Odyse is pointless because his phone is out of order.
A. It’s worth not calling Odyse since his phone is out of order.

Trang 2
B. It’s no use to call Odyse since his phone is out of order.
C. It’s useless call Odyse as his phone is out of order.
D. There is no point in calling odyse as his phone is out of order.
Mark the letter A, B, C or D on your answer sheet to indicate the sentence that best combines each
pair of sentences in the following questions.
Question 25. Randy doesn’t play rugby. Neither does Lucy
A. Neither Randy nor Lucy doesn’t play rugby.
B. Neither Randy nor Lucy play rugby.
C. Neither Randy nor Lucy plays rugby.
D. Either Randy or Lucy plays rugby.
Question 26. Nguyen Huu Kim Son broke the SEA Games record in the men’s 400-metre individual
medley event. He is very proud of it.
A. He is very proud of what he broke the SEA Games record.
B. He takes pride in breaking the SEA Games record.
C. He is interested in the SEA Games record.
D. He is very proud of he broke the SEA Games record.
Mark the letter A, B, C or D on your answer sheet to indicate the most suitable response to
complete each of the following exchanges.
Question 27. This situation happened in a restaurant
- Customer: “Excuse me!”
- Waiter: “Yes, sir. How can I help you?”
- Customer: “I don’t want to make a scene but there’s a fly in my soup.”
- Waiter: “_____________”
A. I’m so sorry! I will get you another one.
B. What can I do to help?
C. You are right. I will get you another one.
D. You could be right but I don’t think it is a fly.
Question 28. Alice and Mary have just finished watching a movie.
- Alice: “Endgame is such a wonderful movie”
- Mary: “_____________”
A. I couldn’t agree more. B. I don’t like your saying.
C. I didn’t say anything. D. I’m a huge fan of Marvel.
Mark the letter A, B, C, or D on your answer sheet to indicate the word or phrase that is CLOSEST
in meaning to the underlined part in each of the following questions.
Question 29. One of the biggest issues that many victims of negligent behavior encounter is difficulty in
determining whether or not an action had foreseeable consequences.

Trang 3
A. predictable B. unpredictable C. ascertainable D. computable
Question 30. We’re really close friends but we just cannot see eye to eye on politics.
A. not see well B. not share the same views about
C. not understand D. not care for
Mark the letter A, B, C, or D on your answer sheet to indicate the word or phrase that is
OPPOSITE in meaning to the underlined part in each of the following questions.
Question 31. The research findings were reliable since modern technology was used to increase the
precision of the sampling procedure.
A. inaccuracy B. exactness C. insecurity D. flexibility
Question 32. Despite the traffic hold-ups, we were able to arrive at the airport in the nick of the time
just before the check-in counter closed.
A. in a terrible condition B. with all out luggage
C. at the very last moment D. with much time to spare
Read the following passage and mark the letter A, B, C or D on your answer sheet to indicate the
correct word for each of the blanks from 33 to 37.
The knock-on effect of volunteering on the lives of individuals can be profound. Voluntary work helps
foster independence and imparts the ability to deal with different situations, often simultaneously, thus
teaching people how to (33) _____________their way through different systems. It therefore brings
people into touch with the real world; and, hence, equips them for the future.
Initially, young adults in their late teens might not seem to have the expertise or knowledge to impart to
others that say a teacher or agriculturalist or nurse would have, (34) _____________they do have many
skills that can help others. And in the absence of any particular talent, their energy and enthusiasm can be
harnessed for the benefit of their fellow human beings, and ultimately themselves. From (35)
_____________this, the gain to any community no matter how many volunteers are involved is (36)
_____________.
Employers will generally look favorably on people (37) _____________have shown an ability to work as
part of a team. It demonstrates a willingness to learn and an independent spirit, which would be desirable
qualities in any employee.
Question 33. A. give B. work C. put D. take
Question 34. A. so B. but C. or D. for
Question 35. A. all B. none C.above D. both
Question 36. A. unattainable B. immeasurable C. undetectable D. impassible
Question 37. A. which B. whose C. who D. what
Read the following passage and mark the letter A, B, C, or D on your answer sheet to indicate the
correct answer to each of the questions from 38 to 42.

Trang 4
Insects’ lives are very short and they have many enemies, but they must survive long enough to breed
and perpetuate their kind. The less insect-like they look, the better their chance of survival. To look
"inedible" by imitating plants is a way frequently used by insects to survive. Mammals rarely imitate
plants, but many fish and invertebrates do.
The stick caterpillar is well named. It is hardly distinguishable from a brown or green twig. This
caterpillar is quite common and can be found almost anywhere in North America. It is also called
"measuring worm" or "inchworm." It walks by arching its body, then stretching out and grasping the
branch with its front feet then looping its body again to bring the hind fed forward, when danger
threatens, the stick caterpillar stretches its body away from the branch at an angle and remains rigid and
still, like a twig, until the danger has passed.
Walking sticks, or stick insects, do not have to assume a rigid, twig-like pose to find protection: they
look like inedible twigs in any position. There are many kinds of walking sticks, ranging in size from
the few inches of the North American variety to some tropical species that may be over a foot long,
when at rest their front legs are stretched out, heightening their camouflage. Some of the tropical
species are adorned with spines or ridges, imitating the thorny bushes or trees in which they live.
Leaves also seem to be a favorite object for insects to imitate. Many butterflies can suddenly disappear
from view by folding their wings and sitting quietly among the plants that they resemble.
Question 38. what is the main idea of the passage?
A. Caterpillars that live in trees
B. The feeding habits of insects
C. How some insects imitate plants to survive
D. Insects that are threatened with extinction
Question 39. which of the following does the word “enemies” in line 1 refer to?
A. plants looking like insects B. extreme weather conditions
C. creatures that eat insects D. insects looking like plants
Question 40. According to the passage, how does a stick caterpillar make itself look like a twig?
A. By holding its body stiff and motionless
B. By looping itself around a stick
C. By changing the colour of its skin
D. By laying its body flat against a branch
Question 41. Which of the following is the antonym of the word “inedible” in paragraph 3?
A. eatable B. colourful C. beautiful D. moving
Question 42. How can butterflies make themselves invisible?
A. By hiding under the leaves
B. By disappearing from the view
C. By folding wings and sitting still among the leaves with similar colours

Trang 5
D. By flying among colourful flowers
Read the following passage and mark the letter A, B, C, or D on your answer sheet to indicate the
correct answer to each of the questions from 43 to 50.
There are two basic types of glaciers, those that flow outward in all directions with little regard for any
underlying terrain and those that are confined by terrain to a particular path.
The first category of glaciers includes those massive blankets that cover whole continents,
appropriately called ice sheets. There must be over 50,000 square kilometers of land covered with ice
for the glacier to qualify as an ice sheet, when portions of an ice sheet spread out over the ocean, they
form ice shelves.
About 20,000 years ago the Cordilleran Ice sheet covered nearly all the mountains in southern Alaska,
western Canada, and the western United States. It was about 3 kilometers deep at its thickest point in
northern Alberta. Now there are only two sheets left on Earth, those covering Greenland and Antarctica.
Any domelike body of ice that also flows out in all directions but covers less than 50,000 square
kilometers is called an ice cap. Although ice caps are rare nowadays, there are a number in northeastern
Canada, on Baffin Island, and on the Queen Elizabeth Islands.
The second category of glaciers includes those of a variety of shapes and sizes generally called
mountain or alpine glaciers. Mountain glaciers are typically identified by the landform that controls
their flow. One form of mountain glacier that resembles an ice cap in that it flows outward in several
directions is called an ice field. The difference between an ice field and an ice cap is subtle.
Essentially, the flow of an ice field is somewhat controlled by surrounding terrain and thus does not
have the domelike shape of a cap. There are several ice fields in the Wrangell. St. Elias, and Chugach
mountains of Alaska and northern British Columbia.
Less spectacular than large ice fields are the most common types of mountain glaciers: the cirque and
valley glaciers. Cirque glaciers are found in depressions in the surface of the land and have a
characteristic circular shape. The ice of valley glaciers, bound by terrain, flows down valleys, curves
around their corners, and falls over cliffs.
Question 43. what does the passage mainly discuss?
A. Where major glaciers are located
B. How glaciers shape the land
C. The different kinds of glaciers
D. How glaciers are formed
Question 44. The word “terrain” in paragraph 1 could best be replaced by____________.
A. the seabed B. area of land C. countryside D. prairie
Question 45. It can be inferred from paragraph 2 that ice sheets are so named
because____________
A. they are thicker in some areas than the others

Trang 6
B. they are identified by the landform that controls their flow
C. they cover large areas of land
D. they are confined to cirque glaciers
Question 46. According to the passage, where was the Cordilleran Ice Sheet thickest?
A. Alaska B. Antarctica C. Greenland D. Alberta
Question 47. According to paragraph 5, ice fields resemble ice caps in which of the following
ways?
A. Their flow B. Their texture C. Their location D. Their shape
Question 48. The word “subtle” in paragraph 5 could best be replaced by____________.
A. slight B. substantial C. regional D. obvious
Question 49. The word “their” in last paragraph refers to____________.
A. ice fields B. cirque glaciers C. valley glaciers D. valleys
Question 50. All of the following are alpine glaciers EXCEPT____________.
A. cirque glaciers B. ice caps C. ice fields D. Valley glaciers

Trang 7
Đáp án
1-A 2-C 3-D 4-B 5-B 6-A 7-A 8-A 9-C 10-D
11-B 12-A 13-B 14-B 15-C 16-D 17-D 18-C 19-C 20-D
21-B 22-D 23-A 24-D 25-C 26-B 27-A 28-A 29-A 30-B
31-A 32-D 33-B 34-B 35-A 36-B 37-C 38-C 39-C 40-A
41-A 42-C 43-C 44-B 45-C 46-D 47-A 48-A 49-D 50-B

LỜI GIẢI CHI TIẾT


Question 1: Đáp án A
Đáp án A đúng vì phần gạch chân của đáp án A đọc là âm /aɪ/ còn phần gạch chân của các đáp án khác
được đọc là âm /ɪ/
A. climate /ˈklaɪmət/ (n): khí hậu
B. examine /ɪɡˈzæmɪn/ (v): kiểm tra
C. invitation /ˌɪnvɪˈteɪʃn/ (n): lời mời
D. interview /ˈɪntəvjuː/ (n): cuộc phỏng vấn/ (v): phỏng vấn
Question 2. Đáp án C
Đáp án C đúng vì phần gạch chân của đáp án C đọc là âm / aɪ / còn phần gạch chân của các đáp án còn lại
được đọc là âm /i/
A. society /səˈsaɪəti/ (n): xã hội
B. geography /dʒiˈɒɡrəfi/ (n): địa lý học
C. dry /draɪ/ (adj): khô cạn
D. sandy /ˈsændi/ (adj): có cát
Question 3. Đáp án D
Đáp án D đúng vì đáp án D trọng âm rơi vào âm tiết thứ nhất còn 3 đáp án còn lại trọng âm rơi vào âm
tiết thứ hai
A. dismiss /dɪsˈmɪs/ (v): giải tán, sa thải
B. destroy /dɪˈstrɔɪ/ (v): phá hủy
C. display /dɪˈspleɪ/ (v): trưng bày
D. district /ˈdɪstrɪkt/ (n): quận, huyện
❖ For review
Hầu hết động từ có hai âm tiết thì trọng âm rơi vào âm tiết thứ hai; hầu hết danh từ, tính từ có hai âm tiết
thì trọng âm rơi vào âm tiết nhất
Question 4. Đáp án B
Đáp án B đúng vì đáp án B trọng âm rơi vào âm tiết thứ nhất còn 3 đáp án còn lại trọng âm rơi vào âm tiết
thứ ba.
A. entertain /ˌentəˈteɪn/ (v): giải trí

Trang 8
B. candidate /ˈkændɪdət/ (n): thí sinh
C. referee /ˌrefəˈriː/ (n): trọng tài
D. afternoon /ˌɑːftəˈnuːn/ (n): buổi chiều
❖ For review
Từ có 3 âm tiết, trọng âm sẽ rơi vào âm tiết cuối nếu tận cùng kết thúc bằng các hậu tố như -ain, -oon, -
ee,..
Tù có 3 âm tiết trọng âm sẽ rơi vào âm tiết đầu tiên nếu tận cùng kết thúc bằng các hậu tố như -ate, -ize, -
fy
Question 5. Đáp án B
Ta thấy trong câu xuất hiện một hành động đang xảy ra trong quá khứ (play) thì bị một hành động khác
xen vào (got home) → như vậy động từ cần chia quá khứ tiếp diễn để diễn tả hành động đang xảy ra trong
quá khứ. Như vậy ta chọn đáp án B.
Tạm dịch. Khi Bill về nhà thì bọn trẻ đang chơi ngoài sân
Question 6. Đáp án A
Ta có cấu trúc manage to V: cố gắng làm việc gì đó. Như vậy ta chọn được đáp án A
Tạm dịch. Bạn đã tìm thấy cuốn sách đang tìm chưa?
Question 7. Đáp án A
Chúng ta dựa vào cụm cannot he → suy đoán logic xảy ra trong quá khứ → phù hợp về nghĩa nhất là must
have + P2. Như vậy ta chọn được đáp án A.
Tạm dịch. Đó không thể là câu chuyện có thật được. Anh ta chắc chắn đã bịa ra nó.
❖ For review
Đối với các động từ khuyết thiếu hoàn thành, chúng ta có một số cấu trúc cơ bản và cách dùng như sau:
- S + should (not) have + P2: Đáng lẽ phải...: dùng để chỉ một việc nên/ không nên làm ở quá khứ.
- S + must have + P2: Chắc hẳn là đã...: dùng để thể hiện suy luận logic dựa trên sự việc xảy ra ở quá khứ
dưới dạng câu khẳng định.
- S + can’t/ couldn’t have + P2: Không thể đã...: dùng để chỉ một sự việc chắc chắn không thể xảy ra trong
quá khứ
- S + could/may/might have + P2/ been V-ing: Có thể là đã...: chỉ một sự việc có thể xảy ra ở quá khứ
- S + needn’t have + P2: Đã không cần phải...: diễn tả một hành động không cần xảy ra trong quá khứ
Question 8. Đáp án A
Ta thấy perform (v): biểu diễn, là một ngoại động từ nhưng phía sau không có tân ngữ → phải ở dạng bị
động → loại C, D vì không đúng cấu trúc câu bị động (be + P2). Trong câu ta thấy có cụm từ chỉ thời gian
next weekend chỉ tương lai gần → đáp án đúng của câu là A
Tạm dịch. Bản giao hưởng số 5 của Beethoven sẽ được biểu diễn vào tuần tới
Question 9.

Trang 9
Đây là câu tường thuật lại câu kể. Căn cứ vào từ “when” và nghĩa của cả câu. Trong câu trực tiếp dùng
động từ ở thì quá khứ tiếp diễn kết hợp với quá khứ đơn thì khi chuyển sang câu tường thuật ta không cần
đổi thì động từ.
Tạm dịch: Cô ấy bảo rằng cô ấy đã nhìn thấy một vụ tai nạn khi cô ấy đang tới trường.
Question 10.
Trong câu trên ta thấy Tom và Jack đóng vai trò làm tân ngữ trong câu nên ta cần dùng đại từ quan hệ
“whom” cho cả hai trường hợp
Tạm dịch. Cô ấy muốn có Tom người mà cô ấy quý mến như một người bạn đồng hành nhưng cô ấy cũng
có cả Jack người mà cô ấy chẳng ưa.
Question 11. Đáp án B
Đây là câu đảo ngữ với cụm từ “Scarcely... when”
- Scarcely had + S + P2 when S + V (past simple): vừa mới...thì....
→ Loại C và D vì “I had got” chưa đảo ngữ
→ Loại A vì phương án A sử dụng đảo ngữ sau “when”
Tạm dịch. Tôi vừa mới ra khỏi phòng thì chuông cửa reo.
Question 12.
Câu trên sử dụng câu hỏi đuôi có sử dụng động từ nêu quan điểm, ý kiến. Câu có động từ nêu quan điểm,
ý kiến thường có 2 mệnh đề, mệnh đề 1 (đứng trước) và mệnh đề 2 (đứng sau), nếu chủ ngữ mệnh đề 1 là
ngôi “I” thì phần hỏi đuôi sẽ hỏi lại của mệnh đề 2 → won’t he
Tạm dịch. Tôi nghĩ anh ta sẽ tham gia cùng chúng ta, phải thế không?
Question 13.
Ta có significant (adj): có ý nghĩa, quan trọng → chỗ trống cẩn điền là một danh từ → loại A và D.
Ngoài ra trong câu ta còn có cụm từ “in the economy”- trong nền kinh tế → ta có thể suy luận ta cần một
danh từ chỉ vật, không phải danh từ chỉ người
A. develop (v): phát triển
B. developments (n): bước phát triển
C. developers (n): chuyên viên phát triển
D. developed (adj): đã phát triển
Tạm dịch. Trong hơn 10 năm qua, chúng ta đã chứng kiến những bước phát triển quan trọng trong nền
kinh tế của nước nhà.
Question 14.
Trong câu trên ta thấy có sự xuất hiện của các từ load (n): trọng tải. Khi diễn tả trọng tải tối đa của
phương tiện ta sẽ sử dụng cụm từ “max load”
A. top (adj): cao nhất
B. max (n): tối đa, cực độ
C. peak (n): đỉnh, chóp

Trang 10
D. high (adj) : cao
Tạm dịch. Tải trọng tối đa của chiếc ô tô này là 1 tấn.
Question 15.
Câu trên ta thấy cả 4 đáp án đều có nét nghĩa tương đồng nhau, tuy nhiên ta thấy trong câu xuất hiện từ
most....includes- đang được chia số ít. Trong 4 phương án chỉ có phương án C là danh từ không đếm được
nên khi kết hợp với most động từ chia số ít còn 3 phương án còn lại đều là danh từ đếm được nên khi kết
hợp với most phải ở dạng số nhiều
A. device (n): thiết bị
B. tool (n): công cụ, đổ dùng (cầm tay, làm vườn..)
C. equipment (n): trang thiết bị, dụng cụ, đồ dùng (chuyến đi, nhà máy...)
D. instrument (n): nhạc cụ
Tạm dịch. Phần lớn các trang thiết bị văn phòng hiện đại bao gồm máy fax, máy photo và hệ thống điện
thoại.
Question 16.
Ta có thành ngữ “take it for granted”: cho là điều dĩ nhiên
A. take st into consideration: tính đến cái gì, xem xét cái gì
B. take st into account: lưu tâm, tính đến cái gì
C. take it easy: đừng căng thẳng
Tạm dịch. Dĩ nhiên là anh ấy luôn đúng giờ
Question 17.
Ta có thành ngữ “hit the nail on the head”: Đoán trúng, nói đúng ý
A. put your foot in it: gây nhầm lẫn, làm bối rối, phạm một sai lầm
B. killed two birds with one stone: một công đôi việc
C. put two and two together: hiển nhiên, chứng cứ rõ ràng
Tạm dịch. Đó chính xác là những gì tôi muốn nói Tom ạ, cậu đoán đúng rồi.
Question 18.
Ta loại được B và D vì away và up không đi với động từ trade. Trong câu ta thấy có cụm từ for a new
one: lấy cái mới → C: trade st in: đổi cũ lấy mới
A. trade st off: từ bỏ cái gì như một thỏa hiệp
Tạm dịch. Ô tô của tôi ngày càng trục trặc. Tôi nghĩ phải bán nó đi để mua cái mới.
Question 19.
Câu này liên quan đến cách sử dụng mạo từ “the”. Chúng ta thấy mạo từ the sẽ được dùng trước tên một
số quốc gia có các từ như “Republic”, “Kingdom”, “States”, hoặc tên ở dạng số nhiều (như the
Dominican Republic, the USA, the UK, the Philippines). Như vậy trong câu này ta thấy ta cần sửa lại là
“the United States”
Tạm dịch. Tôi đã đến thăm Mexico và Hoa Kỳ vào năm ngoái.

Trang 11
Question 20. Đáp án D
Chúng ta có cấu trúc S + regret + (not) V-ing/ (not) having P2: Hối tiếc đã (không) làm gì.
Như vậy ta thấy ở đây đáp án D chưa chính xác, ta cần sửa thành not having gone
Tạm dịch. Hôm qua anh ấy nói rằng anh ấy tiếc vì đã không tham gia triển lãm tuần trước.
Question 21. Đáp án B
Câu trên sử dụng cấu trúc thức giả định, ta có
- S + would rather/ sooner + (not)+ V than V
Như vậy đáp án B sẽ sửa lại thành “not meet”
Tạm dịch. Mẹ của Minh thích gặp trực tiếp bạn bè của bà ấy hơn là gọi điện thoại.
Question 22. Đáp án D
Dịch câu đề. Mặc dù có vẻ rất thân thiện nhưng anh ấy không được tin tưởng
Dịch đáp án.
A. Anh ấy quá thân thiện để không được tin tưởng
B. Tuy anh ấy có vẻ thân thiện nhưng anh ấy không được tin tưởng
C. Anh ấy có thể có nhiều bạn nhưng anh ấy không được tin tưởng
D. Tuy anh ấy có vẻ thân thiện nhưng anh ấy không được tin tưởng
Loại đáp án A và C vì nghĩa không phù hợp với câu gốc. Câu B sai vì đã sử dụng sai cấu trúc với
however. Cấu trúc đúng của however khi thể hiện sự nhượng bộ là:
- However +adv/ adj+ S + V, S + V
Vì vậy chỉ có phương án D là chính xác
Question 23. Đáp án A
Dịch câu đề. Lần cuối cùng tôi gặp cô ấy là 3 năm trước.
Dịch đáp án.
A. Tôi đã không gặp cô ấy 3 năm rồi.
B. Khoảng 3 năm trước, tôi đã từng gặp cô ấy
C. Tôi thường gặp cô ấy khoảng 3 năm trước
D. Tôi gặp cô ấy 3 năm trước và sẽ không bao giờ gặp cô ấy nữa.
Ta có cấu trúc: S + haven’t/ hasn’t + P2 + since/ for.. . = The last time (when) S + V (past simple) + was +
a period of time + ago. Như vậy đáp án chính xác là đáp án A
Question 24. Đáp án D
Dịch câu đề. Gọi cho odsye không được đâu vì điện thoại của cậu ấy đang hỏng.
Dịch đáp án.
A. Gọi cho Odyse là vô ích vì điện thoại của anh ấy đang hỏng
B. Vô ích khi mà gọi cho Odyse vì điện thoại của anh ấy đang hỏng
C. Gọi cho Odyse không được đâu vì điện thoại của anh ấy đang hỏng
D. Gọi cho Odyse không được đâu vì điện thoại của anh ấy đang hỏng

Trang 12
Khi diễn tả việc làm một việc gì đó là vô ích, không đáng ta có thể sử dụng các cấu trúc sau:
- It’s not worth + V-ing
- It’s no use/ no good + V-ing
- It’s useless + V-ing/ to V
- There is no point in + V-ing
Như vậy là chỉ có đáp án D là chính xác vì 3 phương án còn lại đang sử dụng sai cấu trúc.
Question 25. Đáp án C
Dịch câu đề. Randy không chơi bóng bầu dục. Lucy cũng thế.
Dịch đáp án.
A. Cả Randy và Lucy đều chơi bóng bầu dục
B. Cả Randy và Lucy đều không chơi bóng bầu dục.
C. Cả Rancy và Lucy đều không chơi bóng bầu dục.
D. Cả Randy và Lucy đều chơi bóng bầu dục.
Ở đây ta dùng cặp liên từ tương quan để nối hai câu đơn. Neither... nor.... Tuy nhiên động từ sau nor sẽ ở
dạng khẳng định và được chia theo chủ ngữ gần nhất → như vậy chỉ có đáp án C là chính xác
Question 26. Đáp án B
Dịch câu đề. Nguyen Huu Kim Son đã phá kỉ lục SEA Games ở nội dung bơi 400m hỗn hợp nam. Cậu ấy
rất tự hào về điều đó.
Dịch đáp án.
A. Anh ấy rất tự hào về thứ mà anh ấy đã phá kỉ lục SEA Games.
B. Anh ấy rất tự hào về việc phá kỉ lục SEA Games
C. Anh ấy thích thú về kỉ lục SEA Games.
D. Anh ấy rất tự hào về việc phá kỉ lục SEA Games.
Chúng ta có cấu trúc take pride in + N/V-ing: Tự hào về việc gì → Đáp án đúng cho câu này là đáp án B
Đáp án A chưa chính xác vì truyền tải sai ý của câu gốc
Đáp án C chưa chính xác vì chuyển tải sai ý câu gốc
Đáp án D sử dụng sai cấu trúc: proud of + V-ing/Noun
Question 27. Đáp án A
Tình huống xảy ra ở một nhà hàng
Khách hàng: Xin lỗi
Phục vụ: Vâng thưa ngài. Tôi có thể giúp gì cho ngài?
Khách hàng: Tôi không muốn gây chuyện nhưng mà có một con ruồi trong bát súp của tôi
Phục vụ:_____________
A. Tôi xin lỗi. Tôi sẽ đem cho ngài 1 bát súp khác.
B. Vậy tôi có thể làm gì để giúp ngài
C. Rất đúng. Tôi sẽ đem cho ngài 1 bát súp khác

Trang 13
D. Có thể ngài nói đúng nhưng tôi không nghĩ đây là một con ruồi.
Dựa vào tình huống chúng ta thấy đây là một lời phàn nàn → đáp án đúng là phương án A. Phương án B
thường sử dụng khi chúng ta bày tỏ sự cảm thông tới ai đó. Phương án C thường dùng khi chúng ta tán
thành ý kiến với ai đó. Phương án D thường sử dụng khi chúng ta không đồng tình với ý kiến của ai đó.
Question 28. Đáp án A
Alice và Mary vừa xem xong một bộ phim.
Alice: Endgame là một bộ phim thật tuyệt.
Mary: ____________
Ta thấy trong tình huống này Alice đang đưa ra một lời nhận xét về bộ phim Endgame và để đáp lại thì
Mary sẽ bày tỏ ý kiến quan điểm của mình. Như vậy đáp án A là đáp án chính xác.
A. Tớ không thể đồng ý hơn được nữa → rất đồng tình
B. Tớ không thích cậu nói như vậy.
C. Tớ đã không nói gì hết
D. Tớ là fan ruột của Marvel.
Question 29. Đáp án A
Đáp án A đúng vì ta có foreseeable (adj) có nghĩa là có thể lường trước = predictable (adj). Đây là câu hỏi
tìm từ đồng nghĩa nên chọn đáp án A
A. predictable (adj): có thể đoán được >< B. unpredictable (adj)
C. ascertainable (adj): có thể tìm ra một cách chính xác
D. computable (adj): có thể tính toán được
Tạm dịch. Một trong những vấn đề lớn nhất mà nhiều nạn nhân của hành vi bất cẩn gặp phải là khó xác
định được hành động đó có những hậu quả có thể lường trước hay không
❖ Note
Với những từ có hậu tố -able thì thường mang ý nghĩa có khả năng hoặc có thể
Question 30. Đáp án B
Đáp án B đúng vì ta có not see eye to eye with sb (on st) = not share the same views with sb (about st):
không đồng quan điểm với ai đó về cái gì. Đây là câu hỏi tìm từ đồng nghĩa nên ta chọn phương án B
A. not see well: không nhìn được
C. not understand: không hiểu
D. not care for: không quan tâm
Tạm dịch. Chúng tôi thật sự là những người bạn thân nhưng chúng tôi không cùng quan điểm về chính trị
Question 31. Đáp án A
Đáp án A đúng vì ta có precision (n) = exactness (n): sự chính xác >< inaccuracy (n): sự không chính
xác. Đây là câu hỏi tìm từ trái nghĩa nên ta chọn phương án A
C. insecurity (n): sự không đảm bảo, thiếu cảm giác an toàn
D. flexibility (n): sự linh hoạt linh động

Trang 14
Tạm dịch. Kết quả của cuộc nghiên cứu rất đáng tin vì công nghệ hiện đại đã được sử dụng để tăng tính
chính xác của quá trình lấy mẫu thử.
Question 32. Đáp án D
Đáp án D đúng vì ta có in the nick of the time = at the very last moment: sát giờ, đến vào phút chót ><
with much time to spare: đến sớm, còn thừa nhiều thời gian. Đây là câu hỏi tìm từ trái nghĩa nên ta chọn
phương án D
A. in a terrible condition: trong điều kiện tồi tệ
B. with all our luggage: với tất cả hành lý của chúng tôi
Tạm dịch. Mặc dù giao thông tắc nghẽn nhưng chúng tôi vẫn đến sân bay vào phút chót trước khi quầy
check-in đóng cửa
Question 33.
Ta có cụm từ work one’s way through st: kiên trì với một nhiệm vụ công việc.
Voluntary work helps foster independence and imparts the ability to deal with different situations, often
simultaneously, thus teaching people how to work their way through different systems.
Tạm dịch. Công việc tình nguyện giúp bồi dưỡng tính tự lập và truyền cho ta khả năng đối phó với các
tình huống khác nhau, thường là một cách đồng thời, từ đó bằng những hệ thống khác nhau dạy mọi
người cách làm sao để kiên trì làm việc.
Question 34. Đáp án B
Ta thấy mối quan hệ giữa hai vế câu trước và sau chỗ trống là quan hệ tương phản. Vế trước ta thấy xuất
hiện các cụm từ “young adults .. .not seem to have expertise or knowledge” còn vế sau ta lại thấy các cụm
từ “they do have many skills”. Như vậy để thể hiện quan hệ tương phản đối lập ta chọn được phương án
B. Phương án A đứng trước mệnh đề chỉ kết quả. Phương án C thường nối hai vế có quan hệ ngang hàng.
Phương án D dùng để chỉ mục đích.
Initially, young adults in their late teens might not seem to have the expertise or knowledge to impart to
others that say a teacher or agriculturalist or nurse would have, but they do have many skills that can help
others.
Tạm dịch. Ban đầu những người trẻ có thể không có chuyên môn hay kiến thức để truyền bá cho người
khác như một giáo viên, một nhà nông nghiệp hay một y tá, nhưng họ lại có nhiều kĩ năng có thể giúp
người khác
Question 35.
Ta có cụm từ “from all this”: từ những điều này. Ta thấy phía trước chỗ trống liệt kê rất nhiều những
đóng góp của các bạn tình nguyện viên và chỗ trống này là liên từ bắt đầu cho câu kết đoạn.
Question 36. Đáp án B
Dựa vào nghĩa của từ và những phân tích từ câu 35 ta chọn được B là đáp án đúng.
A. unattainable (adj): không thể đạt được, không thể chạm tới
B. immeasurable (adj): không thể đong đếm được

Trang 15
C. undetectable (adj): không thể dò được không thể khám phá ra
D. impassible (adj): không thể vượt qua được
From all this, the gain to any community no matter how many volunteers are involved is immeasurable.
Tạm dịch. Từ những điều này, ta thấy những gì mà mỗi cộng đồng nhận được cho dù các tình nguyện
viên tham gia như thế nào đi nữa đều không thể đong đếm được.
Question 37. Đáp án C
Đáp án C. Ở đây ta cần một đại từ quan hệ thay thế cho danh từ chỉ người people và làm chủ ngữ của
mệnh đề quan hệ → chỉ có phương án C là phù hợp
Employers will generally look favorably on people who have shown an ability to work as part of a team.
Tạm dịch. Các nhà quản lý thường thiên về những người thể hiện được khả năng làm việc nhóm.
Question 38. Đáp án C
Câu hỏi. Ý chính của đoạn văn là gì?
A. Sâu bướm sống trên cây
B. Thói quen kiếm mồi của côn trùng
C. Cách một số côn trùng ngụy trang giống thực vật để sinh tồn
D. Những loài côn trùng bị đe dọa tuyệt chủng
Đây là câu hỏi hỏi ý chính của văn bản. Trong đoạn 1 dòng 3, ta tìm thấy thông tin:
To look "inedible" by imitating plants is a way frequently used by insects to survive.
Tạm dịch. Để trông có vẻ không ăn được bằng cách giả mạo giống cây cối là cách thường được côn trùng
sử dụng để tồn tại.
Ngoài ra khi đọc lướt toàn bộ vàn bản ta thấy văn bản không viết về một loại côn trùng cụ thể nào mà viết
về nhiều loài khác nhau nhưng có đặc tính giống nhau đấy là ngụy trang để tránh kẻ thù. Như vậy đáp án
C hợp lý hơn cả.
Question 39. Đáp án C
Câu hỏi. Từ “enemies” ở dòng 1 đề cập đến điều nào sau đây:
A. cây cối trông giống côn trùng
B. điều kiện thời tiết cực đoan
C. những loài sinh vật ăn côn trùng
D. côn trùng trông giống cây cối
Đây là một câu hỏi liên quan đến từ vựng. Với loại câu hỏi này chúng ta tìm thông tin xung quanh từ
vựng đó là được. Từ vựng xuất hiện ở dòng đầu tiên đoạn 1
Insects’ lives are very short and they have many enemies but they must survive long enough to breed and
perpetuate their kind.
Tạm dịch. Vòng đời của côn trùng rất ngắn và chúng có nhiều kẻ thù, nhưng chúng phải tồn tại đủ lâu để
sinh sản và duy trì nòi giống. Trong câu trên ta còn thấy xuất hiện liên từ but, thể hiện quan hệ tương
phản, một về câu ta thấy có cụm “must survive”- phải sống sót → vế còn lại cần có yếu tố chứa đựng sự

Trang 16
tương phản với điều này. Từ enemy (n): kẻ thù, trong văn bản này chính là những loài ăn côn trùng đe
dọa đến sự sống của côn trùng → đáp án đúng là C
Question 40. Đáp án A
Câu hỏi. Theo đoạn văn, sâu bướm gậy làm cho nó giống cành cây như thế nào?
A. giữ cho cơ thể cứng đờ và không chuyển động
B. cuộn cơ thể quanh một cái gậy
C. thay đổi màu da
D. giữ cơ thể phẳng trên một cành cây
Chúng ta tìm thấy thông tin trong đoạn 2 dòng 5-6.
When danger threatens, the stick caterpillar stretches its body away from the branch at an angle and
remains rigid and still, like a twig, until the danger has passed.
Tạm dịch. Khi gặp nguy hiểm, sâu bướm gậy kéo dài cơ thể nó ra khỏi cành cây ở một góc nhất định và
làm cho cơ thể nó cứng đờ, đứng yên giống như một cái gậy cho đến khi mối đe dọa đã qua.
Như vậy ta chọn được đáp án A
Question 41. Đáp án A
Câu hỏi. Đâu là từ trái nghĩa của “inedible” trong đoạn 3
A. eatable (adj): có thể ăn được
B. colourful (adj): nhiều màu sắc
C. beautiful (adj): đẹp
D. moving (adj): di chuyển
Chúng ta tìm thấy thông tin trong đoạn 3, dòng 1-2:
Walking sticks, or stick insects, do not have to assume a rigid, twig-like pose to find protection: they
look like inedible twigs in any position.
Tạm dịch: Cái gậy biết đi, hoặc côn trùng gậy, không cần phải khoác lên mình vẻ ngụy tạo cứng nhắc,
giống như cành cây con để tìm sự bảo vệ: chúng trông giống như những cành cây không thể ăn được dù ở
bất kì vị trí nào. Từ inedible mang nghĩa không ăn được. Câu hỏi yêu cầu tìm từ trái nghĩa → đáp án A là
phương án chính xác
Question 42. Đáp án C
Câu hỏi. Bướm có thể khiến mình không bị nhìn thấy như thế nào?
A. giấu mình sau lá
B. biến mất khỏi tầm nhìn
C. gập cánh và ngồi yên giữa những chiếc lá có màu giống cánh
D. bay xung quanh những bông hoa nhiều màu sắc
Đây là câu hỏi chi tiết, trong câu hỏi ta tìm thấy key word là từ butterflies và ta thấy đoạn cuối xuất hiện
thông tin cần tìm ở dòng 1-2:

Trang 17
Many butterflies can suddenly disappear from view by folding their wings and sitting quietly among the
plants that they resemble.
Tạm dịch: Nhiều con bướm có thể đột nhiên biến mất khỏi tầm nhìn bằng cách gấp đôi đôi cánh của
chúng và ngồi yên lặng giữa những chiếc lá giống với chúng.
Như vậy ta thấy đáp án C là phương án chính xác. Phương án B có được đề cập đến trong đoạn nhưng
chưa phải là cách ngụy trang của loài bướm.
Question 43. Đáp án C
Câu hỏi: Văn bản trên chủ yếu nói về vấn đề gì?
A. Các sông băng lớn nằm ở đâu
B. Các sông băng hình thành vùng đất như thế nào
C. Các loại sông băng khác nhau
D. Các sông băng được hình thành như thế nào
Chúng ta thấy ngay trong câu đầu tiên, văn bản đã đề cập đến các loại sông băng cơ bản: There are two
basic types of glaciers, those that flow outward in all directions with little regard for any underlying
terrain and those that are confined by terrain to a particular path.
Tạm dịch: Sông băng được chia làm hai loại cơ bản, một loại do không bị ảnh hưởng bởi những địa hình
nằm dưới nó, có thể chảy về mọi hướng, còn loại còn lại bị cố định bởi địa hình, chỉ có thể chảy theo một
đường cụ thể riêng biệt.
Khi đọc toàn bộ bài, chúng ta cũng thấy văn bản đề cập đến các loại sông băng khác nhau → đáp án C
chính xác
Question 44. Đáp án B
Câu hỏi. Từ “terrain” ở đoạn 1 có thể thay thế bằng _____________
A. đáy biển B. vùng đất
C. vùng quê D. thảo nguyên
Ta tìm thấy thông tin câu trả lời ở đoạn 1
dòng đầu tiên, giống câu hỏi 43. Trong câu này ta thấy terrain (n) có nghĩa địa thế, địa hình, địa vật. Cụm
từ thay thế phù hợp nhất chỉ có thể là đáp án B
Question 45. Đáp án C
Câu hỏi. Từ đoạn 2 có thể suy ra rằng ice sheets được đặt tên như vậy bởi vì____________.
A. Chúng dày hơn những nơi khác ở một số khu vực
B. Chúng được xác định bởi dạng địa hình kiểm soát dòng chảy của chúng
C. Chúng che phủ các khu đất rộng lớn
D. Chúng được giới hạn trong những hố băng
Ta tìm thấy trông tin ở đoạn 2 dòng 1-2:
The first category of glaciers includes those massive blankets that cover whole continents, appropriately
called ice sheets.

Trang 18
Tạm dịch. Loại sông băng đầu tiên là những loại giống như những tấm phủ khổng lồ bao trùm toàn bộ các
lục địa, thường được gọi là những mảng băng. Trong đoạn trên ta thấy cụm từ cover whole continents gần
nghĩa nhất với large areas of land → đáp án đúng là C
Question 46. Đáp án D
Câu hỏi. Theo đoạn văn, mảng băng Cordilleran dày nhất ở đâu?
A. Alaska B. Antarctica
C. Greenland D. Alberta
Đây là một câu hỏi hỏi chi tiết liên quan đến thông tin cố định. Để tìm câu trả lời ta chỉ cần tìm thông tin
cố định này trong văn bản. Đáp án sẽ nằm xung quanh khu vực ta vừa khoanh vùng thông tin.
Với câu hỏi này ta tìm thấy thông tin ở đoạn 3, dòng 2-3:
It was about 3 kilometers deep at its thickest point in northern Alberta.
Tạm dịch: Vị trí dày nhất của mảng băng này, ở phía bắc Alberta có độ sâu vào khoảng 3km
Như vậy ta chọn được đáp án D
Question 47. Đáp án A
Câu hỏi. Theo đoạn 5, những cánh đồng băng giống như những chỏm băng theo cách nào sau đây?
A. dòng chảy của chúng
B. kết cấu của chúng
C. vị trí của chúng
D. hình dạng của chúng
Ta tìm thấy thông tin trong đoạn 5, dòng 2-4: One form of mountain glacier that resembles an ice cap in
that it flows outward in several directions is called an ice field.
Tạm dịch: Một dạng của sông băng trên núi, có thể chảy ra mọi hướng giống như mảng băng, được gọi là
đồng băng
Trong đoạn trên ta thấy có từ khóa là flows outward → đáp án A đúng.
Question 48. Đáp án A
Câu hỏi. Từ “subtle” ở đoạn 5 có thể được thay thế bằng____________
A. slight (adj): nhẹ
B. substantial (adj): đáng kể
C. regional (adj): theo vùng
D. obvious (adj): rõ ràng
Ta tìm thấy thông tin trong đoạn 2, dòng 2-4: One form of mountain glacier that resembles an ice cap in
that it flows outward in several directions is called an ice field. The difference between an ice field and an
ice cap is subtle.
Tạm dịch: Một dạng của sông băng trên núi, có thể chảy ra mọi hướng giống như mảng băng, được gọi là
đồng băng. Sự khác biệt giữa đồng bằng và mảng băng là nhỏ thôi.
Như vậy chỉ có phương án A mới có thể thay thế cho tính từ subtle (adj): nhỏ, ít

Trang 19
Question 49. Đáp án D
Câu hỏi. Từ “their” ở đoạn cuối ám chỉ____________
A. Những đồng băng
B. những hố băng
C. những thung lũng băng
D. những thung lũng
Chúng ta tìm thấy thông tin ở đoạn cuối, 2 dòng cuối:
The ice ofvalley glaciers, bound by terrain, flows down valleys, curves around their corners, and falls
over cliffs.
Tạm dịch: Băng trên thung lũng sông băng, bị ràng buộc bởi địa hình, chảy xuống thung lũng, uốn cong
tại những góc thung lũng, rồi rơi xuống trên vách đá.
Tình từ sở hữu their thường để thay thế cho sở hữu của một danh từ đứng trước ở số nhiều. Trong câu
này chính là phương án D
Question 50. Đáp án B
Câu hỏi. Tất cả những cái sau đây là các sông băng trên núi trừ_____________
A. hố băng B. chỏm băng
C. đồng băng D. thung lũng băng
Dạng câu hỏi lựa chọn kiểu này chúng ta nên để làm sau cùng, khi đã hiểu nội dung chính của văn bản và
nắm được sơ bộ các vùng thông tin quan trọng trong văn bản. Chúng ta có thể dùng phép loại trừ với câu
hỏi này, đáp án A, C, D đều thuộc về nhóm mountain glaciers hay còn gọi là alpine glaciers. Ta thấy
thông tin trong đoạn 5, dòng 1-4:
The second category of glaciers includes those of a variety of shapes and sizes generally called mountain
or alpine glaciers. Mountain glaciers are typically identified by the landform that controls their flow. One
form of mountain glacier that resembles an ice cap in that it flows outward in several directions is called
an ice field.
Tạm dịch: Loại thứ 2 là loại sông băng đa dạng về hình dạng, kích thước, thường được gọi là sông băng
trên núi hoặc sông băng núi cao (liên quan đến núi Alps). Các sông băng trên núi thường được xác định
bởi dạng địa hình kiểm soát dòng chảy của chúng. Một dạng của sông băng trên núi, có thể chảy ra mọi
hướng giống như mảng băng, được gọi là đồng băng.
Và thông tin ở đoạn cuối, dòng 1-2:
Less spectacular than large ice fields are the most common types of mountain glaciers: the cirque and
valley glaciers.
Tạm dịch: Không hùng vĩ bằng những đồng băng rộng lớn là những loại sông băng trên núi phổ biến
nhất: những hố băng và thung lũng băng.
Như vậy đáp án B không được đề cập là sông băng

Trang 20
ĐỀ SỐ 12 ĐỀ THI THỬ TỐT NGHIỆP THPT
NĂM HỌC: 2020 – 2021
MÔN: TIẾNG ANH
Thời gian làm bài: 60 phút; không kể thời gian phát đề

Mark the letter A, B, C, or D on your answer sheet to indicate the word whose underlined part
differs from the other three in pronunciation in each of the following questions.
Question 1. A. theatre B. thunder C. therefore D. throughout
Question 2. A. apologises B. invites C. roofs D. certificates
Mark the letter A, B, C or D on your answer sheet to indicate the word that differs from the rest in
the position of the main stress in each of the following questions.
Question 3. A. sector B. lawyer C. workforce D. prefer
Question 4. A. diversity B. biography C. biology D. fundamental
Mark the letter A, B, C or D on your answer sheet to indicate the correct answer to each of the
following questions.
Question 5. I don’t know much about computer, so I asked the assistant for____________advice.
A. a little B. many C. a few D. a lot
Question 6. You should know that everyone in this office____________busy planning the dance for a
week.
A. is B. has been C. are D. have been
Question 7. The children were made____________up the mess they had left before they could watch
TV.
A. clear B. to clear C. clearing D. cleared
Question 8. Most headaches____________with aspirin unless they are too severe.
A. can treat B. can be treating C. can be treated D. can treats
Question 9. I wish I____________able to accept that role, but I was preparing for another
play at the time.
A. were B. was C. had been D. have been
Question 10. Everyone thought she would accept the offer. ____________, she turned it down
immediately.
A. Therefore B. Moreover C. However D. Hitherto
Question 11. I would have cooked something special if I____________you were coming.
A. knew B. know C. have known D. had known
Question 12. Women have proved repeatedly that they are equal and often superior____________men
in almost every field.
A. to B. than C. as D. over

Trang 1
Question 13. There is still widespread____________against older people in the job market.
A. discriminate B. discriminatory C. discrimination D. discriminating
Question 14. The arrested man is suspected of____________government funds for himself.
A.robbing B. appropriating C. appreciating D. confiscating
Question 15. It was hard to____________the temptation to watch the late-night show even though I
was so tired then.
A. defy B. resist C. refuse D. oppose
Question 16. The company managed to beat the ____________on delivering its new system.
A. team B. other C. time D. clock
Question 17. Even though she had been studying hard, she had only a____________chance of passing
the exam.
A. slim B. narrow C. thin D. light
Question 18. I ran in a marathon last week, but I was not fit enough. I____________after 15
kilometres.
A. dropped out B. moved in C. showed off D. closed down
Mark the letter A, B, C, or D on your answer sheet to indicate the underlined part that needs
correction in each of the following questions.
Question 19. Keith had so interesting and creative plans that everyone wanted to work on his
committee.
A. wanted B. plans C. so interesting D. on
Question 20. Air pollution, together with littering, are causing many problems in our large, industrial
world.
A. in our large B. with C. many D. are
Question 21. A paragraph is a portion of a text consists of one or more sentences related to the same
idea.
A. sentences B. consists of C. to D. A paragraph
Mark the letter A, B, C or D on your answer sheet to indicate the sentence that is closest in
meaning to each of the following questions.
Question 22. Greater use of public transport would cut the amount of pollution from cars.
A. Were more people to use public transport, cars would stop releasing exhaust into the atmosphere.
B. If more people use public transport, it will cut the amount of pollution from cars.
C. If public transport was widely used, people would no longer suffer from pollution from cars.
D. If more people used public transport, there would be less pollution from cars.
Question 23. Mark delayed writing the book until he had done a lot of research.
A. Mark did a lot of research after he finished writing the book.
B. It was only when Mark had written the book that he did a lot of research.

Trang 2
C. Mark delayed writing the book as he had already done any research.
D. Only after Mark had done a lot of research did he begin to write the book.
Question 24. “Would you like to come out to dinner with me tonight, Jenny?” Paul said.
A. Paul suggested that Jenny go out to dinner with him that night.
B. Paul insisted on Jenny going out to dinner with him that night.
C. Paul invited Jenny to go out to dinner with him that night.
D. Pau offered Jenny to go out to dinner with him that night
Mark the letter A, B, C or D on your answer sheet to indicate the sentence that best combines
each pair of sentences in the following questions.
Question 25. Lee gave up her job. She planned to continue her education.
A. Lee’s education was interrupted since she wanted to find a job.
B. Lee gave up her job with the aim at continuing her education.
C. Lee gave up her job in case she continued her education.
D. Lee gave up her job with a view to continuing her education.
Question 26. This is a rare opportunity. You should take advantage of it to get a better job.
A. You should take advantage of this rare opportunity, for which is to get a better job.
B. This is a rare opportunity what you should take advantage of to get a better job.
C. You should take advantage of this opportunity, that is rare to get a better job.
D. This is a rare opportunity that you should take advantage of to get a better job.
Mark the letter A, B, C or D on your answer sheet to indicate the most suitable response to
complete each of the following exchanges.
Question 27. Mark and Anne are talking after the class.
Mark: “Guess what? I’ve been recruited to be a member of Green Dream Volunteer Group.”
Anne: “____________”
A. Good luck next time!
B. That’s great! Congratulations!
C. That’s the least thing I could do for you.
D. It doesn’t make sense to me.
Question 28. Amy and Jacob are talking about the two-day excursion at the end of the school year.
Amy: “You look so sad. ____________”
Jacob: “I couldn’t get my parents’ permission to stay the night away from home.”
A. Can you help me? B. How do you deal with it?
C. What should I do? D. what’s the problem?
Mark the letter A, B, C, or D on your answer sheet to indicate the word or phrase that is
CLOSEST in meaning to the underlined part in each of the following questions.

Trang 3
Question 29. It was late at night, the wind was howling and when she heard the knock on the door, she
almost jumped out of her skin.
A. was surprised B. was asleep C. was terrified D. was delighted
Question 30. Wild animals use various methods to ward off predators and their natural enemies.
A. rebel B. befriend C. deter D. attack
Mark the letter A, B, C, or D on your answer sheet to indicate the word or phrase that is
OPPOSITE in meaning to the underlined part in each of the following questions.
Question 31. We should grow more trees so that they can absorb more carbon dioxide from the
atmosphere.
A. take in B. emit C. consume D. cut off
Question 32. We have achieved considerable results in the economic field, such as high economic
growth, stability and significant poverty alleviation over the past few years.
A. eradication B. aggravation C. prevention D. reduction
Read the following passage and mark the letter A, B, C or Don your answer sheet to indicate the
correct word for each of the blanks from 33 to 37.
Everyone wants to reduce pollution. But the pollution (33) ____________is as complicated as it is
serious. It is complicated because much pollution is caused by things that benefit people. (34)
____________, exhaust from automobiles causes a large percentage of air pollution. But the automobile
provides transportation for millions of people. Factories discharge much of the material that pollutes the
air and water but factories give (35) ____________to a large number of people.
Thus, to end or greatly reduce pollution immediately, people would have to (36) ____________using
many things that benefit them. Most people do not want to do that, of course. But pollution can be
gradually reduced in several ways. Scientists and engineers can work to find ways to lessen the amount
of pollution that such things as automobiles and factories cause. Governments can pass and enforce
laws (37) ____________require businesses and traffic to stop, or to cut down on certain polluting
activities.
Question 33. A. event B. accident C. work D. problem
Question 34. A. As a result B. However C. Therefore D. For example
Question 35. A. employed B. employment
C. unemployed D. unemployment
Question 36. A. start B. stop C. enjoy D. continue
Question 37. A. whom B. that C. whose D. who
Read the following passage and mark the letter A, B, C, or D on your answer sheet to indicate the
correct answer to each of the questions from 38 to 42.
By adopting a few simple techniques, parents who read to their children can considerably increase their
children's language development. It is surprising, but true. How parents talk to their children makes a

Trang 4
big difference in the children's language development. If a parent encourages the child to actively
respond to what the parent is reading, the child's language skills increase.
A study was done with two or three-year-old children and their parents. Half of the thirty children
participants were in the experimental study; the other half acted as the control group. In the
experimental group, the parents were given a two-hour training session in which they were taught to
ask open-ended questions rather than yes-no questions. For example, the parent should ask, "What is
the doggy doing?" rather than, "Is the doggy running away?" Experimental parents were also instructed
how to expand on their children's answer, how to suggest alternative possibilities, and how to praise
correct answers.
At the beginning of the study, the children did not differ on levels of language development, but at the
end of one month, the children in the experimental group were 5.5 months ahead of the control group
on a test of verbal expression and vocabulary. Nine months later, the children in the experimental group
still showed an advance of 6 months over the children in the control group.
Question 38. Parents can give great help to their children's language development
by____________them.
A. responding to B. reading to C. adopting D. experimenting
Question 39. what does the word "they" in the second paragraph refer to?
A. Participants B. Children C. Questions D. Parents
Question 40. During the training session, experimental parents were taught to____________.
A. use yes-no questions B. study many experiments
C. ask open-ended questions D. give correct answers
Question 41. what was the major difference between the control group and the experimental one
in the study?
A. The number of participants. B. The books that were read.
C. The age of the children. D. The training that parents received.
Question 42. What conclusion can be drawn from this passage?
A. Children who read actively always act six months earlier than those who don't.
B. The more children read, the more intelligent they become.
C. Two or three-year-old children can be taught to read actively.
D. Children's language skills increase when they are required to respond actively.
Read the following passage and mark the letter A, B, C, or D on your answer sheet to indicate the
correct answer to each of the questions from 43 to 50.
The elements other than hydrogen and helium exist in such small quantities that it is accurate to say that
the universe is somewhat more than 25 percent helium by weight and somewhat less than 75 percent
hydrogen.

Trang 5
Astronomers have measured the abundance of helium throughout our galaxy and in other galaxies as
well. Helium has been found in old stars, in relatively young ones, in interstellar gas, and in the distant
objects known as quasars. Helium nuclei have also been found to be constituents of cosmic rays that
fall on the earth (cosmic rays are not really a form of radiation; they consist of rapidly moving
particles of numerous different kinds). It doesn’t seem to make very much difference where the helium
is found. Its relative abundance never seems to vary much. In some places, there may be slightly more
of it; in others, slightly less, but the ratio of helium to hydrogen nuclei always remains about the same.
Helium is created in stars. In fact, nuclear reactions that convert hydrogen to helium are responsible for
most of the energy that stars produce. However, the amount of helium that could have been produced in
this manner can be calculated, and it turns out to be no more than a few percent. The universe has not
existed long enough for this figure to be significantly greater. Consequently, if the universe is
somewhat more than 25 percent helium now, then it must have been about 25 percent helium at a time
near the beginning.
However, when the universe was less than one minute old, no helium could have existed. Calculations
indicate that before this time temperatures were too high and particles of matter were moving around
much too rapidly. It was only after the one- minute point that helium could exist. By this time, the
universe had cooled so sufficiently that neutrons and protons could stick together. But the nuclear
reactions that led to the formations of helium went on for only relatively short time. By the time the
universe was a few minutes old, helium production had effectively ceased
Question 43. What does the passage mainly explain?
A. Why hydrogen is abundant.
B. How stars produce energy.
C. When most of the helium in the universe was formed.
D. The difference between helium and hydrogen.
Question 44. According to the passage, helium is____________.
A. difficult to detect.
B. the oldest element in the universe.
C. the second-most abundant element in the universe.
D. the most prevalent element in quasars.
Question 45. The word “constituents” is closest in meaning to____________.
A. causes B. relatives C. components D. targets
Question 46. Why does the author mention “cosmic rays”?
A. To explain how the universe began.
B. As an example of an unsolved astronomical puzzle.
C. To explain the abundance of hydrogen in the universe.
D. As part of a list of things containing helium.

Trang 6
Question 47. The word “they” refers to____________ .
A. radiation B. constituents C. cosmic rays D. particles
Question 48. The word “vary” is closest in meaning to____________.
A. include B. mean C. stretch D. change
Question 49. The creation of helium within stars____________.
A. cannot be measured
B. produces hydrogen as a by-product
C. produces energy
D. causes helium to be much more abundant in old stars than in young stars
Question 50. Most of the helium in the universe was formed____________ .
A. in a very short time
B. in interstellar space
C. before most of the hydrogen
D. during the first minute of the universes existence

Trang 7
Đáp án
1-C 2-A 3-D 4-D 5-A 6-B 7-B 8-C 9-C 10-C
11-D 12-A 13-C 14-B 15-B 16-D 17-A 18-A 19-C 20-D
21-B 22-D 23-D 24-C 25-D 26-D 27-B 28-D 29-C 30-C
31-B 32-B 33-D 34-D 35-B 36-B 37-B 38-B 39-D 40-C
41-D 42-D 43-C 44-C 45-C 46-B 47-C 48-D 49-C 50-A

LỜI GIẢI CHI TIẾT


Question 1: Đáp án C
Đáp án C đúng vì phần gạch chân của đáp án C là âm /ð/. Các đáp án còn lại có phần gạch chân là âm /θ/.
A. theatre /ˈθɪətə(r)/ (n): rạp chiếu phim
B. thunder /ˈθʌndə(r)/ (n): tiếng sấm
C. therefore /ˈðeəfɔː(r)/ (adv nối câu): vì thế/ cho nên
D. throughout /θruːˈaʊt/ (prep): xuyên suốt
Question 2. Đáp án A
Đáp án A đúng vì phần gạch chân của đáp án A là âm /ɪz/. Các đáp án còn lại có phần gạch chân là âm /s/.
A. apologises /əˈpɒlədʒaɪz/ (v): xin lỗi
B. invites /ɪnˈvaɪts/ (n): mời
C. roofs /ru:fs/ (n): mái nhà
D. certificates /səˈtɪfɪkət/ (n): chứng chỉ
* Mở rộng: Quy tắc phát âm đuôi -s/-es
-s/-es được phát âm là: Khi tận cùng của từ là: Ví dụ
/s/ Các phụ âm vô thanh: /k/, /θ/, /f/, stops, laughs, accepts, months
/p/, /t/
/ɪz/ Các âm: /s/, /z/, /ʃ/, /ʒ/, /tʃ/, /dʒ/ kisses, washes, matches, changes,
buzzes
/z/ Các âm hữu thanh còn lại styles, intends, orphans

Question 3: Đáp án D
Đáp án D đúng vì đáp án D có trọng âm rơi vào âm tiết thứ 2. Các đáp án còn lại có trọng âm rơi vào âm
tiết thứ nhất.
A. sector /ˈsektə(r)/ (n): phấn/ bộ phận
B. lawyer /ˈlɔɪə(r)/ (n): luật sư
C. workforce /ˈwɜːkfɔːs/ (n): số nhân viên/ lực lượng lao động
D. prefer /prɪˈfɜː(r)/ (v): thích hơn/ ưu tiên hơn
Question 4. Đáp án D

Trang 8
Đáp án D đúng vì đáp án D có trọng âm rơi vào âm tiết thứ 3. Các đáp án còn lại có trọng âm rơi vào âm
tiết thứ 2.
A. diversity /daɪˈvɜːsəti/ (n): sự đa dạng
B. biography /baɪˈɒɡrəfi/ (n): tiểu sử/ lý lịch
C. biology /baɪˈɒlədʒi/ (n): sinh học
D. fundamental /ˌfʌndəˈmentl/ (a): căn bản/ cơ bản
Question 5. Đáp án A
Đáp án A - câu hỏi về lượng từ
Danh từ “advice” - lời khuyên - là danh từ không đếm được nên hai đáp án B và C bị loại (many và a few
đi với danh từ đếm được số nhiều). Đáp án D - a lot - cũng bị loại vì thiếu “of”. Chỉ còn lại đáp án A
đúng, a little đi với danh từ không đếm được.
Tạm dịch: Tôi không biết nhiều về máy tính, vậy nên tôi hỏi xin trợ lý của tôi một ít lời khuyên.
Question 6. Đáp án B
Đáp án B - câu hỏi thì động từ
“For a week” là trạng ngữ chỉ thời gian và là dấu hiệu nhận biết của thì hoàn thành, nên đáp án A và C (is
và are là hiện tại đơn) bị loại. “Everyone” là đại từ bất định và là chủ ngữ số ít, nên đáp án D - have been
(chia số nhiều) bị loại. Chỉ còn lại đáp án B đúng.
Tạm dịch: Anh nên biết rằng mọi người trong văn phòng đã bận rộn lên kế hoạch cho buổi vũ hội cả tuần
nay.
Question 7. Đáp án B
Đáp án B - câu hỏi câu bị động với to V
Động từ “make” có cấu trúc chủ động là : S + make sb/ sth do sth: khiến ai đó làm gì.
Cấu trúc này chuyển về thể bị động là: S + be + made + to do sth.
Xét 4 đáp án, chỉ có đáp án B - to clear - là đáp án phù hợp.
Tạm dịch: Mấy đứa trẻ bị bắt đi dọn dẹp đống bừa bộn mà chúng để lại trước khi chúng có thể xem Ti Vi.
* Mở rộng: Tất cả các cấu trúc chủ động có dạng: S + V1 + sb/sth + do sth
Đều được chuyển sang thể bị động có dạng: S + be + P2 (V1) + to do sth.
NGOẠI TRỪ: Trường hợp bị động với let : S + be + let + V_bare inf
Question 8. Đáp án C
Đáp án C - câu hỏi câu bị động với động từ khuyết thiếu
Động từ “treat” - chữa trị - là ngoại động từ ( là loại động từ luôn cần có danh từ làm tân ngữ theo sau).
Tuy nhiên, trong câu bài cho, sau chỗ trống không có danh từ làm tân ngữ, nên “treat” phải chia ở dạng bị
động.
Xét 4 đáp án, chỉ có đáp án C - can be treated - là phù hợp bởi dạng bị động với động từ khuyết thiếu luôn
là: Modal verb + be + P2.

Trang 9
Tạm dịch: Hầu hết các chứng đau đầu đều có thể được chữa trị với thuốc giảm đau aspirin nếu như chúng
không quá nghiêm trọng.
Question 9. Đáp án C
Đáp án C - câu hỏi giả định
Mệnh đề theo sau “wish” - ước rằng - được chia ở thức giả định.
Xét thời gian của câu này ở quá khứ do có mệnh đề “but I was preparing for another play at the time.”
chia ở quá khứ tiếp diễn, nên mệnh đề sau “wish” được chia ở thức giả định loại 3, tức là thì quá khứ
hoàn thành hoặc quá khứ hoàn thành tiếp diễn.
Vì vậy, đáp án C - had been (quá khứ hoàn thành) là phù hợp.
Tạm dịch: Tôi ước rằng tôi đã có thể nhận vai đó, nhưng khi đó tôi đang phải tập cho một vở kịch khác.
* Mở rộng: Ngoài “wish”, mệnh đề theo sau “if only” cũng chỉ mong ước và được chia ở thức giả định
(loại 2 hoặc loại 3) - tức là thì quá khứ đơn/ quá khứ tiếp diễn (loại 2) và quá khứ hoàn thành/ quá khứ
hoàn thành tiếp diễn (loại 3).
Question 10. Đáp án C
Đáp án C - câu hỏi trạng từ nối
Câu hỏi nghĩa của trạng từ nối. Căn cứ vào nghĩa của các trạng từ nối để chọn đáp án C - however.
A. Therefore (adv nối câu): vì thế/ cho nên
B. Moreover (adv nối câu): hơn thế/ ngoài ra
C. However (adv nối câu): tuy nhiên/ tuy vậy
D. Hitherto (adv): đến thời điểm này
Tạm dịch: Tất cả mọi người đều cho rằng cô ấy sẽ chấp nhận đề nghị đó. Tuy nhiên, cô ấy từ chối nó
ngay lập tức.
Question 11. Đáp án D
Đáp án D - câu hỏi câu điều kiện
“Could/ would/ should/ might + (not) + have + P2” được dùng trong mệnh đề chính của câu điều kiện loại
3, vậy nên câu đề bài cho là càu điều kiện loại 3 - cũng là câu giả định loại 3. Động từ ở câu giả định loại
3 được chia ở quá khứ hoàn thành hoặc quá khứ hoàn thành tiếp diễn, nên đáp án D - had known (chia ở
thì quá khứ hoàn thành) là đáp án đúng.
Tạm dịch: Đáng lẽ tôi sẽ nấu thêm một vài món đặc biệt nếu tôi biết bạn sắp tới.
Question 12. Đáp án A
Đáp án A - câu hỏi giới từ
Tính từ “superior” - tốt hơn/ thượng đẳng hơn - đi kèm với giới từ “to”. Vậy nên, đáp án A - to - là đáp án
phù hợp.
Tạm dịch: Phụ nữ đã nhiều lần chứng mình rằng họ ngang bằng và thường thì tốt hơn đàn ông trong hầu
hết mọi lĩnh vực.
* Mở rộng: Hầu hết các tính từ có hậu tố “-ior” đều đi với giới từ “to”. Ví dụ: superior/ inferior/ junior...

Trang 10
Question 13. Đáp án C
Đáp án C - câu hỏi từ loại
Cấu trúc “there + be + Noun” được dùng để chỉ sự tồn tại của một sự vật/ sự việc.
Trong câu bài cho “there is still widespread_____________ ” (với “still” là trạng từ, và “widespread” là
tính từ) nên theo cấu trúc, chỗ trống còn thiếu một danh tù.
Xét 4 đáp án, đáp án C - discrimination (sự phân biệt/ kì thị) là danh từ và là đáp án phù hợp.
A. discriminate (v): phân biệt đối xử/ kì thị
B. discriminatory (a): phân biệt đối xử
C. discrimination (n): sự phân biệt đối xử/ sự kì thị
D. discriminating (a): kĩ lưỡng/ kĩ tính.
Tạm dịch: vẫn còn tồn tại nhiều sự kì thị đối với người lớn tuổi trong thị trường việc làm.
Question 14. Đáp án B
Đáp án B - câu hỏi từ vựng
Câu này chọn đáp án đúng phù hợp với nghĩa và kết hợp từ. Đáp án B - appropriating (biển thủ/ lấy trộm)
là đáp án phù hợp.
A. rob (v): ăn trộm - đi với cấu trúc: Rob sb of sth
B. appropriate (v): biển thủ/ lấy trộm (có một appropriate là tính từ mang nghĩa: phù hợp)
C. apprecitate (v): cảm kích/ đánh giá cao
D. confiscate (v): tịch thu/ thu vào công quỹ
Tạm dịch: Người đàn ông bị bắt bị tình nghi biển thủ công quỹ cho riêng mình.
Question 15. Đáp án B
Đáp án B - câu hỏi từ vựng
Câu này chọn đáp án đúng phù hợp với nghĩa và kết hợp từ. Đáp án B - resist (chống lại/ chống đối), theo
kết hợp từ (collocation), đi với danh từ “temptation” và phù hợp với nghĩa của câu.
A. defy (v): không tuân thủ/ vi phạm
B. resist (v): chống lại/ chống đối
C. refuse (v): từ chối
D. oppose (v): phản đối
Tạm dịch: Việc chống lại cám dỗ của việc xem chương trình đêm muộn là rất khó mặc dù tôi đã rất mệt
vào lúc đó.
Question 16. Đáp án D
Đáp án D - câu hỏi thành ngữ
Thành ngữ “beat the clock” - hoàn thành trước thời hạn/ làm xong trước hạn.
Xét 4 đáp án, chỉ có thể chọn được đáp án D - clock.
A. team (n): đội
B. other (n)/ (a): khác/ cái khác/ người khác

Trang 11
C. time (n): thời gian
D. clock (n): đồng hồ treo tường
Tạm dịch: Công ty này đã cố gắng hoàn thành trước thời hạn việc đưa ra hệ thống mới của mình.
Question 17. Đáp án A
Đáp án A - câu hỏi từ vựng
Câu này chọn đáp án đúng phù hợp với nghĩa và kết hợp từ.
Đáp án A - slim (gầy gò/ nhỏ bé), theo kết hợp từ (collocation), đi với danh từ “chance” và phù hợp với
nghĩa của câu.
A. slim (a): gầy gò/ nhỏ bé
B. narrow (a): hẹp
C. thin (a): mảnh khảnh/ gầy
D. light (a): nhẹ
Tạm dịch: Mặc dù cô ấy học rất chăm chỉ, cô ấy chỉ có một khả năng nhỏ để vượt qua kì thi.
Question 18. Đáp án A
Đáp án A - câu hỏi phrasal verb
Câu hỏi này cần chọn một đáp án có cụm động từ (phrasal verb) phù hợp nghĩa nhất. Chọn A - dropped
out vì:
A. drop out: bỏ cuộc
B. move in: dọn vào ở
C. show off: khoe ra/ khoe khoang
D. close down: đóng cửa
Tạm dịch: Tôi tham gia vào một cuộc chạy việt giã vào tuần trước, nhưng tôi không đủ sức khỏe và bỏ
cuộc sau khi chạy được 15 km.
Question 19. Đáp án C
Mệnh đề trạng ngữ chỉ kết quả với “so” và “such” thường có dạng:
Với “so”: ... so + adj/ adv + that + clause.
Với “such”: ... such + (a/an) + (adj) +Noun + that + clause.
Câu đề bài cho có danh từ “plans” sau hai tính từ “interesting” và “creative”, nên vị trí của “so” phải được
thay bằng “such”. Vậy nên, đáp án C - so → such.
Tạm dịch: Keith có những kế hoạch thú vị và sáng tạo đến nỗi mà ai cũng muốn làm việc trong ủy bản
của cô ấy.
Question 20. Đáp án D
“Air pollution” là danh từ không đếm được và là chủ ngữ số ít nên động từ theo sau nó phải được chia ở
số ít. Vậy nên, đáp án D - are → is.
Tạm dịch: Ô nhiễm không khí, cùng với việc xả rác bừa bãi, đang gây ra nhiều vấn đề trong thế giới công
nghiệp hóa rộng lớn của chúng ta.

Trang 12
* Mở rộng: Các cụm “with, along with, as well as, together with, accompanied by, besides, in addition
to...” thực chất ra là các giới từ. Vì vậy, các cụm này kết hợp với danh từ theo sau, ví dụ như trên đề bài là
“together with littering”, tạo thành một cụm giới từ - có vai trò làm trạng ngữ trong câu, nên động từ
trong câu sẽ không bị ảnh hưởng bởi các cụm này bởi động từ chỉ chia theo chủ ngữ chứ không chia theo
trạng ngữ.
Question 21. Đáp án B
“A text” là danh từ làm tân ngữ trong câu, vậy nên động từ theo sau nó không thể chia ở dạng bình
thường như “consists”. Trong trường hợp này, động từ “consist” cần phải chia ở dạng mệnh đề quan hệ
rút gọn.
Vậy, đáp án B - consists of → consisting of.
Question 22. Đáp án D
Dịch đề bài: Việc sử dụng các phương tiện giao thông công cộng nhiều hơn sẽ cắt giảm lượng ô nhiễm
phát thải từ ô tô riêng (Câu đề bài dùng “would” - ý nghĩa giả định - thực chất câu này là câu điều kiện
loại 2, ngụ ý: nếu dùng phương tiện công cộng nhiều hơn thì lượng ô nhiễm từ ô tô con sẽ giảm - nhưng
thực tế thì không như thế).
A. Nếu có nhiều người sử dụng phương tiện công cộng hơn, ô tô riêng sẽ ngưng phát thải khói vào bầu
khí quyển. → sai nghĩa.
B. Nếu có nhiều người sử dụng phương tiện công cộng hơn, điều này sẽ cắt giảm lượng ô nhiễm phát thải
từ ô tô riêng → đúng ý nhưng sai loại câu điều kiện (loại 1, bài cho loại 2).
C. Nếu các phương tiện giao thông công cộng được sử dụng rộng rãi, con người sẽ không phải chịu đựng
ô nhiễm phát thải từ ô tô riêng nữa. → sai nghĩa/ và nếu xét chặt chẽ theo ngữ pháp chính thống thì sai cấu
trúc vì câu điều kiện loại 2 có “be” luôn là “were”.
D. Nếu có nhiều người sử dụng phương tiện công cộng hơn, ô nhiễm phát thải từ ô tô riêng sẽ ít đi. →
đúng nghĩa, đúng cấu trúc.
Đáp án D đúng về nghĩa và cấu trúc câu điều kiện loại 2. Các đáp án còn lại sai nghĩa hoặc sai cấu trúc
Question 23. Đáp án D
Dịch đề bài: Mark hoãn việc viết sách lại cho đến khi đã xong việc nghiên cứu kĩ lưỡng, (ngụ ý: Nghiên
cứu kĩ lưỡng xong mới viết sách).
A. Mark nghiên cứu rất kĩ lưỡng sau khi viết xong quyển sách. → sai nghĩa
B. Chỉ khi Mark đã viết xong quyển sách anh ta mới nghiên cứu kĩ lưỡng. → sai nghĩa
C. Mark hoãn viết quyển sách lại vì anh ấy đã nghiên cứu rất kĩ lưỡng. → sai nghĩa
D. Chỉ sau khi Mark đã nghiên cữu kĩ lưỡng, anh ấy mới bắt đầu viết sách. → đúng
Đáp án D đúng về nghĩa theo ngụ ý đề bài cho. Các đáp án khác còn lại sai nghĩa.
Question 24. Đáp án C
Dịch đề bài: “Em có muốn ra ngoài ăn tối với anh hôm nay không Jenny?” Paul hỏi. (Đây là cấu trúc
dùng để mời).

Trang 13
A. Paul gợi ý rằng Jenny nên ra ngoài ăn tối với anh ấy hôm đó. → sai ý
B. Paul nằng nặc muốn Jenny ra ngoài ăn tối với anh ấy hôm đó. → sai ý
C. Paul mời Jenny ra ngoài ăn tối với anh ấy hôm đó. → đúng
D. Không dịch vì sai cấu trúc: Động từ “offer” không có cấu trúc “offer sb to do sth”.
Đáp án C đúng ý đề bài cho. Các đáp án còn lại sai ý hoặc sai cấu trúc.
Question 25. Đáp án D
Dịch đề bài: Lee từ bỏ công việc của mình. Cô ấy có kế hoạch tiếp tục việc học. (ngụ ý: Ngưng công việc
để đi học lại).
A. Việc học của Lee bị gián đoạn vì cô ấy muốn tìm một công việc. → sai nghĩa
B. Không dịch vì sai cấu trúc: “with the aim of + sth/ V-ing” chứ không có cấu trúc “with the aim at...”
C. Lee từ bỏ công việc của mình để phòng trường hợp cô ấy tiếp tục việc học. → sai nghĩa
D. Lee từ bỏ công việc với mục đích tiếp tục việc học. → đúng
Đáp án D đúng nghĩa và cấu trúc. Các đáp án lại sai nghĩa hoặc cấu trúc.
* Mở rộng: “with a view to sth/ V-ing” và “with the aim of sth/ V-ing” là cấu trúc dùng để chỉ mục đích.
Question 26. Đáp án D
Dịch đề bài: Đây là một cơ hội hiếm có. Cậu nên tận dụng nó để có được một công việc tốt hơn.
A. Không dịch vì sai cấu trúc: “for which” là cụm giới từ - làm trạng ngữ - không làm được chủ ngữ.
B. Không dịch vì sai cấu trúc: “what” không xuất hiện trong mệnh đề quan hệ, thay “what” bằng “which”
thì đúng cấu trúc.
C. Không dịch vì sai cấu trúc: “that” không xuất hiện trong mệnh đê' quan hệ không xác định (tức là
không theo sau dấu phẩy).
D. Đây là một cơ hội hiếm có mà cậu cần phải tận dụng để có được một công việc tốt hơn. → đúng.
Chỉ duy nhất đáp án D đúng về nghĩa và cấu trúc
Question 27. Đáp án B
“Guess what? I’ve been recruited to be a member of Green Dream Volunteer Group.” - “Biết chuyện gì
không? Tớ mới vừa trúng tuyển làm thành viên của Hội Tình nguyên Giấc mơ Xanh đấy.” - đây là câu
thông báo tin mừng.
Đối với lời thông báo tin mừng, lời đáp lại tốt nhất là chúc mừng và một câu biểu cảm sự vui mừng.
Xét 4 đáp án, đáp án B phù hợp nhất.
A. Chúc may mắn lần sau! - câu trả lời cho lời thông báo không may mắn/ tin xấu ở lần này.
B. Thật tuyệt! Chúc mừng bạn! - dùng để đáp lại tin mừng
C. Đó là điều tối thiểu tôi có thể làm cho bạn. - dùng để đáp lại lời cám ơn vì đã được giúp đỡ.
D. Tôi không hiểu. → không phù hợp về nghĩa.
* Mở rộng: “Sense” có hai cấu trúc thường dùng:
1/ Sb make sense of sth: sb hiểu được sth.
2/ Sth make sense to sb: sth có ý nghĩa/ có thể hiểu được với sb.

Trang 14
Question 28. Đáp án D
Amy: “You look so sad._____________” -
“Cậu có vẻ buồn._____________”
Jacob: “I couldn’t get my parents’ permission to stay the night away from home.”- “Tớ không thể xin
phép bố mẹ để ngủ ở ngoài một đêm.”
Căn cứ vào nghĩa của 4 đáp án để chọn đáp án D mang nghĩa phù hợp.
A. Cậu giúp tôi một việc được không?
B. Cậu giải quyết việc đó thế nào?
C. Tôi nên làm gì?
D. Có vấn đề gì thế?
Question 29. Đáp án C
“Jump out of one’s skin” - cực kì sợ hãi/ lo lắng”. Chọn đáp án C - was terrified: cực kì sợ hãi - là gần
nghĩa nhất với từ đề bài cho.
A. was surprised: ngạc nhiên
B. was asleep: ngủ
C. was terrified: rất sợ hãi
D. was delighted: vui vẻ/ hài lòng
Tạm dịch: Đêm về khuya, gió gào thét, và khi cô ấy nghe thấy tiếng gõ cửa, cô ấy gần như sợ hãi tột
cùng.
Question 30. Đáp án C
“Ward off” - bảo vệ bản thân khỏi... Chọn đáp án C - deter (v): ngăn cản/ gây khó khăn/ ngăn chặn - là
gần nghĩa nhất với từ đề bài cho.
A. rebel (v): nổi dậy chống lại
B. befriend (v): kết bạn
C. deter (v): ngăn cản/ gây khó khăn. Thường dùng theo cấu trúc: deter sb from doing sth.
D. attack (v): tan công
Tạm dịch: Các loài động vật hoang dã sử dụng nhiều phương pháp để ngăn các loài săn mồi và các loài
thiên địch.
Question 31. Đáp án B
“Absorb” (v) - hấp thụ. chọn đáp án B - emit (v): phát thải/ thải ra - là trái nghĩa với từ để bài cho.
A. take in (phrs verb): hấp thu
B. emit (v): thải ra/ phát thải
C. consume (v): tiêu thụ/ sử dụng
D. cut off (phrs verb): cắt bỏ
Tạm dịch: Chúng ta nên trồng nhiều cây xanh hơn để chúng có thể hấp thụ khi CO2 từ bầu khí quyển.
Question 32. Đáp án B

Trang 15
“Alleviation” (n) - giảm nhẹ đi/ sự làm giảm. Chọn đáp án B - aggravation (n): sự làm nghiêm trọng
thêm/ làm trầm trọng thêm - là trái nghĩa với từ đề bài cho.
A. eradication (n): sự tiêu diệt/ sự loại bỏ
B. aggravation (n): sự làm nghiêm trọng thêm/ làm trầm trọng thêm.
C. prevention (n): sự ngăn chặn
D. reduction (n): sự làm giảm/ sự hạn chế
Tạm dịch: Chúng ta đã đạt được những kết quả đáng kể trong lĩnh vực kinh tế như là tốc độ tăng trưởng
cao, sự ổn định, và xóa đói giảm nghèo đáng kể trong những năm qua.
Question 33. Đáp án D
A. event (n): sự kiện
B. accident (n): tai nạn
C. work (n): công việc
D. problem (n): vấn đề
Câu hỏi từ vựng. Căn cứ vào nghĩa để chọn đáp án D - problem.
Trích bài: Everyone wants to reduce pollution. But the pollution problem is as complicated as it is
serious.
Tạm dịch: Tất cả mọi người đều muốn giảm thiểu ô nhiễm. Nhưng vấn đề ô nhiễm cũng phức tạp như
tính nghiêm trọng của nó vậy.
Question 34. Đáp án D
A. As a result: vậy nên
B. However: tuy nhiên/ tuy vậy
C. Therefore: vì vậy/ cho nên
D. For example: ví dụ như
Câu hỏi ngữ pháp và từ vựng. Vị trí đứng đầu câu và cách câu bằng dấu phẩy là trạng ngữ. Căn cứ vào
nghĩa của 4 đáp án, chọn đáp án phù hợp là đáp án D - For example.
Trích bài: It is complicated because much pollution is caused by things that benefit people.
For example, exhaust from automobiles causes a large percentage of air pollution. But the automobile
provides transportation for millions of people.
Tạm dịch: Vấn đề này phức tạp bởi vì phần lớn ô nhiễm được gây ra bởi những thứ tạo ra lợi ích cho con
người. Ví dụ như khí thải từ xe ô tô gây ra một phần lớn ô nhiễm không khí. Nhưng ô tô cung cấp phương
tiện di chuyển cho hàng triệu người.
Question 35. Đáp án B
A. employed (a): có việc làm
B. employment (n): công việc
C. unemployed (a): thất nghiệp
D. unemployment (n): sự thất nghiệp

Trang 16
Câu hỏi từ vựng. Căn cứ vào nghĩa để chọn đáp án B - employment.
Trích bài: Factories discharge much of the material that pollutes the air and water but factories give
employment to a large number of people.
Tạm dịch: Các nhà máy thải ra phần lớn các chất gây ô nhiễm không khí và nước, nhưng các nhà máy lại
cung cấp việc làm cho nhiều người.
Question 36. Đáp án B
A. start (v): bắt đầu + V-ing
B. stop (v): dừng + V-ing
C. enjoy (v): thích thú + V-ing
D. continue (v): tiếp tục + V-ing
Câu hỏi về từ vựng. Căn cứ vào nghĩa để chọn đáp án B - stop.
Trích bài: Thus, to end or greatly reduce pollution immediately, people would have to stop using many
things that benefit them.
Tạm dịch: Vì thế cho nên, để chấm dứt hoặc giảm mạnh ô nhiễm ngay lập tức, con người sẽ phải dừng sử
dụng nhiều thứ mang lại lợi ích cho họ.
Question 37. Đáp án B
Câu hỏi ngữ pháp. Mệnh đề quan hệ xác định theo sau để bổ sung thông tin cho danh từ “laws”: luật pháp
- chỉ vật nên phải được bắt đầu bởi “which” hoặc “that”. Xét 4 đáp án, chỉ có thể chọn B - that là phù hợp.
Trích bài: Governments can pass and enforce laws that require businesses and traffic to stop, or to cut
down on certain polluting activities.
Tạm dịch: Các chính phủ có thể thông qua và áp đặt các điều luật yêu cẩu các doanh nghiệp và các
phương tiện giao thông phải dừng lại hoặc cắt giảm các hoạt động gây ô nhiễm.
Question 38. Đáp án B
Dịch đề bài: Các bậc cha mẹ có thể giúp cho con trẻ của mình phát triển khả năng ngôn ngữ bằng cách
____________chúng.
A. Đáp lời B. Đọc sách cho
C. Nhận nuôi D. Làm thí nghiệm
Câu hỏi lấy thông tin.
* Hướng dẫn làm bài:
Đối với loại bài tìm thông tin, cần chọn ra một vài từ khóa ở đáp án cũng như ở câu hỏi rồi tìm những từ
khóa đó ở trên bài đọc. Khi đã tìm được đoạn chứa từ khóa trên bài đọc, cẩn đọc kĩ và chọn lọc những
thông tin nào ủng hộ đáp án. Sẽ chỉ có một đáp án được thông tin trong bài ủng hộ, và đó là đáp án đúng.
Trích thông tin trong bài: “By adopting a few simple techniques, parents who read to their children can
considerably increase their children's language development.”

Trang 17
Tạm dịch: Bằng cách áp dụng một vài kĩ thuật đơn giản, các bậc cha mẹ, những người đọc sách cho con
nghe, có thể tăng tốc độ phát triển khả năng ngôn ngữ của con mình đáng kể. → việc đọc sách cho con có
thể giúp tăng khả năng ngôn ngữ của con trẻ.
Vậy đáp án B phù hợp.
Question 39. Đáp án D
Dịch đề bài: Từ “they” ở đoạn hai nói tới cái gì?
A. Những người tham gia
B. Con trẻ
C. Các câu hỏi
D. Các bậc cha mẹ
* Hướng dẫn làm bài: Đối với loại câu hỏi này, đọc ngược lại 1-2 câu trước nó để tìm từ mà nó đang
thay thế.
Trích thông tin trong bài: “In the experimental group, the parents were given a two-hour training session
in which they were taught to ask open-ended questions...”
Tạm dịch: Ở nhóm thực nghiệm, các phụ huynh được cung cấp các buổi đào tạo dài hai tiếng, ở đó họ
được dạy để đặt các câu hỏi theo hướng gợi mở...” → “they” được dùng để nói đến các bậc phụ huynh.
Đáp án D là phù hợp.
Question 40. Đáp án C
Dịch đề bài: Trong các buổi đào tạo, phụ huynh nhóm thực nghiệm được dạy để _____________
A. sử dụng câu hỏi yes-no
B. nghiên cứu các thực nghiệm
C. đặt câu hỏi theo hướng gợi mở
D. đưa ra câu trả lời đúng
Câu hỏi lấy thông tin. Hướng dẫn làm bài có ở câu 38.
Trích thông tin trong bài: “In the experimental group, the parents were given a two-hour training session
in which they were taught to ask open-ended questions...”
Tạm dịch: Ở nhóm thực nghiệm, các phụ huynh được cung cấp các buổi đào tạo dài hai tiếng, ở đó họ
được dạy để đặt các câu hỏi theo hướng gợi mở...” → như vậy, các phụ huynh trong nhóm thực nghiệm
được dạy đặt câu hỏi theo hướng gợi mở.
Đáp án C là phù hợp.
Question 41. Đáp án D
Dịch đề bài: Khác biệt chính giữa nhóm kiểm soát và nhóm thực nghiệm trong nghiên cứu là gì?
A. Số lượng người tham gia
B. Những cuốn sách được đọc
C. Tuổi của những đứa trẻ
D. Những phiên đào tạo mà họ được nhận

Trang 18
Câu hỏi lấy thông tin. Hướng dẫn làm bài có ở câu 38.
Trích thông tin trong bài: “In the experimental group, the parents were given a two-hour training
session in which they were taught to ask open-ended questions...”
Tạm dịch: Ở nhóm thực nghiệm, các phụ huynh được cung cấp các buổi đào tạo dài hai tiếng, ở đó họ
được dạy để đặt các câu hỏi theo hướng gợi mở...” → như vậy, phụ huynh trong nhóm thực nghiệm được
cung cấp các buổi đào tạo.
Bài không nói gì đến phụ huynh trong nhóm kiểm soát. → sự khác biệt lớn nhất là phụ huynh của nhóm
thực nghiệm được đào tạo. Đáp án D là phù hợp
Question 42. Đáp án D
Dịch đề bài: Kết luận nào dưới đây có thể được rút ra từ bài đọc này?
A. Những đứa trẻ chủ động đọc luôn có hành vi già hơn sáu tháng so với những đứa trẻ không chủ động
đọc.
B. Những đứa trẻ đọc càng nhiều thì chúng càng trở nên thông minh.
C. Những đứa trẻ 2 hoặc 3 tuổi có thể được dạy để đọc một cách chủ động.
D. Kĩ năng ngôn ngữ của những đứa trẻ sẽ phát triển khi chúng bị yêu cầu phải trả lời lại một cách chủ
động.
Câu hỏi suy luận.
* Hướng dẫn làm bài:
Đối với loại câu hỏi suy luận, đọc kĩ 4 đáp án và tìm thông tin hỗ trợ trong bài. Chắc chắn chỉ có một đáp
án được thông tin trong bài hỗ trợ, các đáp án còn lại đều sai hoặc không có thông tin trong bài. Lưu ý
phải tìm thông tin dựa vào từ khóa, và không dùng kiến thức bản thân để trả lời loại câu hỏi này.
Trích thông tin trong bài: “If a parent encourages the child to actively respond to what the parent is
reading, the child's language skills increase”
Tạm dịch: Nếu một bậc phụ huynh khuyến khích con mình đáp lại một cách chủ động với những thứ mà
họ đọc cho chúng, kĩ năng ngôn ngữ của đứa bé sẽ tăng lên. → thông tin ủng hộ đáp án D.
Đáp án D là phù hợp.
Question 43. Đáp án C
Dịch đề bài: Bài đọc này chủ yếu giải thích điều gì?
A. Vì sao Hydro lại nhiều như vậy.
B. Các ngôi sao sinh ra năng lượng thế nào
C. Phần lớn lượng Heli trong vũ trụ được tạo ra thế nào.
D. Sự khác biệt giữa Heli và Hydro.
Câu hỏi ý chính của bài.
* Hướng dẫn làm bài: Đối với loại câu hỏi tìm ý chính/ tiêu đề của đoạn văn, việc cần làm là:
1/ Đọc 2-3 câu đầu tiên của bài vì những câu đó là câu thể hiện chủ đề của bài viết.
2/ Đọc lướt qua nội dung của bài để chắc chắn mình xác định ý chính đúng.

Trang 19
Ở bài này, câu đầu tiên, cũng là đoạn đầu tiên, giới thiệu tỉ lệ giữa Hydro và Heli trong vũ trụ. Các đoạn
sau đều dành để nói về nơi có thể tìm thấy Heli (đoạn 2), và Heli được tạo ra thế nào (đoạn 3 và 4). Có
thể thấy, phần lớn dung lượng của bài đọc nói về việc Heli trong vũ trụ được tạo ra thế nào.
Vậy nên, đáp án C là phù hợp.
Question 44. Đáp án C
Dịch đề bài: theo như bài này, Heli...
A. khó phát hiện
B. là nguyên tố cổ xưa nhất trong Vũ trụ
C. là nguyên tố dồi dào thứ hai trong Vũ trụ.
D. là nguyên tố phổ biến nhất ở các vật thể giống sao
Câu hỏi lấy thông tin.
* Hướng dẫn làm bài:
Đối với loại bài tìm thông tin, cần chọn ra một vài từ khóa ở đáp án cũng như ở câu hỏi rồi tìm những từ
khóa đó ở trên bài đọc. Khi đã tìm được đoạn chứa từ khóa trên bài đọc, cần đọc kĩ và chọn lọc những
thông tin nào ủng hộ đáp án. Sẽ chỉ có một đáp án được thông tin trong bài ủng hộ, và đó là đáp án đúng.
Trích thông tin trong bài: “.. .it is accurate to say that the universe is somewhat more than 25 percent
helium by weight and somewhat less than 75 percent hydrogen.”
Tạm dịch: ... chính xác có thể nói rằng vũ trụ này có trọng lượng hơn 25% một xíu là Heli và ít hơn 75%
một xíu là Hydro. Như vậy, Hydro chiếm khoảng 75% và là nhiều nhất, Heli chiếm khoảng 25% còn lại,
và nhiều thứ 2.
Vậy, đáp án C là phù hợp.
Question 45. Đáp án C
Dịch câu hỏi: Từ “constituents” gần nghĩa nhất với....
A. causes (n): nguyên nhân/ nguyên do
B. relatives (n): họ hàng
C. components (n): thành tố cấu tạo
D. target (n): mục tiêu
Câu hỏi từ vựng. “Constituents” (n): thành tố cấu thành. Chọn đáp án C - components: thành tố cấu thành
- là gần nghĩa nhất với từ mà bài cho.
Question 46. Đáp án D
Dịch câu hỏi: Tại sao tác giả lại nhắc đến “tia vũ trụ”?
A. Để giải thích cách vũ trụ khai sinh
B. Như một ví dụ về một câu hỏi thiên văn chưa được trả lời.
C. Để giải thích về sự dồi dào của Hydro trong Vũ trụ.
D. Như một phần trong danh sách những thứ có chứa Heli.
Câu hỏi lấy thông tin. Hướng dẫn làm bài có ở câu 44.

Trang 20
Trích thông tin bài: “Helium has been found in old stars, in relatively young ones, in interstellar gas, and
in the distant objects known as quasars. Helium nuclei have also been found to be constituents of cosmic
rays that fall on the earth...”
Tạm dịch: Heli được tìm thấy ở các ngôi sao già, ở những ngôi sao khá trẻ, ở khí liên sao, và trong cả
những vật thể xa xôi được biết đến với cái tên là vật thể giống sao. Hạt nhân Heli cũng được nhận thấy là
thành tố cấu thành của tia vụ trụ chạm tới trái đất...
Như vậy, tia vũ trụ được nhắc đến như một trong những nơi có thể tìm thấy Heli.
Vậy, đáp án D là phù hợp.
Question 47. Đáp án C
Dịch đề bài: Từ “they” nói đến...
A. phóng xạ
B. thành tố cấu thành
C. tia vũ trụ
D. các hạt
* Hướng dẫn làm bài:
Đối với loại câu hỏi này, đọc ngược lại 1-2 câu trước nó để tìm từ mà nó đang thay thế. Trích thông tin
trong bài: “Cosmic rays are not really a form of radiation; they consist of rapidly moving particles of
numerous different kinds.”
Tạm dịch: Các tia vũ trụ thực ra không phải một loại phóng xạ; chúng bao gồm các hạt vật chất thuộc
nhiều loại di chuyển nhanh chóng.
Như vậy, từ “they” nói đến các tia vũ trụ. Đáp án C là phù hợp.
Question 48. Đáp án D
Dịch câu hỏi: Từ “vary” gần nghĩa nhất với...
A. include (v): bao gồm
B. mean (v): nghĩa là
C. stretch (v): kéo căng ra
D. change (v): thay đổi
Câu hỏi từ vựng. “Vary” (v): thay đổi theo tình huống. Chọn đáp án D - change (v): thay đổi - là gần
nghĩa nhất với từ mà bài cho.
Question 49. Đáp án C
Dịch đề bài: Sự tạo thành Heli ở các ngôi sao____________
A. không thể đo lường được
B. sinh ra Hydro như một sản phẩm phụ
C. sinh ra năng lượng
D. khiến Heli trở nên dồi dào hơn nhiều ở các ngôi sao già so với các ngôi sao trẻ
Câu hỏi lấy thông tin. Hướng dẫn làm bài có ở câu 44.

Trang 21
Trích thông tin bài: “Helium is created in stars. In fact, nuclear reactions that convert hydrogen to helium
are responsible for most of the energy that stars produce.”
Tạm dịch: Heli được tạo ở các ngôi sao. Trong thực tế, các phản ứng hạt nhân chuyển Hydro thành Heli
chịu trách nhiệm sinh ra phần lớn năng lượng mà các ngôi sao sinh ra. → phản ứng chuyển Hydro thành
Heli sinh ra phần lớn năng lượng của các ngôi sao.
Vậy, đáp án C là phù hợp.
Question 50. Đáp án A
Dịch đề bài: Hầu hết Heli trong vũ trụ được tạo ra____________
A. trong một thời gian rất ngắn
B. trong khoảng không liên sao
C. trước khi phần lớn Hydro được sinh ra
D. trong phút tồn tại đầu tiên của Vũ trụ.
Câu hỏi lấy thông tin. Hướng dẫn làm bài có ở câu 44.
Trích thông tin bài: “But the nuclear reactions that led to the formations of helium went on for only
relatively short time. By the time the universe was a few minutes old, helium production had effectively
ceased.”
Tạm dịch: Nhưng phản ứng hạt nhân tạo ra Heli chỉ tiếp diễn trong một quãng thời gian khá ngắn. Cho
đến lúc Vũ trụ tồn tại được một vài phút, quá trình sản xuất Heli đã hoàn toàn dừng lại.
Như vậy, các phản ứng sinh ra Heli chỉ kéo dài vài phút, khá ngắn.
Vậy nên, đáp án A là phù hợp.

Trang 22
ĐỀ SỐ 13 ĐỀ THI THỬ TỐT NGHIỆP THPT
NĂM HỌC: 2020 – 2021
MÔN: TIẾNG ANH
Thời gian làm bài: 60 phút; không kể thời gian phát đề

Mark the letter A, B, C, or D on your answer sheet to indicate the word whose underlined part
differs from the other three in pronunciation in each of the following questions.
Question 1. A. stone B. zone C. phone D. none
Question 2. A. university B. unique C. unit D. undo
Mark the letter A, B, C or D on your answer sheet to indicate the word that differs from the rest in
the position of the main stress in each of the following questions.
Question 3. A. release B. offer C. amaze D. believe
Question 4. A. classify B. flexible C. sensitive D. tomato
Mark the letter A, B, C or D on your answer sheet to indicate the correct answer to each of the
following questions.
Question 5. The examination was not very difficult, but it was_____________ long.
A. so much B. too much C. very much D. much too
Question 6. Our industrial output_____________from $2 million in 2002 to $4 million this year.
A. rises B. has risen C. was rising D. rose
Question 7. We had a lot of fun_____________at the picnic.
A. to play B. playing C. played D. plays
Question 8. We_____________to take a taxi. Otherwise we’ll be late for the meeting.
A. would rather B. had better C. must have D. will have
Question 9. Some film stars_____________difficult to work with.
A. are said be B. are said to be C. say to be D. said to be
Question 10. the long jumper_____________the pole-vaulter win the prize.
A. Both-and B. Neither-nor C. Either-or D. Not only-but also
Question 11. _____________restaurant you pick is fine with me.
A. Whichever B. Whenever C. whoever D. wherever
Question 12. Now the manager is no longer as indifferent_____________criticism as he used to be.
A. against B. to C. towards D. with
Question 13. Does Mr. Mike bring his farm_____________to the local market every day?
A. productively B. productive C. product D. produce
Question 14. In population growth Latin America_____________first, Africa second, and Asia third.
A. places B. ranks C. numbers D. stands
Question 15. The referee_____________the coin to decide which team would kick the ball first.

Trang 1
A. caught B. threw C. cast D. tossed
Question 16. Sorry for being late. I was_____________in the traffic for more than an hour.
A. carried on B. held up C. put off D. taken after
Question 17. Even if you are rich, you should save some money for a_____________day.
A. windy B. rainy C. foggy D. snow
Question 18. “What I’ve got to say to you now is strictly_____________and most certainly not for
publication” said the government to the reporter.
A. off the record B. for the time being
C. by the way D. on record
Mark the letter A, B, C, or D on your answer sheet to indicate the underlined part that needs
correction in each of the following questions.
Question 19. The Canadian Shield is a huge, rocky region who curves around Hudson Bay like a giant
horseshoe.
A. a huge B. who C. around D. like
Question 20. Almost all of the people appeared on television wear makeup.
A. Almost B. of C. appeared D. wear
Question 21. The original World Cup trophy was given permanent to Brazil to honour that country’s
record third world cup title in Mexico in 1970.
A. The original World Cup trophy B. permanent
C. that country’s record D. in Mexico
Mark the letter A, B, C or D on your answer sheet to indicate the sentence that is closest in
meaning to each of the following questions.
Question 22. Studying all night is good for neither your grades nor your health.
A. Studying all night is good for your grades.
B. Studying all night does not help at all.
C. Studying all night is good for your health
D. Studying all night is helpful to you
Question 23. You should not keep bad company under any circumstances.
A. In no circumstances should you be friends with bad people.
B. Under no circumstances should you not keep bad company.
C. Under any circumstances shouldn’t you make friends with bad people.
D. In no circumstances should you keep your company because it is bad.
Question 24. Although he was disabled, he was quite confident the first time he practiced this sport.
A. Though a disabled boy, he was quite confident the first time he practiced this sport.
B. Though he was unable to walk, but the first time he practiced this sport, he was quite confident
C. In spite of being quite confident the first time he practiced this sport, he was disabled

Trang 2
D. He was quite confident when practicing this sport for the first time despite of a disabled boy.
Mark the letter A, B, C or Don your answer sheet to indicate the sentence that best combines each
pair of sentences in the following questions.
Question 25. My grandmother is very old. She can’t do the chores herself.
A. My grandmother is very old that she cant do the chores herself.
B. My grandmother is too old to do the chores herself.
C. My grandmother is such old that she cant do the chores herself.
D. My grandmother is too old to not do the chores herself.
Question 26. She turned the radio on at 7.30. She was still listening to it when her mother came
home at 9.30.
A. She had been listening to the radio at 7.30
B. She had been listening to the radio since 7.30
C. She had been listening to the radio after 7.30
D. She had been listening to the radio by 7.30
Mark the letter A, B, C or D on your answer sheet to indicate the most suitable response to
complete each of the following exchanges.
Question 27. Hana and Rina are talking about Lucy.
- Hanna: “Lucy has lost her purse somewhere this morning!”
- Rina: “_____________”
A. Tough luck B. Congratulations!
C. Never mind. D. I will take it for her
Question 28. Lucia wants to borrow his friends bike.
- Lucia: “Would you mind if I used your bike?”
- Friend: “_____________”
A. I don’t want to use your bike. B. Sure, go ahead.
C. Sorry, no, I wont do it. D. Please accept it with my best wishes.
Mark the letter A, B, c, or D on your answer sheet to indicate the word or phrase that is
CLOSEST in meaning to the underlined part in each of the following questions.
Question 29. We had never experienced such discourtesy towards the elderly as it occurred at this
center
A. politeness B. rudeness C. encouragement D. measurement
Question 30. Just by coincidence, I met my old classmate after 20 years.
A. sight B. surprise C. chance D. luck
Mark the letter A, B, C, or D on your answer sheet to indicate the word or phrase that is
OPPOSITE in meaning to the underlined part in each of the following questions.
Question 31. His thoughtless comments made her very angry.

Trang 3
A. kind B. honest C. thoughtful D. pleasant
Question 32. I could not see what she was doing. It was so dark there.
A. make out B. make up C. make for D. make from
Read the following passage and mark the letter A, B, C or D on your answer sheet to indicate the
correct word for each of the blanks from 33 to 37.
The story of gold is an adventure involving kings, queens, pirates, explorers, conquerors, and
the native people they conquered. Throughout history, gold has (33) _____________a magic spell over
those it touched. Gold is beautiful and rare; a soft shiny metal that can be moulded into many shapes. It
has been used for money, jewelry, and to decorate special buildings such as palaces and places of
worship. (34) _____________the precious metal was discovered, prospectors rushed to mine it, starting
new cities and countries as they went. Gold and the people who love it have helped shape the world we
live in today. Gold is one of many elements, or substances that cannot be changed by normal chemical
means, that are found in the Earth's crust. Gold has a warm, sunny colour and (35) _____________it
does not react with air, water, and most chemicals, its shine never fades. In its natural (36)
_____________, gold is soft and easily shaped, when heated to 1,062 Celsius it melts and can be
poured (37) _____________moulds to form coins, gold bars, and other objects. Stories have been told,
movies made and legends born about the discovery of the world's great gold deposits. It is a saga of
dreams, greed, ambition and exploration.
Question 33. A. knitted B.folded C. woven D. sewn
Question 34. A. Whoever B. However C. Wherever D. Whatever
Question 35. A. despite B. because C. yet D. so
Question 36. A. position B. size C. condition D.shape
Question 37. A. into B. forward C. with D. at
Read the following passage and mark the letter A, B, C, or D on your answer sheet to indicate the
correct answer to each of the questions from 38 to 42.
Tsunami is a Japanese word which means harbour wave and is used as the scientific term for seismic
sea wave generated by an undersea earthquake or possibly an undersea landside or volcanic eruption,
when the ocean floor is tilted or offset during an earthquake, a set of waves is created similar to the
concentric waves generated by an object dropped into the water. Most tsunamis originate along the
Ring of Fire, a zone of volcanoes and seismic activity, 32,500 km long that encircles the Pacific Ocean.
Since 1819, about 40 tsunamis have struck the Hawaiian Islands.
A tsunami can have wavelengths, or widths, of 100 to 200 km, and may travel hundreds of kilometers
across the deep ocean, reaching speeds of about 725 to 800 kilometres an hour. Upon entering shallow
coastal waters, the wave, which may have been only about half a metre high out at sea, suddenly grows
rapidly. When the wave reaches the shore, it maybe 15 m high or more. Tsunamis have tremendous

Trang 4
energy because of the great volume of water affected. They are capable of obliterating coastal
settlements.
Tsunamis should nót be confused with storm surges, which are domes of water that rise underneath
hurricanes or cyclones and cause extensive coastal flooding when the storms reach land. Storm surges
are particularly devastating if they occur at high tide. A cyclone and accompanying storm surge skilled
an estimated 500,000 people in Bangladesh in 1970. The tsunami which struck south and southeast Asia
in late 2004 killed over 200 thousand people.
Question 38. What does the word “concentric” in paragraph 1 mean?
A. wavy B. having many centres
C. having a common centre D. a ring
Question 39. what is the greatest speed of tsunami travelling across the deep ocean?
A. 200 kilometres an hour B. 700 kilometres an hour
C. 800 kilometres an hour D. 150,000 kilometres an hour
Question 40. How are tsunami capable of obliterating coastal settlements?
A. They have tremendous energy due to the great volume of water affected.
B. They are a metre high or more.
C. They travel hundreds of kilometres
D. They can strike the shore fifteen metres high
Question 41. Which of the following is NOT true?
A. Tsunami only occurs in Asia
B. A cyclone along with storm surges happened in Asia in 1970.
C. Storm surges are domes of water rising underneath hurricanes or cyclones.
D. Storm surges cause extensive coastal flooding.
Question 42. what is the passage mainly about?
A. Where tsunamis originate.
B. Damage caused by tsunamis.
C. Facts about tsunamis.
D. How tremendous the energy of a tsunami is.
Read the following passage and mark the letter A, B, C, or D on your answer sheet to indicate the
correct answer to each of the questions from 43 to 50.
There are many theories of aging, but virtually all fall into the category of being hypotheses with a
minimum of supporting evidence. One viewpoint is that aging occurs as the body's organ systems
become less efficient. Thus failures in the immune system, hormonal system, and nervous system could
all produce characteristics that we associate with aging. Following a different vein, many current
researchers are looking for evidence at the cellular and sub cellular level.

Trang 5
It has been shown that cells such as human fibroblasts (generalized tissue cells) grown in culture divide
only a limited number of times and then die. (Only cancer cells seem immortal in this respect).
Fibroblast cells from an embryo divide more times than those taken from an adult. Thus some
researchers believe that aging occurs at the cellular level and is part of the cell's genetic makeup. Any
event that disturbs the cell's genetic machinery such as mutation, damaging chemicals in the cell's
environment, or loss of genetic material, could cause cells to lose their ability to divide and thus bring
on aging. Other theories of aging look at different processes. Chronological aging refers to the passage
of time since birth and is usually measured in years. While chronological age can be useful in
estimating the average status of a large group of people, it is a poor indicator of an individual person's
status because there is a tremendous amount of variation from one individual to the next in regard to the
rate at which biological age changes occur. For example, on the average, aging results in people losing
much of their ability to perform strenuous activities, yet some elderly individuals are excellent
marathon runners.
Another type of aging is cosmetic aging, which consists of changes in outward appearance with
advancing age. This includes changes in the body and changes in other aspects of a person's
appearance, such as the style of hair and clothing, the type of eyeglasses, and the use of a hearing aid.
Like chronological aging, it is frequently used to estimate the degree to which other types of aging have
occurred. However, it is an inaccurate indicator for either purpose because of variation among
individuals and because a person's appearance is affected by many factors that are not part of aging,
including illness, poor nutrition, and exposure to sunlight
Question 43. What is the best title for this passage?
A. Different Processes of Aging
B. Outstanding Characteristics Associated with Aging
C. Theories of Aging: Well Proven Hypotheses
D. Theories of Aging: Poorly Supported Hypotheses
Question 44. The author points out that cancer cells_____________
A. seem to live forever
B. divide and then die
C . divide more in embryos than in adults
D. lose their ability to divide
Question 45. The phrase “an embryo” in paragraph 2 is closest in meaning to_____________.
A. a descendant B. an infant C. a parent D. an internal organ
Question 46. The word “strenuous” in paragraph 3 is closest in meaning to_____________.
A. basic B. troublesome C. mental D. intense
Question 47. As explained in this passage, the theory of aging which examines the cellular level
would NOT assign which of the following as a cause of aging?

Trang 6
A. Mutation B. Deterioration into the body’s immune system
C. Loss of genetic material D. Chemical damage from the environment
Question 48. According to the passage, chronological aging is not a good indicator of an
individual’s status regarding aging because_____________ .
A. elderly people are often athletic
B. there is individual variation in the rate of biological aging
C. strenuous activities are not good measures of age
D. it is difficult to get accurate records of birth dates
Question 49. In the last paragraph, the word “this” refers to_____________
A. cosmetic aging B. type of aging C. outward appearance D. advancing age
Question 50. The author implies all of the following about cosmetic aging EXCEPT_____________
A. It does not occur at the same rate for all people
B. It is a poor indicator of chronological age
C. Illness, poor nutrition, and exposure to sunlight cause aging to occur
D. It is described by changes in outward appearance

Trang 7
Đáp án
1-D 2-D 3-B 4-D 5-D 6-B 7-B 8-D 9-B 10-A
11-A 12-B 13-D 14-B 15-D 16-B 17-B 18-A 19-B 20-C
21-B 22-B 23-A 24-A 25-B 26-B 27-A 28-B 29-B 30-C
31-C 32-A 33-C 34-C 35-B 36-C 37-A 38-C 39-C 40-A
41-A 42-C 43-D 44-A 45-B 46-D 47-B 48-B 49-A 50-C

LỜI GIẢI CHI TIẾT


Question 1: Đáp án D
Đáp án D đúng vì phần gạch chân của đáp án D đọc là âm /ʌ/ còn phần gạch chân của các đáp án khác
được đọc là âm / əʊ /
A. stone /stəʊn/ (n): hòn đá
B. zone /zəʊn/ (n): vùng, khu vực
C. phone /fəʊn/ (n): điện thoại/; (v): gọi điện
D. none /nʌn/ (pronoun): không một cái gì
Question 2. Đáp án D
Đáp án D đúng vì phần gạch chân của đáp án D đọc là âm / ʌ / còn phần gạch chân của các đáp án còn lại
được đọc là âm /ju:/
A. university /ˌjuːnɪˈvɜːsəti/ (n): đại học
B. unique /juˈniːk/ (adj): lạ, độc nhất
C. unit /ˈjuːnɪt/ (n): đơn vị, bài (học)
D. undo /ʌnˈduː/ (v): hủy, tháo, cởi
Question 3. Đáp án B
Đáp án B đúng vì đáp án B trọng âm rơi vào âm tiết thứ nhất còn 3 đáp án còn lại trọng âm rơi vào âm tiết
thứ hai.
A. release /rɪˈliːs/ (v): thả, phóng thích
B. offer /ˈɒfə(r)/ (v): đề nghị
C. amaze /əˈmeɪz/ (v): làm kinh ngạc
D. believe /bɪˈliːv/ (v): tin tưởng
Question 4. Đáp án D
Đáp án D đúng vì đáp án D trọng âm rơi vào âm tiết thứ hai còn ba đáp án còn lại trọng âm rơi vào âm
tiết thứ nhất.
A. classify /ˈklæsɪfaɪ/ (v): phân loại
B. flexible /ˈfleksəbl/ (adj): linh hoạt
C. sensitive /ˈsensətɪv/ (adj): nhạy cảm
D. tomato /təˈmɑːtəʊ/ (n): quả cà chua

Trang 8
❖ For review
Các từ có 3 âm tiết thì trọng âm của từ thường rơi vào âm tiết thứ 3 từ dưới lên nếu từ có tận cùng bằng
hậu tố -fy, -ible, -tive, -ate
Các từ có 3 âm tiết thì trọng âm của tù thường rơi vào âm tiết thứ hai từ dưới lên nếu từ có tận cùng bằng
hậu tố -o/ow.
Question 5. Đáp án B
Ta cần trạng từ chỉ lượng để bổ sung ý nghĩa cho tính từ “long”. Ta có phương án A và C bổ trợ cho động
từ, phương án B bổ trợ cho danh từ
Chỉ có phương án D là bổ trợ cho tính từ
Tạm dịch. Bài thi không quá khó nhưng quá dài.
❖ For review
Much được dùng như trạng từ bổ sung ý nghĩa cho động từ, tính từ. Có thể đứng trước các từ như more,
too, đứng sau các từ như so, very, too.
Ví dụ. We love children so/ very much. Chúng tôi rất yêu trẻ con
The film is (much) too long - Bộ phim này dài quá.
Question 6. Đáp án B
Trong câu trên ta thấy có từ chỉ thời gian “this year” và 2002, có nghĩa hành động xảy ra từ quá khứ và
kéo dài cho đến hiện tại → sử dụng thời hiện tại hoàn thành. Trong 4 phương án chỉ có phương án B là
phù hợp.
Tạm dịch. Sản lượng công nghiệp của chúng ta đã tăng từ 2 triệu đô la Mỹ vào năm 2002 lên 4 triệu đô
vào năm nay.
Question 7. Đáp án B
Ta có cấu trúc: (have) a lot of fun + V-ing: vui thích khi làm việc gì đó. Như vậy ta chọn được đáp án B.
Tạm dịch. Chúng tôi đã chơi rất vui trong chuyến dã ngoại.
Question 8. Đáp án D
Ta thấy sau vị trí cần điền là “to take” → loại A và B vì would rather và had better không dùng với to V.
Đáp án C không đúng vì must không đi cùng với have to → đáp án đúng là D
Tạm dịch. Có lẽ chúng ta phải bắt taxi. Nếu không chúng ta sẽ bị muộn họp.
❖ For review
+ Had better là động từ khuyết thiếu dùng để đưa ra lời khuyên, kiến nghị cho một tình huống cụ thể ở
hiện tại hoặc tương lai
Cấu trúc. S+ had better + (not) V (bare infinitive). Ai đó nên/ không nên làm gì
+ Would rather là cấu trúc sử dụng trong câu giả định để diễn đạt điều mong muốn thực hiện được trong
một tình huống cụ thể
Cấu trúc.
- Mong muốn ở hiện tại/ tương lai.
Trang 9
S + would rather + (not) V (bare infinitive) + (than V-bare infinitive)
S1 + would rather + S2 + Ved/ didn’t V
- Mong muốn ở quá khứ.
S+ would rather+ (not) have P2
S1+ would rather+ S2+ had(not) P2
Question 9. Đáp án B
Dễ dàng nhận thấy câu trên thuộc trường hợp bị động đặc biệt của các động từ tường thuật/ nêu ý kiến →
ta chọn đáp án B do ba đáp án còn lại sử dụng sai cấu trúc.
❖ For review
Đối với cấu trúc bị động của các động từ tường thuật nêu ý kiến thì ta có 2 cách để biến đổi cấu trúc bị
động
Câu chủ động.
S1+ V1 (tường thuật, ý kiến) that S2+ V2
Câu bị động.
Cách 1: It + passive V1 that S2+ V2
Cách 2: S2 + passive V1 (chia theo V1) + to- infinitive: nếu hành động ở mệnh đề that xảy ra đồng thời
hoặc sau hành động ở mệnh đề chính.
S2 + passive V1 (chia theo V1 + to be V-ing: nếu hành động ở mệnh đề that diễn tả sự tiếp diễn (hiện tại
tiếp diễn/ quá khứ tiếp diễn/ ...) S2 + passive V1 (chia theo V1) + to have Vpp: nếu hành động ở mệnh đề
that xảy ra trước hành động trong mệnh đề chính.
Question 10. Đáp án A
Trong câu trên ta thấy động từ “win”- vị ngữ của câu đang chia số nhiều → cần chủ ngữ số nhiều. Trong 4
phương án trên chỉ có phương án A. Both-and cần động từ số nhiều. Các phương án còn lại, động từ sẽ
chia theo chủ ngữ gần nó nhất (chủ ngữ thứ 2)- trong trường hợp này là the pole-vaulter (n) - vận động
viên nhảy sào
Tạm dịch. Cả vận động viên nhảy xa và vận động viên nhảy sào đều có giải.
Question 11. Đáp án A
Trong câu trên ta thấy có động từ to be “is” làm vị ngữ trong câu → Toàn bộ phần phía trước từ vị trí chỗ
trống cần điền từ đến “you pick” sẽ đóng vai trò như chủ ngữ trong cầu. Ta nhận ra trong “chủ ngữ” này
lại chứa đựng một cụm chủ-vị khác chính là “you pick”. Như vậy có nghĩa đoạn từ vị trí cần điền từ đến
“you pick” là mệnh đề danh từ. Bên cạnh đó ta thấy sau chỗ trống cần điền từ là danh từ “restaurant”.
Trong 4 phương án chỉ có phương án A có thể đứng trước một danh từ, 3 phương án còn lại không đi
cùng danh từ.
Whichever + noun: bất kì cái nào
Tạm dịch. Bất kì nhà hàng nào bạn chọn đều hợp với tôi.
Question 12. Đáp án B

Trang 10
Trong câu trên chúng ta thấy có cụm từ “indifferent to st” có nghĩa lãnh đạm, thờ ơ với điều gì. Như vậy
ta chọn được đáp án B.
Tạm dịch. Bây giờ, người quản lí không còn thờ ơ với những lời chỉ trích như trước đây.
Question 13.
Ta có cấu trúc “bring st to st” - cung cấp/ đem cái gì đi đâu → chỗ trống cần điền sẽ là một danh từ để tạo
ra cụm danh từ ghép với “farm”. Ta có farm produce (n): nông sản
A. productively (adv): sản xuất nhiều, có năng suất cao
B. productive (adj): có năng suất
C. product (n): sản phẩm
D. produce (n): nông sản/ (v): sản xuất
Tạm dịch. Có phải hàng ngày ông Mike đem nông sản ra chợ bán không?
❖ Note:
produce (n) và produce (v) khác nhau về cách phát âm và trọng âm.
- produce /prəˈdjuːs/ (v): sản xuất
- produce /prəˈdjuːs/ (n): nông sản
Question 14. Đáp án B
Trong câu trên ta thấy có sự xuất hiện của các từ chỉ thứ tự như “first, second, third” → chỉ có đáp án B là
phù hợp.
A. places (v): đặt, để
B. ranks (v): xếp thứ
C. numbers (v): đánh số
D. stands (v): đứng
Tạm dịch, vể tăng dân số, châu Mỹ La tinh đứng thứ nhất, châu Phi thứ hai và châu Á thứ ba.
Question 15. Đáp án D
Câu trên ta thấy xuất hiện từ khóa referee (n)
- trọng tài và coin (n) - đồng xu vì vậy ta chọn đáp án D
A. caught (v- past simple form): bắt được
B. threw (v- past simple form): ném
C. cast (v- past simple form): quăng, ném, liệng
D. tossed (v- past simple form): tung lên
Tạm dịch. Trọng tài tung đồng xu để quyết định xem đội nào giao bóng trước.
Question 16. Đáp án B
Đáp án B vì cụm động từ (phrasal verb) hold up: cản trở, trì trệ, kẹt. Ở đây có nghĩa là “bị kẹt xe”
A. carried on (phrasal verb): tiếp tục
C. put off (phrasal verb): hoãn lại
D. taken after (phrasal verb): giống

Trang 11
Tạm dịch. Xin lỗi vì đến muộn. Tôi bị kẹt xe hơn một tiếng đồng hồ.
Question 17. Đáp án B
Đáp án B. Ta có thành ngữ “save st for a rainy day” - dành dụm/ phòng khi đau ốm
A. windy (adj): có gió
C. foggy (adj): sương mù
D. snowy (adj): có tuyết
Tạm dịch. Cho dù bạn có giàu có đi chăng nữa, bạn cũng nên dành dụm tiền bạc phòng khi đau ốm.
Question 18. Đáp án A
Đáp án A. Ta có thành ngữ “off the record” - không chính thức
B. for the time being: trong thời gian này, tạm thời
C. by the way: nhân tiện, tiện thể
D. on record: chính thức, được ghi danh
Tạm dịch. Một nhân viên chính phủ nói với phóng viên. “Những gì tôi nói với cậu bây giờ hoàn toàn chưa
chính thức và không được công bố rộng rãi”
Question 19.
Câu này đã sử dụng sai từ đại từ quan hệ. Đại từ quan hệ “who” không dùng để thay thế cho danh từ chỉ
vật “region”. Cần sửa thành “which” hoặc “that”.
Tạm dịch. Canada Shield là một khu vực đá rộng lớn, uốn lượn quanh Vịnh Hudson giống như một chiếc
móng ngựa khổng lồ.
Question 20.
Động từ “appear” là nội động từ, không chia ở dạng bị động. Trong câu trên, “appear” là dạng thức rút
gọn của mệnh đề quan hệ, chia ở dạng chủ động phải là “appearing”, tương đương với mệnh đề đầy đủ là
“who/that appear”.
Tạm dịch. Hầu hết mọi người xuất hiện trên truyền hình đều trang điểm.
Question 21.
Câu trên là đang sử dụng sai loại từ. Chúng ta thấy để bổ sung nghĩa cho động từ, ta không thể dùng tính
từ mà phải dùng trạng từ. Trong trường hợp này permanent được sử dụng không chính xác do bổ sung ý
nghĩa cho động từ give → sửa thành permanently
Tạm dịch. Chiếc cúp World Cup đầu tiên đã được trao vĩnh viễn cho Brazil để vinh danh kỉ lục 3 lần vô
địch của nước này ở Mexico năm 1970.
Question 22. Đáp án B
Dịch câu đề. Việc học cả đêm không tốt cho cả điểm số và sức khỏe của bạn
Dịch đáp án.
A. Việc học cả đêm tốt cho điểm số cửa bạn
B. Việc học cả đêm chẳng có ích lợi tẹo nào cả
C. Việc học cả đêm tốt cho sức khỏe của bạn

Trang 12
D. Việc học cả đêm rất có ích cho bạn
Đáp án đúng của câu hỏi này là đáp án B. Chúng ta có cấu trúc neither...nor.... cả 2 cũng không
Đáp án A, C, D sai vì truyền đạt sai ý của câu gốc
Question 23. Đáp án A
Dịch câu đề. Anh không nên giao du với những kẻ xấu trong bất cứ hoàn cảnh nào.
Dịch đáp án.
A. Không bao giờ nên kết bạn với kẻ xấu.
B. Trong bất cứ hoàn cảnh nào bạn nên giao du với kẻ xấu
C. Trong bất cứ hoàn cảnh nào bạn không nên giáo du với kẻ xấu
D. Trong bất cứ hoàn cảnh nào thì anh không nên giữ công ty lại vì nó tệ
Đáp án đúng của câu hỏi này là A. Chúng ta có cấu trúc đảo ngữ.
In/ Under no circumstances + aux (trợ động từ) + S+ V: Không bao giờ...
Đáp án B sai vì diễn đạt ý nghĩa trái ngược với câu gốc
Đáp án C sai vì ta không dùng cấu trúc đảo ngữ với under any circumstances
Đáp án D sai vì sai nghĩa hoàn toàn so với câu gốc.
Question 24. Đáp án A
Dịch câu đề. Mặc dù anh ấy bị khuyết tật nhưng lần đầu tập môn thể thao này anh ấy khá tự tin
Dịch đáp án.
A. Dù khuyết tật nhưng anh ấy khá tự tin khi lần đầu tập môn thể thao này.
B. Dù anh ấy không thể đi được nhưng lần đầu tiên tập môn thể thao này anh ấy khá tự tin
C. Dù khá tự tin khi lần đầu tiên tập môn thể thao này nhưng anh ấy bị khuyết tật
D. Anh ấy khá tự tin khi tập môn thể thao này lần đầu tiên dù bị khuyết tật
Đáp án của câu hỏi này là đáp án A. Đây là câu hỏi liên quan đến mệnh đề trạng ngữ chỉ sự nhượng bộ.
Với mệnh đề trạng ngữ chỉ sự nhượng bộ bắt đầu bằng “Although S1+V1, S2+V2” ta có thể viết lại bằng.
Despite N1, S2+ V2 hoặc In spite of N1, S2+ V2
Bên cạnh đó đối với các trường hợp chung chủ ngữ (S1 và S2 là một người) thì ta có thể rút gọn mệnh đề
trạng ngữ thành Although/ Though + adj/ N, S+V
Question 25. Đáp án B
Dịch câu đề. Bà tôi rất già. Bà không thể tự làm việc nhà được
Dịch đáp án.
A. Không dịch vì sai cấu trúc: không có cấu trúc “very adj that...”
B. Bà tôi quá già nên không thể tự làm việc nhà
C. Không dịch vì sai cấu trúc: such thì đằng sau phải là một danh từ, nhưng trong câu lại là tính từ “old”
D. Không dịch vì sai cấu trúc chỗ “to not do”
Đáp án của câu hỏi này là đáp án B. Chúng ta có cấu trúc.
S+V+ too+ adj + toV: quá.. .nên không thể...

Trang 13
Các phương án còn lại sử dụng sai cấu trúc
Question 26. Đáp án B
Dịch câu đề. Cô ấy mở đài lúc 7h30. Khi mẹ về nhà lúc 9h30 thì cô ấy vẫn còn nghe đài
Dịch đáp án.
A. Cô ấy đã nghe đài lúc 71130
B. Cô ấy đã nghe đài từ 7h30
C. Cô ấy đã nghe đài sau 7h30
D. Cô ấy đã nghe đài cho đến 7h30
Đáp án đúng của câu hỏi này là đáp án B. Chúng ta thấy trong câu gốc có cụm was still listening. Khi
muốn diễn tả một hành động xảy ra liên tục trong quá khứ và muốn nhấn mạnh tính liên tục của hành
động đó thì ta sẽ dùng quá khứ hoàn thành tiếp diễn với cấu trúc S+ had been + V-ing
Các phương án còn lại chưa thể hiện được tính liên tục của hành động
Question 27. Đáp án A
Hana và Rina đang nói chuyện về Lucy.
Hana: Lucy vừa mới để quên ví của cô ấy ở đâu đó sáng nay
Rina: ___________
A. Thật không may
B. Chúc mừng nhé
C. Đừng bận tâm
D. Để tôi đem nó về cho cô ấy
Chúng ta thấy trong tình huống trên, Hana đang nói về một sự việc không tốt lắm, ta dựa vào cụm từ “has
lost”-bị mất đồ. Như vậy người nghe khi nghe về một sự việc không tốt thường sẽ bày tỏ sự thông cảm và
chia sẻ. Trong trường hợp này ta sẽ chọn phương án A. Phương án B thường dùng khi ta muốn chúc
mừng ai đó, phương án C được sử dụng khi đáp lại lời cảm ơn còn phương án D thường dùng để thể hiện
lời hứa.
Question 28. Đáp án B
Lucia muốn mượn xe của một người bạn và cô ấy đã sử dụng cấu trúc xin phép bắt đầu bằng “Would you
mind if I used your bike?”- Bạn có phiền không nếu tôi dùng chiếc xe của bạn? Để đáp lại lời xin phép, ta
thường sử dụng cấu trúc go ahead- cứ tự nhiên. Vì vậy đáp án của câu hỏi này là đáp án B
A. Tôi không muốn dùng xe đạp của bạn- câu này chúng ta thường sử dụng khi muốn hỏi ý kiến của ai đó
C. Xin lỗi, không, tôi sẽ không làm thế- với cấu trúc sử dụng will/ wont thường thể hiện lời hứa
D. Làm ơn hãy nhận nó đi với tất cả tấm lòng của tôi- chúng ta có thể sử dụng cấu trúc này khi muốn ai
đó chấp nhận điều gì
Question 29. Đáp án B
Đáp án B đúng vì ta có discourtesy (n) có nghĩa là không lịch sự = rudeness (n). Đây là câu hỏi tìm từ
đồng nghĩa nên chọn đáp án B

Trang 14
A. politeness (n): phép lịch sự
B. rudeness (n): sự thô lỗ
C. encouragement (n): sự khuyến khích
D. measurement (n): số đo, kích thước
Tạm dịch. Chúng tôi chưa bao giờ thấy hành động bất kính với người lớn tuổi nào như sự việc đã xảy ra ở
trung tâm này
Question 30. Đáp án C
Đáp án C đúng vì ta có by coincidence = by chance - tình cờ. Đây là câu hỏi tìm từ đồng nghĩa nên ta
chọn phương án C
A. sight (n): tầm nhìn, việc nhìn thấy, trông thấy
B. surprise (n): sự ngạc nhiên, bất ngờ
C. chance (n): sự tình cờ, sự may rủi
D. luck (n): sự may mắn
Tạm dịch. Chỉ tình cờ mà thôi, tôi đã gặp lại một người bạn cũ sau 20 năm.
Question 31. Đáp án C
Đáp án C đúng vì ta có thoughtless (adj) - khinh xuất, thiếu suy nghĩ >< thoughtful (adj) - chín chắn, thận
trọng. Đây là câu hỏi tìm từ trái nghĩa nên ta chọn phương án C
A. kind (adj): tử tế, tốt bụng
B. honest (adj): trung thực, chân thật
C. thoughtful (adj): chín chắn, thận trọng
D. pleasant (adj): dễ chịu, vừa ý, vui vẻ
Tạm dịch. Lời bình luận vô tư của anh ấy làm cho cô ấy tức giận
Tips
Các tính từ kết thúc bằng hậu tố -ful thường có tính từ trái nghĩa kết thúc bằng hậu tố -less
Ví dụ. careful (adj): cẩn thận >< careless (adj): bất cẩn
Meaningful (adj): có ý nghĩa >< meaningless (adj): vô nghĩa
Question 32. Đáp án A
Đáp án A đúng vì ta có make out (phrasal verb): nhìn thấy, nhận ra >< not see: không nhìn thấy. Đây là
câu hỏi tìm từ trái nghĩa nên ta chọn phương án A
A. make out (phrasal verb): nhìn thấy, nhận ra
B. make up (phrasal verb): trang điểm
C. make for (phrasal verb): tiến lại gần...
D. make from (phrasal verb): làm từ...
Tạm dịch. Tôi không thể nhìn thấy những gì cô ấy đang làm. Trời quá tối.
❖ Note
Sự khác nhau giữa (be) made from và (be) made of

Trang 15
- (be) made from st: được sử dụng khi chất liệu bị thay đổi trong quá trình tạo ra một vật nào đó
Ví dụ. Paper is made from wood. Giấy được làm từ gỗ (nhưng khi thành sản phẩm là giấy thì chất liệu gỗ
đã bị thay đổi, không còn là gỗ ban đầu nữa)
- (be) made of st: được sử dụng khi chất liệu không bị thay đổi trong quá trình tạo ra một vật nào đó
Ví dụ. This table is made of wood. Chiếc bàn này được làm bằng gỗ (Khi thành sản phẩm là cái bàn thì
chất liệu ban đầu là gỗ vẫn giữ nguyên, không bị thay đổi)
Question 33.
Ta có cụm từ weave a spell (over sb): lôi cuốn, hấp dẫn
Throughout history, gold has woven a magic spell over those it touched.
Tạm dịch. Trong suốt tiến trình lịch sử, vàng có một sức hút thần kì với những ai chạm vào nó
Question 34. Đáp án C
Đoạn văn đang bàn đến sức hấp dẫn của vàng nên căn cứ vào nghĩa của đáp án ta thấy phương án C phù
hợp nhất
A. Whoever: bất cứ ai
B. However: tuy nhiên
C. Wherever: bất cứ đâu
D. Whatever: bất cứ điều gì
Wherever the precious metal was discovered, prospectors rushed to mine it, starting new cities and
countries as they went.
Tạm dịch. Bất cứ nơi nào kim loại quý này được phát hiện, những người đi tìm vàng đều đổ xô đi khai
thác, đi đến đâu họ hình thành nên các thành phố và làng mạc đến đó.
Question 35. Đáp án B
Ta thấy mối quan hệ giữa hai vế câu là quan hệ nguyên nhân kết quả. Để thể hiện điểu này thì chỉ có
phương án B phù hợp do chỗ trống cần điền từ đứng trước mệnh đề chỉ nguyên nhân.
A. Despite: mặc dù- dùng cho mệnh đề chỉ sự nhượng bộ
C. Yet: cho dù vậy- liên từ thường dùng để nhấn mạnh sự tương phản mạnh mẽ, có thể gây bất ngờ ngạc
nhiên cho mọi người
D. So: nên- liên từ đứng trước mệnh đề chỉ kết quả
Gold has a warm, sunny colour and because it does not react with air, water, and most chemicals, its
shine never fades.
Tạm dịch. Vàng có màu ấm, ánh vàng như màu nắng và bởi vì nó không phản ứng với không khí, nước và
hầu hết các chất hóa học nên độ sáng của nó không bao giờ giảm đi.
Question 36.
Ta có cụm từ natural condition (n): điều kiện tự nhiên
In its natural condition, gold is soft and easily shaped.
Tạm dịch. Trong điều kiện tự nhiên, vàng mềm và dễ tạo hình.

Trang 16
Question 37.
Ở đây ta có cụm động từ “pour into”: đổ vào
When heated to 1,062 Celsius it melts and can be poured into moulds to form coins, gold bars, and other
objects.
Tạm dịch. Khi nung nóng lên đến 1062 độ C nó tan chảy và có thể đổ vào khuôn để đúc thành tiền đồng,
vàng thỏi và các đồ vật khác
Question 38. Đáp án C
Câu hỏi. Từ “concentric” ở đoạn 1 có nghĩa là gì?
A. Gợn sóng
B. Có nhiều trung tâm
C. Có 1 tâm chung
D. Cái vòng, nhẫn, đai, vành đai
Đây là một câu hỏi liên quan đến từ vựng, với câu hỏi này chúng ta chỉ cần đọc thông tin xung quanh từ
vựng đã cho là có thể tìm ra câu trả lời.
Và chúng ta có thể tìm thấy thông tin trong đoạn 1 dòng 3-4.
When the ocean floor is tilted or offset during an earthquake, a set of waves is created similar to the
concentric waves generated by an object dropped into the water.
Tạm dịch. Khi bề mặt đáy đại dương bị nghiêng hoặc nứt ra trong suốt quá trình xảy ra động đất, một tập
hợp các cơn sóng giống như các vòng tròn đồng tâm sẽ hình thành nếu có một vật thể rơi xuống nước.
Trong câu văn này, chúng ta cũng có thể dựa vào thông tin “waves generated by an object dropped into
the water”- sóng tạo ra khi một vật thể rơi xuống nước để đoán nghĩa của từ.
Từ concentric (adj): đồng tâm, có nghĩa giống với phương án C “having a common centre”
Question 39. Đáp án C
Câu hỏi. Tốc độ lớn nhất của sóng thần trong lòng đại dương là bao nhiêu
A. 200 km/ giờ B. 700 km/ giờ
C. 800 km/ giờ D. 150, 000 km/ giờ
Đối với các câu hỏi hỏi về thông tin cố định (là các thông tin không thay đổi trong văn bản ví dụ số liệu,
tên riêng của người, địa danh, số năm..), thì cách làm rất đơn giản đó là đọc lướt thật nhanh toàn bộ văn
bản, xác định các thông tin cố định trong văn bản, có thể sử dụng các kí hiệu khác nhau để phân biệt các
loại thông tin khác nhau, ví dụ gạch chân các thông tin về năm, khoanh tròn các tên riêng chỉ người...
Khi trả lời ta sẽ dễ dàng nhìn vào hệ thống kí hiệu đó và có thể nhanh chóng tìm ra đáp án. Cùng với các
câu hỏi liên quan đến từ vựng thì các câu hỏi về thông tin cố định cũng là các câu hỏi chúng ta có thể dễ
dàng ghi điểm và ưu tiên trả lời trước.
Chúng ta tìm thấy đáp án ở đoạn 2, dòng 1-2. A tsunami can have wavelengths, or widths, of 100 to 200
km, and may travel hundreds of kilometers across the deep ocean, reaching speeds of about 725 to 800
kilometres an hour.

Trang 17
Tạm dịch. Sóng thần có thể có độ dài hoặc chiều rộng từ 100-200 km và có thể di chuyển hàng trăm cây
số dưới lòng sâu đại dương, đạt tốc độ khoảng 725 đến 800km/h. Như vậy đáp án C là đáp án đúng
Question 40. Đáp án A
Câu hỏi. Sóng thần có khả năng hủy diệt các khu dân cư ven bờ như thế nào?
A. Chúng có năng lượng khủng khiếp do khối lượng nước khổng lồ bị tác động mạnh
B. Chúng cao một mét hoặc hơn
C. Chúng di chuyển hàng trăm cây số
D. Chúng có thể đánh mạnh vào bờ với độ cao 15m
Chúng ta tìm thấy thông tin trong đoạn 2 dòng 4-5.
Tsunamis have tremendous energy because of the great volume of water affected. They are capable of
obliterating coastal settlements.
Tạm dịch. Sóng thần có năng lượng khủng khiếp bởi một khối lượng nước khổng lồ bị tác động mạnh.
Chúng có khả năng xóa sổ hoàn toàn các khu dân cư ven biển.
Như vậy ta chọn được đáp án A.
Question 41. Đáp án A
Câu hỏi. Điều nào sau đây không đúng
A. Sóng thần chỉ xảy ra ở châu Á
B. Một cơn lốc xoáy cùng với bão dâng đã xảy ra ở châu Á vào năm 1970
C. Bão dâng là các vòm nước dâng cao bên dưới các cơn bão hoặc lốc xoáy.
D. Bão dâng gây ra ngập lụt ven biển trên diện rộng
Đây là câu hỏi lựa chọn - một trong những loại câu hỏi gây mất thời gian nhất trong bài đọc. Tuy nhiên ở
câu hỏi này, chúng ta có thể sử dụng kiến thức nền sẵn có thông qua báo chí truyền hình là có thể đoán
được phương án.
Ở đây ta thấy phương án A là phương án chúng ta lựa chọn vì sự thật chúng ta thấy sóng thần đã từng xảy
ra rất nhiều nơi trên thế giới chứ không phải mỗi châu Á.
Như vậy có thể thấy việc mở rộng kiến thửc nền, kiến thức xã hội là một trong những công việc chúng ta
nên làm hàng ngày để vừa nâng cao vốn từ nhưng đồng thời nâng cao hiểu biết- và đây cũng là một trong
những mẹo làm bài chúng ta có thể áp dụng để tìm ra các phương án không hợp logic.
Còn trong văn bản này, chúng ta có thể tìm thông tin ở đoạn 1, dòng 4-6.
Most tsunamis originate along the Ring of Fire, a zone of volcanoes and seismic activity, 32,500 km long
that encircles the Pacific Ocean.
Tạm dịch. Hầu hết các cơn sóng thần đều bắt nguồn từ Vành Đai Lửa- khu vực hoạt động của núi lửa và
các cơn địa chấn, trải dài 32,500 km bao quanh Thái Bình Dương → như vậy có nghĩa không chỉ xảy ra ở
châu Á. Ngoài ra thông tin ở các phương án khác cũng tìm thấy trong bài.
Ý B được diễn đạt ở đoạn 3, dòng 3-4. A cyclone and accompanying storm surge skilled an estimated
500,000 people in Bangladesh in 1970

Trang 18
Tạm dịch. Một trận lốc xoáy và cơn bão đi kèm đã làm thiệt mạng khoảng 500,000 người dân ở
Bangladesh vào năm 1970.
Ý C, D được tìm thấy trong đoạn 3, dòng 1 -2. Tsunamis should not be confused with storm surges, which
are domes of water that rise underneath hurricanes or cyclones and cause extensive coastal flooding when
the storms reach land.
Tạm dịch. Sóng thần không nên bị nhầm lẫn với hiện tượng bão dâng- là những vòm nước phát sinh bên
dưới các cơn bão hoặc lốc xoáy, và gây lũ lụt trên diện rộng khi đổ bộ vào bờ
Question 42. Đáp án C
Câu hỏi. Ý chính của bài là gì?
A. Sóng thần bắt nguồn từ đâu
B. Tổn thất do sóng thần gây ra
C. Những thông tin về sóng thần
D. Một trận sóng thần mạnh thế nào
Đây là câu hỏi hỏi ý chính - chúng ta nên để trả lời cuối cùng sau khi đã nắm được toàn bộ văn bản.
Chúng ta thấy đoạn 1 nói về nơi bắt nguồn của sóng thần - thông tin nằm cuối đoạn 1 với từ “originate”.
Đoạn 2 nói về các tính chất vật lý của sóng thần như chiều cao, độ dài, tốc độ.. .Đoạn 3 nói về thiệt hại do
sóng thần gây ra như là gây lũ lụt diện rộng, thiệt hại về người.. .như vậy ta thấy chỉ có đáp án C mới khái
quát nội dung chính của toàn bài
Question 43.
Câu hỏi. Tiêu đề phù hợp nhất cho bài đọc là gì?
A. Những quá trình khác nhau của sự lão hóa
B. Những đặc điểm nổi bật của lão hóa
C. Các giả thuyết được chứng minh một cách rõ ràng về sự lão hóa
D. Các giả thuyết ít được củng cố về sự lão hóa
Chúng ta có thể tìm thấy đáp án của câu hỏi này ngay câu đầu tiên của bài đọc: There are many theories
of aging, but virtually all fall into the category of being hypotheses with a minimum of supporting
evidence.
Tạm dịch. Có nhiều học thuyết về sự lão hóa nhưng thực tế là có rất ít bằng chứng chứng minh cho các
học thuyết này
Question 44. Đáp án A
Câu hỏi. Tác giả chỉ ra rằng các tế bào ung thư_____________.
A. Có vẻ sẽ tồn tại mãi mãi
B. Phân chia rồi chết
C. Phân chia ở trẻ sơ sinh nhiều hơn ở người lớn
D. Mất khả năng phân chia
Ta tìm thấy thông tin câu trả lời ở đoạn 2, dòng 2: Only cancer cells seem immortal in this respect.

Trang 19
Tạm dịch. Chỉ có các tế bào ung thư là có vẻ bất diệt trong việc này.
Ta có từ immortal (adj) - bất tử, bất diệt diễn tả ý tương đồng với “live forever” trong đáp án A
Question 45. Đáp án B
Câu hỏi. Cụm từ “an embryo” trong đoạn 2 gần nghĩa nhất với_____________
A. một người con, người cháu, hậu duệ
B. một đứa trẻ sơ sinh
C. một người cha/ người mẹ
D. một bộ phận bên trong cơ thể
Ta tìm thấy thông tin chứa từ vựng này trong đoạn 2, dòng 2-3. Fibroblast cells from an embryo divide
more times than those taken from an adult.
Tạm dịch. Các nguyên bào sợi lấy từ một đứa trẻ sơ sinh phân chia nhiều lần hơn các nguyên bào sợi lấy
từ người trưởng thành. Trong câu này ta thấy có sự so sánh “than” giữa hai đối tượng, một đối tượng
chính là an embryo và một đối tượng là an adult- người trưởng thành → Ta có thể dễ dàng suy luận đáp án
đúng là B
Question 46.
Câu hỏi. Từ “strenuous” ở đoạn 3 gần nghĩa nhất với_____________.
A. basic (adj): căn bản
B. troublesome (adj): gây rắc rối
C. mental (adj): thuộc về tinh thần
D. intense (adj): mạnh, có cường độ lớn
Thông tin nằm trong đoạn 3, dòng 5-6: For example, on the average, aging results in people losing much
of their ability to perform strenuous activities, yet some elderly individuals are excellent marathon
runners.
Tạm dịch. Ví dụ, nhìn chung thì lão hóa làm con người mất đi khả năng thực hiện các hoạt động cần
nhiều sức mạnh, tuy nhiên có những người cao tuổi lại là những người chạy đường dài cự phách.
Question 47. Đáp án B
Câu hỏi. Như được giải thích trong bài đọc, học thuyết lão hóa nghiên cứu ở mức độ tế bào không cho
dấu hiệu nào dưới đây là một nguyên nhân gây lão hóa
A. Sự đột biến
B. Sự hỏng hóc của hệ thống miễn dịch
C. Sự thất thoát nguyên liệu gen
D. Những hóa chất có hại từ môi trường Với câu hỏi này ta có thể dùng phép loại trừ, ta tìm thấy thông
tin trong đoạn 2 dòng 4-6: Any event that disturbs the cell's genetic machinery such as mutation,
damaging chemicals in the cell's environment, or loss of genetic material, could cause cells to lose their
ability to divide and thus bring on aging.

Trang 20
Tạm dịch. Bất kì việc gì gây ảnh hưởng đến bộ máy gen tế bào như đột biến, hóa chất có hại xâm nhập
vào môi trường tế bào hoặc sự thất thoát các nguyên liệu gen có thể khiến cho tế bào mất đi khả năng
phân chia và dẫn tới sự lão hóa.
Ta thấy thông tin trong các phương án A, C, D đều được nhắc đến → chỉ còn đáp án B là không được đề
cập
Question 48. Đáp án B
Câu hỏi. Theo đoạn văn, ước lượng độ lão hóa bằng tuổi không phải là một chỉ số chính xác để xác định
tình trạng tuổi tác của một người bởi vì____________
A. Những người già thường rất khỏe
B. Có sự khác nhau giữa các cá nhân về tốc độ lão hóa sinh học
C. Các hoạt động cần dùng sức không phải là một phương pháp đo tuổi chính xác
D. Rất khó để xác định được chính xác thông số ngày sinh
Ta tìm thấy thông tin trong đoạn 3, dòng 2-4: While chronological age can be useful in estimating the
average status of a large group of people, it is a poor indicator of an individual person's status because
there is a tremendous amount of variation from one individual to the next in regard to the rate at which
biological age changes occur.
Tạm dịch. Trong khi sự đo đạc lão hóa bằng tuổi có thể hữu dụng trong việc ước lượng tình trạng sức
khỏe trung bình của một nhóm nhiều người, nhưng nó lại là một chỉ số kém hiệu quả khi xác định tình
trạng sức khỏe của một cá nhân bởi vì những thay đổi về lão hóa sinh học sẽ xảy ra khác nhau và rất đa
dạng giữa người này với người kia
Như vậy giữa người này và người kia có sự khác nhau về tốc độ lão hóa sinh học, ta chọn đáp án B.
Question 49. Đáp án A
Câu hỏi. Từ “this” ở đoạn cuối ám chỉ____________
A. Lão hóa ngoại hình
B. Loại lão hóa
C. Ngoại hình
D. Tuổi tác tăng
Chúng ta tìm thấy thông tin ở đoạn cuối dòng 1-2: Another type of aging is cosmetic aging, which
consists of changes in outward appearance with advancing age. This includes changes in the body and
changes in other aspects of a person's appearance, such as the style of hair and clothing, the type of
eyeglasses, and the use of a hearing aid.
Tạm dịch. Một dạng khác của lão hóa là lão hóa ngoại hình, loại này gồm có những thay đổi về mặt ngoại
hình khi tuổi tác tăng. Nó bao gồm những thay đổi trong cơ thể và những thay đổi về diện mạo như kiểu
tóc, quần áo, kiểu kính mắt và việc phải sử dụng máy trợ thính.

Trang 21
Ta thấy đại từ chỉ định nếu làm chủ ngữ thường để thay thế cho thông tin xuất hiện phía trước. Ở câu văn
trước ta thấy đối tượng được nhắc đến là cosmetic aging. Như vậy có thể suy luận đáp án của câu hỏi này
là A
Question 50. Đáp án C
Câu hỏi. Tác giả ám chỉ tất cả những điều lão hóa ngoại hình dưới đây trừ_____________
A. Nó không xảy ra với mức độ như nhau cho tất cả mọi người
B. Nó là một chỉ số kém hiệu quả để xác định độ tuổi
C. Bệnh tật, dinh dưỡng kém và việc tiếp xúc vái ánh nắng mặt trời khiến cho lão hóa xảy ra
D. Nó được mô tả bởi các thay đổi ở hình thức bên ngoài
Chúng ta tìm thấy thông tin trong đoạn cuối dòng 5-7: because a person's appearance is affected by many
factors that are not part of aging, including illness, poor nutrition, and exposure to sunlight.
Tạm dịch, bởi vì ngoại hình của con người có thể bị tác động bởi nhiều yếu tố không phải là lão hóa bao
gồm bệnh tật, dinh dưỡng kém và việc tiếp xúc với ánh mặt trời.
Như vậy ta thấy các yếu tố như bệnh tật, dinh dưỡng kém và việc tiếp xúc với ánh nắng mặt trời không
phải là nguyên nhân của lão hóa → Chọn đáp án C

Trang 22
ĐỀ SỐ 16 ĐỀ THI THỬ TỐT NGHIỆP THPT
NĂM HỌC: 2020 – 2021
MÔN: TIẾNG ANH
Thời gian làm bài: 60 phút; không kể thời gian phát đề

Mark the letter A, B, C, or D on your answer sheet to indicate the word whose underlined part
differs from the other three in pronunciation in each of the following questions.
Question 1. A. expand B. stagnant C. vacancy D. applicant
Question 2. A. examine B. employ C. exact D. species
Mark the letter A, B, C or D on your answer sheet to indicate the word that differs from the rest in
the position of the main stress in each of the following questions.
Question 3. A. attention B. financial C. guarantee D. courageous
Question 4. A. profit B. modern C. typhoon D. feature
Mark the letter A, B, C or D on your answer sheet to indicate the correct answer to each of the
following questions.
Question 5. Some encyclopedias provide___________on specific fields such as music or philosophy.
A. information B. informations C. informing D. informativation
Question 6. This time last Friday, we___________to Moscow.
A. flew B. had flown C. were being flown D. were flying
Question 7. My boss expects the report___________on time.
A. to be completed B. to complete C. completing D. being completed
Question 8. The language centre offers courses of various levels, such as elementary, intermediate
and___________.
A. advance B. advancement C. advanced D. advancing
Question 9. ___________5 months for the fridge to be delivered, I decide to cancel the order.
A. Waiting B. Waited C. Having waited D. Had waited
Question 10. The clothes of my sister are more expensive___________of my mother.
A. than this B. than that C. than those D. than these
Question 11. In a formal interview, it is essential to maintain good eye___________with the
interviewers.
A. link B. connection C. touch D. contact
Question 12. It is___________of businessmen to shake hands in formal meetings.
A. familiar B. typical C. ordinary D. common
Question 13. Yesterday, John told me that he wished he___________harder in high school then
university might not be so difficult for him.

Trang 1
A. studied B. would study C. had studied D. studies
Question 14. Peter was the last applicant___________by that interviewer.
A. to interview B. interviewing C. to be interviewing D. to be interviewed
Question 15. He fixed a metal ladder to the wall below his window___________escape if there was a
fire.
A. to B. not to C. so as not D. so that
Question 16. He came___________a lot of criticism for the remarks he made in a television interview.
A. in for B. over C. out of D. off
Question 17. Jane has never enjoyed going to the ballet or the opera. They are not really
her___________.
A. piece of cake B. cup of tea C. biscuit D. chip off the old block
Question 18. I always get in my stomach before visiting the dentist. ___________
A. worms B. butterflies C. crabs D. mosquitoes
Mark the letter A, B, C, or D on your answer sheet to indicate the underlined part that needs
correction in each of the following questions.
Question 19. The keynote speaker started with some complementary remarks about the organizers of
the conference, and then proceeded with her speech.
A. The keynote speaker B. complementary remarks
C. organizers D. proceeded
Question 20. when I got home, Irene lay in bed watching a blockbuster movie.
A. got home B. lay C. watching D. blockbuster
Question 21. The graduated assistant informed us that the first exam was the most difficult of the two.
A. graduated B. informed C. the first D. most difficult
Mark the letter A, B, C or D on your answer sheet to indicate the sentence that is closest in
meaning to each of the following questions.
Question 22. In Vietnam, football is more popular than basketball.
A. In Vietnam, basketball is not as popular as football.
B. In Vietnam, basketball is more popular than football.
C. In Vietnam, football is not as popular as basketball.
D. In Vietnam, football is as popular as basketball.
Question 23. It usually takes her an hour to drive to work.
A. She never spends an hour driving to work.
B. She used to drive to work in an hour.
C. She doesn’t usually drive to work in an hour.

Trang 2
D. She usually spends an hour driving to work.
Question 24. It is believed that modern farming methods have greatly improved farmers’ lives.
A. People believe that there is little improvement in farmers’ lives thanks to modern farming
methods.
B. Modern farming methods were believed to have greatly improved farmers’ lives.
C. Modern farming methods are believed to have had negative effects on farmers’ lives.
D. Farmers are believed to have enjoyed a much better life thanks to modern farming methods.
Mark the letter A, B, C or D on your answer sheet to indicate the sentence that best combines
each pair of sentences in the following questions.
Question 25. We gain more knowledge about how to stay safe online. We worry about the threats
of cybercrime less.
A. The more knowledge about how to stay safe online we gain, the less we worry about the threats of
cybercrime.
B. The more we stay online to gain safety knowledge, the less we worry about the threats of
cybercrime.
C. The more knowledge about how to stay safe online we gain, the more we worry about the threats
of cybercrime.
D. The more we know about how to stay safe online, we worry about the threats of cybercrime less.
Question 26. The new contract sounds good. However, it seems to have some problems.
A. In spite of the fact that the new contract sounds good, it appears to have some problems.
B. In spite of its sound, the new contract appears to have some problems.
C. In order to get less problem, the new contract should improve its sound.
D. If it had much less problems, the new contract would sound better.
Mark the letter A, B, C or D on your answer sheet to indicate the most suitable response to
complete each of the following exchanges.
Question 27. Liz and Jenifer are talking about the gift Jenifer brought.
- Liz: “Thanks for the nice gift you brought to us!”
- Jenifer: “__________ ”
A. Not at all, don’t mention it.
B. Welcome! It’s very nice of you.
C. Alright! Do you know how much it costs?
D. Actually speaking, I myself don’t like it.
Question 28. Ben has done something by accident.
- Ben: “___________”

Trang 3
- Jane: “Never mind”
A. Sorry for staining your carpet. Let me have it cleaned
B. Would you mind going to dinner next Sunday?
C. Thank you for being honest with me.
D. Congratulations! How wonderful!
Mark the letter A, B, C, or D on your answer sheet to indicate the word or phrase that is
CLOSEST in meaning to the underlined part in each of the following questions.
Question 29. The football final has been postponed until next Sunday due to the heavy snowstorm.
A. cancelled B. changed C. delayed D. continued
Question 30. The discovery of the new planet was regarded as a major breakthrough in astronomy.
A. promised B. doubted C. considered D. refused
Mark the letter A, B, C, or D on your answer sheet to indicate the word or phrase that is
OPPOSITE in meaning to the underlined part in each of the following questions.
Question 31. People nationwide have acted without hesitation to provide aids for the victims in the
disaster-stricken areas.
A. awareness B. uncertainty C. reluctance D. willingness
Question 32. Any student who neglects his or her homework is unlikely to do well at school.
A. puts off B. looks for C. attends to D. approves of
Read the following passage and mark the letter A, B, C or D on your answer sheet to indicate the
correct word for each of the blanks from 33 to 37.
Back in the 1960s and 1970s, the world was becoming more aware of the destructive effects of industry
(33) ___________. the environment and people were starting to think seriously about ways of
protecting the environment. One man who was particularly affected by this subject was Gerard Morgan-
Grenville. As Morgan-Grenville travelled round earning his living as a gardener, he noticed signs of
damage that was being done to the countryside around him. It wasn’t long before Morgan-Grenville
decided that he had to do something about this situation. He felt that if people could be shown a better
way of living then maybe they would be interested enough to try to protect their (34)
___________environment.
Mr. Morgan-Grenville decided to set up a project (35) ___________would prove what was happening
to our surroundings and what could be done about it. So, in 1975, Morgan-Grenville created the Centre
for Alternative Technology (CAT) in a village in Wales.
The main aim of CAT is to search for an ecologically better way of living by using technology which
(36) ___________no harm to the environment. One of the most important things CAT did initially was
to explore and demonstrate a wide range of techniques and to point out which ones had the least

Trang 4
destructive results on the world around us. (37) ___________, CAT provides information and advice to
people all over Britain and all over the world. If more and more individuals are informed about how
much damage our modern lifestyle is causing to the planet, maybe more of them would be prepared to
look for practical solutions to environmental problems.
Question 33. A. for B. to C. with D. on
Question 34. A. worthless B. valueless C. precious D. priceless
Question 35. A. that B. whose C. who D. this
Question 36. A. gets B. makes C. plays D. does
Question 37. A. Therefore B. However C. Moreover D. Although
Read the following passage and mark the letter A, B, C, or D on your answer sheet to indicate the
correct answer to each of the questions from 38 to 42.
The custom of paying a bride price before marriage is still a well-established part of many African
cultures. In paying a bride price, the family of the groom must provide payment to the family of the
bride before the marriage is allowed. The bride price can vary greatly from culture to culture in Africa.
In the Zulu and Swazi tribes of southern Africa, the bride price often takes the form of cattle. In
Western African, kola nuts, shells, and other goods are often used for the payment of the bride price.
The actual payment of money sometimes takes place, but the payment of goods is more frequent. The
amount of paid in a bride price can also vary. In modern times, the bride price is occasionally quite
small and its value is mainly symbolic. However, the bride price can still be quite high, especially
among prominent or highly traditional families.
There are a number of justifications used to explain the payment of bride price. The first is that the
bride price represents an acknowledgement of the expense the bride’s family has gone in order to raise
her and bring her up as a suitable bride for the groom. It also represents payment for the loss of a
family member, since the bride will officially become a member of her husbands family and will leave
her own. On a deeper level the bride price represents payment for the fact that the bride will bring
children into the family of the groom, thereby increasing the wealth of the family. This concept is
reinforced by the fact that the bride price must often be returned if the bride fails to bear children.
The payment of the bride price has quite a number of effects on African society. First, the payment of
bride price acts to increase the stability of African family structures. Sons are dependent on their fathers
and older relatives to help them pay the bride price of their wives, and this generally leads to greater
levels of obedience and respect. The negotiations between the two families concerning the bride price
allow the parents and other family members to meet and get to know one another before the marriage.
Finally, since the bride price must often be repaid in case of divorce, the bride’s family often works to
make sure that any marital problems are solved quickly. Bride prices also work as a system of wealth

Trang 5
distribution in African cultures. Wealthier families can afford to support the marriage of their son, and
thus their wealth is transferred to other families.
Question 38. What is the topic of the passage?
A. The most common payments of the bride price in African cultures.
B. The effects of the practice of bride price on African society.
C. The practice of bride price in many African cultures.
D. The financial problems arising after marriage.
Question 39. The word “prominent” in paragraph 1 is closet in meaning to____________.
A. famous B. impoverished C. two-parent D. affluent
Question 40. The word “It” in paragraph 2 refers to____________.
A. Justification B. The expense of raising the bride
C. An acknowledgement D. The bride price
Question 41. It can be inferred from the paragraph 2 that____________.
A. After marriage, married women never see their parents again.
B. The family of the groom pays the bride price on the wedding day.
C. African society puts more value on men than women.
D. African families attach great importance to children.
Question 42. Which of the following statement is NOT true, according to the passage?
A. With the practice of paying the bride price, African men would not respect their family members.
B. The initial negotiations over the bride price provide opportunities for the two families to get to
know each other.
C. Animals are an acceptable form of payment when it comes to paying the bride price.
D. The bride’s family has to return the bride price to the groom’s in case the couple are unable to
have children or get divorced.
Read the following passage and mark the letter A, B, C, or D on your answer sheet to indicate the
correct answer to each of the questions from 43 to 50.
Contrary to popular belief, one does not have to be trained programmer to work online. Of course, there
are plenty of jobs available for people with high-tech computer skills, but the growth of new media has
opened up a wide range of Internet career opportunities requiring only a minimal level of technical
expertise. Probably one of the most well-known online job opportunities is the job of webmaster.
However, it is hard to define one basic job description for this position. The qualifications and
responsibilities depend on what tasks a particular organization needs a webmaster to perform.
To specify the job description of a webmaster, one needs to identify the hardware and software that the
website the webmaster will manage is running on. Different types of hardware and software require

Trang 6
different skill sets to manage them. Another key factor is whether the website will be running internally
(at the firm itself) or externally (renting shared space on the company servers). Finally, the
responsibilities of a webmaster also depend on whether he or she will be working independently, or
whether the firm will provide people to help. All of these factors need to be considered before one can
create an accurate webmaster job description.
Webmaster is one type of Internet career requiring in-depth knowledge of the latest computer
applications. However, there are also online jobs available for which traditional skills remain in high
demand. Content jobs require excellent writing skills and a good sense of the web as a “new media”.
The term “new media” is difficult to define because it encompasses a constantly growing set of new
technologies and skills, specifically, it includes websites, email, Internet technology, CD-ROM, DVD,
streaming audio and video, interactive multimedia presentations, e-books, digital music, computer
illustration, video games, virtually reality, and computer artistry.
Additionally, many of today’s Internet careers are becoming paid-by-the-job professions. With many
companies having to downsize in tough economic items, the outsourcing and contracting of freelance
workers online has become common business practice. The Internet provides an infinite pool of buyers
from around the world with whom freelancers can contract their services. An added benefit to such
online jobs is that freelancers are able to work on projects with companies outside their own country of
residence.
How much can a person make in these kinds of career? As with many questions related to todays
evolving technology, there is no simple answer. There are many companies willing to pay people with
technical Internet skills salaries well above $70,000 a year. Generally, webmasters start at about
$30,000 per year, but salaries can vary greatly. Freelance writers working online have been known to
make between $40,000 and $70,000 per year.
Question 43. What is the best title for this passage?
A. The definition of “new media” B. Internet jobs
C. The job of Webmasters D. People with Internet skills
Question 44. According to the passage, which of the following is TRUE of webmasters?
A. They work either independently or collaboratively.
B. The duties they perform stay almost unchanged.
C. Their jobs require a minimal level of expertise
D. They hardly support hardware and software products.
Question 45. The word “identity” in paragraph 2 is closest in meaning to____________
A. find out B. pick up on C. come across D. look into
Question 46. The word “them” in paragraph 2 refers to____________.

Trang 7
A. websites B. tasks C. hardware and software D. skill sets
Question 47. Which of the followings is NOT true about the job of the freelance writers?
A. It may involve working with foreign companies.
B. It is considered a “content” job.
C. There is a high demand for traditional skills.
D. It requires deep knowledge of computer applications.
Question 48. The word “downsize” in paragraph 5 most likely means____________.
A. dismiss workers B. decrease salary C. go bankrupt D. win new contracts
Question 49. According to the passage, all of the followings are true EXCEPT____________.
A. A basic job description for a webmaster is not easy to define.
B. A webmaster does not have to get himself updated with computer science.
C. “New media” does not encompass writing skills.
D. Online jobs for workers with basic computer skills are available.
Question 50. It can be inferred from the passage that____________.
A. only well-trained workers are advised to work online.
B. online workers can work full-time online.
C. becoming a webmaster is really easy.
D. workers with limited computer skills cannot make good money.

Trang 8
Đáp án
1-C 2-D 3-C 4-C 5-A 6-D 7-A 8-C 9-C 10-C
11-D 12-B 13-C 14-D 15-A 16-A 17-B 18-B 19-B 20-B
21-D 22-A 23-D 24-D 25-A 26-A 27-A 28-A 29-C 30-C
31-D 32-C 33-D 34-D 35-A 36-D 37-C 38-C 39-A 40-D
41-D 42-A 43-B 44-A 45-A 46-C 47-D 48-A 49-B 50-B

LỜI GIẢI CHI TIẾT


Question 1: Đáp án C
Đáp án C đúng vì phần gạch chân của đáp án C đọc là âm /eɪ/ còn phần gạch chân của các đáp án khác
được đọc là âm /æ/
A. expand /ɪkˈspænd/ (v): mở rộng
B. stagnant /ˈstæɡnənt/ (adj): ứ đọng, trì trệ, uể oải
C. vacancy /ˈveɪkənsi/ (n): vị trí việc làm còn trống, phòng còn trống (trong khách sạn)
D. applicant /ˈæplɪkənt/ (n): người xin việc
Question 2. Đáp án D
Đáp án D đúng vì phần gạch chân của đáp án D đọc là âm /i :/ còn phần gạch chân của các đáp án còn lại
được đọc là âm /ɪ/
A. examine /ɪɡˈzæmɪn/ (v): sát hạch, khám
B. employ /ɪmˈplɔɪ/ (v): tuyển dụng
C. exact /ɪɡˈzækt/ (adj): chính xác
D. species /ˈspiːʃiːz/ (n): loài
Question 3. Đáp án C
Đáp án C đúng vì đáp án C trọng âm rơi vào âm tiết thứ ba, còn ba đáp án còn lại trọng âm rơi vào âm tiết
thứ hai
A. attention /əˈtenʃn/ (n): sự chú ý
B. financial /faɪˈnænʃl/ (adj): tài chính
C. guarantee /ˌɡærənˈtiː/ (n): sự đảm bảo
D. courageous /kəˈreɪdʒəs/ (adj): gan dạ
❖ For review
Các từ có ba âm tiết thì trọng âm của từ thường rơi vào âm tiết thứ hai từ dưới lên nếu từ có tận cùng
bằng hậu tố -ion, -ial, -eous
Các từ có ba âm tiết thì trọng âm của từ thường rơi vào âm cuối nếu từ có tận cùng bằng hậu tố -ee, -eer, -
ette, -ese, -ain, -oon,....

Trang 9
Question 4. Đáp án C
Đáp án C đúng vì đáp án C trọng âm rơi vào âm tiết thứ hai, còn ba đáp án còn lại trọng âm rơi vào âm
tiết thứ nhất.
A. profit /ˈprɒfɪt/ (n): lợi nhuận
B. modern /ˈmɒdn/ (adj): hiện đại
C. typhoon /taɪˈfuːn/ (n): bão nhiệt đới
D. feature /ˈfiːtʃə(r)/ (n): điểm đặc trưng
❖ For review
Đa số danh từ và tính từ có hai âm tiết thì trọng âm rơi vào âm tiết thứ nhất.
Trọng âm của từ rơi vào âm tiết cuối nếu từ có tận cùng bằng hậu tố -oon
Question 5. Đáp án A
Ta có “provide st”: cung cấp điểu gì → Ta cần một danh từ ở vị trí cần điền.
information (n): thông tin - là một danh từ không đếm được nên sẽ không có số nhiều.
Trong 4 đáp án chỉ có phương án A là chính xác. Phương án D sai vì không có từ này trong tiếng Anh.
Tạm dịch: Một số quyển bách khoa toàn thư cung cấp thông tin về các lĩnh vực cụ thể như âm nhạc hoặc
triết học.
Question 6. Đáp án D
Cụm từ “This time last Friday” cho thấy hành động xảy ra tại một thời điểm cụ thể trong quá khứ → ta sử
dụng thì quá khứ tiếp diễn. Mặt khác, động từ trong câu được chia ở chủ động → đáp án D đúng
Tạm dịch: Vào giờ này thứ sáu tuần trước, chúng tôi đang bay tới Mát-xcơ-va
Question 7. Đáp án A
Ta có cấu trúc: expect + Obj+ to + V/ to be + P2 → loại C và D
Ta thấy complete (v): hoàn thành - là một ngoại động từ, trong câu này thiếu tân ngữ → chỗ trống cẩn
điền phải ở dạng bị động → đáp án A đúng
Tạm dịch: Sếp của tôi mong rằng bản báo cáo sẽ được hoàn thành đúng thời gian.
Question 8. Đáp án C
Ta thấy liên từ “and” dùng để nối các từ cùng dạng: elementary và intermediate đều là tính từ → từ cần
điền cũng phải là một tính từ → loại A và B. Đáp án D loại vì không phù hợp về nghĩa.
A. advance (n): tiến bộ, tiến lên
B. advancement (n): sự tiến bộ
C. advanced (adj): nâng cao, cao cấp
D. advancing (adj): tiến tới, tiến lên
Tạm dịch: Trung tâm ngoại ngữ đó cung cấp các khóa học ở nhiều trình độ như là sơ cấp, trung cấp và
nâng cao.

Trang 10
Question 9. Đáp án C
Câu trên là trường hợp rút gọn mệnh đề chung chủ ngữ.
- Nếu hai hành động ở hai mệnh đề xảy ra đồng thời hay được chia cùng thì với nhau, ta lược bỏ chủ từ ở
một mệnh đề rồi sau đó chia động từ trong mệnh đề đó ở dạng V-ing.
- Nếu hai hành động ở hai mệnh đề xảy ra theo thứ tự trước sau, ta lược bỏ chủ từ của mệnh để có hành
động xảy ra trước rồi sau đó chia động từ trong mệnh đề đó ở dạng Having + P2. Như vậy ta giữ lại hai
đáp án A và C.
Ở đây ta có hai hành động là “wait” và “decide”. Dễ dàng nhận thấy trong câu trên hành động “wait” là
hành động xảy ra trước còn “decide” là hành động xảy ra sau → đáp án đúng là C.
Tạm dịch: Phải chờ 5 tháng để chiếc tủ lạnh được chuyển đến nên tôi quyết định hủy đơn hàng.
Question 10. Đáp án C
Câu trên sử dụng hiện tượng so sánh hơn nhưng không muốn nhắc lại đối tượng so sánh. Trong trường
hợp này chúng ta có thể dùng “that” để thay thế cho danh từ số ít và “those” để thay thế cho danh từ số
nhiều; “clothes” đang được để ở dạng số nhiều → chọn đáp án C
Tạm dịch: Quần áo của chị tôi đắt hơn quần áo của mẹ tôi.
Question 11.
Ta có cụm từ “eye contact”: sự giao tiếp bằng mắt
A. link (n): mối liên lạc
B. connection (n): sự liên quan, mối quan hệ
C. touch (n): sự chạm, sờ
D. contact (n): sự giao tiếp, sự tiếp xúc
Tạm dịch: Trong một cuộc phỏng vấn trang trọng, cần duy trì giao tiếp tốt bằng mắt với người phỏng vấn.
Question 12. Đáp án B
Trong câu trên chúng ta thấy có cụm từ “shake hands”- bắt tay- đây là một hành động có tính đặc thù, tiêu
biểu trong các cuộc họp hoặc gặp mặt chính thức. Xét về nghĩa thì chỉ có phương án B là phù hợp.
A. familiar (adj): quen thuộc, thân thuộc
B. typical (adj): đặc thù, tiêu biểu
C. ordinary (adj): thường, bình thường
D. common (adj): chung, thông thường, phổ biến
Tạm dịch: Việc doanh nhân bắt tay tại các cuộc gặp chính thức là một hành động đặc thù.
Question 13. Đáp án C
Trong câu trên ta thấy có sử dụng cấu trúc của thức giả định. Từ “yesterday” cho thấy đây là điều ước trái
với quá khứ, như vậy ta sẽ sử dụng cấu trúc: S + wish+ S + had P2 - Giá mà làm việc gì. Như vậy chỉ có
đáp án C là phù hợp.

Trang 11
Tạm dịch: Hôm qua, John bảo tôi rang anh ấy ước giá mà hồi cấp 3 anh ấy học chăm chỉ hơn thì việc vào
đại học sẽ không quá khó khăn.
Question 14. Đáp án D
Câu trên sử dụng mệnh đề quan hệ được rút gọn. Với mệnh đề quan hệ được rút gọn, nếu trước danh từ có
mệnh đề quan hệ có các từ chỉ thứ tự như first, last.. .thì mệnh đề đó sẽ được rút gọn thành “to V”. Trong
trường hợp này “interview” là một ngoại động từ nhưng không có tân ngữ theo sau nên sẽ ở dạng bị động
“được phỏng vấn”. Như vậy vị trí chỗ trống cần điền sẽ có dạng “to be P2”. Đáp án đúng chỉ có thể là đáp
án D
Tạm dịch: Peter là ứng viên cuối cùng được phỏng vấn
Question 15. Đáp án A
Câu trên sử dụng mệnh đề trạng ngữ chỉ mục đích. Chúng ta có một số cách dùng mệnh đề trạng ngữ chỉ
mục đích như sau:
- to V/ not to V/ in order (not) to V/ so as (not) to V
- in order that/ so that + S + will/ would/ can/ could ... + V(bare-infinitive)
Đáp án B sai vì không phù hợp nghĩa của câu.
Đáp án C sai về cấu trúc, theo cấu trúc trên thì phải là so as not to mới chính xác.
Đáp án D cũng sai về cấu trúc vì sau so that sẽ là một mệnh đề.
Tạm dịch: Anh ta đã gắn một cái thang kim loại vào tường dưới cửa sổ để thoát hiểm khi có hỏa hoạn.
Question 16. Đáp án A
Vì cụm động từ (phrasal verb) “come in for” - hướng, nhận. Ở đây có nghĩa là “nhận nhiều lời chỉ trích”
B. come over: đến thăm, ghé thăm
C. come out of: thoát khỏi...
D. come off: thành công, xảy ra
Tạm dịch: Ông ấy nhận nhiều lời chỉ trích cho những bình luận của ông ấy trong một cuộc phỏng vấn trên
truyền hình.
Question 17. Đáp án B
Ta có thành ngữ “cup of tea”: thứ mình thích
A. piece of cake: việc ngon ơ
C. biscuit (n): bánh quy
D. chip off the old block: người có tính cách giống bố
Tạm dịch: Jane chưa bao giờ hứng thú với đi xem kịch và ba lê. Bởi đó không phải là thứ mà cô ấy thích
Question 18. Đáp án B
Ta có thành ngữ “get butterflies in one’s stomach”: cảm thấy bồn chồn
A. worm (n): con mọt

Trang 12
C. crab (n): con cua
D. mosquito (n): con muỗi
Tạm dịch: Tôi luôn cảm thấy bồn chồn trước khi đến gặp nha sĩ.
Question 19.
Câu này đã sử dụng sai tính từ, ý của người viết là “những lời nhận xét khen ngợi” nhưng đã dùng nhầm
sang một tính từ khác. Ở đây ta cần sửa complementary thành complimentary.
- complementary (adj): bù vào, bổ sung
- complimentary (adj): khen ngợi, ca ngợi
Tạm dịch: Diễn giả đã bắt đầu buổi diễn thuyết bằng những lời nhận xét khen ngợi cho ban tổ chức hội
nghị, sau đó mới bắt đầu bài nói của cô ấy.
Question 20.
Đáp án B đang chia sai thời của động từ. Ở đây thể hiện hành động đang xảy ra trong quá khứ “đang nằm
trên giường xem phim” thì một hành động khác xen vào “tôi về nhà”
→ phương án B phải sửa lại thành “was lying”.
Tạm dịch: Khi tôi về nhà thì Irene đang nằm trên giường xem một bộ phim bom tấn.
Question 21.
Câu trên là hiện tượng so sánh giữa 2 đối tượng (“first exam” và “of the two” - trong hai kì thi), do vậy
cần dùng dạng thức so sánh hơn. Sửa “most difficult" thành “more difficult"
Tạm dịch: Trợ giảng đã thông báo cho chúng tôi rằng trong hai kì thi thì kì thi đầu tiên khó hơn.
Question 22. Đáp án A
Dịch câu đề: Ở Việt Nam, bóng đá phổ biến hơn bóng rổ.
Dịch đáp án.
A. Ở Việt Nam, bóng rổ không phổ biến bằng bóng đá.
B. Ở Việt Nam, bóng rổ phổ biến hơn bóng đá.
C. Ở Việt Nam, bóng đá không phổ biến bằng bóng rổ.
D. Ở Việt Nam, bóng đá phổ biến như bóng rổ.
Đáp án đúng của câu hỏi này là đáp án A. Chúng ta có cấu trúc so sánh không ngang bằng.
S + V (phủ định) + as+ tính từ/ trạng từ + as + N
Đáp án B, C, D sai vì truyền đạt sai ý của câu gốc
Question 23. Đáp án D
Dịch câu đề. Cô ấy thường tốn 1 tiếng để lái xe đi làm.
Dịch đáp án.
A. Cô ấy không bao giờ dành 1 tiếng để lái xe đi làm.
B. Cô ấy từng lái xe đi làm trong vòng 1 tiếng.

Trang 13
C. Cô ấy không thường lái xe đi làm trong vòng 1 tiếng.
D. Cô ấy thường dành 1 tiếng để lái xe đi làm. Đáp án đúng của câu hỏi này là D. Chúng ta có cấu trúc.
It + (take) + sb+ thời gian+ to V - S + (spend)+ thời gian+ V-ing.
Đáp án A sai vì diễn đạt sai ý nghĩa của câu gốc.
Đáp án B sai vì cấu trúc S+ used to V dùng để diễn tả hành động thường làm trong quá khứ nhưng giờ
không làm nữa.
Đáp án C sai vì sai nghĩa so với câu gốc.
Question 24. Đáp án D
Dịch câu đề. Người ta tin rằng các phương pháp trồng trọt hiện đại đã cải thiện đáng kể đời sống của
người nông dân.
Dịch đáp án.
A. Người ta tin rằng chỉ có chút ít cải thiện trong cuộc sống của người nông dân nhờ vào các phương
pháp trồng trọt hiện đại.
B. Các phương pháp trồng trọt hiện đại đã được tin tưởng rằng sẽ cải thiện đáng kể đời sống của người
nông dân
C. Các phương pháp trồng trọt hiện đại được tin là có ảnh hưởng tiêu cực đến đời sống của người nông
dân.
D. Người nông dân được tin là có cuộc sống tốt đẹp hơn nhiều nhờ có các phương pháp trồng trọt hiện
đại.
Đáp án của câu hỏi này là đáp án D. Đây là câu hỏi liên quan đến cấu trúc của câu bị động với động từ
tường thuật.
Chúng ta có cầu chủ động ở dạng: S1 + V1 (các động từ tường thuật như believe, think,..) that S2+V2
Trong đó:
S1 + V1 là mệnh đề chính
S2 + V2 là mệnh đề that (that - clause).
Để chuyển câu chủ động trên sang câu bị động thì thông thường có 2 cách chuyển chính như sau.
Cách 1. Chuyển chủ ngữ trong câu chủ động lên làm chủ ngữ trong câu bị động như thông thường.
Cấu trúc:
S2 + passive V1 (chia theo V1) + to-infinitive: nếu hành động ở mệnh để that xảy ra đồng thời hoặc sau
hành động ở mệnh đề chính.
S2 + passive V1 (chia theo V1) + to be V-ing: nếu hành động ở mệnh đề that diễn tả sự tiếp diễn (hiện tại
tiếp diễn/ quá khứ tiếp diễn/...)
S2 + passive V1 (chia theo V1) + to have Vpp: nếu hành động ở mệnh đề that xảy ra trước hành động trong
mệnh đề chính.

Trang 14
Cách 2. Sử dụng chữ ngữ giả “It"
It + passive V1 + that + S2 + V2
Trong câu hỏi này đề bài đang sử dụng cấu trúc bị động sử dụng chủ ngữ giả “It” (Cách 2).
Ở đây phương án D sử dụng cấu trúc bị động với tân ngữ farmers làm chủ ngữ (cách 1).
Đáp án A sai vì chuyển tải sai ý nghĩa so với câu gốc
Đáp án B sai vì chia sai thời của động từ so với câu gốc
Đáp án C sai vì chuyển tải sai ý nghĩa so với câu gốc
Chỉ có phương án D là vừa sử dụng đúng cấu trúc ngữ pháp và đổng thời chuyển tải đúng ý nghĩa của cầu
gốc
Question 25.
Dịch câu đề. Chúng ta biết nhiều hơn về cách giữ an toàn khi lên mạng. Chúng ta sẽ bớt lo lắng hơn về
những mối đe dọa từ tội phạm mạng.
Dịch đáp án.
A. Chúng ta càng biết về cách giữ an toàn khi lên mạng nhiều bao nhiêu thì chúng ta sẽ ít phải lo lắng về
những mối đe dọa từ tội phạm mạng bấy nhiêu
B. Chúng ta càng lên mạng để tìm hiểu kiến thức an toàn bao nhiêu thì chúng ta sẽ ít phải lo lắng vể
những mối đe dọa từ tội phạm mạng bấy nhiêu.
C. Chúng ta càng biết về cách giữ an toàn khi lên mạng nhiều bao nhiêu thì chúng ta sẽ càng phải lo lắng
về những mối đe dọa từ tội phạm mạng bấy nhiêu
D. Chúng ta càng biết cách giữ an toàn khi lên mạng nhiều bao nhiêu thì chúng ta sẽ ít lo lắng về những
mối đe dọa từ tội phạm mạng hơn.
Đáp án của câu hỏi này là đáp án A. Chúng ta có cấu trúc so sánh kép:
The + comparative + S+V, the comparative +S+V: càng.. .thì càng...
Trong 4 phương án chỉ có phương án A là truyền đạt đúng ý nghĩa của câu gốc và sử dụng đúng cấu trúc
để nối hai câu đơn thành một câu ghép.
Phương án B và C sai vì truyền đại sai ý nghĩa của câu gốc
Phương án D sai vì sử dụng sai cấu trúc so sánh kép.
Question 26. Đáp án A
Dịch câu đề. Hợp đồng mới nghe có vẻ rất được. Tuy nhiên nó dường như có một vài vấn đề.
Dịch đáp án.
A. Mặc dù thực tế là hợp đồng mới nghe có vẻ rất được nhưng nó dường như có một vài vấn đề.
B. Mặc dù âm thanh của nó nhưng hợp đồng mới dường như có một vài vấn đề.
C. Để có ít vấn đề xảy ra hơn thì hợp đồng mới nên cải thiện âm thanh của nó.
D. Giả sử có ít vấn đề hơn thì hợp đồng mới nghe có vẻ ổn hơn.

Trang 15
Đáp án đúng của câu hỏi này là đáp án A. Chúng ta có cấu trúc mệnh đề nhượng bộ với “in spite of”:
In spite of/ Despite + N/ V-ing, S+V.
In spite of/ Despite+ the fact that + S+V, S+V.
Đáp án B sai vì truyền đạt sai ý nghĩa của câu gốc
Đáp án C sai vì truyền đạt sai ý nghĩa của câu gốc, ở đây câu gốc không nhấn mạnh vào mục đích của
hành động.
Đáp án D sai vì sử dụng sai cấu trúc của mệnh đề nhượng bộ và câu gốc không truyền tải ý nghĩa của câu
điều kiện
Question 27. Đáp án A
Liz và Jenifer đang nói chuyện về món quà mà Jenifer đem tới. Đây là câu cảm ơn được bắt đầu bởi
“Thanks for...”. Cảm ơn vì món quà tuyệt vời bạn đã mang đến cho chúng tôi
A. Không có gì đâu, đừng khách sáo.
B. Không có gì! Bạn tốt quá.
C. Đúng rồi! Bạn có biết nó bao nhiêu tiền không?
D. Thật sự mà nói thì bản thân tôi khống thích nó lắm.
Để đáp lại lời cảm ơn chúng ta sẽ sử dụng cấu trúc “Not at all, don’t mention it”.
Đáp án đúng của câu hỏi này là đáp án A
Question 28. Đáp án A
Ben vừa vô tình gây ra một chuyện và từ khóa là lời đáp cho câu xin lỗi: Never mind = No big deal:
không vấn đề gì, không sao đâu, đừng bận tâm.
Vì vậy đáp án của câu hỏi này là đáp án A: Xin lỗi vì đã làm bẩn tấm thảm của bạn. Để tôi đem nó đi giặt.
→ Để xin lỗi một ai đó chúng ta có thể bắt đầu bằng Sorry for...
B. Bạn có phiền không nếu đi ăn tối vào chủ nhật tới?
C. Cảm ơn vì đã chân thành với tôi.
D. Chúc mừng! Thật tuyệt quá!
Question 29. Đáp án C
Đáp án C đúng vì ta có postpone (v) có nghĩa là hoãn lại = delay (v). Đây là câu hỏi tìm từ đồng nghĩa
nên chọn đáp án C
A. cancelled: hủy bỏ
B. changed: thay đổi
C. delayed: hoãn, làm chậm trễ
D. continued: tiếp tục
Tạm dịch: Trận chung kết bóng đá bị hoãn đến chủ nhật tuần tới do bão tuyết lớn
Question 30. Đáp án C

Trang 16
Đáp án C đúng vì ta có: (to) regard st1 as st2 = (to) consider st1 st2: coi cái gì là cái gì. Đây là câu hỏi tìm
từ đồng nghĩa nên ta chọn phương án C
A. promised: hứa hẹn
B. doubted: nghi ngờ
C. considered: cân nhắc
D. refused: từ chối
Tạm dịch: Sự phát hiện ra hành tinh mới đó được coi như một bước đột phá lớn trong ngành thiên văn
học.
Question 31. Đáp án D
Đáp án D đúng vì ta có hesitation (n) = reluctance (n): sự lưỡng lự, do dự >< willingness (n): sự sẵn lòng,
sẵn sàng làm điều gì. Đây là câu hỏi tìm từ trái nghĩa nên ta chọn phương án D
A. awareness (n): sự nhận thức, sự hiểu biết
B. uncertainty (n): sự không chắc chắn
C. reluctance (n): sự lưỡng lự, sự do dự
D. willingness (n): sự sẵn lòng, sẵn sàng làm điều gì
Tạm dịch: Mọi người trên phạm vi toàn quốc hành động không do dự cung cấp cứu trợ cho nạn nhân ở
vùng bị ảnh hưởng của thảm họa.
Question 32. Đáp án C
Đáp án C đúng vì ta có neglect (v): sao nhãng bỏ bê >< attend to (phrasal verb): chăm sóc, phục vụ.
Đây là câu hỏi tìm từ trái nghĩa nên ta chọn phương án C
A. puts off (phrasal verb): hoãn lại
B. looks for (phrasal verb): tìm kiếm
C. attends to (phrasal verb): chăm sóc, phục vụ, chú tâm
D. approves of (phrasal verb): tán thành, ủng hộ
Tạm dịch: Bất kỳ học sinh nào lơ là việc làm bài tập về nhà sẽ khó có thể học tốt ở trường.
Question 33.
Ta có cụm từ effect on/ upon sb/ st: ảnh hưởng đến ai cái gì.
Back in the 1960s and 1970s, the world was becoming more aware of the destructive effects of industry
on the environment and people were starting to think seriously about ways of protecting the environment.
Tạm dịch: Quay trở lại những năm 1960 và 1970, thế giới ngày càng nhận thức được tác động tàn phá của
ngành công nghiệp đối với môi trường và mọi người bắt đầu suy nghĩ nghiêm túc về các cách bảo vệ môi
trường.
Question 34.
Ta căn cứ vào nghĩa của câu để chọn đáp án

Trang 17
A. worthless (adj): không có giá trị
B. valueless (adj): không có giá trị
C. precious (adj): quý giá
D. priceless (adj): vô giá
He felt that if people could be shown a better way of living then maybe they would be interested enough
to try to protect their priceless environment.
Tạm dịch. Ông cảm thấy rằng nếu mọi người được chỉ ra cách sống tốt hơn thì có thể họ sẽ quan tâm đến
việc bảo vệ môi trường vô giá của họ
Question 35.
Trong trường hợp này, ta cần điền một đại từ quan hệ làm chủ ngữ thay thế cho “a project” là danh từ chỉ
vật. Trong 4 phương án thì chỉ có phương án A là đại từ quan hệ dùng thay thế cho danh từ chỉ người và
vật, thay cho who, whom, which trong mệnh đề quan hệ xác định.
Mr. Morgan-Grenville decided to set up a proj ect that would prove what was happening to our
surroundings and what could be done I about it.
Tạm dịch: Ông Morgan-Grenville đã quyết định lập một dự án chứng minh điều gì đang xảy ra với môi
trường xung quanh chúng ta và những gì chúng ta có thể làm được.
Question 36.
Ta có cấu trúc do harm (to...) làm hại đến...
The main aim of CAT is to search for an ecologically better way of living by using technology which
does no harm to the environment.
Tạm dịch: Mục đích chính của CAT là tìm kiếm một lối sinh thái tốt hơn bằng việc sử dụng công nghệ
không gây hại cho môi
Question 37. Đáp án C
Ở đây ta cần chọn từ nối hai câu văn với nhau. Trong trường hợp này, ta thấy từ phù hợp là moreover do
hai câu văn có quan hệ bổ sung ý nghĩa. Ở câu văn trước ta thấy xuất hiện các hành động mà CAT thực
hiện như là “explore”, “demonstrate”, “point out”. Ở câu văn sau, ta lại thấy xuất hiện thêm một hoạt
động nữa đó là “provide information and advice”. Trong 4 đáp án chỉ có phương án C- moreover. Hơn thế
nữa, ngoài ra- được dùng để bổ sung thêm thông tin.
A. Therefore: Do đó- chỉ quan hệ nguyên nhân, hệ quả
B. However: Tuy nhiên- chỉ quan hệ tương phản, nhượng bộ
D. Although: Mặc dù- chỉ quan hệ tương phản, nhượng bộ
Câu văn trước: One of the most important things CAT did initially was to explore and demonstrate a
wide range of techniques and to point out which ones had the least destructive results on the world around
us.

Trang 18
Tạm dịch: Một trong những điều quan trọng nhất mà ban đầu CAT đã làm đó là để khám phá và thực hiện
một loạt các kĩ thuật và để tìm ra kĩ thuật nào có ít tác hại nhất đến môi trường xung quanh chúng ta.
Câu văn sau: Moreover, CAT provides information and advice to people all over Britain and all over the
world.
Tạm dịch: Hơn nữa, CAT cung cấp thông tin và lời khuyên cho mọi người trên khắp nước Anh và trên
toàn thế giới.
Question 38. Đáp án C
Đây là câu hỏi hỏi ý chính của đoạn văn, ta tìm thấy từ khóa “topic of passage” trong câu hỏi - Chủ đề
của bài đọc là gì?
A. Những cách nộp sính lễ ở các nền văn hóa châu Phi.
B. Những ảnh hưởng của phong tục nộp sính lễ trong xã hội châu Phi.
C. Phong tục nộp sính lễ ở nhiều nền văn hóa châu Phi.
D. Những vấn đề tài chính nảy sinh sau hôn nhân.
Đối với câu hỏi yêu cầu tìm nội dung chính, ý chính hoặc chủ đề của bài đọc, ta cần làm 3 bước sau.
Bước 1. Đọc câu văn đầu tiên của bài đọc, do đây thường chứa nội dung chính- main ideas của cả văn
bản.
The custom of paying a bride price before marriage is still a well-established part of many African
cultures.
Tạm dịch. Phong tục thách cưới trước hôn nhân vẫn là một phần tồn tại lâu dài trong nhiều nền văn hóa
châu Phi
Như vậy ban đầu ta có thể thấy nội dung chính của văn bản này đó là nói đến sự tồn tại lâu dài của phong
tục thách cưới trong nhiều nền văn hóa châu Phi
Bước 2. Đọc nhanh toàn bộ văn bản. Đây là thời gian giúp các em được cung cấp thêm thông tin và xác
định xem liệu đoạn văn có nói đến chủ đề mà mình dự đoán khi đọc câu đầu hay không. Trong văn bản
này, chúng ta thấy xuyên suốt văn bản xuất hiện rất nhiều các địa danh và nền văn hóa như là Zulu,
Swazi.., như vậy suy đoán ban đầu của ta đã chính xác
Bước 3. Đọc câu hỏi, các phương án lựa chọn; loại phương án sai và chọn phương án đúng. Trong 4
phương án cung cấp, chúng ta thấy chỉ có phương án C là phù hợp nhất với phân tích. Phương án A sai vì
cách nộp sính lễ chỉ được nhắc đến trong một số câu văn ở đoạn 1. Phương án B sai vì những ảnh hưởng
của phong tục này chỉ là nội dung chính của một đoạn trong văn bản - đoạn 3, không phải nội dung của
toàn bộ văn bản. Phương án D không chính xác do “problems” được đề cập đến trong một phần nhỏ của
đoạn văn số 3, không phải toàn bộ văn bản

Trang 19
Những câu hỏi ý chính thường là những câu hỏi đòi hỏi chúng ta phải nắm được nội dung chủ đạo của
toàn bộ bài văn, vì vậy để tiết kiệm thời gian, chúng ta nên để làm sau, sau khi chúng ta đã nắm sơ qua
nội dung của văn bản nhờ việc trả lời các câu hỏi chi tiết khác.
Question 39.
Câu hỏi. Từ “prominent” gần nghĩa nhất với từ nào?
A. famous (adj): nổi tiếng
B. impoverished (adj): rất nghèo
C. two-parent (adj): có cả cha lẫn mẹ
D. affluent (adj): giàu có
Đây là một câu hỏi liên quan đến từ vựng, với câu hỏi này chúng ta chỉ cần đọc thông tin xung quanh từ
vựng đã cho là có thể tìm ra câu trả lời: In modern times, the bride price is occasionally quite small and
its value is mainly symbolic. However, the bride price can still be quite high, especially among prominent
or highly traditional families.
Tạm dịch: Trong thời hiện đại, tiền thách cưới đôi khi khá nhỏ và chủ yếu mang giá trị biểu tượng. Tuy
nhiên, tiền thách cưới vẫn có thể khá cao, đặc biệt là trong các gia đình nổi tiếng hoặc lâu đời.
Từ “prominent” trong trường hợp này gần nghĩa nhất với phương án A. prominent (adj): nổi tiếng, quan
trọng
Question 40. Đáp án D
Câu hỏi. Từ “It” trong đoạn 2 đề cập đến_____________
A. Sự biện minh
B. Chi phí để nuôi lớn cô dâu
C. Sự công nhận
D. Tiền thách cưới
Đây là câu hỏi suy luận liên quan đến đại từ nhân xưng- từ dùng để thay thế cho danh từ đã xuất hiện
trước đó vì không muốn nhắc lại danh từ ấy. Như vậy ta chỉ cần đọc câu văn xuất hiện phía trước là có thể
tìm ra đáp án.
The first is that the bride price represents an acknowledgement of the expense the brides family has gone
in order to raise her and bring her up as a suitable bride for the groom. It also represents payment for the
loss of a family member, since the bride will officially become a member of her husbands family and will
leave her own.
Tạm dịch: Đầu tiên là tiền thách cưới thể hiện cho sự thừa nhận những chi phí của gia đình cô dâu đã bỏ
ra để nuôi lớn chăm bẵm cô để trở thành 1 cô dâu phù hợp cho chú rể. Nó cũng tượng trưng cho tiền bồi
thường vì mất một thành viên trong gia đình, vì cô dâu sẽ chính thức trở thành thành viên của gia đình
chồng, và rời khỏi gia đình mình.

Trang 20
Nhìn vào phần thông tin được gạch chân, chú ý các cụm từ như “represent” và “also represent”, ta thấy
chỉ có phương án D là chính xác
Question 41.
Câu hỏi. Có thể suy luận từ đoạn 2 rằng____________
A. Sau khi kết hôn, phụ nữ sẽ không bao giờ gặp lại cha mẹ họ
B. Gia đình chú rể nộp sính lễ vào ngày cưới.
C. Xã hội châu Phi đánh giá giá trị của nam giới cao hơn phụ nữ
D. Các gia đình châu Phi rất coi trọng con trẻ.
Đây là câu hỏi liên quan đến đoạn văn 2 nên khi đọc ta sẽ chỉ tập trung vào đoạn văn này. Ta thấy trong
đoạn 2, dòng 4-7 có viết. On a deeper level the bride price represents payment for the fact that the bride
will bring children into the family of the groom, thereby increasing the wealth of the family. This concept
is reinforced by the fact that the bride price must often be returned if the bride fails to bear children.
Tạm dịch: Ở một mức độ sâu xa hơn, tiền thách cưới là khoản thanh toán cho thực tế là cô dâu sẽ sinh con
cái cho gia đình chú rể, qua đó làm gia tăng sự giàu có của gia đình. Quan niệm này được chứng minh bởi
thực tế tiền thách cưới thường phải được trả lại nếu cô dâu không sinh được con.
Dựa vào phần thông tin cuối cùng “bride price must often be returned if the bride fails to bear children” -
tiền thách cưới thường được trả lại nếu cô dâu không sinh được con
→ chỉ có đáp án D là phù hợp
Question 42. Đáp án A
Câu hỏi. Khẳng định nào sau đây là không đúng theo bài đọc?
A. Do có tục nộp tiền thách cưới, đàn ông châu Phi không tôn trọng các thành viên trong gia đình họ.
B. Các cuộc thương lượng đầu tiên về tiền thách cưới tạo cơ hội cho hai gia đình hiểu nhau.
C. Khi nộp tiền thách cưới có thể nộp bằng động vật.
D. Gia đình của cô dâu phải trả lại tiển thách cưới cho nhà chú rể trong trường hợp cặp đôi không có con
hoặc li dị.
Đây là câu hỏi lựa chọn- cũng là dạng câu hỏi ta nên để làm sau cùng, sau khi đã hiểu nội dung chính
cũng như định vị được các vùng thông tin.
Chúng ta thấy trong đoạn 3, dòng 2-4 viết. Sons are dependent on their fathers and older relatives to help
them pay the bride price of their wives, and this generally leads to greater levels of obedience and respect.
Tạm dịch: Con trai phụ thuộc vào cha và những người thân lớn tuổi để giúp họ trả tiền thách cưới để lấy
vợ, và điều này thường khiến con cái ngoan ngoãn và tôn trọng cha mẹ hơn. Cụm từ “leads to greater
levels of obedience and respect”- khiến con cái ngoan ngoãn và tôn trọng cha mẹ hơn, giúp chúng ta xác
định đáp án A là đáp án không đúng theo bài đọc
Question 43. Đáp án B

Trang 21
Câu hỏi. Tiêu đề phù hợp nhất cho bài đọc là gì?
A. Định nghĩa về “new media”
B. Những công việc liên quan đến Internet
C. Công việc của các quản trị web.
D. Những người có kĩ năng về Internet Đây là câu hỏi đòi hỏi chúng ta cần đọc toàn bộ bài, chú ý vào câu
văn đầu tiên mỗi đoạn văn do tiêu đề thường chứa đựng nội dung chủ đề chính của toàn bài và nên để trả
lời sau cùng. Trong văn bản này chúng ta thấy xuất hiện các cụm từ như webmaster (n)- quản trị web,
content jobs (n)- công việc viết nội dung, freelance workers online (n)- những người làm việc trực
tuyến...như vậy ta thấy tiêu đề phù hợp nhất chính là đáp án B. Các phương án A, C, D đều được nhắc đến
trong bài nhưng chỉ là ý nhỏ, không bao quát được toàn bộ bài đọc
Question 44. Đáp án A
Câu hỏi. Theo đoạn văn, điều nào sau đây đúng về quản trị web?
A. Họ làm việc độc lập hoặc có sự cộng tác
B. Các nhiệm vụ mà họ thực hiện hầu như không thay đổi
C. Nghề của họ đòi hỏi trình độ chuyên môn tối thiểu
D. Họ hiếm khi ủng hộ các sản phẩm phần cứng và phần mềm
Ta tìm thấy thông tin câu trả lời ở đoạn 2, dòng 4-5: Finally, the responsibilities of a webmaster also
depend on whether he or she will be working independently, or whether the firm will provide people to
help.
Tạm dịch: Cuối cùng, trách nhiệm của một quản trị viên web cũng phụ thuộc vào việc họ làm việc độc lập
hay công ty sẽ cử người hỗ trợ.
Cặp liên từ “whether.. .or” cho thấy phương án lựa chọn, cụm từ “provide people to help” động nghĩa với
trạng từ collaboratively - cộng tác trong phương án A
Question 45. Đáp án A
Câu hỏi. Từ “identify” ở đoạn 2 gần nghĩa nhất với____________
A. find out (phrasal verb): hỏi/ đọc để có được thông tin
B. pick up on (phrasal verb): phát hiện
C. come across (phrasal verb): đụng phải, vô tình gặp
D. look into (phrasal verb): kiểm tra
Ta tìm thấy thông tin chứa từ vựng này trong đoạn 2, dòng 1-2. To specify the job description of a
webmaster, one needs to identify the hardware and software that the website the webmaster will manage
is running on.

Trang 22
Tạm dịch: Để xác định mô tả công việc của một quản trị web, người ta cần xác định xem website mà
quản trị viên sẽ quản lý đang chạy phần cứng và phần mềm nào. Như vậy thì đáp án cho câu hỏi này chỉ
có thể là đáp án A
Question 46. Đáp án C
Câu hỏi. Từ “them” ở đoạn 2 ám chỉ____________
A. websites (n): trang web
B. tasks (n): công việc, nhiệm vụ
C. hardware and software (n. ph.): phần cứng và phần mềm
D. skill sets (n. ph.): bộ kĩ năng
Thông tin nằm trong đoạn 2, dòng 1-3: To specify the job description of a webmaster, one needs to
identify the hardware and software that the website the webmaster will manage is running on. Different
types of hardware and software require different skill sets to manage them.
Tạm dịch: Để xác định mô tả công việc của một quản trị web, người ta cần xác định xem website mà
quản trị viên sẽ quản lý đang chạy phần cứng và phần mềm nào. Các loại phần cứng và phần mềm khác
nhau đòi hỏi những bộ kĩ năng khác nhau để quản lý chúng.
Như vậy có thể thấy tân ngữ “them” trong trường hợp này để thay thế cho “phần cứng và phần mềm”-
đáp án C
Question 47.
Câu hỏi. Điều nào sau đây không đúng khi nói về công việc của các nhà sáng tạo nội dung tự do?
A. Nó có thể bao gồm các công việc liên quan đến các công ty nước ngoài
B. Nó được coi là công việc “làm nội dung”
C. Việc này đòi hỏi những kĩ năng truyền thống
D. Nó yêu cầu các kiến thức chuyên sâu về ứng dụng máy tính
Với câu hỏi lựa chọn này, ta sẽ dùng phép loại trừ. Phương án B, C được để cập đến trong đoạn văn 3,
dòng 2-3: However, there are also online jobs available for which traditional skills remain in high
demand. Content jobs require excellent writing skills and a good sense of the web as a “new media”.
Tạm dịch. Tuy nhiên cũng có nhiều công việc trực tuyến đòi hỏi những kĩ năng truyền thống. Các công
việc sáng tạo nội dung hay viết lách đòi hỏi kĩ năng viết tốt và có kiến thức về web như một “phương tiện
truyền thông mới”.
Phương án A được đề cập trong đoạn 5, dòng 4-5: An added benefit to such online jobs is that freelancers
are able to work on projects with companies outside their own country of residence.
Tạm dịch. Một lợi ích nữa cho các công việc trực tuyến đó là những người làm nghề tự do có thể làm các
dự án cho các công ty nước ngoài.

Trang 23
Phương án D không đúng do công việc đòi hỏi kiến thức chuyên sâu về máy tính là quản trị web, thông
tin đề cập trong đoạn 3, dòng 1: Webmaster is one type of Internet career requiring in-depth knowledge of
the latest computer applications.
Tạm dịch. Quản trị web là một loại nghề nghiệp trên Internet đòi hỏi kiến thức chuyên sâu về ứng dụng
máy tính mới nhất.
Question 48. Đáp án A
Câu hỏi. Từ “downsize” trong đoạn văn 5 gần nghĩa nhất với____________.
A. Sa thải nhân viên
B. Cắt giảm lương
C. Phá sản
D. Kiếm được hợp đồng mới
Ta tìm thấy thông tin trong đoạn 5, dòng 1 -3: Additionally, many of today s Internet careers are
becoming paid-by-the-job professions. With many companies having to downsize in tough economic
items, the outsourcing and contracting of freelance workers online has become common business practice.
Tạm dịch. Ngoài ra, nhiều nghề nghiệp trên Internet ngày nay đang trở thành nghề trả lương theo công
việc. Với nhiều công ty phải giảm bớt nhân viên do kinh tế khó khăn, việc gia công và kí kết hợp đồng
lao động tự do trực tuyến đã trở thành thực tiễn kinh doanh phổ biến.
Trong đoạn trên ta thấy có cụm “in tough economic items”- trong khi kinh tế khó khăn, theo logic thì khi
kinh tế khó khăn, các công ty thường cắt giảm nhân viên → đáp án A đúng
Question 49. Đáp án B
Câu hỏi. Theo đoạn văn, tất cả những điều sau đây đều đúng, trừ_____________
A. Rất khó để đưa ra 1 bản mô tả công việc cho nghề quản trị web
B. Một quản trị web không cần cập nhật kiến thức về khoa học máy tính
C. “Phương tiện truyền thông mới” không bao gồm kĩ năng viết
D. Những công việc trực tuyến cho người lao động có kĩ năng tối thiểu về máy tính luôn có sẵn
Ta tìm thấy thông tin trong đoạn 3, dòng 1: Webmaster is one type of Internet career requiring in-depth
knowledge of the latest computer applications.
Tạm dịch. Quản trị web là một loại nghề nghiệp trên Internet đòi hỏi kiến thức chuyên sâu về ứng dụng
máy tính mới nhất. Như vậy có thể thấy nội dung của câu B là không đúng
Question 50. Đáp án B
Câu hỏi. Có thể suy luận ra từ đoạn văn là____________
A. Chỉ có nhân viên được đào tạo bài bản mới được khuyên nên làm việc trực tuyến
B. Nhân viên trực tuyến có thể làm việc toàn thời gian trên mạng
C. Để thành một quản trị web thì thật sự rất dễ

Trang 24
D. Nhân viên có kĩ năng máy tính hạn chế không kiếm được nhiều tiền.
Phương án A không chính xác vì ta thấy thông tin ngay câu đầu tiên của văn bản: Contrary to popular
belief, one does not have to be trained programmer to work online.
Tạm dịch. Trái ngược với điều mọi người vẫn nghĩ, không phải cứ phải là một lập trình viên được đào tạo
bài bản mới có thể làm việc trực tuyến.
Phương án C sai vì ta thấy thông tin được nhắc đến trong đoạn 3, dòng 1: Webmaster is one type of
Internet career requiring in- depth knowledge of the latest computer applications.
Tạm dịch. Quản trị web là một loại nghề nghiệp trên Internet đòi hỏi kiến thức chuyên sâu về ứng dụng
máy tính mới nhất. Như vậy thì đây không phải là một công việc đơn giản, dễ làm.
Phương án D sai vì ta tìm thấy thông tin trong đoạn cuối, dòng 1-2: How much can a person make in
these kinds of career? As with many questions related to today’s evolving technology, there is no simple
answer.
Tạm dịch. Một người có thể kiếm được bao nhiêu khi làm những công việc này? Cũng như nhiều câu hỏi
liên quan đến sự phát triển của công nghệ ngày nay, không phải đơn giản để có thể trả lời. Như vậy có
nghĩa với các công việc không đòi hỏi nhiều kĩ năng hay kiến thức về công nghệ ứng dụng máy tính thì
nhân viên cũng có cơ hội kiếm nhiều tiền. Dùng phép loại trừ thì ta chọn được phương án B

Trang 25
ĐỀ SỐ 17 ĐỀ THI THỬ TỐT NGHIỆP THPT
NĂM HỌC: 2020 – 2021
MÔN: TIẾNG ANH
Thời gian làm bài: 60 phút; không kể thời gian phát đề

Mark the letter A, B, C, or D on your answer sheet to indicate the -word whose underlined part
differs from the other three in pronunciation in each of the following questions.
Question 1. A. comb B. come C. dome D. home
Question 2. A. volunteer B. trust C. fuss D. judge
Mark the letter A, B, C, or D on your answer sheet to indicate the word that differs from the other
three in the position of primary stress in each of the following questions.
Question 3. A. sleepy B. trophy C. facial D. exact
Question 4. A. authority B. necessity C. academic D. commercially
Mark the letter A, B, C, or D on your answer sheet to indicate the correct answer to each of the
following questions.
Question 5. He was not offered the job because he knew____________about Information Technology.
A. a little B. little C. many D. a few
Question 6. Graham was disappointed because he____________for the bus for an hour.
A. was waiting B. waited C. has been waiting D. had been waiting
Question 7. Sally hoped____________to join the private club. She could make important business
contacts there.
A. inviting B. being invited C. to invite D. to be invited
Question 8. ____________the homework, he was allowed to go out with his friends.
A. finishing B. finish C. to finish D. having finished
Question 9. The country is rapidly losing its workers as____________people are emigrating.
A. the most and the most B. the more and the more
C. more and more D. most and most
Question 10. It is absolutely essential that she____________ head office in advance.
A. arrive B. arrives C. will arrive D. must arrive
Question 11. We have just visited disadvantaged children in an orphanage____________in Bac Ninh
Province.
A. located B. locating C. which locates D. to locate
Question 12. Peter works hard at everything he does. His brother, ____________, seldom puts out
much effort.
A. on the other hand B. otherwise C. furthermore D. consequently

Trang 1
Question 13. John's____________and efficiency at the company led to his promotion to Sales
Manager.
A. punctuality B. punctual C. punctuate D. punctually
Question 14. We truly respected our father and always____________by his rule.
A. submitted B. obeyed C. complied D. abode
Question 15. We were quite impressed by the____________students who came up with the answer to
our question almost instantly.
A. absent-minded B. big-headed C. quick-witted D. bad-tempered
Question 16. I am sure your sister will lend you a sympathetic____________when you explain the
situation to her.
A. eye B. ear C. arm D. finger
Question 17. My cousin was nervous about being interviewed on television, but he____________to the
occasion wonderfully.
A. raised B. rose C. fell D. faced
Question 18. You looked exhausted. I think you’ve____________more than you can handle.
A. turned on B. taken up C. turned up D. taken on

Mark A, B, C, or D on your answer sheet to indicate the underlined part that needs correction in
each of the following questions.
Question 19. The more tired you are, the more hard you can concentrate.
A. more tired B. you are C. more hard D. concentrate
Question 20. The ocean probably distinguishes the earth from other planets of the solar system, for
scientists believe that large bodies of water are not existing on the other planets.
A. probably B. for C. are not existing D. from
Question 21. There were considerate amounts of money wasted on large building projects.
A. considerate B. amounts C. wasted D. building
Mark the letter A, B, C, or D on your answer sheet to indicate the sentence that is closest in
meaning to each of the following sentences.
Question 22. I thought I should not have stayed at home yesterday.
A. I regretted staying at home yesterday. B. I regretted for staying at home yesterday.
C. I regret for staying at home yesterday. D. I regret to stay at home yesterday.
Question 23. They don’t let the workers use the office telephone for personal calls.
A. They don’t allow using the office telephone to call personal secretaries.
B. They don’t allow workers to use the office telephone.
C. The office telephone is used by workers personally.
D. They don’t let the office telephone be used for personal purpose by workers

Trang 2
Question 24. “ You should have finished the report by now.” John told his secretary.
A. John reproached his secretary for not having finished the report.
B. John said that his secretary had not finished the report.
C. John reminded his secretary of finishing the report.
D. John scolded his secretary for not having finished the report.
Mark the letter A, B, C, or Don your answer sheet to indicate the sentence that best combines
each pair of sentences in the following questions
Question 25. He hadn’t eaten anything since morning. He took the food eagerly.
A. He took the food eagerly although he had eaten a lot since dawn.
B. He took the food eagerly for he had eaten nothing since dawn.
C. He had eaten nothing since dawn so that he took the food eagerly.
D. He took the food so eagerly that he had eaten nothing since dawn.
Question 26. Julie had a training course in alternative medicine. She was able to help the man out
of danger.
A. Much as Julie had a training course in alternative medicine, she was able to help the man out of
danger.
B. Having trained in alternative medicine, Julie was able to help the man out of danger.
C. But for a training course in alternative medicine, Julie could have helped the man out of danger.
D. Despite training in alternative medicine, Julie was able to help the man out of danger.
Mark the letter A, B, C or D on your answer sheet to indicate the most suitable response to
complete each of the following exchanges.
Question 27. Maria and Diana are talking about the evening.
Maria: “ Thanks for the lovely evening”
Diana: “____________”
A. I’m glad you enjoyed it B. Oh, That’s right
C. No, it’s not good D. Yes, it’s really great
Question 28. Tom came late for the meeting with Barbara.
Tom: “ Sorry, I’m late. The traffic was terrible”
Barbara: “____________”
A. My pleasure B. Don’t worry.
C. I wish I could but I’m sorry D. Well, let me see.
Mark the letter A, B, C or D on your answer sheet to indicate the word(s) CLOSEST in meaning
to the underlined word(s) in each of the following questions.
Question 29. Since the death of Laura's father, her mother has become a breadwinner to support the
family.
A. a person who bakes bread every morning

Trang 3
B. a bakery-owner
C. a person who delivers bread to make money
D. a person who goes out to work to earn money
Question 30. Peter is the black sheep of the family, so he is never welcomed there.
A. a beloved member B. a bad and embarrassing member
C. the only child D. the eldest child
Mark the letter A, B, C or D on your answer sheet to indicate the sentence that is OPPOSITE in
meaning to each of the following questions.
Question 31. The loss of his journals had caused him even more sorrow than his retirement from the
military six years earlier.
A. grief B. joy C. comfort D. sympathy
Question 32. As a newspaper reporter, she always wanted to get information at first hand
A. indirectly B. directly C. easily D. slowly
Read the following passage and mark the letter A, B, C, or D on your answer sheet to indicate the
correct word or phrases that best fits each of the numbered blanks.
SPORTS IN SOCIETY
The position of sport in today's society has changed out of all recognition. People no longer seem to
think of sports as ‘just a game’ - to be watched or played for the (33) ____________of enjoyment.
Instead, it has become big business worldwide. It has become accepted practice for leading companies
to provide sponsorship. TV companies pay large sums of money to screen important matches or
competitions. The result has been huge rewards for athletes, some of (34) ____________are now very
wealthy, particularly top footballers, golfers and tennis players. (35) ____________, it is not unusual
for some athletes to receive large fees on top of their salary, for advertising products or making
personal appearances.
A trend towards shorter working hours means that people generally tend to have more free time, both to
watch and to take in sporting activity; sport has become a significant part of the recreation industry that
we now rely (36) ____________to fill our leisure hours. Professional sport is a vital part of that
industry, providing for millions of (37) ____________people all over the world.
Question 33. A. advantage B. good C. benefit D. sake
Question 34. A. whose B. whom C. who D. that
Question 35. A. In addition B. However C. In contrast D. Therefore
Question 36. A. for B. with C. on D. in
Question 37. A. ordinary B. mighty C. extremist D. abnormal
Read the following passage and choose A, B, C or D to indicate the correct answer to each of the
following questions.

Trang 4
In many ways, the increasingly rapid pace of climate change is a direct result of the growth of the
human population. In the last 100 years, the world population has more than tripled, from just under 2
billion at the beginning of the century to nearly 7 billion today. In addition, the average person uses
more energy and natural resources than the average person one hundred years ago, meaning that the
rates of consumption are actually much higher than just the increase in population would imply. For
example, it took the world 125 years to use the first one trillion barrels of oil. The next trillion barrels
will be used in less than 30 years, which is almost 5 times as fast, not three.
All of these activities: food production, energy usage, and the use of natural resources, contribute to
climate change in some way. The greater amounts of oil and other fuels burned to create energy release
chemicals which add to global warming. In order to produce more food, farmers cut down trees to gain
more land for their fields. In addition, we cut down trees to build the houses needed for a larger
population. Those trees are an essential part of controlling global warming; others are too numerous to
mention.
In addition to a growing population, the world also has a population that desires a higher standard of
living than in the past, and a higher standard of living requires the use of even more natural resources.
A look at one country will provide a clear example of this fact. China is the worlds most populous
nation, with 1.3 billion people. Currently, the standard of living for most of those people is far below
that of people in first world nations. Therefore, the average Chinese citizen uses far fewer natural
resources and less energy than the average citizen of the US or Japan. But China is growing in power,
and more of its citizens are beginning to expect a first world lifestyle. If every Chinese person attains a
first world lifestyle, the amount of energy and natural resources needed in the world will double, even if
the standard of living in every other nation on Earth remains the same as it is today.
Question 38. How many years did it take the world years to use the first one trillion barrels of oil?
A. 100 years B. 125 years C. 30 years D. 7 years
Question 39. The word “consumption” in the passage is closest in meaning to___________.
A. development B. usage C. population D. increase
Question 40. According to the passage, which of these activities does NOT contribute to climate
change in some way?
A. food production B. energy usage
C. wild animals hunting D. natural resources consumption
Question 41. According to the passage, how does food production contribute to global warming?
A. Producing more food leads to growth in the world population.
B. Food production uses many chemicals which add to global warming.
C. Food production requires that the forests be cleared to create farmland.
D. Food production decreases the ability of the air to release heat..
Question 42. According to the passage, how does the standard of living affect global warming?

Trang 5
A. Higher standards of living are better for the environment.
B. First world nations create less population than developing nations.
C. The use of natural resources is directly related to the standard of living.
D. High standards of living lead to increases in world population.
Read the following passage and choose the letter A, B, C, or D to indicate the answer to each of the
following questions.
Carnegie Hall, the famous concert hall in New York City, has again undergone a restoration. While this is
not the first, it is certainly the most extensive in the building’s history. As a result of this new restoration,
Carnegie Hall once again has the quality of sound that it had when it was first built.
Carnegie Hall owes its existence to Andrew Carnegie, the wealthy owner of a steel company in the late
1800s. The hall was finished in 1891 and quickly gained a reputation as an excellent performing arts hall
where accomplished musicians gained fame. Despite its reputation, however, the concert hall suffered
from several detrimental renovations over the years. During the Great Depression, when fewer people
could afford to attend performances, the directors sold part of the building to commercial businesses. As a
result, a coffee shop was opened in one corner of the building, for which the builders replaced the brick
and terra cotta walls with windowpanes. A renovation in 1946 seriously damaged the acoustical quality of
the hall when the makers of the film Carnegie Hall cut a gaping hole in the dome of the ceiling to allow
for lights and air vents. The hole was later covered with short curtains and a fake ceiling
but the hall never sounded the same afterwards.
In 1960, the violinist Isaac Stern became involved in restoring the hall after a group of real estate
developers unveiled plans to demolish Carnegie Hall and build a high-rise office building on the site.
This threat spurred Stern to rally public support for Carnegie Hall and encourage the City of New York
to buy the property. The movement was successful, and the concert hall is now owned by the city. In
the current restoration, builders tested each new material for its sound qualities, and they replaced the
hole in the ceiling with a dome. The builders also restored the outer walls to their original appearance
and closed the coffee shop. Carnegie has never sounded better, and its prospects for the future have
never looked more promising.
Question 43. This passage is mainly about___________.
A. changes to Carnegie Hall
B. the appearance of Carnegie Hall
C. Carnegie Hall’s history during the Great Depression
D. damage to the ceiling in Carnegie Hall
Question 44. The word “it” in the first paragraph refers to___________.
A. Carnegie Hall B. New York City C. a restoration D. a plan
Question 45. what major change happened to the hall in 1946?
A. The acoustic dome was damaged.

Trang 6
B. Space in the building was sold to commercial businesses.
C. The walls were damaged in an earthquake.
D. The stage was renovated.
Question 46. Who was Andrew Carnegie?
A. A violinist B. An architect
C. A steel mill owner D. Mayor of New York City
Question 47. What was Isaac Stern’s relationship to Carnegie Hall?
A. He made the movie “Carnegie Hall” in 1946.
B. He performed on opening night in 1891.
C. He tried to save the hall, beginning in 1960.
D. He opened a coffee shop in Carnegie Hall during the Depression
Question 48. which of the following is closest in meaning to the word detrimental in paragraph 2?
A. dangerous B. trivial C. impressive D. damaging
Question 49. which of the following is closest in meaning to the word “unveiled” in paragraph 3?
A. announced B. restricted C. overshadowed D. located
Question 50. How does the author seem to feel about the future of Carnegie Hall?
A. ambiguous B. guarded C. optimistic D. negative
Đáp án
1-B 2-A 3-D 4-C 5-B 6-D 7-D 8-D 9-C 10-A
11-A 12-A 13-A 14-D 15-C 16-B 17-B 18-B 19-C 20-C
21-A 22-A 23-D 24-A 25-B 26-B 27-A 28-B 29-D 30-B
31-B 32-A 33-D 34-B 35-A 36-C 37-A 38-B 39-B 40-C
41-C 42-C 43-A 44-C 45-A 46-C 47-C 48-D 49-A 50-C

LỜI GIẢI CHI TIẾT


Question 1: Đáp án B
Phần gạch chân của đáp án B đọc là /ʌ/ còn phần gạch chân của các phương án còn lại đọc là âm /əʊ/
A. comb /kəʊm/ (n/v): cái lược/chải đầu
B. come /kʌm/ (v): đi đến, đi tới
C. dome /dəʊm/ (n): mái vòm
D. home /həʊm/ (n): nhà
Lưu ý: Khi tra từ điển bạn thấy /əʊ/ hay /oʊ/ thì đều là một âm và cách đọc giống nhau.
Question 2. Đáp án A
Phần gạch chân của đáp án A đọc là âm /ə/, còn phần gạch chân của các phương án còn lại được đọc là
âm /ʌ/.
A. volunteer /ˌvɒlənˈtɪə(r)/ (n/v): sự tình nguyện, tình nguyện viên/ tình nguyện

Trang 7
B. trust /trʌst/ (n/v): sự tin tưởng/ tin tưởng
C. fuss /fʌs/ (n): sự kích động, gây rối
D. judge /dʒʌdʒ/ (v): đánh giá
Question 3. Đáp án D
Trọng âm đáp án D rơi vào âm tiết số hai, còn các phương án còn lại trọng âm rơi vào âm thứ nhất.
A. sleepy /ˈsliːpi/ (adj): buồn ngủ
B. trophy /ˈtrəʊfi/ /'troʊfi/ (n): chiếc cúp
C. facial /ˈfeɪʃl/ (adj): thuộc về khuôn mặt
D. exact /ɪɡˈzækt/ (adj): chính xác
Lưu ý: Thường thì danh từ và tính từ có hai âm tiết trọng âm rơi hầu hết vào số một, tuy nhiên ở đây
“exact” là trường hợp ngoại lệ, cần làm nhiều và tìm tòi bổ sung các trường hợp ngoại lệ khác nữa như
các tính từ: polite, alone, alive danh từ: machine, mistake, police, idea...
Question 4. Đáp án C
Trọng âm đáp án C rơi vào âm tiết số 3, còn các phương án còn lại trọng âm rơi số hai.
A. authority /ɔːˈθɒrəti/ (n): quyền lực, có thẩm quyền
B. necessity /nəˈsesəti/ (n): sự cần thiết
C. academic /ˌækəˈdemɪk/ (adj): có tính chất học thuật, (thuộc) học viện, (thuộc) trường đại học
D. commercially /kəˈmɜːʃəli/ (adv): về phương diện thương mại
Lưu ý: Đuôi ty, gy, cy, phy, fy thường là các duỗi trọng âm sẽ rơi âm tiết số ba tính từ dưới lên. Ta có
đuôi “ate” cũng vậy.
Đuôi “ic”,“cial”,“tial”,“tion”,“sion”, “cian” trọng âm rơi âm tiết ngay trước.
Question 5.
Kiến thức được hỏi: Lượng từ
Trước hết ta đi tìm hiểu cách dùng của các phương án trên.
“many”: nhiều; đi với danh từ số nhiều đếm được:
a few / few / a little / little
- A few và few đi với danh từ số nhiều đếm được
- A little và little đi với danh từ không đếm được.
- A few và a little mang chiều hướng tích cực/ khẳng định “ có ít nhưng vẫn đủ dùng”
- Few và little mang chiều hướng phủ định “quá ít không đủ dùng”
Eg. I have a few foreign friends, (có một vài người bạn)
She feels lonely because she has (too) few friends, (có quá ít bạn bè)
I have a little money to buy books, (có ít tiền nhưng đủ để mua sách)
Sorry, I have little money to buy an ice cream, (có quá ít tiền không đủ để mua kem).
* Ngoài ra, chúng ta còn biết thêm “a little” còn là một đại từ và “little” vừa là đại từ vừa là trạng từ.

Trang 8
Trong câu trên “knew” đóng vai trò là một nội động từ nên ta loại tất cả các trường hợp là đại từ đi (sau
nội động từ không có tân ngữ) mà phải dùng một trạng từ ở chỗ trống này, như vậy chỉ có “little” là phù
hợp nhất, đóng vai trò là trạng từ trong câu này
Tạm dịch. Anh ấy không được nhận việc đó bởi anh ta biết quá ít về Công nghệ thông tin.
Question 6. Đáp án D
Kiến thức được hỏi: Thì động từ
Ta dùng thì quá khứ hoàn thành tiếp diễn để diễn tả một hành động xảy ra trước một hành động/một thời
điểm khác trong quá khứ và nhấn mạnh hành động đó đã xảy ra được bao lâu rôi hành động kia mới xảy
ra.
Đáp án D - had been waiting.
Tạm dịch: Graham rất thất vọng vì đứng đợi xe bus cả tiếng đồng hồ.
Question 7. Đáp án D
Kiến thức: Bị động với động từ theo sau là To V.
Ta biết động từ hope + to V, thêm nữa ta cần xác định thêm có yếu tố bị động trong câu này nên sử dụng
cấu trúc hope + to be P2. Điều này tương ứng đáp án D.
Tạm dịch: Sally hi vọng sẽ được mời tham dự câu lạc bộ kín đó. Cô ấy sẽ có thể có được những mối quan
hệ quan trọng cho công việc.
Question 8. Đáp án D
Kiến thức được hỏi: Rút gọn mệnh đề cùng chủ ngữ dùng Having P2.
Khi hai mệnh đề cùng chủ ngữ, và câu muốn nhấn mạnh hành động phía trước được hoàn thành xong
trước rồi hành động phía sau mới xảy ra thì chúng ta dùng công thức: Having +P2, S+Ved. Đáp án D.
Tạm dịch: Sau khi hoàn thành xong bài tập về nhà, anh ấy được phép đi chơi với những người bạn của
mình.
Question 9. Đáp án C
Kiến thức được hỏi: so sánh kép
More and more + N: càng ngày càng nhiều cái gì
More and more + adj: càng ngày càng thế nào
Trong các phương án duy nhất C đúng.
Tạm dịch: Đất nước đang dần dần bị sụt giảm số lượng người lao động vì ngày càng nhiều người di cư.
Question 10: Đáp án A
Kiến thức được hỏi: Thức giả định. Nhận dạng cấu trúc câu thức giả định.
It is + Adj+ that+ S + V (động từ nguyên thể không to)
Tính từ thể hiện độ cần thiết và quan trọng: important, necessary, essential, vital, crucial, imperative,
urgent... Trong câu có chứa từ khóa “essential” nên dùng “arrive” làm đáp án. Chọn A.
Tạm dịch: Việc cô ấy phải sớm đến trụ sở chính là vô cùng quan trọng.
Question 11. Đáp án D

Trang 9
Kiến thức được hỏi: Rút gọn mệnh đề quan hệ.
Mệnh đề chủ động N + Ving (lược bỏ ĐTQH, chuyển V chính sang dạng Ving)
Mệnh đề bị động N + P2 (lược bỏ ĐTQH và to be, giữ động từ P2)
Khi nói cái gì tọa lạc/ nằm ở đâu ta dùng: Sth be located in/at somewhere. Câu này viết đầy đủ là: .. .in an
orphanage which is located in Bac Ninh Province. Dùng rút gọn bị động ta có: ...in an orphanage located
in Bac Ninh Province
Đáp án A. located
Tạm dịch: Chúng tôi vừa mới đến thăm các trẻ em bị thiệt thòi ở một trại trẻ mồ côi ở tỉnh Bắc Ninh.
Question 12. Đáp án A
Kiến thức được hỏi. Từ nối
A. on the other hand: mặt khác (nối các câu, mệnh đề nghĩa đối lập)
B. otherwise: nếu không thì
C. furthermore: hơn nữa, nối các câu, mệnh đề ý bổ sung thông tin
D. consequently: kết quả là, hậu quả là
Theo ý dịch câu chúng ta cần từ nối ý tương phản nên chọn A.
Tạm dịch: Peter luôn làm việc chăm chỉ trong mọi thứ anh ta làm. Anh trai anh ấy thì ngược lại, chẳng
mấy khi chịu nỗ lực gì cả.
Question 13. Đáp án A
Kiến thức được hỏi: Loại từ
punctuality (n): sự đúng giờ
punctual (adj): đúng giờ
punctuate (v): đánh dấu chấm câu
punctually (adv): một cách đúng giờ
Vị trí chỗ trống này cần một danh từ, vì phía trước có sở hữu cách Johns và phía sau sau chữ “and” là
một danh từ. Vì vậy chọn đáp án A.
Tạm dịch: Sự đúng giờ và làm việc hiệu quả của John ở công ty đã giúp anh ấy được thăng chức trở thành
Giám đốc kinh doanh.
Question 14. Đáp án D
Kiến thức được hỏi: Lựa chọn từ.
submit (v): nộp
obey (somebody/something): tuân theo
comply (with something): tuân theo
abide by something: tuân theo (quá khứ là “abode”).
Vì vậy đáp án là D. abide ghép với từ “by” có trong câu tạo thành abide by.
Tạm dịch: chúng tôi rất tôn trọng cha mình và luôn nghe theo nguyên tắc của ông.
Question 15. Đáp án C

Trang 10
Kiến thức được hỏi: Lựa chọn từ.
absent-minded (adj): đãng trí
big-headed (adj): tự phụ
quick-witted (adj): nhanh trí; thông minh
bad-tempered (adj): dễ nổi nóng
Phù hợp với nghĩa của câu nhất là đáp án C.
Tạm dịch: Chúng tôi đã khá ấn tượng bởi những sinh viên nhanh trí đã đưa ra câu trả lời cho câu hỏi của
chúng tôi gần như ngay lập tức.
Question 16. Đáp án B
Kiến thức được hỏi: Thành ngữ
Ta có cụm “lend an ear to sb/sth”: lắng nghe một cách chân thành. Đáp án B.
Tạm dịch: Tôi chắc rằng chị bạn sẽ lắng nghe bạn một cách đồng cảm khi bạn giải thích tình huống này
với cô ấy.
Question 17. Đáp án B
Kiến thức được hỏi: Thành ngữ
Ta có cụm “rise to the occasion": có khả năng đối phó với tình thế khó khăn bất ngờ. Đáp án B.
Tạm dịch: Anh họ tôi lo lắng vế việc phỏng vấn trên TV, nhưng anh ấy đã ứng phó rất tốt.
Question 18. Đáp án B
Kiến thức được hỏi: Phrasal verb
turn on: bật
take up: đảm nhiệm, gánh vác công việc
turn up: xuất hiện take on: thuê
Đáp án B là phù hợp với nghĩa của câu nhất.
Tạm dịch: Trông cậu thật mệt mỏi. Tớ nghĩ cậu đang gánh vác nhiều hơn những gì cậu có thể xử lý.
Question 19. Đáp án C
Lỗi sai ở cấu trúc so sánh kép: càng.. .càng
Cấu trúc: The + comparative (+ N) + S + V, + the + comparative (+ N) + S + V
So sánh hơn với tính từ ngắn: short adj + er
So sánh hơn vái tính từ dài: more + long adj
Sửa: the more hard → the harder. Đáp án C.
Tạm dịch: Bạn càng mệt mỏi, bạn càng khó tập trung hơn.
Question 20. Đáp án C
Lỗi sai về thì.
Sửa: are not existing → do not exist. Đáp án C. exist (v): tồn tại
Động từ “exist” không được chia ở thì tiếp diễn.

Trang 11
Tạm dịch. Đại dương là đặc điểm phân biệt trái đất với các hành tinh khác trên hệ mặt trời, vì các nhà
khoa học tin rằng vùng nước lớn không tồn tại trên các hành tinh khác.
Question 21.
Lỗi sai từ vựng, câu đã dùng từ sai.
considerate (adj): tận tâm, tận tình
Phải sửa thành: considerable (adj) - đáng kể
Đáp án A. considerate → considerable
Chú ý: amounts không sai vì amount có thể để ở hình thức số nhiều bằng cách thêm “s” sau nó.
Tạm dịch: Có nhiều khoản tiền bị lãng phí đáng kể cho các dự án xây dựng lớn
Question 22. Đáp án A
Dịch câu gốc: Tôi nghĩ tôi đã không nên ở nhà hôm qua.
Dịch các phương án:
A. Tôi hối tiếc vì đã ở nhà tối qua.
B. Sai ngữ pháp (không có cấu trúc regret for doing)
C. Sai ngữ pháp (không có cấu trúc regret for doing)
D. Tôi tiếc khi phải ở nhà tối qua
Regret + Ving: hối hận, hối tiếc làm gì
Regret + to V: hối tiếc khi phải làm gì
A là câu gần nghĩa nhất với câu gốc.
Question 23. Đáp án D
Dịch câu gốc: Họ không cho công nhân sử dụng điện thoại văn phòng cho mục đích cá nhân.
Dịch các phương án:
A. Họ không cho phép sử dụng điện thoại văn phòng để gọi thư ký riêng.
B. Họ không cho công nhân sử dụng điện thoại văn phòng.
C. Điện thoại văn phòng được sử dụng bởi công nhân một cách cá nhân.
D. Họ không cho phép công nhân dùng điện thoại văn phòng cho mục đích cá nhân.
Rõ ràng D là hợp nghĩa với câu gốc nhất.
Question 24. Đáp án A
Dịch câu gốc: John nói với thư ký. “ Cô đáng lẽ ra nên hoàn thành bài báo cáo lúc này rồi.”
Dịch các phương án:
A. John đã phê bình cô thư ký vì không hoàn thành bài báo cáo.
B. John nói rằng cô thư ký đã không hoàn thành bài báo cáo
C. John nhắc nhở cô thư ký vể việc hoàn thành báo cáo đúng thời hạn.
D. John la mắng cô thư ký vì đã không hoàn thành bài báo cáo.
Phương án A dùng động từ dẫn phê bình là phù hợp với văn cảnh câu gốc nhất.
Question 25. Đáp án B

Trang 12
Dịch câu gốc và các phương án:
Anh ấy đã không ăn bất cứ thứ gì từ sáng nay. Anh ta ăn thức ăn một cách đầy háo hức.
A. Anh ta ăn thức ăn một cách đầy háo hức mặc dù anh ấy đã ăn nhiều từ lúc bình minh.
B. Anh ta ăn thức ăn một cách đầy háo hức vì anh ấy đã không ăn gì từ lúc bình minh.
C. Anh ấy đã không ăn bất cứ thứ gì từ lúc bình minh để mà anh ta ăn thức ăn một cách đây háo hức.
D. Anh ta ăn thức ăn một cách quá háo hức đến nỗi mà anh ấy đã không ăn gì từ lúc bình minh.
B là hợp nghĩa vái câu gốc nhất, các phương án khác sai khác về ngữ nghĩa.
Question 26.
Dịch câu gốc và các phương án:
Julie đã có một khóa đào tạo về y học cổ truyền. Cô ấy có thể giúp người đàn ông thoát khỏi nguy hiểm.
A. Dù Julie đã có một khóa đào tạo về y học cổ truyền, cô đã có thể giúp người đàn ông ra nguy hiểm.
B. Đã được đào tạo về y học cổ truyền, Julie đã có thể giúp người đàn ông thoát khỏi nguy hiểm.
C. Nếu không có một khóa đào tạo về y học cổ truyền, Julie đã có thể giúp người đàn ông thoát khỏi nguy
hiểm.
D. Mặc dù đã được đào tạo về y học cổ truyền, Julie đã có thể giúp người đàn ông thoát khỏi nguy hiểm.
Question 27. Đáp án A
Maria và Diana đang nói chuyện về buổi tối hôm nay.
Maria: “Cảm ơn cậu, tối nay thật tuyệt”
Diana: “_____________”
A. Mình vui vì cậu đã thích.
B. Ồ, được rồi.
C. Không, nó không ngon.
D. Ừm, nó rất tuyệt.
Để trả lời cho một câu cảm ơn, thì A là cách trả lời phù hợp nhất.
Question 28. Đáp án B
Tom đã tới muộn trong buổi hẹn với Barbara. Tom: “ Xin lỗi, tớ đến trễ. Đường tắc kinh khủng”.
Barbara: “____________”
A. Niềm hân hạnh của tớ.
B. Đừng lo lắng gì cả.
C. Tớ hi vọng tớ có thể, nhưng tớ xin lỗi.
D. Ồ, để tớ xem nào.
Để đối đáp với một lời xin lỗi thì phương án B là phù hợp nhất.
Question 29. Đáp án D
Dữ liệu của câu: Kể từ khi cha của Laura chết, mẹ cô đã trở thành breadwinner để nuôi cả gia đình. Từ
văn cảnh ta có thể hiểu breadwinner (n) - người trụ cột trong gia đình
A. người nướng bánh mì mỗi sáng

Trang 13
B. chủ tiệm bánh
C. người giao bánh mì để kiếm tiền
D. người đi làm để kiếm tiền
→ a person who goes out to work to earn money = breadwinner. Đáp án D.
Tạm dịch: Kể từ khi cha của Laura chết, mẹ cô đã trở thành người trụ cột gia đình.
Question 30. Đáp án B
Dữ liệu của câu: Peter là the black sheep của gia đình, nên anh ấy không bao giờ được chào đón
Là người không được chào đón, nên ta có thể đoán định rằng phần gạch chân mang nghĩa tiêu cực.
Và the black sheep - thành viên cá biệt, người bị cho là tồi tệ và đáng xấu hổ.
A. một thành viên được yêu quý
B. một thành viên tồi tệ và đáng xấu hổ
C. con một
D. con cả
→ a bad and embarrassing member = the black sheep. Đáp án B.
Tạm dịch: Peter là thành viên cá biệt của gia đình, nên anh ấy không bao giờ được chào đón.
Question 31. Đáp án B
Dữ liệu của câu: Việc đánh mất nhật kí của mình khiến ông ấy sorrow hơn cả việc xuất ngũ 6 năm trước.
sorrow (n): nỗi buồn (danh từ của từ sorry).
grief (n): nỗi đau buồn
joy (n): niềm vui
comfort (n): sự thoải mái
sympathy (n): sự đồng cảm
sorrow (nỗi buồn) trái nghĩa với joy (niềm vui). Đáp án B.
Tạm dịch: Việc đánh mất nhật kí của mình khiến ông ấy buồn hơn cả việc xuất ngũ 6 năm trước.
Question 32.
Dữ liệu của bài: Là một phóng viên báo chí, cô ấy luôn muốn nhận được thống tin at first hand.
Logic: Phóng viên thì muốn tiếp cận thông tin như thế nào?
“At first hand”: trực tiếp
• indirectly (adv): một cách gián tiếp
• directly (adv): trực tiếp
• easily (adv): dễ dàng
• slowly (adv): chậm rãi
→ at first hand (trực tiếp) trái nghĩa với indirectly (một cách gián tiếp)
Tạm dịch: Là một phóng viên báo chí, cô ấy luôn muốn nhận được thông tin một cách trực tiếp.
Question 33: Đáp án D
Ta có cụm: for the sake of something: vì cái gì. Đáp án D

Trang 14
The position of sport in today's society has changed out of all recognition. People no longer seem to think
of sports as ‘just a game’ - to be watched or played for the sake of enjoyment.
Tạm dịch: Vị trí của thể thao trong xã hội hiện nay đã thay đổi tất cả nhận thức của chúng ta. Mọi người
không còn nghĩ đến các môn thể thao như ‘chỉ là một trò chơi để xem hoặc chơi chỉ vì sự vui thích.
Các phương án khác:
A. advantage: ưu điểm
B. good: tốt
C. benefit: lợi ích
Question 34. Đáp án B
Câu hỏi yêu cầu chọn đại từ quan hệ đúng.
Nhận thấy rằng trước chỗ trống cần điền ĐTQH có giới từ “of” mà ta luôn có: giới từ + which / whom
- which: thay thế cho danh từ chỉ vật
- whom: thay thế cho danh từ chỉ người
Athletes(n): vận động viên là N chỉ người vì vậy ta chọn B.
Các phương án còn lại không thể đứng sau giới từ of.
The result has been huge rewards for athletes, some of whom are nowvery wealthy, particularly top
footballers, golfers and tennis players.
Tạm dịch: Kết quả là có những phần thưởng khổng lồ dành cho cho các vận động viên, một số người mà
hiện đang rất giàu có, đặc biệt là các cầu thủ bóng đá, tay golf và các tay vợt hàng đầu
Question 35. Đáp án A
A. In addition: thêm vào đó
B. However: tuy nhiên
C. In contrast: ngược lại
D. Therefore: vì vậy
Phần gạch chân cần điền từ nối để nối giữa ý trước và ý sau của bài. Ý trước: Những giải thưởng khổng lồ
khiến một số vận động viên giàu có “The result has been huge rewards for athletes, some of whom are
now very wealthy, particularly top footballers, golfers and tennis players.”
Ý sau: nó cũng không lấy gì làm bất thường khi có một số vận động viên nhận lương cao do quảng cáo
sản phẩm hoặc hiện diện mang tính cá nhân: “it is not unusual for some athletes to receive large fees on
top of their salary, for advertising products or making personal appearances”
Ý nối ở đây cùng đề cập đến thu nhập cao của vận động viên nên dùng ”In addition - thêm vào đó” là
hợp lí. Đáp án A.
Question 36. Đáp án C
Ta có cụm : (to) rely on: dựa vào. Đáp án C.

Trang 15
A trend towards shorter working hours means that people generally tend to have more free time, both to
watch and to take in sporting activity; sport has become a significant part of the recreation industry that
we now rely on to fill our leisure hours.
Tạm dịch: Xu hướng về giờ làm việc ngắn hơn có nghĩa là mọi người thường có xu hướng có nhiều thời
gian rảnh hơn, cả để xem và tham gia các hoạt động thể thao; thể thao đã trở thành một phần quan trọng
của ngành công nghiệp giải trí mà hiện nay chúng ta dựa vào để lấp đầy thời gian rảnh rỗi của mình.
Question 37. Đáp án A
Câu hỏi từ vựng.
A. ordinary (adj): bình thường
B. mighty (adj): mạnh mẽ
C. extremist (n) người cực đoan
D. abnormal (adj): không bình thường ordinary people: người bình thường. Đáp án A.
Professional sport is a vital part of that industry, providing for millions of ordinary people all over the
world.
Tạm dịch: Thể thao chuyên nghiệp là một phần quan trọng của ngành công nghiệp đó, cung cấp cho hàng
triệu người dân bình thường trên khắp thế giới.
READING 1
Đoạn 1: Sự thay đổi khí hậu là kết quả trực tiếp từ việc tăng dân số. Đồng thời, việc tăng lượng dùng tài
nguyên thiên nhiên của mỗi một cá nhân cũng gây áp lực lên nguồn tài nguyên thiên nhiên.
Đoạn 2: Các hoạt động của con người và những hệ lụy kèm theo dẫn đến thay đổi khí hậu- sự nóng lên
của toàn cầu. Ví dụ như: chặt cây phá rừng, khai thác và tiêu thụ dầu và các nhiên liệu khác..
Đoạn 3: Tiêu chuẩn sống của con người tăng lên đồng nghĩa mức tiêu dùng năng lượng cũng tăng lên
càng là gánh nặng với nguồn tài nguyên vốn đã kiệt quệ.
Question 38. Đáp án B
Dịch câu hỏi: Thế giới đã phải mất bao nhiêu năm để dùng hết một nghìn tỷ thùng dầu đầu tiên?
Dịch đáp án:
A. 100 năm B. 125 năm
C. 30 năm D. 7 năm
Đây là câu hỏi chi tiết, key words là “first one trillion barrels of oil”, ta tìm thấy phần thông tin này ở
đoạn 1 dòng 6-7.
“For example, it took the world 125 years to use the first one trillion barrels of oil.”
Ví dụ như, thế giới mất 125 năm để sử dụng hết một nghìn tỷ thùng dầu đầu tiên.
→ Chọn đáp án B
Question 39.
Dịch câu hỏi: Từ “consumption” trong đoạn văn gần nghĩa nhất với từ____________.
Dịch đáp án:

Trang 16
A. Sự phát triển
B. Sự sử dụng, sự tiêu thụ
C. Dân số
D. Sự tăng lên
Câu hỏi từ vựng, chúng ta đọc lại câu chứa từ consumption.
In addition, the average person uses more energy and natural resources than the average person one
hundred years ago, meaning that the rates of consumption are actually much higher than just the increase
in population would imply.
Them vào đó, một người trung bình hiện nay tiêu thụ năng lượng và tài nguyên thiên nhiên nhiều hơn so
với một người trung bình của 100 năm trước, có nghĩa là tỉ lệ tiêu thụ thực sự cao hơn nhiều so với việc
chỉ tính tăng về mặt dân số.
→ Chọn đáp án B
Question 40. Đáp án C
Dịch câu hỏi. Theo như đoạn văn, hành động nào dưới đây KHÔNG gây biến đổi khí hậu?
Dịch đáp án.
A. Sản xuất thức ăn
B. Sử dụng năng lượng
C. Săn bắt động vật hoang dã
D. Tiêu thụ, sử dụng tài nguyên thiên nhiên
Đây là câu hỏi chứa từ phủ định. Cái nào KHÔNG nên chúng ta sẽ phải chọn phương án mà trong bài đọc
không nhắc đến, dùng phương pháp loại trừ.
Thông tin nằm ở những dòng đầu tiên của đoạn 2.
All of these activities: food production, energy usage, and the use of natural resources, contribute to
climate change in some way.
Trong đó các chữ in đậm tương ứng với việc loại bỏ A, B, D.
→ Chọn đáp án C
Question 41. Đáp án C
Dịch câu hỏi: Theo như đoạn văn, việc sản xuất thức ăn góp phần gây ra sự nóng lên toàn cầu như thế
nào?
Dịch đáp án:
A. Việc sản xuất ra nhiều thức ăn dẫn đến sự tăng trưởng dân số thế giới.
B. Việc sản xuất thức ăn sử dụng nhiều loại hóa chất gây ra sự nóng lên toàn cầu.
C. Việc sản xuất thức ăn đòi hỏi phải phá rừng để có nhiều đất trồng trọt hơn.
D. Việc sản xuất thức ăn làm giảm khả năng thoát nhiệt của không khí
Câu hỏi này chúng ta tìm theo keywords “food production contribute to global warming”
Thông tin nằm ở nửa cuối đoạn 2 có các ý sau:

Trang 17
“In order to produce more food, farmers cut down trees to gain more land for their fields” và “Those trees
are an essential part of controlling global warming;”
Để sản xuất ra nhiều thức ăn hơn nữa, những người nông dân đã chặt cây để có thêm nhiều đất đai trồng
trọt... Những cây này là một phần quan trọng trong việc kiểm soát sự nóng lên của toàn cầu.
Do đó hành động chặt cây để có thêm đất trồng nhằm sản xuất thêm thực phẩm đã gây ra sự nóng lên của
toàn cầu, làm biến đổi khí hậu.
→ Chọn đáp án C
Question 42. Đáp án C
Dịch câu hỏi: Theo như đoạn văn, mức sống ảnh hưởng như thế nào đến sự nóng lên toàn cầu?
Dịch đáp án:
A. Mức sống cao hơn là tốt hơn cho môi trường.
B. Những quốc gia tốp đầu tạo ra ít dân số hơn những quốc gia đang phát triển.
C. Việc sử dụng tài nguyên thiên nhiên liên quan trực tiếp với mức sống.
D. Mức sống cao hơn dẫn đến sự tăng trưởng dân số thế giới
Thông tin tìm ở những dòng đầu tiên của đoạn 3.
Trích bài: In addition to a growing population, the world also has a population that desires a higher
standard of living than in the past, and a higher standard of living requires the use of even more natural
resources.
Tạm dịch: Ngoài việc dân số đang tăng lên, thế giới còn có một bộ phận dân số mà mong muốn mức tiêu
chuẩn sống cao hơn so với trước kia, và một mức tiêu chuẩn sống cao hơn yêu cầu việc tiêu thụ nhiều tài
nguyên thiên nhiên hơn.
Tương ứng với ý C, các phương án khác không phù hợp.
READING 2
Đoạn 1: Mở bài: Giới thiệu Carnegie Hall đang trải qua cuộc trùng tu để lấy lại chất lượng âm thanh tốt
nhất của nó.
Đoạn 2: Lịch sử của Carnegie Hall và những cuộc trùng tu gây hư hại nghiêm trọng cho tòa nhà.
Đoạn 3: Carnegie Hall được cứu khỏi nguy cơ phá hủy và đang có một viễn cảnh tương lai tươi sáng.
Question 43. Đáp án A
Dịch câu hỏi: Đoạn văn chủ yếu nói về vần đề____________.
Dịch đáp án:
A. Sự thay đổi đối với Carnegie Hall.
B. Sự xuất hiện của Carnegie Hall.
C. Lịch sử của Carnegie Hall trong suốt thời kì Đại Khủng Hoảng.
D. Sự hư hại của trần nhà Carnegie Hall

Trang 18
Ngay phần mở bài, tác giả đã giới thiệu về một sự thay đổi- sự trùng tu- đang được thực hiện đối với
Carnegie Hall “Carnegie Hall, the famous concert hall in New York City, has again undergone a
restoration”
Xuyên suốt bài, ở các đoạn 2, 3 cũng đề cập đến những lần thay đổi khác đối với Carnegie Hall cụ thể vào
năm 1946 đã có sự thay đổi dẫn đến hư hại nhiều cho tòa nhà. Và cuối bài, tác giả nhắc đến những mặt
vượt trội của sự trùng tu lần này có thể sẽ trả lại cho Carnegie Hall lại một lần nữa có thể trở thành phòng
hòa nhạc tuyệt vời như xưa.
→ Chọn đáp án A
Question 44. Đáp án C
Dịch câu hỏi. Từ “it” ở đoạn đầu chỉ___________
Dịch đáp án.
A. Carnegie Hall
B. Thành phố New York
C. Một cuộc trùng tu
D. Một kế hoạch
Câu hỏi về đại từ thay thế cho danh từ đứng ở trước nên ta quay lại câu chứa từ “it”.
Carnegie Hall, the famous concert hall in New York City, has again undergone a restoration. While this is
not the first, it is certainly the most extensive in the buildings history.
Carnegie Hall, phòng hòa nhạc nổi tiếng ở thành phố New York, lại một lần nữa đang được trùng tu lại.
Dẫu rằng đây không phải lần đầu tiên, nhưng đây chắc chắn là lần trùng tu có phạm vi rộng nhất trong
lịch sử của tòa nhà.
Vì thế it thay thế cho restoration: cuộc trùng tu. → Chọn đáp án C
Question 45. Đáp án A
Dịch câu hỏi: Sự thay đổi chính của tòa nhà vào năm 1946 là gì?
Dịch đáp án:
A. Mái vòm âm thanh bị phá hủy.
B. Không gian bên trong tòa nhà bị bán cho các doanh nghiệp.
C. Các bức tường đã bị phá hủy vì một trận động đất
D. Sân khấu được xây lại.
Câu hỏi thông tin chi tiết, tìm thông tin ở những dòng 8,9, 10 ở đoạn 2, key words là năm 1946
Trích bài: A renovation in 1946 seriously damaged the acoustical quality of the hall when the makers of
the film Carnegie Hall cut a gaping hole in the dome of the ceiling to allow for lights and air vents.
Tạm dịch: Một cuộc trùng tu năm 1946 đã phá hủy nghiêm trọng chất lượng âm thanh của phòng hòa
nhạc khi những nhà làm phim Carnegie Hall đã cắt một cái hố toác ra trên mái vòm của trần nhà để cho
ánh sáng và lỗ thông không khí.

Trang 19
Hệ quả là “the hall never sounded the same afterwards” phòng hòa nhạc không bao giờ nghe tuyệt như
trước được nữa.
Thông tin tương ứng với đáp án A.
Question 46. Đáp án C
Dịch câu hỏi: Andrew Carnegie là ai?
Dịch đáp án:
A. Một nghệ sĩ vi-ô-lông
B. Một kiến trúc sư
C. Một chủ nhà máy thép
D. Thị trưởng thành phố New York
Câu hỏi thông tin chi tiết, ta tìm thấy key words “Andrew Carnegie” ở dòng đầu tiên đoạn 2.
“Andrew Carnegie, the wealthy owner of a steel company in the late 1800s.”
Andrew Carnegie là ông chủ giàu có của một nhà máy thép những năm cuối 1800.
→ Chọn đáp án C
Question 47. Đáp án C
Dịch câu hỏi: Isaac Stern có quan hệ như thế nào với tòa nhà Carnegie Hall?
Dịch đáp án:
A. Ông làm bộ phim “Carnegie Hall” vào năm 1946.
B. Ông đã biểu diễn ở đêm khai mạc năm 1891.
C. Ông đã cố gắng để cứu tòa nhà tù năm 1960.
D. Ông đã mở một tiệm café ở Carnegie Hall trong thời kì Khủng Hoảng.
Ta tìm thông tin chứa key words “Isaac Stern” ở dòng đầu đoạn 3.
Trích bài: In 1960, the violinist Isaac Stern became involved in restoring the hall after a group of real
estate developers unveiled plans to demolish Carnegie Hall and build a high-rise office building on the
site.
Tạm dịch: Năm 1960, Isaac Stern người chơi đàn vĩ cầm đã tham gia vào việc khôi phục lại phòng hòa
nhạc sau khi một nhóm những người làm trong mảng bất động sản đã tiết lộ về kế hoạch sẽ phá hủy
Carnegie Hall và xây dựng một tòa nhà văn phòng cao trọc trời ở khu vực đó
“The movement was successful, and the concert hall is now owned by the city.” Những nỗ lực ấy đã
thành công và bây giờ phòng hòa nhạc thuộc quyền sở hữu của thành phố.
Vì vậy Isaac Stern chính là người đã cứu tòa nhà khỏi bị phá hủy vào năm 1960.
→ Chọn đáp án C
Question 48. Đáp án D
Dịch câu hỏi: Từ nào dưới đây gần nghĩa nhất với từ detrimental trong đoạn 3?
Dịch đáp án:
A. Nguy hiểm

Trang 20
B. tầm thường, không đáng kể
C. ấn tượng
D. gây hại
Câu hỏi về từ vựng, cùng xem lại câu hỏi chứa từ cẩn tìm nghĩa.
Despite its reputation, however, the concert hall suffered from several detrimental renovations over the
years.
Mặc dù danh tiếng của nó, phòng hòa nhạc vẫn phải chịu đựng một vài những lần trùng tu gây hại lớn
trong suốt những năm qua.
→ Chọn đáp án D
Question 49. Chọn đáp án A
Dịch câu hỏi: Từ nào dưới đây gần nghĩa nhất với từ “unveiled” trong đoạn 3?
Dịch đáp án:
A. Được thông báo, tiết lộ.
B. Bị cấm
C. Bị lu mờ
D. Được đặt ở
Câu hỏi về từ vựng, cùng xem lại câu hỏi chứa từ cần tìm nghĩa.
In 1960, the violinist Isaac Stern became involved in restoring the hall after a group of real estate
developers unveiled plans to demolish Carnegie Hall and build a high-rise office building on the site.
Năm 1960, Isaac Stern người chơi đàn vĩ cầm đã tham gia vào việc khôi phục lại phòng hòa nhạc sau khi
một nhóm những người làm trong mảng bất động sản đã tiết lộ về kế hoạch sẽ phá hủy Carnegie Hall và
xây dựng một tòa nhà văn phòng cao trọc trời ở khu vực đó.
→ Chọn đáp án A
Question 50. Đáp án C
Dịch cầu hỏi: Cảm nhận của tác giả vẽ tương lai của Carnegie Hall là?
Dịch đáp án:
A. Mơ hồ B. Được bảo vệ
C. Lạc quan D. Tiêu cực
Cảm nhận của tác giả được thể hiện rõ nhất ở phần mở bài và kết bài.
Mở: Carnegie Hall once again has the quality of sound that it had when it was first built.
Carnegie Hall một lần nữa có được chất lượng âm thanh như nó đã từng có ở ngày đầu xây dựng.
Kết: Carnegie has never sounded better, and its prospects for the future have never looked more
promising.
Carnegie chưa bao giờ nghe tuyệt hơn thế, và viễn cảnh tương lai của nó cũng chưa bao giờ hứa hẹn đến
thế.
Qua đây ta thấy được cảm nhận của tác giả là rất lạc quan về tương lai của Carnegie Hall.

Trang 21
Chọn đáp án C

Trang 22
ĐỀ SỐ 18 ĐỀ THI THỬ TỐT NGHIỆP THPT
NĂM HỌC: 2020 – 2021
MÔN: TIẾNG ANH
Thời gian làm bài: 60 phút; không kể thời gian phát đề

Mark the letter A, B, C or D on your answer sheet to indicate the word whose underlined part
differs from the other three in pronunciation in each of the following questions.
Question 1. A. laugh B. cough C. enough D. though
Question 2. A. breath B. breathe C. with D. soothe
Mark the letter A, B, C or D on your answer sheet to indicate the word that differs from the other
three in position of primary stress in each of the following questions.
Question 3. A. nuclear B. consist C. hello D. prepare
Question 4. A. economic B. solution C. convention D. specific
Mark the letter A, B, C, or D on your answer sheet to indicate the correct answer to each of the
following questions.
Question 5. A complete_____________of terms and conditions should be made before the agreement
is signed.
A. examine B. examining C. examination D. examined
Question 6. I_____________one item early last week, but I am certain that I did not get the receipt at
that time
A. purchase B. purchases C. purchased D. have purchased
Question 7. American_____________to have been discovered by Colombus.
A. is said B. was said C. being said D. says
Question 8. _____________to set the alarm clock, Jenny left for work late this morning.
A. Forget B. To forget C. Having forgotten D. Forgotten
Question 9. Since he was unable to attend the annual meeting, Mr. Yokomate requested that the
minutes_____________to his office.
A. be sent B. were sent C. sent D. send
Question 10. Volunteers should keep in mind values of the charity event in_____________they
participate actively.
A. when B. whose C. that D. which
Question 11. _____________to the airline strikes, Mr. Jones had to postpone his business trip to Rome.
A. Owed B. Because C. Due D. As
Question 12. If she had experience in medical research, she_____________for one of the positions
at the hospital.
A. apply B. could apply C. could have apply D. can apply

Trang 1
Question 13. Those in research positions are responsible for compiling and_____________research
data.
A. store B. storage C. storing D. to store
Question 14. _____________a year, Tarrin Industrial Supply audits the accounts of all of its factories.
A. Once B. Immediately C. Directly D. Yet
Question 15. The timeline for the pathway lighting project was extended to_____________input from
the Environmental Commission.
A. use up B. believe in C. make into D. allow for
Question 16. If you don’t want your business to_____________bankrupt, avoid consulting those
willing to step out on a limb.
A. come B. take C. go D. pay
Question 17. _____________the BPT39 wireless speaker is widely popular, production will be
increased fivefold starting next month.
A. On behalf of B. Whether C. Moreover D. As
Question 18. Jack is very independent, he always paddle his own_____________
A. boat B. canoe C. ship D. yatch
Mark the letter A, B, C or D on your answer sheet to indicate the underlined part that needs
correction in each of the following questions
Question 19. If you came to the party tomorrow, don’t forget to bring your parents.
A. If B. came C. don’t D. to bring
Question 20. Even the CEO had to admit that Prasma Designs’ win was part the result of fortunate
timing.
A. Even B. Prasma Designs’ C. part D. timing
Question 21. To prepare for marathon, I run 15 kilometers three and four times a week.
A. To prepare B. run C. three and four D. times
Mark the letter A, B, C, or D on your answer sheet to indicate the sentence that is closest in
meaning to each of the following questions.
Question 22. “Where were you last night, Mr. Jenkins?” he said.
A. He wanted to know where Mr. Jenkins was the night before.
B. He asked Mr. Jenkins where was he last night.
C. He wanted to know where Mr. Jenkins had been the following night.
D. He asked Mr. Jenkins where he had been the previous night.
Question 23. The burglar was caught red- handed by the police when he was breaking into the
flat.
A. The police caught the burglar to break into the flat.
B. The police caught the burglar when breaking into the flat,

Trang 2
C. When the burglar had broken into the flat, the police caught him at once.
D. The police caught the burglar breaking into the flat.
Question 24. “We’ll go camping as long as the weather is good.”
A. If the weather is fine, we will go camping.
B. The weather is good when we will go camping.
C. If the weather is better, we will go camping.
D. We’ll go camping immediately the weather is good.
Mark the letter A, B, C, or D on your answer sheet to indicate the sentence that best combines
each pair of sentences in the following questions.
Question 25. We are not going to select the ruling party. We are not going to select the opposition
party either.
A. We are going to select neither the ruling party nor the opposition party in the upcoming election.
B. We are not going to select neither the ruling party nor the opposition party in the upcoming
election.
C. We are going to select either the ruling party or the opposition party in the upcoming election.
D. We are not going to select either the ruling party or the opposition party in the upcoming election.
Question 26. The lighthouse was built in the 1600s. It remains standing through hundreds of
years.
A. Having been built in the 1600s, the lighthouse remains standing through hundreds of years.
B. Building in the 1600s, the lighthouse remains standing through hundreds of years.
C. Built in the 1600s, the lighthouse remains standing through hundreds of years.
D. Having built in the 1600s, the lighthouse remains standing through hundreds of years.

Mark the letter A, B, C, or D on your answer sheet to indicate the sentence that best completes
each of the following exchanges.
Question 27. Laura is warning Bob about the house.
- Laura: “Mind your head. The ceiling is low”.
- Bob: “_____________”
A. Don’t mention it. B. Thanks, I’ll remember it.
C. I couldn’t agree more with you. D. I don’t think you’re right.
Question 28. David is talking to Linda after a party.
- David: “Would you like me to give you a ride home?”
- Linda: “_____________”
A. That’s be great, thanks. B. Sorry, you’re not my type.
C. Yes, I’m riding home now D. No, thanks. I don’t like riding.

Trang 3
Mark the letter A, B, C, or D on your answer sheet to indicate the word(s) CLOSEST in meaning
to the underlined word(s) in each of the following questions.
Question 29. The Boy Scouts organisation is dedicated to helping boys become moral and productive
adults.
A. committed B. used C. focused D. interested
Question 30. Father has lost his job,so we’ll have to tighten our belt to avoid getting into debt.
A. earn money B. save money C. sit still D. economize
Mark the letter A, B, C, or D on your answer sheet to indicate the word(s) OPPOSITE in meaning
to the underlined word(s) in each of the following questions.
Question 31. Henry has found temporary job in a factory.
A. eternal B. genuine C. permanent D. satisfactory
Question 32. We had a whale of time as everything was quite fantastic.
A. had little time to play B. had a lot of time to play
C. felt happy D. felt disappointed
Read the following passage and mark the letter A,B, C,or D on your answer sheet to indicate the
correct word or phrase that best fits each of the numbered blanks.
Not everybody recognises the benefits of new developments in communications technology. Indeed,
some people fear that text messaging may actually be having a negative (33) _____________on young
people’s communication and language skills, especially when we hear that primary school children may
be at risk of becoming addicted to the habit. So widespread has texting become, however, that even
pigeons have started doing it. (34) _____________, in this case, it’s difficult to view the results as
anything but positive. Twenty of the birds are about to take to the skies with the task of measuring air
pollution, each (35) _____________with sensor equipment and a mobile phone. The readings made by
the sensors will be automatically converted into text messages and beamed to the Internet - (36)
_____________they will appear on a dedicated ‘pigeon blog’. The birds will also each have a GPS
receiver and a camera to capture aerial photos, and researchers are building a tiny ‘pigeon kit’
containing all these gadgets. Each bird will carry these in a miniature backpack, (37) ____________ ,
that is, from the camera, which will hang around its neck. The data the pigeons text will be displayed in
the form of an interactive map, which will provide local residents with up-to-the-minute information on
their local air quality.
Question 33. A. result B. outcome C. effect D. conclusion
Question 34. A. Therefore B. What’s more C.whereas D. That is
Question 35. A. armed B.loaded C. granted D. stocked
Question 36. A. when B. which C. where D. what
Question 37. A. instead B. except C. apart D. besides

Trang 4
Read the following passage and mark the letter A, B, C or D on your answer sheet to indicate the
correct answer to each of the questions.
The Art World
One of the maj or problems in the art world is how to distinguish and promote an artist. In effect, a
market must be created for an artist to be successful.The practice of signing and numbering individual
prints was introduced by James Abbott McNeill whistler, the nineteenth - century artist best known for
the painting of his mother, called “Arrangement in Grey and Black”, but known to most of us as“
whistler’s Mother”. Whistler’s brother - in - law, Sir Francis Seymour Haden, a less well - known artist,
had speculated that collectors might find prints more attractive if they knew that they were only a
limited number of copies produced. By signing the work in pencil, an artist could guarantee and
personalize each print. As soon as Whistler and Haden began the practice of signing and numbering
their prints, their work began to increase in value, when other artists noticed that the signed prints
commanded higher prices, they began copying the procedure. Although most prints are signed on the
right - hand side in the margin below the image, the placement of the signature is a matter of personal
choice. Indeed, prints have been signed within the image, in any of the margins, or even on the
reverseside of the print, wherever the artist elects to sign it, a signed print is still valued
above an unsigned one, even in the same edition.
above an unsigned one, even in the same edition.
Question 38. Which of the following would be a better title for the passage?
A. Whistlers Mother B.Whistler’s Greatest Works
C. Ihe Practice of Signing Prints D. Coping Limited Edition Prints
Question 39. The word “speculated” in the paragraph 1 could best be replaced by_____________
A. guessed B. noticed C. denied D. announced
Question 40. What was true about the painting of Whistler’s mother?
A. It was painted by Sir Francis Haden
B. Its title was “Arrangement in Grey and Black”
C. It was not one of whistler’s best paintings
D. It was a completely new method of painting
Question 41. The author mentions all of the following as reasons why a collector prefer a signed
print EXCEPT_____________
A. it guarantees the prints authenticity
B. it makes the print more personal
C. it encourages higher prices for the print
D. it limits the number of copies of the print
Question 42. It can be inferred from the passage that artists number their prints_____________
A. as an accounting procedure B. to guarantee a limited edition

Trang 5
C. when the buyer requests it D. at the same place on each of the prints
Read the following passage and mark the letter A,B, C or Don your answer sheet to indicate the
correct answer to each of the questions.
Butterflies are among the most extensively studied insectsan estimated 90 percent of the world’s
species have scientific names. As a consequence, they are perhaps the best group of insects for
examining patterns of terrestrial biotic diversity and distribution. Butterflies also have a favorable
image with the general public.
Hence, they are an excellent group for communicating information on science and conservation issues
such as diversity. Perhaps the aspect of butterfly diversity that has received the most attention over the
past century is the striking difference in species richness between tropical and temperate regions. For
example, in 1875 one biologist pointed out the diversity of butterflies in the Amazon when he
mentioned that about 700 species were found within an hour’s walk, whereas the total number found on
the British islands did not exceed 66, and the whole of Europe supported only 321. This early
comparison of tropical and temperate butterfly richness has been well confirmed. A general theory of
diversity would have to predict not only this difference between temperate and tropical zones, but also
patterns within each region, and how these patterns vary among different animal and plant groups.
However, for butterflies, variation of species richness within temperate or tropical regions, rather man
between them, is poorly understood. Indeed, comparisons of numbers of species among the Amazon
basin, tropical Asia, and Africa are still mostly “personal communication” citations, even for
vertebrates, In other words, unlike comparison between temperate and
tropical areas, these patterns are still in the documentation phase. In documenting geographical
variation in butterfly diversity, some arbitrary, practical decisions are made. Diversity, number of
species, and species richness are used synonymously; little is known about the evenness of butterfly
distribution. The New World butterflies make up the preponderance of examples because they are the
most familiar species. It is hoped that by focusing on them, the errors generated by imperfect and
incomplete taxonomy will be minimized.
Question 43.The word “striking” in paragraph 2 is closest in meaning to_____________
A. physical B. confusing C. noticeable D. successful
Question 44. Butterflies are a good example for communicating information about conservation
issues because they_____________
A. are simple in structure
B. have been given scientific names
C. are viewed positively by people
D. are found mainly in temperate climates
Question 45. The word “exceed” in paragraph 3 is closest in meaning to_____________
A. locate B. allow C. go beyond D. come close to

Trang 6
Question 46.Which of the following is NOT well understood by biologists?
A. European butterfly habitats
B. Differences in species richness between temperate and tropical regions
C. Differences in species richness within a temperate or a tropical region
D. Comparisons of behavior patterns of butterflies and certain animal groups
Question 47. All of the followings are mentioned as being important parts of a general theory of
diversity EXCEPT_____________
A. differences between temperate and tropical zones
B. patterns of distribution of species in each region
C. migration among temperate and tropical zones
D. variation of patterns of distribution of species among different animals and plants
Question 48. Which aspect of butterflies does the passage mainly discuss_____________
A. their physical characteristics B. their adaption to different habitats
C. their names D. their variety
Question 49. The word “they” in paragraph 1 refer to_____________
A. insects B. butterflies C. patterns D. issues
Question 50. The idea “little is known about the eveness of butterfly distribution” in paragraph 5
means that_____________
A. there are many things that we know about butterfly eveness distribution
B. we don’t know anything about butterfly evenness distribution
C. we know much about butterfly evenness distribution
D. we know about butterfly evenness distribution to some extent

Trang 7
Đáp án
1-D 2-A 3-A 4-A 5-C 6-C 7-A 8-C 9-A 10-D
11-C 12-B 13-C 14-A 15-D 16-C 17-D 18-B 19-B 20-C
21-C 22-D 23-D 24-A 25-A 26-C 27-B 28-A 29-A 30-D
31-C 32-D 33-C 34-B 35-A 36-C 37-C 38-C 39-A 40-B
41-D 42-B 43-C 44-C 45-C 46-C 47-C 48-D 49-B 50-B

LỜI GIẢI CHI TIẾT


Question 1: Đáp án D
Đáp án D đúng vì phần gạch chân là âm câm còn phần gạch chân của các đáp án khác được đọc là âm /f/
A. laugh /lɑːf/ (v): cười
B. cough /kɒf/ (v): ho
C. enough /ɪˈnʌf/ (adv): đủ
D. though /ðəʊ/ (conj): mặc dù
Question 2: Đáp án A
Đáp án A đúng vì phần gạch chân là âm /θ/ còn phần gạc chân của các đáp án còn lại được đọc là âm /ð/
A. breath /breθ/ (v): hơi thở
B. breathe /briːð/ (v): thở
C. with /wɪð/ (pre): với
D. soothe /suːð/ (v):làm dịu
Question 3: Đáp án A
Đáp án A đúng vì đáp án A trọng âm rơi vào âm tiết thứ nhất còn ba đáp án còn lại trong âm rơi vào âm
tiết thứ hai.
A. nuclear /’nju:klia/ (a):thuộc hạt nhân
B. consist /kan’sist/ (v): bao gồm
C. hello /hơTou/ (n): xin chào
D. prepare /pri’pco/ (v): chuẩn bị
Question 4: Đáp án A
Đáp án A đúng vì đáp án A trọng âm rơi vào âm tiết thứ ba, còn các đáp án còn lại trọng âm rơi vào âm
tiết thứ hai.
A. economic /ˌiːkəˈnɒmɪk/ (a): kinh tế
B. solution /səˈluːʃn/ (n): giải pháp
C. convention /kənˈvenʃn/ (n): công ước
D. specific /spəˈsɪfɪk/ (a): đặc trưng
Question 5: Đáp án C
Đáp án C - Câu hỏi về loại từ

Trang 8
Mạo từ “a” bắt đầu cụm danh từ. Ở đây chưa có danh từ mà mới có tính từ “complete” nên cần chọn một
danh từ để hoàn thiện cụm danh từ.
A. examine (v): kiểm tra
C. examination (n): sự kiểm tra
B, D lần lượt là phân từ hiện tại và phân từ quá khứ của examine
Tạm dịch: Một cuộc kiểm tra toàn diện các điều khoản và điều kiện nên được tiến hành trước khi hợp
đồng được kí kết
Question 6: Đáp án C
Đáp án C - Câu hỏi về thì động từ
Động từ chia thì Quá khứ Đơn vì có trạng ngữ chỉ thời gian là “last week”.
Purchase (v): mua/ nhập hàng
Tạm dịch: Tôi đã mua một sản phẩm vào đầu tuần trước nhưng tôi chắc chắn là tôi chưa nhận hóa đơn
vào lúc đó
Question 7: Đáp án A
Đáp án A - Câu hỏi về bị động với động từ tường thuật
Câu bị động với động từ tường thuật, có cấu trúc: S + be + P2 (tường thuật) + to V/ to have P2+...
Ta dùng “to have P2” mà không dùng “to V” khi hành động đươc tường thuật xảy ra trước động từ tường
thuật, vì vậy câu này chọn A (ở hiện tại) vì việc Colombus tìm ra châu Mỹ đã ở quá khứ (xảy ra trước).
Tạm dịch: châu Mỹ được cho rằng được phát hiện bởi Columbus
Question 8: Đáp án C
Đáp án C - Câu hỏi về rút gọn mệnh đề Dùng phân từ để nối hai mệnh đề có cùng chủ ngữ. “Forget + to
V” là dạng chủ động nên ta chọn nối bằng phân từ 1. “Having + P2” được coi là phân từ 1, nhưng thêm
“having” để nhấn mạnh việc hành động này xảy ra trước.
Tạm dịch: Quên đặt chuông báo thức, Jenny đi làm muộn sáng nay.
Question 9: Đáp án A
Đáp án A - Câu hỏi về thức giả định
Cấu trúc câu giả định: S + V (mang nghĩa yêu cầu/ sai khiến/ gợi ý/ bắt buộc) + that + S2 + (should) + V
(inf) +...
Tạm dịch: Vì không thể tham gia cuộc họp hàng năm, ông Yokomate đã yêu cầu biên bản cuộc họp được
gửi về văn phòng của mình.
Question 10: Đáp án D
Đáp án D - Câu hỏi về đại từ quan hệ
Mệnh đề quan hệ (which they participate actively in) được đảo giới từ “in” lên đầu.
Charity event là sự việc nên dùng “which” (ngoài ra That cũng không đi với giới từ nên loại C)
Tạm dịch: Các tình nguyện viên nên tâm niệm về giá trị của sự kiện từ thiện mà họ tham gia một cách
tích cực.

Trang 9
Question 11: Đáp án C
Đáp án C - Câu hỏi về liên từ
Due to + N/V-ing: bởi vì lý do gì đó ...
A. owed - sửa thành “owing” thì sẽ đúng vì “owing to + N/V-ing”: Nhờ có cái gì đó
B. because - Because đi vái “of” để có “because of + N/V-ing”: Bởi vì có cái gì đó
D. As - là liên từ nên đi với mệnh đề. As: khi/ bởi vì
Tạm dịch: Bởi vì các vụ đình công ngành hàng không, ông Jones đã phải hoãn chuyến công tác tới Rome.
Question 12: Đáp án B
Đáp án B - Câu hỏi về câu điều kiện
Câu điều kiện loại II, giả sử một việc không có thật ở hiện tại “If + S + Ved, S + would/ could + V”
Tạm dịch: Giá mà (nếu) cô ấy có kinh nghiệm trong ngành nghiên cứu y dược, cô ấy đã có thể ứng tuyển
vào một trong các vị trí trong bệnh viện.
Question 13: Đáp án C
Đáp án C - Câu hỏi về cấu trúc song hành
Sau giới từ (for) cần dùng N hoặc V-ing. Từ còn thiếu được nối song song với “compiling” nên cũng phải
có dạng V-ing.
Tạm dịch: Những người làm vị trí nghiên cứu chịu trách nhiệm cho việc phân loại và lưu trữ dữ liệu khảo
sát.
Question 14: Đáp án A
Câu hỏi nghĩa của từ, chọn A - once (một lần) phù hợp nghĩa.
B. immediately - nghĩa là ngay lập tức (adv)/ ngoài ra còn là liên từ (conj) mang nghĩa “ngay khi” = no
sooner/ hardly.
C. directly (adv): một cách trực tiếp.
D. Yet (adv) - Trạng từ nhấn mạnh, chủ yếu dùng trong câu phủ định và câu hỏi của các thì hoàn thành
Tạm dịch: Mỗi năm một lần, công ty Tarrin Industrial Suppy kiểm tra các tài khoản trên toàn bộ các nhà
máy của nó.
Question 15: Đáp án D
Câu hỏi phrasal verb, chọn D phù hợp nghĩa.
A. use up: dùng hết
B. believe in: tin tưởng
C. make into: chuyển thành
D. allow for: cân nhắc
Tạm dịch: Lịch trình của dự án chiếu sáng lối đi đã được kéo dài để cân nhắc các thông tin từ Uỷ ban Môi
trường.
Question 16: Đáp án C
Câu hỏi kết hợp từ, chọn đáp án C phù hợp tạo thành cụm cố định “go bankrupt”: phá sản.

Trang 10
A. come (v): đến
B. take (v): lấy đi
C. go (v): rời đi/ trở nên
D. pay (v): trả
Tạm dịch: Nếu bạn không muốn doanh nghiệp của mình phá sản, tránh hỏi ý kiến những người luôn có
các ý kiến khác biệt
Question 17: Đáp án D
Đề bài cho hai mệnh đề, cần dùng một liên từ (conj) để nối. Chọn đáp án D phù hợp về nghĩa
A. on behalf of (prep): thay mặt cho
B. whether (conj): liệu rằng có hay không
C. moreover (adv): trạng từ nối: ngoài ra/ hơn nữa
D. As = because = bởi vì
Tạm dịch: Vì dòng tai nghe không dây BPT39 rất được ưa chuộng, việc sản xuất sẽ được tăng gấp 5 bắt
đầu từ tháng sau.
Question 18:
Câu hỏi thành ngữ: “paddle sb’s own canoe”: tự làm công việc của mình
A. boat (n): con tàu
B. canoe (n): ca nô
C. ship (n): tàu lớn đi biển
Tạm dịch : Jack rất tự lập, anh ta luôn tự làm các công việc của mình mà không cần người khác giúp
Question 19:
Came → come
Mệnh đề chính của câu điều kiện là một câu mệnh lệnh nên đây là câu điều kiện loại I, nghĩa là mệnh đề
“if” phải chia ở thì Hiện Tại.
Tạm dịch: Nếu bạn đến dự tiệc ngày mai, đừng quên đưa cả bố mẹ đến nhé.
Question 20: Đáp án C
Đáp án C
Part → partly
“the result of fortunate timing” là cụm danh từ làm bổ ngữ. Trước bổ ngữ và sau động từ “was” ta cần
một trạng từ bổ sung nghĩa cho bổ ngữ.
Tạm dịch: Thậm chí giám đốc điều hành cũng phải thừa nhận rằng chiến thắng của Prasma Designs một
phần là do thời điểm may mắn.
Question 21:
And → or
Câu này đơn thuần sai về nghĩa, sửa thành “or” để phù hợp nghĩa.
Tạm dịch: Để chuẩn bị cho giải ma-ra-tông, tôi chạy 15km 3 hoặc 4 lần một tuần.

Trang 11
Question 22: Đáp án D
Tạm dịch:“Tối qua ông ở đâu, ông Jenkins?” anh ta hỏi.
Khi viết lại câu tường thuật, cần phải lùi một thì (were → had been) và thay đổi trạng từ thời gian
(yesterday → the previous night). Động từ tường thuật của câu hỏi là “asked” hoặc “wanted to know”
Câu A sai vì chưa lùi thì (was)
Câu B sai vì chưa đổi trạng từ thời gian (last night)
Câu C sai vì sai trạng từ thời gian (last night khi chuyển trạng từ dùng the following night là không đúng)
Question 23: Đáp án D
Tạm dịch: Kẻ trộm bị bắt ngay tại trận bởi cảnh sát khi hắn ta đang đột nhập vào trong căn hộ.
Cấu trúc “catch + someone + V-ing”: bắt được ai đó khi họ đang làm một việc bất kì.
Chọn đáp án D phù hợp về nghĩa và cấu trúc với “catch”.
A. Cảnh sát bắt kẻ trộm để đột nhập vào căn hộ → sai nghĩa/ sai cấu trúc.
B. Cảnh sát bắt kẻ trộm khi họ đột nhập vào căn hộ → Sai vì dùng mệnh đề phân từ nối hai câu cùng chủ
ngữ, nghĩa là cảnh sát mới là người đột nhập.
C. Kẻ trộm đột nhập vào căn hộ thì cảnh sát bắt anh ta ngay lập tức → sai nghĩa
D. Cảnh sát bắt được kẻ trộm đang đột nhập vào căn hộ. → đúng.
Question 24: Đáp án A
Tạm dịch: Chúng ta sẽ đi cắm trại miễn là trời đẹp.
“As long as” cũng là một liên từ điều kiện giống như “if”, vậy đây cũng được coi như một câu điều kiện
loại I.
A. Nếu trời đẹp, chúng ta sẽ đi cắm trại → dùng câu ĐK loại I là hợp lí
B. Trời đẹp khi chúng ta sẽ đi cắm trại → sai nghĩa
C. Nếu trời đẹp hơn, chúng ta sẽ đi cắm trại → sai nghĩa
D. Chúng ta sẽ đi cắm trại ngay lập tức khi trời đẹp → sai nghĩa
Question 25: Đáp án A
Tạm dịch: Chúng tôi không chọn đảng cầm quyền. Chúng tôi không chọn đảng đối lập
Để nối song song hai cụm phủ định, ta dùng “neither A nor B”, chọn đáp án A vừa đúng vừa phù hợp
nghĩa.
A. Chúng tôi sẽ không chọn cả đảng đang cầm quyền lẫn đảng đối lập trong cuộc bầu cử sắp tới.
B. Đáp án B không dịch được tương đồng vì dùng 2 phủ định trong câu → sai ngữ pháp và sai nghĩa.
C. Chúng tôi sẽ chọn đảng đang cẩm quyền hoặc đảng đối lập trong kì bầu cử sắp tới → sai nghĩa.
D. Chúng tôi sẽ không chọn một trong hai đảng là đảng đang cầm quyền hoặc đảng đối lập trong kì bầu
cử sắp tới → sai nghĩa.
Mở rộng: Để nối song song với nghĩa A hoặc B, ta dùng “either A or B”.
Question 26:
Tạm dịch: Ngọn hải đăng được xây dựng vào những năm 1600. Nó vẫn tồn tại hàng trăm năm

Trang 12
Dùng phân từ để nối hai mệnh đề cùng chủ ngữ. Động từ “build” là ngoại động từ. Ở 4 đáp án, sau
“build” không có danh từ làm tân ngữ nên phải để ở dạng phân từ 2 (phân từ bị động). Ở câu này hai
mệnh đề có năm 1600 và thì hiện tại rõ ràng không cần phải thể hiện sự trước sau nên không cần dùng
dạng “having + P2”.
A. Sai vì dùng dạng “having + P2”.
B. Sai vì dùng phần từ 1 (chủ động)
C. đúng
D. Sai vì dùng dạng “having + P2” và là phân từ chủ động.
Question 27:
Tạm dịch: Laura đang cảnh báo Bob về căn nhà
Laura: Cẩn thận đầu nhé. Trần nhà thấp lắm
Bob:____________
Dạng câu cảnh báo/ khuyên nhủ nên trả lời bằng một câu cám ơn.
A. Don’t mention it - Dùng để trả lời cho lời cám ơn với nghĩa là “không có gì đâu”.
B. Thanks. I’ll remember it: Cám ơn, mình nhớ rồi - hợp lí về nghĩa
C. I couldn’t agree more - Dùng để tán thành. Thường dùng để trả lời những câu đưa ra ý kiến.
D. I don’t think you are right: Tôi không nghĩ rằng bạn đúng - Dùng để phản đối ý kiến
Question 28:
Tạm dịch: David đang nói chuyện với Linda sau bữa tiệc
David:“Bạn có muốn mình chở bạn về nhà không?”
Linda:____________
Dạng câu đưa ra lời đề nghị giúp đỡ nên chọn đáp án cám ơn hoặc khen ý kiến đó.
A. Thế thì tuyệt quá,cảm ơn nhé → hợp lí về nghĩa
B. Xin lỗi, bạn không phải là gu của tôi
C. Vâng, tôi đang đi về nhà
D. Không, cảm ơn. Tôi không thích cưỡi.
Question 29:
Tạm dich: Tổ chức Hướng Đạo Sinh Nam cống hiến hết mình để giúp đỡ các chàng trai trở thành những
người lớn có đạo đức và làm việc hiệu quả.
A. be dedicated to = be commited to (+ V-ing): Cống hiến làm gì đó
B. used: được sử dụng
C. focused: tập trung
D. interested: yêu thích, quan tâm đến
Question 30:
Tạm dịch: Bố đã mất việc rồi vì thế chúng tôi phải thắt lưng buộc bụng để tránh rơi vào nợ nần “to tighten
one’s belt” (thành ngữ): thắt lưng buộc bụng,tiết kiệm

Trang 13
A. earn money: kiếm tiền
B. save money: tiết kiệm tiền
C. sit still: ngồi yên một chỗ
D. to tighten one’s belt= economize: thắt lưng buộc bụng, tiết kiệm
Question 31:
Tạm dịch: Henry đã tìm được một công việc tạm thời trong một nhà máy
Temporary: Tạm thời
A. eternal(a): bất diệt ,vĩnh cửu
B. genuine(a): thật, đúng như người ta nói
C. permanent: lâu dài, dài hạn >< temporary (a): tạm thời, nhất thời
D. satisfactory(a): hài lòng,vừa lòng, thỏa đáng
Question 32:
Tạm dịch: Chúng tôi rất phấn khởi bởi mọi việc đều khá thuận lợi
“Had a whale of time” (thành ngữ): Tận hưởng sự phấn khích/ phấn khởi.
A. had little time to play: có ít thời gian chơi
B. had a lot of time to play:có nhiều thời gian chơi
C. felt happy:cảm thấy hạnh phúc
D. to feel disappointedưhất vọng >< to have a whale of time: phấn khởi
Question 33:
...text messaging may actually be having a negative _____________ on young people’s communication and
language skills:....nhắn tin có thể thực sự có_____________tiêu cực tới kỹ năng ngôn ngữ và giao tiếp
của những người trẻ tuổi. Giới từ đi sau là “on” nên danh từ cần tìm là “effect”.
A. result (n): kết quả/ thường đi với giới từ “of”
B. outcome (n): đầu ra, kết quả/ thường đi với giới từ “of”
C. effect(n): ảnh hưởng/ đi với giới từ “on”
D. conclusion(n): kết luận/ thường đi với giới từ “of” hoặc liên từ “that” + mệnh đề.
Question 34:
...So widespread has texting become, however, that even pigeons have started doing it.____________, in
this case, it’s difficult to view the results as anything but positive: Tuy nhiên nhắn tin trở nên quá phổ
biến đến nỗi thậm chí những con bồ câu đang bắt đầu nhắn tin. ____________, trong trường hợp này, rất
khó để xem xét các kết quả không có gì khác là tích cực.
Cần dùng một trạng từ nối câu vì chỗ trống nằm giữa một dấu chấm và một dấu phấy. Xét về nghĩa thì
“What’s more” phù hợp.
A. Therefore (adv): vì vậy,do đó
B. What’s more (adv): Quan trọng hơn
C. Whereas (conj): trong khi đó

Trang 14
D. That is - mệnh đề này thiếu, không có nghĩa.
Question 35: Đáp áp A
...each____________with sensor equipment and a mobile phone: mỗi____________với thiết bị cảm ứng
và một chiếc điện thoại di động. Dùng mệnh đề phân từ nối hai câu khác chủ ngữ. Trong trường hợp này
cả 4 phương án đều là phân từ 2 nên chỉ cần xét nghĩa. Đáp án A phù hợp.
A. armed: được trang bị (to be armed with sth)
B. loaded: được nạp, được chất lên (to be loaded with sth)
C. granted: được trợ cấp,được ban cho
D. stocked: dự trữ, chất kho
Question 36:
….The readings made by the sensors will be automatically converted into text messages and beamed to
the Internet____________ - they will appear on a dedicated “pigeon blog”: Những số liệu thu từ máy
cảm ứng sẽ tự động đổi thành tin nhắn và phát vào Internet ______________chúng sẽ xuất hiện ở mục
nhật kí bồ câu.
Câu sau chỗ trống không thiếu thành phần câu nào nên chỉ có thể thêm một trạng ngữ bắt đầu mệnh đề
quan hệ. Chọn where vì Internet là không gian mạng, không dùng để chỉ thời gian.
A. when (adv): khi mà
B. which (pron): cái mà
C. where (adv): nơi mà
D. what (pron): cái mà
Question 37: Đáp án C
Each bird will carry these a miniature backpack, (37) _____________ , that is, from the camera, which
will hang around its neck.
Giới từ đề bài sẵn cho là “from” → chọn C và tạm dịch câu thành: Mỗi chú chim sẽ mang những dụng cụ
này trong một chiếc balo siêu nhỏ được đeo ở cổ bên cạnh chiếc camera.
A. instead of: thay vào đó
B. except for: ngoại trừ
C. apart from: ngoài ra/ bên cạnh
D. besides: bên cạnh đó
Question 38: Đáp án C
Dịch câu hỏi: Điều nào sau đây sẽ là một tiêu đề tốt hơn cho đoạn văn?
The practice of signing and numbering individual prints was introduced by James Abbott McNeill
whistler: Thông lệ kí và đánh số vào các bản vẽ được khởi sướng bởi James Whisler.
Từ sign và print được lặp đi lặp lại nhiều lần trong bài và cả hai đều nói về việc họa sĩ đã đi đến sáng kiến
kí tên vào bản vẽ của mình như thế nào → Chọn C.

Trang 15
A. Mẹ của Whistler → chỉ được nhắc đến như tên bức tranh, không phải là một nội dung xuyên suốt toàn
bài.
B. Các tác phẩm vĩ đại nhất của whistler → bài chỉ nhắc đến một tác phẩm của họa sỹ này, không phải là
một nội dung xuyên suốt toàn bài
D. Sao chép các ấn bản in có giới hạn → không liên quan đến nội dung bài.
Question 39:
Dịch câu hỏi: Từ “speculated” trong đoạn một có thể được thay thế bởi____________
...Sir Francis Seymour Haden, a less well - known artist, had speculated that collectors might find prints
more attractive if they knew that they were only a limited number of copies produced: Sir Haden, một họa
sĩ kém tên tuổi hơn, đã speculated rằng các nhà sưu tầm tranh sẽ đánh giá cao bức tranh hơn nếu như họ
biết rằng chỉ có một số lượng có hạn bản sao được vẽ.
Speculate (v): dự đoán/ tiên đoán
A. guess (v) = speculate (v): dự đoán/ tiên đoán
B. notice (v): nhận thấy
C. deny (v): chối
D. announce (v): tuyên bố
Question 40: Đáp án B
Dịch câu hỏi: Điều gì đã đúng về bức tranh của mẹ Whistler?
...the painting of his mother, called “Arrangement in Grey and Black”, but known to most of us as“
whistler’s Mother” :bức vẽ của mẹ ông, có tên “Sự sắp đặt trong màu xám và đen” nhưng thường được
biết tới dưới cái tên “Mẹ của whistler”
Câu trên cho thấy bức tranh “Whistlers mother” có tên chính thức là “Arrangement of Grey and Black”.
Chọn B đúng.
A. nó được vẽ bởi Sir Francis Haden
B. tên của nó là “Arrangement in Grey and Black”
C. nó không phải là một trong những bức tranh tuyệt nhất của whistler
D. đó là một phương pháp vẽ mới hoàn toàn.
Question 41: Đáp án D
Dịch câu hỏi: Tác giả đề cập đến tất cả những điều sau đây là lý do tại sao một nhà sưu tập thích một bản
in có chữ ký NGOẠI TRỪ?
“…collectors might find prints more attractive if they knew that they were only a limited number of
copies produced. By signing the work in pencil, an artist could guarantee and personalize each print”: các
nhà sưu tầm tranh sẽ đánh giá cao bức tranh hơn nếu như họ biết rằng chỉ có một số lượng có hạn bản sao
được vẽ. Bằng cách kí bút chì vào bức tranh, họa sĩ có thể đảm bảo và cá nhân hóa cho bức tranh.

Trang 16
Và: “... .As soon as whistler and Haden began the practice of signing and numbering their prints, their
work began to increase in value”: Ngay khi whistler và Haden bắt đầu việc kí tên và đánh dấu lên các bản
in, tác phẩm của họ bắt đầu tăng giá”.
Hai đoạn trích dẫn trên không nhắc đến lý do “việc này hạn chế số lượng bản sao”, nên chọn D là đúng.
A. việc này đảm bảo tính xác thực của bức tranh
B. việc này khiến bức tranh trở nên riêng tư hơn
C. Việc này khiến bức tranh có giá cao hơn
D. Việc này hạn chế số lượng bản sao của bức tranh
Question 42: Đáp án B
Dịch câu hỏi: Có thể suy ra từ đoạn văn rằng các nghệ sĩ đánh số bản in của họ_____________?
...collectors might find prints more attractive if they knew that they were only a limited number of copies
produced: các nhà sưu tầm tranh sẽ đánh giá cao bức tranh hơn nếu như họ biết rằng chỉ có một số lượng
có hạn bản sao được vẽ.
Đoạn trích trên cho thấy việc đánh số vào tranh đảm bảo rằng các bản vẽ này là bản có số lượng hữu hạn.
chọn B là phù hợp.
A. như là một thủ tục kế toán
B. đảm bảo bản vẽ này là có hạn
C. khi người mua yêu cầu
D. ở cùng một nơi trên mỗi bản in
Question 43:
Dịch câu hỏi: từ “striking” ở đoạn 2 đồng nghĩa với từ nào?
Thông tin nằm ở: ... Perhaps the aspect of butterfly diversity that has received the most attention over the
past century is the striking difference in species richness between tropical and temperate regions”: Có lẽ
khía cạnh nhận được nhiều sự quan tâm nhất trong thế kỉ vừa rồi về sự đa dạng của bướm là sự khác biệt
đáng chú ý trong số lượng giống loài giữa khu vực nhiệt đới và ôn đới.
Striking (a): Đáng chú ý/ đáng để tâm
A. physical (a): thuộc về thể chất
B. confusing (a): gây khó hiểu
C. noticable (a): đáng chú ý = striking
D. successful (a): thành công
Question 44:
Dịch câu hỏi: Bướm là một ví dụ điển hình để truyền đạt thông tin về các vấn đề bảo tồn vì
chúng___________
Butterflies also have a favorable image with the general public. Hence, they are an excellent group for
communicating information on science and conservation issues such as diversity: loài bướm cũng giữ một

Trang 17
hình ảnh tốt đẹp trong mắt con người. Thế nên, chúng là nhóm động vật tuyệt vời để truyền đạt thông tin
về khoa học và các vấn đề bảo tồn ví dụ như đa dạng sinh học.
Đoạn trích trên cho thấy bướm được chọn làm ví dụ điển hình vì chúng có hình ảnh tốt đẹp trong mắt con
người.
A. đơn giản trong cấu trúc
B. được đặt cho những cái tên khoa học
C. là hình ảnh tích cực trong mắt con người
D. được tìm thấy phần lớn ở khí hậu ôn đới
Question 45:
Dịch câu hỏi: Từ vượt “exceed” trong đoạn 3 có nghĩa gần nhất với_____________
...the total number found on the British islands did not exceed 66: Tổng số được tìm thấy trên đảo Anh
không exceed 66.
Exceed (v): Vượt quá.
A. locate (v): định vị
C. go beyond (phr v): vượt quá = exceed
B. allow (v): cho phép
D. come close to (phrase): đến gần
Question 46: Đáp án C
Dịch câu hỏi: Điều nào sau đây KHÔNG được các nhà sinh học hiểu rõ?
However, for butterflies, variation of species richness within temperate or tropical regions, rather man
between them, is poorly understood: Tuy nhiên, với loài bướm, sự biến đổi về mức độ phong phú của loài
trong vùng ôn đới và nhiệt đới chứ không phải giữa các loài, thì không được hiểu rõ.
Đoạn trích cho thấy sự khác biệt trong số lượng loài nội trong các khu vực ôn đới và nhiệt đới vẫn chưa
được hiểu rõ; “poorly understood” - chưa được hiểu rõ. Chọn C là hợp lý.
A. Môi trường sống của loài bướm châu âu
B. Sự khác biệt độ phong phú của loài giữa các vùng ôn đới và nhiệt đới
C. Sự khác biệt giữa số lượng loài nội trong một khu vực ôn đới hoặc nhiệt đới
D. So sánh hành vi của loài bướm với các nhóm động vật nhất định
Question 47: Đáp án C
Dịch câu hỏi: Tất cả những điều sau đây được đề cập là những phần quan trọng của một lý thuyết chung
về sự đa dạng NGOẠI TRỪ ?
A general theory of diversity would have to predict not only this difference between temperate and
tropical zones, but also patterns within each region, and how these patterns vary among different animal
and plant groups: một lí thuyết tổng quát về sự đa dạng đã phải dự đoán được không chỉ sự khác biệt giữa
các khu vực ôn đới và nhiệt đới mà còn các kiểu mẫu theo từng vùng và có sự biến đổi của những kiểu
mẫu giữa các nhóm động vật thực vật khác nhau.

Trang 18
Cấu trúc “not only..but also ...’’(không những mà còn) đã đưa ra 3 phần quan trọng của lí thuyết này.
Trong đó không đề cập đến “Việc di cư trong khu vực ôn đới và nhiệt đới” vậy nên chọn C là hợp lý.
A. sự khác biệt giữa khu vực ôn đới và hàn đới
B. kiểu phân bố giống loài ở từng khu vực
C. sự di cư trong khu vực ôn đới và nhiệt đới
D. những khác biệt trong kiểu phân bố giống loài trong các loại động vật và thực vật khác nhau
Question 48: Đáp án D
Dịch câu hỏi: Những khía cạnh của bướm mà đoạn văn chủ yếu thảo luận?
Butterflies are among the most extensively studied insects—an estimated 90 percent of the world’s
species have scientific names. As a consequence, they are perhaps the best group of insects for
examining patterns of terrestrial biotic diversity and distribution: Bướm là một trong những loài côn trùng
được nghiên cứu rộng rãi nhất, ước tính 90% các loài trên thế giới có tên khoa học. Kết quả là, chúng có
lẽ là nhóm côn trùng tốt nhất để kiểm tra các mô hình phần bố và phân bố sinh học trên cạn.
Đoạn trích cho thấy nội dung của đoạn văn chủ yếu thảo luận về sự đa dạng của bướm, chọn D là hợp lý.
A. đặc điểm thể chất của chúng
B. sự thích của chúng đối với các môi trường khác nhau
C. tên của chúng
D. sự đa dạng của chúng
Question 49: Đáp án B
Dịch câu hỏi: Từ “they” trong đoạn 1 chỉ?
Butterflies also have a favorable image with the general public. Hence, they are an excellent group for
communicating information
Từ “they” ở đây sẽ liên quan đến danh từ được nhắc đến trước đó.
Dựa vào nghĩa của 2 câu trên ta thấy đáp án đúng là B.
A. các loại côn trùng
B. các loại bướm
C. các kiểu
D. các vấn đề
Question 50:
Dịch câu hỏi: Ý kiến “little is known about the eveness of butterfly distribution” trong đoạn 5 là?
little is known about the eveness of butterfly distribution: Chúng ta không biết gì về sự đồng đều
trong sự phân bố của bướm.
Đoạn trích cho thấy chúng ta không biết gì về sự đồng đều trong việc phân bố của loài bướm. Chọn B là
hợp lý.
A. có nhiều điều chúng ta biết về sự phân bố đồng đều của bướm
B. chúng ta không biết gì về sự phân bố đồng đều của bướm

Trang 19
C. chúng ta biết nhiều về sự phân bố đồng đều của bướm
D. chúng ta biết về sự phân bố đồng đều của bướm ở một mức độ nào đó

Trang 20
ĐỀ SỐ 19 ĐỀ THI THỬ TỐT NGHIỆP THPT
NĂM HỌC: 2020 – 2021
MÔN: TIẾNG ANH
Thời gian làm bài: 60 phút; không kể thời gian phát đề

Mark the letter A, B, C, or D on your answer sheet to indicate the word whose underlined part
differs from the other three in pronunciation in each of the following questions.
Question 1. A. workshop B. worm C. worry D. wordless
Question 2. A. borrowed B. helped C. dismissed D. booked
Mark the letter A, B, C or D on your answer sheet to indicate the word that differs from the rest in
the position of the main stress in each of the following questions.
Question 3. A. dissolve B. household C. confide D. approach
Question 4. A. determine B. argument C. counterpart D. marvelous
Mark the letter A, B, C or D on your answer sheet to indicate the correct answer to each of the
following questions.
Question 5. A few days ago, ____________problem arise. However, until now, we haven’t found any
solutions to it yet.
A. a B. an C. the D. ∅
Question 6. Keith deserves____________in the conference for all the help she has given us.
A. to mention B. mention C. mentioning D. mentioned
Question 7. This is not____________the last one we stayed in.
A. as a comfortable hotel as B. an as comfortable hotel as
C. a as comfortable hotel as D. as comfortable a hotel as
Question 8. There are dozens of TV channels, ____________operate 24 hours a day.
A. some B. some of which C. some of those D. some of them
Question 9. Thirty-one people have been injured in____________incidents throughout the day.
A. violent B. violence C. violently D. violate
Question 10. what chemical is this? It's____________a horrible smell.
A. giving over B. giving off C. giving down D. giving up
Question 11. Mary will have finished all her work ____________
A. as soon as her boss returned B. until her boss will return
C. by the time her boss returns D. when her boss will return
Question 12. So little____________about mathematics that the lecture was beyond completely
beyond me.
A. I have known B. I knew C. do I know D. did I know

Trang 1
Question 13. It is probably impossible for life to ever exist on Venus____________its intense surface
heat.
A. because B. although C. despite D. due to
Question 14. If you cannot improved sales figures this month, you will be____________sacked.
A. promise B. conduct C. regulate D. deliver
Question 15. At the meeting last week, the finance director____________the figures for the previous
year.
A. produced B. represented C. purchased D. commended
Question 16. I think Trump can easily win the election. He is a man of the____________.
A. community B. society C. country D. people
Question 17. Overcrowded slums are often considered as breeding ground____________crime.
A. of B. to C. for D. towards
Question 18. The military court____________ the major for failing to do his duty.
A. scolded B. reprimanded C. humiliated D. ridiculedƯ
Mark the letter A, B, C, or D on your answer sheet to indicate the underlined part that needs
correction in each of the following questions.
Question 19. A severe illness when she was just nineteen months old deprived the well- known writer
and lecturer Helen Keller from both her sight and hearing.
A. just B. deprived C. when D. from both
Question 20. He didn’t know who it was and couldn’t imagine why they did it, but there must have
been somebody that started the rumor which he was from London and very wealthy.
A. which B. who C. why D. that
Question 21. It is believed that in the near future robots will be used to doing things such as cooking.
A. such as B. in the near future
C. It is believed D. be used to doing
Mark the letter A, B, C or D on your answer sheet to indicate the sentence that is closest in
meaning to each of the following questions.
Question 22. It would have been better if he had told us his new address.
A. He might have told us his new address.
B. He should have told us his new address.
C. He shouldn’t have told us his new address.
D. It doesn’t matter that he didn’t tell us his new address.
Question 23. She knows more about it than I do.
A. I know as much about it as she does.

Trang 2
B. She knows as much about it as I do.
C. I don’t know as much about it as she does.
D. She doesn’t know as much about it as I do.
Question 24. “Why don’t you participate in the volunteer work in summer?”, said Sophie.
A. Sophie suggested me to participate in the volunteer work in summer.
B. Sophie asked me why not participate in the volunteer work in summer.
C. Sophie suggested my participating in the volunteer work in summer.
D. Sophie made me participate in the volunteer work in summer.
Mark the letter A, B, C or D on your answer sheet to indicate the sentence that best combines
each pair of sentences in the following questions.
Question 25. We arrived at the airport. We realized our passports were still at home.
A. It was until we arrived at the airport that we realized our passports were still at home.
B. We arrived at the airport and realized that our passports are still at home.
C. Not until had we arrived at the airport we realized our passports were still at home.
D. Not until we arrived at the airport did we realize that our passports were still at home.
Question 26. Nam was so rude to them last night. Now he feels regretful.
A. Nam regrets to have been so rude to them last night.
B. Nam regrets having so rude to them last night.
C. Nam wishes he hadn’t been so rude to them last night.
D. Nam wishes he weren’t so rude to them last night.
Mark the letter A, B, C or D on your answer sheet to indicate the most suitable response to
complete each of the following exchanges.
Question 27. - Laura: “What a lovely cat you have!”
- Maria: “____________.”
A. Of course not, it’s not costly
B. Thank you. Thank you. My father bought it for me
C. I think so
D. No problem
Question 28. - Janet: "Do you feel like going to the cinema this evening?"
- Susan: “____________’’
A. I don't agree, I'm afraid B. You’re welcome
C. That would be great D. I feel very bored
Mark the letter A, B, C, or D on your answer sheet to indicate the word or phrase that is
CLOSEST in meaning to the underlined part in each of the following questions.

Trang 3
Question 29. she got up late and rushed to the bus stop.
A. came into B. went leisurely C. dropped by D. went quickly
Question 30. School uniform is compulsory in most of Vietnamese schools.
A. divided B. paid C. required D. depended
Mark the letter A, B, C, or D on your answer sheet to indicate the word or phrase that is
OPPOSITE in meaning to the underlined part in each of the following questions.
Question 31. I must have a watch since punctuality is imperative in my new job.
A. being courteous B. being cheerful C. being efficient D. being late
Question 32. My sister lives in Alaska, so we can only see each other once in a blue moon.
A. occasionally B. rarely C. at night D. frequently
Read the following passage and mark the letter A, B, C or D on your answer sheet to indicate the
correct word for each of the blanks from 33 to 37.
VIRTUAL DOCTORS
Clare Harrison rarely falls ill and hates going to the doctors when she does. So when she recently (33)
____________out in a painful rash down one side of her body she emailed her symptoms, (34)
____________also included a (35) ____________fever, to e-doc, the internet medical service. Two
hours later she was diagnosed as having shingles (Herpes Zoster) by her online doctor, who prescribed
a special cleansing solution for the rash and analgesics to help relieve the pain.
Health advice is now the second most popular topic that people search for on the internet, and online
medical consultation is big business. Sites vary enormously in what they offer, with services ranging
from the equivalent of a medical agony aunt to a live chat with a doctor via email. They are clearly (36)
____________a demand from people who are too busy or, in some cases, too embarrassed to discuss
their medical (37) ____________with their general practitioners.
Question 33. A. worked B. passed C. came D. ran
Question 34. A. whose B. which C. who D. that
Question 35. A. small B. weak C. mild D. calm
Question 36. A. serving B. meeting C. creating D. establishing
Question 37. A. harm B. story C. hardship D. complaint
Read the following passage and mark the letter A, B, C, or D on your answer sheet to indicate the
correct answer to each of the questions from 38 to 42.
Dolphins are one of the most intelligent species on the planet, which makes them a very interesting
animal to scientists. In their natural habitats, dolphins use various vocalization techniques. They
whistle and squeak to recognize members of their pod, identify and protect their young, and call out

Trang 4
warnings of danger. They also make clicking sounds used for echolocation to find food and obstacles in
dark and murky waters.
Amazingly, the whistling sound that the bottlenose dolphin makes has been found to have a similar
pattern to human language. They always make conversational sounds when they greet each other. If you
listen to dolphins' squeaks and squeals, it will sound like they are having a conversation.
Dolphins usually use both sound and body language to communicate with each other. It is through
gesture and body language, however, that most of their communication with humans comes. Dolphins
can be trained to perform complicated tricks. This suggests they have a high level of intelligence and
communication capacity. If they work for a long time with a trainer, they are able to recognize and
understand human commands.
A lot of dolphin communication has been studied using dolphins in captive environments. These studies
have been criticized because some marine biologists believe that dolphins living in aquariums or
research centers cannot be considered "normal." Even so, most believe that studying dolphin
communication in captivity is useful for beginning to understand the complexity of dolphin
communication. After all, dolphins are one of the most intelligent animals. Their ability to
communicate is impressive and worthy of study.
Question 38. What would be the most suitable title for the passage?
A. Communications in dolphins
B. Intelligent mammals in captivity
C. Dolphins' social tendencies with humans
D. Complex patterns in dolphin life
Question 39. In line 2, the word vocalization is closest in meaning to____________.
A. making gestures B. creating words C. producing sounds D. closing eyes
Question 40. According to paragraph 4, what do some marine biologists think about captive
dolphins?
A. Their relationships are too personal with marine biologists.
B. They can't be considered accurate subjects for biological studies.
C. They can communicate exactly the same as humans do.
D. They easily demonstrate how dolphins act in the wild.
Question 41. All of the following are true about dolphin communication EXCEPT____________
A. dolphins' squeals and squeaks sound conversational to the human ear
B. dolphins' clicking sounds are sometimes used to greet humans
C. communication using sounds and gestures occurs between dolphins
D. body language and gestures are used for communication with humans

Trang 5
Question 42. In line 17, the word “most” refers to ____________
A. the majority of marine biologists B. a few bottlenose dolphins
C. a lot of bottlenose dolphins D. the minority of marine biologists
Read the following passage and mark the letter A, B, C, or D on your answer sheet to indicate the
correct answer to each of the questions from 43 to 50.
What does it take to graduate from university with a First?
Mark McArdle, first-class degree holder from the University of Lancaster, tells how he did it.
Don’t spend too much time at the student lounge, do turn up for most lectures and tutorials and do
submit all coursework - eventually. That, I was told by a PhD student during freshers’ week, was all I
needed to do to get a 2:2. For a 2:1, I’d require a better attendance record and have to work harder, but
not at the expense of being cut off from civilization. And for a First I would have to become some sort
of social outcast, go to even lecture and tutorial (scribbling notes madly), spend every waking moment
immersed in academic books, and be among the last to be thrown out of the university library at 10 pm
closing time.
Well, I did not give up my life for study. I didn't attend every lecture and tutorial. I didn’t write down
every word spoken in lectures. I didn’t get 80% or more in every essay, project, test or exam. I was
usually behind with my reading and occasionally mystified by the syllabus. Sometimes I couldn't be
bothered to go to university and stayed at home instead. But I always knew where I was what I had to
do and what not to bother with. And I always worked hard on the things that counted: assignments and
exams.
Getting a degree is about learning, but it isn’t just about learning biology, history, English or whatever.
It’s about understanding what you need to succeed - what, in fact, the university
wants from you and what you will get in return. You have to have a feel for the education market and
really sell your inspirations, what does the lecturer want? what is the essay marker searching for? Some
students try to offer something not wanted. Others want to give very little - they steal the thoughts of
others and submit them as their own. But they all want to be rewarded. Exchange, but don’t steal, and
you’ll get a degree.
I could guarantee every book on my reading list was out on long loan from the university library within
five seconds of the list being issued. This was worrying at first, but I quickly learned that it was
impossible to read all of the books on an average reading list anyway. I sought shortcuts. Collections of
selected readings or journal articles were excellent sources that often saved me the bother of reading the
original texts. References in books dragged me all over the place but with all the courses I had to do,
there wasn't enough time to be dragged too far. I would flick through the book, read the introduction,

Trang 6
note any summaries, look at diagrams, skim the index, and read any conclusions. I plucked out what
was needed and made my escape.
I revised by discarding subject areas I could not face revising, reading, compiling notes, and then
condensing them onto one or two sheets of A4 for each subject area. Leading up to the exam, I would
concentrate on just the condensed notes and rely on my memory to drag out the detail behind them
when the time came. I didn’t practice writing exam questions, although it was recommended. I prefer to
be spontaneous and open-minded. I don't want pre-formed conclusions filling my mind.
And nor should you; there is no secret to getting a First - this is just an account of how I got my First.
Be a happy student by striking the right balance between working and enjoying yourself. Take what you
do seriously and do your best. And, no matter what you do don’t forget to appreciate every day of your
university studies: it is one of the greatest periods of your life.
Glossary:
University degree classifications in the UK:
- First class
- Upper second (2:1)
- Lower second (2:2)
- Third class (3)
Question 43. What is the main purpose of the passage?
A. To describe learning experience of an excellent student
B. To emphasize the importance of higher education
C. To suggest ways to deal with assignments at university
D. To point out challenges of studying at university
Question 44. The PhD students who spoke to the writer____________.
A. exaggerate the need to work hard B. succeeded in scaring the writer
C. was uncertain how to help the writer D. thought the writer would get a First
Question 45. In the third paragraph, the writer warns against____________.
A. offering money to academic staff B. trying to second-guess lectures
C. expecting to be rewarded D. plagiarism in essays and exams
Question 46. The word “immersed” in paragraph 1 is closest in meaning to____________.
A. ploughed B. absorbed C. sunk D. dipped
Question 47. Why did the writer not practice writing exam questions?
A. He thought the practice was rather boring.
B. He wanted to answer exam questions critically.
C. He was advised not to.

Trang 7
D. He thought it might prejudice staff against him.
Question 48. The word “flick through” in paragraph 4 is closest in meaning to____________.
A. search quickly B. skim C. borrow D. read carefully
Question 49. All of the following are mentioned about factors that help the writer to succeed at
university EXCEPT ____________.
A. an understanding of what was required
B. regular attendance at lectures
C. selective reading
D. well-organizing revision
Question 50. It can be inferred from what the write said in the last paragraph that____________ .
A. student’s top priority is to pass exams
B. teachers should set goals for students right from the first year
C. students should follow his recipe for success
D. students should make the most of being a student

Trang 8
Đáp án
1-C 2-A 3-B 4-A 5-A 6-C 7-D 8-B 9-A 10-B
11-C 12-D 13-D 14-D 15-A 16-D 17-C 18-B 19-D 20-A
21-D 22-B 23-C 24-C 25-D 26-C 27-B 28-C 29-D 30-C
31-D 32-D 33-C 34-B 35-C 36-B 37-D 38-A 39-C 40-B
41-B 42-A 43-A 44-A 45-D 46-B 47-B 48-B 49-B 50-D

LỜI GIẢI CHI TIẾT


Question 1: Đáp án C
Đáp án C đúng vì phần gạch chân của đáp án C là âm /ʌ/. Các đáp án còn lại có phần gạch chân là âm /ɜ:/.
A. workshop /ˈwɜːkʃɒp/ (n): hội thảo
B. worm /wɜːm/ (n): sâu/ giun
C. worry /ˈwʌri/ (n)/ (v): mối lo/ lo lắng
D. wordless /ˈwɜːdləs/ (a): không nói nên lời
* Mở rộng: worry có hai phiên âm, là / 'wʌri/ và /'wɜ:ri/. Tuy nhiên, trong trường hợp này, ba đáp án còn
lại đều có phần gạch chân là âm /ɜ:/ nên từ worry cần có phần gạch chân là âm /ʌ/ để đảm bảo đáp án.
Question 2. Đáp án A
Đáp án A đúng vì phần gạch chân của đáp án A là âm /d/. Các đáp án còn lại có phần gạch chân là âm /t/.
A. borrowed /ˈbɒrəʊ/: mượn
B. helped /helpt/: giúp đỡ
C. dismissed /dɪsˈmɪst/: giải tán
D. booked /bʊk/: đặt vé/ đặt trước
* Mở rộng: xem lại cách phát âm đuôi “ed” đã có ở các đề trước.
Question 3. Đáp án B
Đáp án B đúng vì đáp án B có trọng âm rơi vào âm tiết thứ nhất. Các đáp án còn lại có trọng âm rơi vào
âm tiết thứ 2.
A. dissolve /dɪˈzɒlv/ (v): hòa tan/ tan ra
B. household /ˈhaʊshəʊld/ (n): hộ gia đình
C. confide /kənˈfaɪd/ (v): tiết lộ/ tâm sự/ thổ lộ
D. approach /əˈprəʊtʃ/ (v)/ (n): tiếp cận/ cách tiếp cận
Question 4. Đáp án A
Đáp án A đúng vì đáp án A có trọng âm rơi vào âm tiết thứ 2. Các đáp án còn lại có trọng âm rơi vào âm
tiết thứ nhất.
A. determine /dɪˈtɜːmɪn/ (v): xác định

Trang 9
B. argument /ˈɑːɡjumənt/ (n): sự cãi vã/ tranh luận/ luận điểm
C. counterpart /ˈkaʊntəpɑːt/ (n): người đồng cấp/ người đồng vị trí
D. marvelous /ˈmɑːvələs/ (a): kì diệu/ thần kì
Question 5. Đáp án A
Đáp án A - câu hỏi mạo từ
“Problem” là danh từ đếm được số ít, nên trước nó buộc phải có một mạo từ hoặc một định từ phù hợp,
nên đáp án D bị loại.
“Problem” được nhắc đến lần đầu trong câu, cũng không phải là danh từ mà mọi người đều đã biết đến,
nên nó chưa xác định, vậy cũng không đi với đáp án C - the.
“Problem” bắt đầu bằng một phụ âm (P) nên sẽ đi với mạo từ ở đáp án A - a.
Tạm dịch: Một vài ngày trước một vấn đề nổi lên. Tuy nhiên, đến hiện tại chúng tôi vẫn chưa tìm ra cách
giải quyết nó.
Question 6. Đáp án C
Đáp án C - câu hỏi bị động
“Deserve” có cấu trúc khá đặc biệt.
Ở thể chủ động, cấu trúc của nó là: S + deserve + to V...
Ở thể bị động, động từ này có hai cách dùng:
1/ S + deserve + to be + P2
2/ S + deserve + V-ing
Xét 4 đáp án, có thể thấy đáp án D và B bị loại vì không theo cấu trúc của “deserve”.
Động từ “mention” là ngoại động từ (loại động từ phải có một danh từ theo sau làm tân ngữ), tuy nhiên
trong trường hợp này đề bài không cho danh từ nào làm tân ngữ ở sau chỗ trống, nên buộc phải chia động
từ “mention” ở thể bị động.
Vì vậy, đáp án C - mentioning đúng vì theo cấu trúc bị động của “deserve”
Tạm dịch: Keith xứng đáng được nhắc tên ở hội nghị vì tất cả những gì cô ấy đã giúp chúng tôi.
* Mở rộng: ngoài “deserve”, các động từ “want”, “need”, và “require” cũng có cách dùng ở thể bị động
như trên:
1/ S + deserve/ want/ need/ require + to be + P2
2/ S + deserve/ want/ need/ require + V-ing
Question 7. Đáp án D
Đáp án D - câu hỏi so sánh
Với so sánh tính từ ngang bằng, khi tính từ đứng ở vị trí bổ sung thông tin cho danh từ số ít, cấu trúc được
dùng sẽ là “as + adj + a/ an + Noun + as”
Vậy nên, xét 4 đáp án chỉ có đáp án D - as comfortable a hotel as - là đáp án đúng.

Trang 10
Tạm dịch: Khách sạn này không thoải mái bằng khách sạn lần trước chúng tôi ở.
* Mở rộng: Ngoài “as”, các trạng từ “rather”, “so”, “quite”, và “too” cũng có cấu trúc như trên khi đi
kèm với tính từ để bổ sung thông tin cho danh từ số ít: As/ so/ too/ quite/ rather + adj + a/an + Noun
Question 8. Đáp án B
Đáp án B - câu hỏi mệnh đề quan hệ
Mệnh đề “There are dozens of TV channels” là một câu đơn đầy đủ, nếu không có các liên từ hoặc các
cấu trúc ngữ pháp đặc biệt khác để nối với mệnh đề sau, thì phải đặt dấu chấm (“.”) sau mệnh đề đó. Vì
đề bài cho dấu phẩy (“,”) sau mệnh đề đó cho nên các đáp án A, C, D bị loại vì ba đáp án đó không chứa
liên từ và cũng không có các cấu trúc đặc biệt để nối, muốn chọn 3 đáp án này thì sau mệnh đề đầu tiên
phải là dấu chấm
Chỉ còn lại đáp án D - some of which (cấu trúc mệnh đề quan hệ không xác định) là phù hợp vì có thể nối
hai mệnh đề lại.
Tạm dịch: Có hàng tá những kênh ti vi, một vài trong số đó hoạt động 24 giờ/ ngày.
Question 9. Đáp án A
Đáp án A - câu hỏi từ loại
“In” là giới từ. Sau giới từ, thường là danh từ hoặc danh động từ.
Sau chỗ trống đã là danh từ “incidents” (tai nạn), cho nên chỗ trống cần điền chỉ có thể là một tính từ để
bổ sung thông tin cho danh từ “incidents”.
Xét 4 đáp án, đáp án A - violent (a) là phù hợp.
A. violent (a): bạo lực/ thô bạo/ mãnh liệt
B. violence (n): sự bạo lực/ thô bạo
C. violently (adv): một cách thô bạo/ bạo lực
D. violate (v): vi phạm
Tạm dịch: 31 người đã bị thương trong các vụ tai nạn thảm khốc trong suốt ngày hôm nay.
Question 10. Đáp án B
Đáp án B - câu hỏi phrasal verb
Câu hỏi này cần chọn một đáp án có cụm động từ (phrasal verb) phù hợp nghĩa nhất .Chọn B - giving off
vì:
A. giving over: ngưng làm việc phiền toán
B. giving off: thải ra/ tỏa ra mùi, nhiệt độ, khói, ánh sáng...
C. giving down - không tồn tại
D. giving up: từ bỏ/ bỏ cuộc/ ngưng lại
Tạm dịch: Đây là chất hóa học gì? Nó đang tỏa ra một mùi kinh khủng.
Question 11. Đáp án C

Trang 11
Đáp án C - câu hỏi mệnh đề trạng ngữ
Mệnh đề trạng ngữ chỉ thời gian thường được bắt đầu bằng các liên từ: when/ while/ before/ after/ as soon
as/ by the time/ until... Đối với các câu ở thì tương lai, mệnh đề trạng ngữ chỉ thời gian đi kèm chỉ được
phép chia ở các thì hiện tại.
Vậy nên, chỉ còn đáp án C - by the time her boss returns (chia ở thì hiện tại) là phù hợp.
Tạm dịch: Mary sẽ hoàn thành xong toàn bộ công việc không trễ hơn lúc sếp cô ấy quay về.
* Mở rộng: “By + Mốc thời gian” hoặc “By the time + mệnh đề” là dấu hiệu nhận biết các thì hoàn thành
hoặc hoàn thành tiếp diễn.
Căn cứ vào “mốc thời gian” hoặc “mệnh đề” ở hiện tại, quá khứ, hoặc tương lai để chia thành các thì hiện
tại, quá khứ, hoặc tương lai hoàn thành/ hoàn thành tiếp diễn.
Ví dụ: By yesterday - chia ở quá khứ hoàn thành/ hoàn thành tiếp diễn
Question 12. Đáp án D
Đáp án D - câu hỏi đảo ngữ
“Little” (adv): ít, gần như không - là một trạng từ mang nghĩa phủ định. Đối với những câu có trạng ngữ
phủ định được đảo lên đầu câu và không cách câu bằng dấu phẩy (“,”), cần phải sử dụng đảo ngữ. Vì vậy,
các đáp án A và B bị loại do không đảo ngữ.
Câu này có mệnh đề sau chia ở thì quá khứ, nên đáp án phải chọn là đáp án D - did I know (chia ở quá
khứ) và loại đáp án C - do I know (chia ở hiện tại).
Tạm dịch: Tôi biết quá ít về toán đến mức mà bài giảng này nằm ngoài tầm đối với tôi.
* Mở rộng: Ngoài ra, câu này còn có thể giải thích theo cấu trúc đảo ngữ của mệnh đề trạng ngữ chỉ kết
quả:
- Cấu trúc bình thường: S + V + so +adv/ adj... + that...
- Cấu trúc đảo ngữ: So + adv/adj + trợ động từ + S + (V) + ... + that...
Question 13. Đáp án D
Đáp án D - câu hỏi từ nối
Sau chỗ trống cần điền là một cụm danh từ “its intense surface heat”. Để nối danh từ vào câu, giới từ
được sử dụng.
Hai đáp án A - because (bởi vì) và B - although (mặc dù) bị loại vì cả hai là liên từ.
Xét thêm yếu tố về nghĩa, có thể loại được đáp án C - despite (prep): mặc dù - và chọn đáp án D - due to
(prep): do có/ bởi vì.
Tạm dịch: Có lẽ sự sống không thể có khả năng tồn tại trên sao Kim do sức nóng bề mặt khủng khiếp của
nó.
Question 14. Đáp án D
Đáp án D - câu hỏi từ vựng

Trang 12
Câu này chọn đáp án đúng phù hợp nghĩa và kết hợp từ. Đáp án D - deliver trong trường hợp này có nghĩa
là “đảm bảo thực hiện/ giữ được cam kết” - là phù hợp nghĩa với câu đề bài cho.
A. promise (v): hứa
B. conduct (v): tiến hành/ dẫn điện
C. regulate (v): điều chỉnh/ chỉnh đốn
D. deliver (v): giao hàng/ đảm bảo thực hiện/ giữ được cam kết thực hiện
Tạm dịch: Nếu anh không thể giữ được cam kết tăng doanh số tháng này, anh sẽ bị sa thải.
Question 15. Đáp án A
Đáp án A - câu hỏi từ vựng
Câu này chọn đáp án đúng phù hợp nghĩa và kết hợp từ. Đáp án A - produce trong trường hợp này có
nghĩa là “xuất trình/ trình bày” - là phù hợp nghĩa với câu đề bài cho.
A. produced (v): sản xuất/ xuất trình/ trình bày
B. represented (v): đại diện
C. purchased (v): mua
D. commended (v): khen ngợi
Tạm dịch: Trong buổi họp tuần trước, giám đốc tài chính đã trình bày những con số của năm ngoái.
Question 16. Đáp án D
Đáp án D - câu hỏi thành ngữ
“A man of the people” (thành ngữ): người hiểu được nguyện vọng của người dân.
Xét 4 đáp án, chỉ có thể chọn được đáp án D - people.
A. community (n): cộng đồng
B. society (n): xã hội
C. country (n): đất nước
D. people (n): con người/ nhân dân
Tạm dịch: Tôi nghĩ Trump có thể dễ dàng thắng cuộc bầu cử này. Ông ta là người thấu hiểu nguyện vọng
nhân dân.
Question 17. Đáp án C
Đáp án C - câu hỏi giới từ
Danh từ “breeding ground”: nơi phát sinh ra - đi với giới từ “for”.
Vậy nên, chỉ có thể chọn đáp án C - for là phù hợp.
Tạm dịch: Những khu ổ chuột đông đúc thường được coi là nơi phát sinh ra tội phạm.
Question 18. Đáp án B
Đáp án B - câu hỏi từ vựng

Trang 13
Câu này chọn đáp án đúng phù hợp nghĩa và kết hợp từ. Đáp án B - reprimanded - có nghĩa là “khiển
trách” và thường được dùng trong hoàn cảnh khiển trách các quan chức, trong các văn bản chính thức có
hiệu lực pháp lý.
Các đáp án còn lại có nghĩa không phù hợp.
A. scolded (v): mắng/ khiển trách - dùng trong văn cảnh thường ngày (bố mẹ/ thầy cô mắng con/ học
sinh)
B. reprimanded (v): khiển trách - dùng trong văn cảnh chính thức, có hiệu lực pháp lý
C. humiliated (v): làm cho xấu hổ/ nhạo báng
D. ridiculed (v): cười nhạo
Tạm dịch: Tòa án quân đội khiển trách vị thiếu tá vì không thể hoàn thành nhiệm vụ của mình.
Question 19. Đáp án D
Động từ “deprive sb/sth of sth”: tước đoạt cái gì khỏi ai đó. Vậy nên, giới từ “from” phải sửa thành “of”.
Đáp án D - from both → of both.
Tạm dịch: Một căn bệnh hiểm nghèo lúc cô ấy mới tròn 19 tuần tuổi đã tước đi của người giảng viên nổi
tiếng Helen Keller cả thị giác và thính giác.
Question 20. Đáp án A
Mệnh đề “Which he was from London and very healthy” không phải là mệnh để quan hệ mà là một mệnh
đề đóng vai trò bổ sung thông tin cho danh từ theo cấu trúc: N + That + mệnh đề.
Điều này có thể thấy được bởi mệnh đề “he was from London and very wealthy” là một mệnh đề đã đầy
đủ về nghĩa, không thể thêm bất cứ một đại từ/ trạng từ/ định từ quan hệ nào vào trước nó nữa.
Vậy, A - which → that.
Tạm dịch: Cậu ta không hề biết đó là ai và không tưởng tượng được vì sao họ làm như vậy, nhưng phải có
ai đó tung ra tin đồn rằng cậu ta đến từ London và rất giàu có.
Question 21. Đáp án D
Động từ “use” có cấu trúc chủ động là: use sth to do sth: sử dụng sth để làm gì đó
Cấu trúc này chuyển thành bị động sẽ là: sth be used to do sth.
Ngoài ra, tính từ “used” lại có cấu trúc là: be used to doing: quen với việc làm gì đó.
Trong câu này, cấu trúc bị động với “use” đang được sử dụng vì chủ ngữ của nó là “robot”.
Vậy nên, đáp án D - be used to doing → be used to do thì phù hợp nghĩa hơn.
Tạm dịch: Người ta tin rằng trong tương lai gần, robot sẽ được sử dụng để làm những việc như nấu
nướng.
Question 22. Đáp án B
Dịch đề bài: Chuyện có lẽ đã tốt hơn nếu lúc đó anh ấy nói cho chúng tôi địa chỉ của anh ấy. (ngụ ý: đáng
lẽ ra nên nói địa chỉ).

Trang 14
A. Anh ấy đáng ra đã có thể đã nói cho chúng tôi địa chỉ của anh ấy. → sai nghĩa
B. Anh ấy đang ra nên nói cho chúng tôi biết địa chỉ của anh ầy. → đúng.
C. Anh ấy đáng lẽ ra không nên nói cho chúng tôi biết địa chỉ của anh ấy. → sai nghĩa
D. Anh ấy không nói cho chúng tôi dịa chỉ của anh ấy cũng không sao. → sai nghĩa
Chọn đáp án B vì đáp án B phù hợp với nghĩa và ngụ ý của đề bài. Các đáp án khác sai nghĩa.
Question 23. Đáp án C
Dịch đề bài: Cô ấy biết nhiều về chuyện đó hơn tôi.
A. Tôi biết về chuyện đó nhiều ngang với cô → sai nghĩa
B. Cô ấy biết về chuyện đó nhiều ngang với tôi. → sai nghĩa
C. Tôi biết về chuyện đó không nhiều bằng cô ấy. → đúng
D. Cô ấy biết về chuyện đó không nhiều bằng tôi. → sai nghĩa
Chọn đáp án C vì đáp án C có nghĩa đúng. Các đáp án khác sai nghĩa.
Question 24. Đáp án C
Dịch đề bài: “Sao cậu không tham gia vào hoạt động tình nguyện mùa hè này?”, Sophie nói.
A. Không dịch vì sai cấu trúc: động từ suggest không có cấu trúc suggest sb to do sth.
B. Không dịch vì sai cấu trúc: “not participate” không chia theo thì và ngôi
C. Sophie gợi ý tôi tham gia các hoạt động tình nguyện vào mùa hè. → đúng
D. Sophie khiến tôi tham gia các hoạt động tình nguyện vào mùa hè → sai nghĩa.
Chọn đáp án C vì đáp án C đúng cấu trúc và đúng nghĩa. Các đáp án còn lại sai nghĩa hoặc cấu trúc.
Question 25. Đáp án D
Dịch đề bài: Chúng tôi đến sân bay. Chúng tôi nhận ra hộ chiếu vẫn đang ở nhà. (ngụ ý: đến sân bay rồi
mới nhớ là hộ chiếu để quên ở nhà).
A. Không dịch vì sai cấu trúc: cấu trúc đúng là “it was not until + mệnh đề”: mãi cho đến lúc....
B. Không dịch vì sai cấu trúc: “are” chia ở hiện tại là sai, cần phải chia là “were” ở quá khứ.
C. Không dịch vì sai cấu trúc: “Not until + mệnh đề” đứng đẩu câu thì phải dùng đảo ngữ.
D. Cho đến khi chúng tôi đến sân bay thì chúng tôi mới nhận ra hộ chiếu vẫn còn đang ở nhà. → đúng.
Chọn đáp án D vì nó là đáp án đúng duy nhất.
Question 26. Đáp án C
Dịch đề bài: Tối qua Nam đã thật thô lỗ với họ. Giờ đây anh ấy cảm thấy thật hối tiếc, (ngụ ý: hối tiếc vì
đêm qua đã quá thô lỗ).
A. Không dịch vì sai cấu trúc: “regret + to V” - tiếc vì sắp phải làm gì, nghĩa là hành động “to V” chưa
diễn ra, không thể để “to have P2” - mang nghĩa hoàn thành vào đây được.
B. Không dịch vì sai cấu trúc: “rude” là tính từ, không đi với động từ “have”. Cần sửa “having” thành
“having been”.

Trang 15
C. Nam ước rằng tối qua mình đã không thô lỗ với họ như vây. → đúng.
D. Không dịch vì sai cấu trúc: Ước cho một chuyện ở quá khứ (vì có trạng ngữ chỉ thời gian là “last
night”) thì phải dùng thức giả định loại 3 (động từ chia ở quá khứ hoàn thành/ hoàn thành tiếp diễn). Vậy
nên “were” (quá khứ đơn) phải chia thành “had been” (quá khứ hoàn thành). Chọn đáp án C vì nó là đáp
án đúng duy nhất.
Question 27. Đáp án B
“What a lovely cat you have: Bạn có một con mèo đáng yêu thế! - đây là một lời khen. Để đáp lại lời
khen, cần cám ơn và nói một vài lời khách sáo. Vì vậy, đáp án B phù hợp.
A. Dĩ nhiên là không, nó đắt lắm
B. Cám ơri bạn. Bố mình mua nó cho mình
C. Mình nghĩ vậy
D. Không vấn đề gì
Question 28.
"Do you feel like going to the cinema this evening?" - Cậu có muốn đi xem phim tôi nay không? - đây là
một lời mời mọc, rủ rê. Đối với loại câu này, lời đáp lại thường sẽ là: That’s great/ nice/ excellent, hoặc
that’s a great idea... hoặc I would love/ like to...
Nếu muốn từ chối, vẫn trả lời như trên, nhưng thêm “but + lý do từ chối”.
Vậy nên, chọn đáp án C là phù hợp.
A. Tôi e rằng tôi không đồng tình với cậu được
B. Không có gì cậu ạ
C. Thật tuyệt
D. Tôi đang chán lắm cậu ạ
Question 29. Đáp án D
“Rush” (v) - đi vội vàng, gấp gáp. Chọn đáp án D - went quickly: đi vội vàng, nhanh - là gần nghĩa nhất
với từ đề bài cho.
A. came into: thừa kế
B. went leisurely: đi thong dong
C. dropped by: ghé thăm/ ghé chơi
D. went quickly: đi nhanh/ đi vội vàng
Tạm dịch: Cô ấy ngủ dậy muộn và đi vội vàng đến trạm xe buýt.
Question 30. Đáp án nC
“Compulsory” (a): bắt buộc. Chọn đáp án C - required (a): bắt buộc/ yêu cầu phải có - là gần nghĩa nhất
với từ đề bài cho.
A. divided: bị chia ra

Trang 16
B. paid: đã được trả
C. required: bắt buộc/ yêu cầu phải có
D. depended: (quá khứ/ phân từ 2) phụ thuộc
Tạm dịch: Đồng phục trường là điều bắt buộc phải có ở hầu hết các trường học ở Việt Nam.
Question 31. Đáp án D
“Punctuality” (n): sự đúng giờ. chọn đáp án D - being late (danh động từ): trễ/ muộn giờ - là trái nghĩa với
từ bài cho.
A. being courteous: lịch sự/ lịch thiệp
B. being cheerful: vui vẻ
C. being efficient: hiệu quả/ tiết kiệm
D. being late: trễ/ muộn giờ
Tạm dịch: Tôi cần phải có một chiếc đồng hồ bởi sự đúng giờ là thứ bắt buộc với công việc mới của tôi.
Question 32. Đáp án D
“Once in a blue moon” (thành ngữ): hiếm khi xảy ra. chọn đáp án D - frequently (adv): thường xuyên - là
trái nghĩa với thành ngữ bài cho.
A. occasionally: thi thoảng
B. rarely: hiếm khi
C. at night: nửa đêm
D. frequently: thường xuyên
Tạm dịch: chị tôi sống ở Alaska, vì thế nên chúng tôi hiếm khi gặp nhau.
Question 33.
A. worked (v): làm việc
B. passed (v): vượt qua
C. came (v): đi về
D. ran (v): chạy
Câu hỏi từ vựng. Chọn đáp án C ta có phrasal verb “come out in sth”: bị nổi đầy mụn... trên da.
Các đáp án khác không kết hợp thành phrasal verb với “out in”.
Trích bài: she recently came out in a painful rash down one side of her body.
Tạm dịch: Gần đây cô ấy bị nổi mụn ban đỏ từ trên xuống dưới khắp một bên cơ thể.
Question 34. Đáp án B
Câu hỏi đại từ quan hệ. “Symptoms” - triệu chứng - là danh từ chỉ vật. Hơn nữa, trong mệnh đề quan hệ
còn đang khuyết vị trí chủ ngữ trước động từ “included”, vậy nên chọn B - which (làm chủ ngữ thay cho
danh từ chỉ vật).

Trang 17
Trích bài: she emailed her symptoms, which also included a mild fever, to e-doc, the internet medical
service.
Tạm dịch: Cô ấy gửi mail các triệu chứng, bao gồm cả một cơn sốt nhẹ đến một bác sỹ điện tử, là một
dịch vụ y tế trên internet.
Question 35. Đáp án C
A. small (a): nhỏ bé
B. weak (a): yếu ớt
C. mild (a): nhẹ/ dễ chịu
D. calm (a): bình tĩnh
Câu hỏi từ vựng. Chọn đáp án C - mild là phù hợp nghĩa và kết hợp từ (mild fever: sốt nhẹ).
Trích bài: she emailed her symptoms, which also included a mild fever, to e-doc, the internet medical
service.
Tạm dịch: Cô ấy gửi mail các triệu chứng, bao gồm cả một cơn sốt nhẹ đến một bác sỹ điện tử, là một
dịch vụ y tế trên internet.
Question 36. Đáp án B
A. serving (v): phục vụ
B. meeting (v): gặp/ đáp ứng
C. creating (v): tạo ra
D. establishing (v): thiết lập
Câu hỏi từ vựng. Chọn đáp án B - meeting là phù hợp với nghĩa và kết hợp từ (meet demand: đáp ứng nhu
cầu).
Trích bài: They are clearly meeting a demand from people who are too busy...
Tạm dịch: Họ rõ ràng đang đáp ứng được nhu cầu của những người quá bận...
Question 37. Đáp án D
A. harm (n): tác hại
B. story (n): câu chuyện
C. hardship (n): sự khó khăn
D. complaint (n): chứng bệnh/ bệnh tật
Câu hỏi từ vựng. Căn cứ vào nghĩa để chọn đáp án D - complaint.
Trích bài: ...people who are too busy or, in some cases, too embarrassed to discuss their medical
complaint with their general practitioners.
Tạm dịch: ... những người quá bận hoặc trong một vài trường hợp là quá xấu hổ nên không thể thảo luận
tình trạng bệnh tình của mình với bác sỹ khu vực.
Question 38. Đáp án A

Trang 18
Dịch đề bài: Tiêu đề phù hợp nhất cho đoạn văn là gì?
A. Hành vi giao tiếp của cá heo
B. Động vật có vú thông minh trong điều kiện nuôi nhốt
C. Xu hướng xã hội của cá heo với con người
D. Các đặc điểm phức tạp trong cuộc sống của cá heo
Câu hỏi tiêu đề đoạn văn, thực chất là một loại câu hỏi ý chính của đoạn văn.
Đối với loại cầu hỏi tìm ý chính/ tiêu đề của đoạn văn, việc cần làm là:
1/ Đọc 2-3 câu đầu tiên của bài vì những câu đó là câu thể hiện chủ đề của bài viết.
2/ Đọc lướt qua nội dung của bài để chắc chắn mình xác định ý chính đúng.
Có thể thấy, tất cả các đoạn văn trong bài đọc này đều nói đến khả năng và hành vi giao tiếp của cá heo,
vì vậy đáp án A là phù hợp.
Question 39. Đáp án C
Dịch đề bài: Ở dòng 2, từ “vocalization” gần nghĩa nhất với....
A. tạo ra cử chỉ B. tạo ra từ
C. tạo ra âm thanh D. nhắm mắt
Câu hỏi từ vựng.
“Vocalization” (n): phát ra âm thanh. Chọn đáp án C - producing sounds - là phù hợp với từ đề bài cho.
Question 40. Đáp án B
Theo đoạn 4, một số nhà sinh vật biển nghĩ gì về cá heo nuôi nhốt?
A. Mối quan hệ giữa chúng quá riêng tư với các nhà sinh học biển.
B. Chúng không thể được coi là đối tượng chính xác cho các nghiên cứu sinh học.
C. Chúng có thể giao tiếp giống hệt như con người.
D. Chúng dễ dàng thể hiện cách cá heo hành động trong tự nhiên.
Câu hỏi tìm thông tin.
* Hướng dẫn làm bài:
Đối với loại bài tìm thông tin, cần chọn ra một vài từ khóa ở đáp án cũng như ở câu hỏi rồi tìm những từ
khóa đó ở trên bài đọc. Khi đã tìm được đoạn chứa từ khóa trên bài đọc, cần đọc kĩ và chọn lọc những
thông tin nào ủng hộ đáp án. Sẽ chỉ có 1 đáp án được thông tin trong bài ủng hộ, và đó là đáp án đúng.
Trích thông tin trong bài: “...some marine biologists believe that dolphins living in aquariums or research
centers cannot be considered "normal."
Tạm dịch: “...một số nhà sinh vật biển tin rằng cá heo sống trong bể cá hoặc trung tâm nghiên cứu không
thể được coi là "bình thường".
Vậy, đáp án B là phù hợp vì được thông tin trong bài ủng hộ.
Question 41. Đáp án B

Trang 19
Dịch đề bài: Tất cả những điều sau đây là đúng về hành vi giao tiếp cá heo, NGOẠI TRỪ...
A. Tiếng ré và tiếng rít của cá heo nghe như đang có một cuộc trò chuyện đối với tai con người
B. Những tiếng click của cá heo đôi khi được sử dụng để chào hỏi con người
C. Hành vi giao tiếp sử dụng âm thanh và cử chỉ xuất hiện giữa các con cá heo
D. Ngôn ngữ cơ thể và cử chỉ được sử dụng cho việc giao tiếp với con người
Câu hỏi tìm ý sai hoặc tìm ý đúng.
Đối với loại câu hỏi tìm ý sai hoặc tìm ý đúng, cần phải đọc hết cả 4 đáp án và tìm thông tin trong bài ủng
hộ các đáp án đó. Nếu đáp án nào không có thông tin ủng hộ hoặc ngược lại với thông tin trong bài, thì đó
là đáp án sai. Việc tìm thông tin ủng hộ đáp án phải dựa vào từ khóa của đáp án và trên bài đọc. Tuyệt đối
không được dùng suy luận từ kiến thức của bản thân để trả lời loại câu hỏi này mà phải dựa hết vào bài
đọc.
Trích thông tin trong bài: “They also make clicking sounds used for echolocation to find food and
obstacles in dark and murky waters.”
Tạm dịch: “Chúng cũng tạo ra những tiếng click để sử dụng cho việc định vị bằng âm thanh để tìm thức
ăn và chướng ngại vật trong vùng nước tối và âm u.” thông tin không ủng hộ đáp án B.
Vậy, đáp án B là phù hợp.
Question 42. Đáp án A
Dịch đề bài:
Ở dòng 17, từ “most” chỉ...
A. Phần lớn các nhà sinh vật biển
B. Một vài con cá heo mũi chai
C. Rất nhiều cá heo mũi chai
D. Thiểu số các nhà sinh vật học biển
* Hướng dẫn làm bài:
Đối với loại câu hỏi này, đọc ngược lại 1-2 câu trước nó để tìm từ mà nó đang thay thế. Trích thông tin
trong bài: “These studies have been criticized because some marine biologists believe that dolphins living
in aquariums or research centers cannot be considered "normal." Even so, most believe that studying
dolphin communication in captivity is useful for beginning to understand the complexity of dolphin
communication.”
Tạm dịch: Những nghiên cứu này đã bị chỉ trích vì một số nhà sinh vật biển tin rằng cá heo sống trong bể
cá hoặc trung tâm nghiên cứu không thể được coi là "bình thường". Mặc dù vậy, hầu hết đều tin rằng
nghiên cứu hành vi giao tiếp của cá heo trong điều kiện nuôi nhốt là hữu ích để bắt đầu hiểu được sự phức
tạp trong sự giao tiếp cá heo.
Như vậy, “most” ở đây chỉ hầu hết các nhà sinh vật biển. → đáp án A phù hợp.

Trang 20
Glossary:
Phân loại bằng đại học ở Anh:
- First class: Bằng hạng Nhất
- Upper second (2:1): Bằng hạng 2:1
- Lower second (2:2): Bằng hạng 2:2
- Third class (3): Bằng hạng 3
Question 43. Đáp án A
Dịch đề bài: Mục đích chính của đoạn văn là gì?
A. Để mô tả kinh nghiệm học tập của một sinh viên xuất sắc
B. Để nhấn mạnh tầm quan trọng của giáo dục đại học
C. Để đề xuất các cách để giải quyết các bài tập ở trường đại học
D. Để chỉ ra những thách thức của việc học tập tại trường đại học
Câu hỏi ý chính của đoạn văn.
Đối với loại câu hỏi tìm ý chính/ tiêu đề của đoạn văn, việc cần làm là:
1/ Đọc 2-3 câu đầu tiên của bài vì những câu đó là câu thể hiện chủ đề của bài viết.
2/ Đọc lướt qua nội dung của bài để chắc chắn mình xác định ý chính đúng
Thông tin về ý chính có thể lấy được tiêu đề phụ của bài đọc: “What does it take to graduate from
university with a First? Mark McArdle, first-class degree holder from the University of Lancaster, tells
how he did it.”
Tạm dịch: Làm gì để được tốt nghiệp đại học với tấm bằng hạng Nhất? Mark McArdle, người giữ bằng
cấp hạng Nhất của Đại học Lancaster, cho biết ông đã làm như thế nào. → Nội dung chính của đoạn văn
là một sinh viên đạt được bằng hạng Nhất chia sẻ lại kinh nghiệm của mình.
Vậy, đáp án A là phù hợp.
Question 44. Đáp án A
Dịch đề bài: Các nghiên cứu sinh tiến sỹ đã nói chuyện với tác giả....
A. Phóng đại sự cần thiết của việc học tập chăm chỉ
B. Đã thành công trong việc khiến nhà văn sợ hãi
C. Không chắc chắn làm thế nào để giúp nhà văn
D. Nghĩ rằng tác giả sẽ có được tấm bằng hạng Nhất
Câu hỏi tìm thông tin.
* Hướng dẫn làm bài:
Đối với loại bài tìm thông tin, cần chọn ra một vài từ khóa ở đáp án cũng như ở câu hỏi rồi tìm những từ
khóa đó ở trên bài đọc. Khi đã tìm được đoạn chứa từ khóa trên bài đọc, cần đọc kĩ và chọn lọc những
thông tin nào ủng hộ đáp án. Sẽ chỉ có 1 đáp án được thông tin trong bài ủng hộ, và đó là đáp án đúng.

Trang 21
Trích thông tin bài: “Don’t spend too much time at the student lounge, do turn up for most lectures and
tutorials and do submit all coursework - eventually. That, I was told by a PhD student during freshers’
week, was all I needed to do to get a 2:2.”
Tạm dịch: Đừng dành quá nhiều thời gian tại phòng chờ sinh viên, phải tham dự hết các bài giảng và các
buổi hướng dẫn và cuối là nộp hết các loại bài tập. Một nghiên cứu sinh nói với tôi điều này, và nó chỉ là
tất cả những gì tôi cần làm để có được bằng hạng 2:2. → Người nghiên cứu sinh này cho rằng, muốn đạt
được bằng hạng Nhất phải học tập thật chăm chỉ, nhưng điều đó là chưa đủ.
Vậy, nghiên cứu sinh này đang làm quá lên sự cần thiết của việc học chăm. → đáp án A là phù hợp.
Question 45. Đáp án D
Dịch đề bài: Trong đoạn thứ ba, người viết cảnh báo về...
A. Đưa tiền cho nhân viên nhà trường
B. Cố gắng đoán trước bài giảng
C. Mong đợi được khen thưởng
D. Đạo văn trong bài tiểu luận và bài kiểm tra
Câu hỏi tìm thông tin.
* Hướng dẫn làm bài có ở câu 44.
Trích thông tin bài: “Some students try to offer something not wanted. Others want to give very little -
they steal the thoughts of others and submit them as their own. But they all want to be rewarded.
Exchange, but don’t steal, and you’ll get a degree.”
Tạm dịch: Một số sinh viên cố gắng đưa ra những điều không cần thiết. Những người khác muốn cho rất
ít - họ đánh cắp ý tưởng của người khác và gửi chúng như của chính họ. Nhưng tất cả họ đều muốn được
khen thưởng. Trao đổi, nhưng không đánh cắp, và bạn sẽ có được một tấm bằng. → Tác giả muốn cảnh
báo không nên đánh cắp ý tưởng của người khác, thực chất đó chính là đạo văn.
Vậy, đáp án D là phù hợp.
Question 46. Đáp án B
Dịch đề bài: Từ “immersed” ở đoạn 1 gần nghĩa với từ...
B. Mải mê
C. Chìm xuống nước
D. Nhúng xuống
Câu hỏi từ vựng. “Immersed” (a): chìm đắm/ mải mê. Chọn đáp án B - absorbed (a): mải mê - là gần
nghĩa nhất với từ đề bài cho.
Question 47. Đáp án B
Dịch đề bài: Tại sao người viết không luyện tập trả lời các câu hỏi thi trước?
A. Ông ấy nghĩ rằng việc luyện tập khá nhàm chán.

Trang 22
B. Ông ấy muốn trả lời các câu hỏi thi một cách có phản biện.
C. Ông ấy được khuyên không nên làm thế.
D. Ông ấy nghĩ rằng nó có thể khiến các nhân viên có định kiến về mình?
Câu hỏi tìm thông tin. Hướng dẫn làm bài có ở câu 44.
Trích thông tin bài: “I didn’t practice writing exam questions, although it was recommended. I prefer to
be spontaneous and open-minded.”
Tạm dịch: Tôi đã không luyện tập các bài thi viết, mặc dù nó được khuyến khích. Tôi thích tự nhiên và
cởi mở. → Ngụ ý rằng tác giả muốn làm bài thi một cách tự nhiên, cởi mở, không gò bó mà có phản biện
Vậy, đáp án B là phù hợp.
Question 48. Đáp án B
Dịch đề bài: Từ “flick through” ở đoạn 4 gần nghĩa nhất với...
A. Tìm kiếm nhanh
B. Đọc lướt
C. Mượn
D. Đọc cẩn thận
Câu hỏi từ vựng. “Flick through” (phrs verb): đọc nhanh, đọc lướt. Chọn đáp án B - skim (v): đọc lướt - là
gần nghĩa nhất với từ đề bài cho.
Question 49. Đáp án B
Dịch đề bài: Tất cả những điều sau đây được đề cập về các yếu tố giúp tác giả thành công tại trường đại
học NGOẠI TRỪ______________.
A. Hiểu về những gì được yêu cầu
B. Tham dự thường xuyên tại các bài giảng
C. Đọc có chọn lọc
D. Tổ chức ôn tập tốt
Câu hỏi tìm ý sai hoặc tìm ý đúng.
Đối với loại câu hỏi tìm ý sai hoặc tìm ý đúng, cần phải đọc hết cả 4 đáp án và tìm thông tin trong bài ủng
hộ các đáp án đó. Nếu đáp án nào không có thông tin ủng hộ hoặc ngược lại với thông tin trong bài, thì đó
là đáp án sai. Việc tìm thông tin ủng hộ đáp án phải dựa vào từ khóa của đáp án và trên bài đọc. Tuyệt đối
không được dùng suy luận từ kiến thức của bản thân để trả lời loại câu hỏi này mà phải dựa hết vào bài
đọc.
Trích thông tin trong bài: “I didn't attend every lecture and tutorial.”
Tạm dịch: Tôi đã không tham dự tất cả các buổi học và các buổi hướng dẫn. → Như vậy tác giả không
thành công nhờ đi học đầy đủ → thông tin không ủng hộ đáp án B.
Question 50. Đáp án D

Trang 23
Dịch đề bài: Có thể suy ra từ những gì tác giả viết ở đoạn cuối là...
A. ưu tiên hàng đầu của sinh viên là vượt qua các kỳ thi
B. Giáo viên nên đặt mục tiêu cho sinh viên ngay từ năm đầu tiên
C. sinh viên nên làm theo công thức của ông ấy để thành công
D. sinh viên nên tận hưởng cuộc đời học sinh của chính mình
Câu hỏi suy luận.
* Hướng dẫn làm bài:
Đối với loại câu hỏi suy luận, đọc kĩ 4 đáp án và tìm thông tin hỗ trợ trong bài. Chắc chắn chỉ có một đáp
án được thông tin trong bài hỗ trợ, các đáp án còn lại đều sai hoặc không có thông tin trong bài.
Lưu ý phải tìm thông tin dựa vào từ khóa, và không dùng kiến thức bản thân để trả lời loại câu hỏi này.
Trích thông tin trong bài: “Be a happy student by striking the right balance between working and enjoying
yourself. Take what you do seriously and do your best. And, no matter what you do don’t forget to
appreciate every day of your university studies: it is one of the greatest periods of your life.”
Tạm dịch: Hãy là một sinh viên vui vẻ, hạnh phúc bằng cách tạo ra sự cân bằng phù hợp giữa làm việc và
tận hưởng chính mình. Hãy làm những gì bạn làm nghiêm túc và làm tốt nhất. Và, dù bạn có làm gì đi
nữa, đừng quên biết ơn mỗi ngày bạn học ở đại học: đó là một trong những giai đoạn tuyệt vời nhất trong
cuộc đời bạn. → Tác giả khuyên tận hưởng quãng đời sinh viên → thông tin ủng hộ đáp án D.

Trang 24
ĐỀ SỐ 20 ĐỀ THI THỬ TỐT NGHIỆP THPT
NĂM HỌC: 2020 – 2021
MÔN: TIẾNG ANH
Thời gian làm bài: 60 phút; không kể thời gian phát đề

Mark the letter A, B, C, or D on your answer sheet to indicate the word whose underlined part
differs from the other three in pronunciation in each of the following questions.
Question 1. A. land B. sandy C. many D. candy
Question 2. A. compete B. intend C. medal D. defend
Mark the letter A, B, C, or D on your answer sheet to indicate the word that differs from the other
three in the position of primary stress in each of the following questions.
Question 3. A. relax B. wonder C. problem D. special
Question 4. A. reflection B. division C. industry D. remember
Mark the letter A, B, C, or D on your answer sheet to indicate the correct answer to each of the
following questions.
Question 5. I think everyone wants to make friends with John. He is___________honest person.
A. a B. an C. the D. no article
Question 6. He___________only three letters to his parents since he joined the army.
A. has written B. wrote C. would write D. had written
Question 7. Jimmy’s low examination scores kept him from___________to the university
A. to admit B. to be admitted C. admitting D. being admitted
Question 8. The company is believed___________a lot of money last year.
A. to lose B. lost C. to have lost D. to be losing
Question 9. When___________as the new manager of the company, Mr. Smith knew he had a lot of
things to do.
A. appointing B. appointed C. appoint D. have appointed
Question 10: ___________his poor English, he managed to communicate his problem very clearly.
A. Because B. Even though C. Because of D. In spite of
Question 11. If I___________a wallet in the street, I’d take it to the police.
A. find B. found C. will find D. would find
Question 12. I assume that you are acquainted___________this subject since you are responsible
writing the accompanying materials.
A. to/for B. with/for C. to/to D. with/with
Question 13. The song has___________been selected for the 2018 World Cup, Russia.
A. office B. officer C. officially D. official

Trang 1
Question 14. Elephants___________the ecosystems they live in, and make it possible for a lot of other
species to survive in those environments as well.
A. obtain B. remain C. maintain D. attain
Question 15. After the fire, the government pledged to implement a program of___________all over
the devastated national park.
A. reforestation B. rebuilding C. replenishment D. reconstruction
Question 16. He was given a medal in___________of his service to the country.
A. gratitude B. recognition C. knowledge D. response
Question 17. I'm going on business for a week, sp I'll be leaving everything___________.
A. on your guards B. up to your eyes
C. in your capable hands D. in the care of you
Question 18. Archaeologists think that massive floods could have___________the dinosaurs.
A. wiped out B. laid off C. put aside D. taken down.
Mark A, B, C, or D on your answer sheet to indicate the underlined part that needs correction in
each of the following questions.
Question 19. The assumption that smoking has bad effects on our health have been proved.
A. smoking B. effects C. on D. have been proved.
Question 20. She had so many luggage that there was not enough room in the car for it.
A. so many B. was C. enough room D. it
Question 21. Alike light waves, microwaves may be reflected and concentrated elements.
A. Alike B. waves C. may be D. concentrated
Mark the letter A, B, C, or D on your answer sheet to indicate the sentence that is closest in
meaning to each of the following sentences.
Question 22. "I didn't break the mobile phone," Lan said.
A. Lan denied breaking the mobile phone.
B. Lan admitted breaking the mobile phone.
C. Lan prevented us from breaking the mobile phone
D. Lan promised to break the mobile phone.
Question 23. The bag was heavy, so we could not take it with us.
A. If the bag was not heavy, we would take it with us.
B. Unless the bag had not been heavy, we would have taken it with us.
C. If had the bag not been heavy, we would have taken it with us.
D. Had the bag not been heavy, we would have taken it with us
Question 24. He was such a wet blanket at the party tonight!
A. He made people at the party wet through.
B. He spoiled other people's pleasure at the party

Trang 2
C. He bought a wet blanket to the party.
D. He was wet through when going home from the party.
Mark the letter A, B, C, or D on your answer sheet to indicate the sentence that best combines
each pair of sentences in the following questions.
Question 25. He was suspected of stealing credit cards. The police have investigated him for days.
A. He has been investigated for days, suspected to have stolen credit cards.
B. Suspecting to have stolen credit cards, he has been investigated for days.
C. Having suspected of stealing credit cards, he has been investigated for days.
D. Suspected of stealing credit cards, he has been investigated for days.
Question 26. The girl packed the vase in polyester foam. She didn’t want it to get broken in the
post.
A. The girl packed the vase in polyester foam so that it wouldn’t get broken in the post.
B. The girl packed the vase in polyester foam so it didn’t get broken in the post.
C. The girl packed the vase in polyester foam so as it wouldn’t get broken in the post.
D. The girl packed the vase in polyester foam for it didn’t get broken in the post.
Mark the letter A, B, C or D on your answer sheet to indicate the most suitable response to
complete each of the following exchanges.
Question 27. David and Cathy are talking about the party next week.
David.: “I’d like to invite you to a party next Sunday.”
Cathy: “___________”
A. Thank you. What time? B. How do you do?
C. You’re entirely welcome. D. I’m glad you like it.
Question 28. Tom and Marry are meeting at the class reunion.
Tom. "Your hairstyle is terrific, Mary" - Mary. “___________"
A. Thanks. That's a nice compliment B. why do you say so?
C. Sorry, I don't like it D. I think so
Mark the letter A, B, C or D on your answer sheet to indicate the word(s) CLOSEST in meaning
to the underlined word(s) in each of the following questions.
Question 29. I strongly recommend that you should take out an insurance policy in the house for your
own peace of mind.
A. to stop your sleeping B. to stop your worrying
C. to stop your thinking D. to stop your believing
Question 30. Every year this charity organization takes on volunteers to support the needy and the
poor.
A. dismisses B. creates C. recruits D. interviews

Trang 3
Mark the letter A, B, C or D on your answer sheet to indicate the sentence that is OPPOSITE in
meaning to each of the following questions.
Question 31. I must have a watch since punctuality is imperative in my new job.
A. being on time B. being cheerful C. being efficient D. being late
Question 32. Jose had a hard time comparing the iPhone to the Samsung phone because to him they
were apples and oranges.
A. containing too many technical details B. very similar
C. completely different D. very complicated
Read the following passage and mark A, B, C, or D to indicate the correct answer to each of the
blanks.
Higher education also provides a competitive edge in the career market. We all know that in the
economic times we are living in today, finding jobs is not guaranteed. The number of people
unemployed is still relatively high, and the number of new career (33) ___________ isn’t nearly
enough to put people in jobs they are seeking. As a job seeker, you’re competing with a high number of
experienced workers (34) ___________ have been out of the workforce for a while and are also seeking
work. (35) ___________, when you have a higher education, it generally equips you for better job
security. Generally speaking, employers tend to value those who have completed college than those
who have only completed high school and are more likely to replace that person who hasn’t (36)
___________a higher education. Furthermore, some companies even go so far as to pay your tuition
because they consider an educated (37) ___________to be valuable to their organization. A college
education is an investment that doesn’t just provide you with substantial rewards. It benefits the hiring
company as well.
Question 33. A. responsibilities B. activities C. opportunities D. possibilities
Question 34. A. who B. where C. whose D. which
Question 35. A. Otherwise B. Moreover C. Therefore D. However
Question 36. A. permitted B. refused C. applied D. received
Question 37. A. employment B. employer C. employee D. unemployed
Read the following passage and choose A, B, C or D to indicate the correct answer to each of the
following questions from 38 to 42.
In this era of increased global warming and diminishing fossil fuel supplies, we must begin to put a
greater priority on harnessing alternative energy sources. Fortunately, there are a number of readily
available, renewable resources that are both cost- effective and earth - friendly. Two such resources are
solar power and geothermal power.
Solar energy, which reaches the earth through sunlight, is so abundant that it could meet the needs of
worldwide energy consumption 6,000 times over. And solar energy is easily harnessed through the use
of photovoltaic cells that convert sunlight to electricity. In the us alone, more than 100, 000 homes are

Trang 4
equipped with solar electric systems in the form of solar panels or solar roof tiles. And in other parts of
the world, including many developing countries, the use of solar system is growing steadily.
Another alternative energy source, which is abundant in specific geographical areas, is geothermal
power, which creates energy by tapping heat from below the surface of the earth. Hot water and steam
that are trapped in underground pools are pumped to the surface and used to run a generator, which
produces electricity. Geothermal energy is 50,000 times more abundant than the entire known supply of
fossil fuel resources and as with solar power, the technology needed to utilize geothermal energy is
fairly simple. A prime example of effective geothermal use in Iceland, a region of high geothermal
activity where there are over 80 percent of private homes, are heated by geothermal power.
Solar and geothermal energy are just two of promising renewable alternatives to conventional energy
sources. The time is long overdue to invest in the development and use of alternative energy on global
scale.
Question 38. what is the main topic of this passage?
A. The benefits of solar and wind power over conventional energy sources.
B. Two types of alternative energy sources that should be further utilized.
C. How energy resources are tapped from nature.
D. Examples of the use of energy sources worldwide.
Question 39. which of the following words could best replace the word “abundant”?
A. a lot B. scarce C. little D. enough
Question 40. The word “it” in the second paragraph refers to_____________
A. solar energy B. the earth C. sunlight D. energy consumption
Question 41. According to the passage, how is solar energy production similar to geothermal
energy production?
A. They both require the use of a generator.
B. They both use heat from the earth’s surface.
C. They both require fairly simple technology.
D. They are both conventional and costly.
Question 42. what best describes the author’s purpose in writing the passage?
A. To warn people about the hazards of fossil fuel use.
B. To convince people of the benefits of developing alternative energy sources.
C. To describe the advantages and disadvantages of alternative energy use.
D. To outline the problems and solutions connected with global warming.
Read the following passage and mark the letter A, B, C, or D on your answer sheet to indicate the
correct answer to each of the following questions from 43 to 50.
According to sociologists, there are several different ways in which a person may become recognized as
the leader of a social group in the United States. In the family, traditional cultural patterns confer

Trang 5
leadership on one or both of the parents. In other cases, such as friendship groups, one or more persons
may gradually emerge as leaders, although there is no formal process of selection. In larger groups,
leaders are usually chosen formally through election or recruitment.
Although leaders are often thought to be people with unusual personal ability, decades of research have
failed to produce consistent evidence that there is any category of “natural leaders.” It seems that there
is no set of personal qualities that all leaders have in common; rather, virtually any person may be
recognized as a leader if the person has qualities that meet the needs of that particular group.
Furthermore, although it is commonly supposed that social groups have a single leader, research
suggests that there are typically two different leadership roles that are held by different individuals.
Instrumental leadership is leadership that emphasizes the completion of tasks by a social group. Group
members look to instrumental leaders to “get things” done. Expressive leadership, on the other hand, is
leadership that emphasizes the collective well¬being of a social group’s member. Expressive leader are
less concerned with the overall goals of the group than with providing emotional support to group
members and attempting to minimize tension and conflict among them. Group members expect
expressive leaders to maintain stable relationships within the group and provide support to individual
members. Instrumental leaders are likely to have a rather secondary relationship to other group
members. They give orders and may discipline group members who inhibit attainment of the groups
goals. Expressive leaders cultivate a more personal or primary relationship to others in the group. They
offer sympathy when someone experiences difficulties or is subjected to discipline, are quick to lighten
a serious moment with humor, and try to resolve issues that threaten to divide the group. As the
differences in these two roles suggest, expressive leaders generally receive more personal affection
from group members; instrumental leaders, if they are successful in promoting group goals, may enjoy
a more distant respect.
Question 43. what does the passage mainly discuss?
A. How leadership differs in small and large groups
B. The role of leaders in social groups
C. The problems faced by leaders
D. How social groups determine who will lead them
Question 44. The passage mentions all of the following ways by which people can become leaders
EXCEPT___________
A. recruitment B. specific leadership training
C. traditional cultural patterns D. formal election process
Question 45. which of the following statements about leadership can be inferred from paragraph
2?
A. person can best learn how to be an effective leader by studying research on leadership.
B. Most people desire to be leaders but can produce little evidence of their qualifications.

Trang 6
C. A person who is an effective leader of a particular group may not be an effective leader in another
group.
D. Few people succeed in sharing a leadership role with another person.
Question 46. The passage indicates that instrumental leaders generally focus on___________
A. sharing responsibility with group members
B. achieving a goal
C. ensuring harmonious relationships
D. identifying new leaders
Question 47. The word “collective” in paragraph 3 is closest in meaning to___________
A. necessary B. group C. particular D. typical
Question 48. It can be understood that___________
A. There is lots of tension and conflict in an election of a leader in the family.
B. There is usually an election to choose leaders in a family as well as in larger groups.
C. It has been said that there must be a set of personal qualities that all leaders have in common.
D. Leaders are sometimes chosen formally or informally.
Question 49. The word “resolve” in paragraph 4 is closest in meaning to___________
A. talk about B. find a solution for
C. avoid repeating D. avoid thinking about
Question 50. Paragraphs 3 and 4 organize the discussion of leadership primarily in terms
of___________
A. examples that illustrate a problem B. narration of events
C. comparison and contrast D. cause and effect analysis

Trang 7
Đáp án
1-C 2-A 3-A 4-C 5-B 6-A 7-D 8-C 9-B 10-D
11-B 12-B 13-C 14-C 15-A 16-B 17-C 18-A 19-D 20-A
21-A 22-A 23-D 24-B 25-D 26-A 27-A 28-A 29-B 30-C
31-D 32-B 33-C 34-A 35-D 36-D 37-C 38-B 39-A 40-A
41-C 42-B 43-B 44-B 45-C 46-B 47-B 48-D 49-B 50-C

LỜI GIẢI CHI TIẾT


Question 1: Đáp án C
Phần gạch chân của đáp án C đọc là /e/ còn phần gạch chân của các phương án khác được đọc là âm /æ/.
A. land /lænd/ (n): đất đai
B. sandy /ˈsændi/ (adj): như cát, phủ cát
C. many /ˈmeni/ (pronoun): nhiều
D. candy /ˈkændi/ (n): kẹo ngọt
Question 2. Đáp án A
Phần gạch chân của đáp án A đọc là /i:/ còn phần gạch chân của các phương án khác đọc là âm /e/.
A. compete /kəmˈpiːt/ (v): cạnh tranh, thi đấu
B. intend /ɪnˈtend/ (v): có ý định
C. medal /ˈmedl/ (n): huy chương
D. defend /dɪˈfend/ (v): phòng thủ, bảo vệ
Question 3. Đáp án A
Trọng âm của đáp án A rơi âm tiết số hai còn các phương án còn lại trọng âm rơi âm thứ nhất.
A. relax /rɪˈlæks/ (v): thư giãn
B. wonder /ˈwʌndə(r)/ (n/v): kì quan, điều kì diệu/băn khoăn, tự hỏi
C. problem /ˈprɒbləm/ (n): vấn đề
D. special /ˈspeʃl/ (adj): đặc biệt
Lưu ý: Danh từ và tính từ có hai âm tiết trọng âm thường rơi vào âm tiết thứ nhất, động từ có hai âm tiết
trọng âm thường rơi vào âm tiết thứ hai.
Question 4. Đáp án C
Trọng âm của đáp án C rơi âm tiết thứ nhất, còn các phương án còn lại trọng âm rơi âm số hai.
A. reflection /rɪˈflekʃn/ (n): sự phản chiếu
B. division /dɪˈvɪʒn/ (n): sự phân chia, chia rẽ
C. industry /ˈɪndəstri/ (n): nền công nghiệp
D. remember /rɪˈmembə(r)/ (v): ghi nhớ, nhớ lại
Lưu ý: Trọng âm của các từ có đuôi “tion/ sion/cian/ic/tial/cial...” nằm ngay trước các âm này.
Question 5. Đáp án B

Trang 8
Kiến thức được hỏi: Mạo từ.
Tính từ “honest” đọc là /ˈɒnɪst/ (do “h” bị câm), âm đầu tiên là nguyên âm nên sử dụng mạo từ chưa xác
định “an”. Đáp án: B.
Tạm dịch. Tôi nghĩ ai cũng muốn làm bạn với John. Anh ấy là một người trung thực.
Question 6. Đáp án A
Kiến thức được hỏi: Thì động từ.
Trong câu chứa mốc thời gian “since he joined the army.” nên ta dùng thì HTHT ở phía trước. Đáp án A.
Cẩu trúc: S+ have/has+ P2 since S+Ved.
Tạm dịch: Anh ấy chỉ viết được ba lá thư cho bố mẹ kể từ khi anh ta gia nhập quân đội.
Lưu ý: Với câu chia thì của động từ, luôn tìm và nhìn vào trạng ngữ thời gian hoặc các từ khóa để xác
định thì, sau đó quan sát kĩ chủ ngữ để chia cho phù hợp.
Question 7. Đáp án D
Kiến thức được hỏi: Bị động với dạng động từ theo sau là Ving.
Sau cấu trúc keep sb from luôn dùng Ving (giữ ai không/ khỏi làm gì); trong câu có kết hợp yếu tố bị
động (be admitted to the university: được nhận vào trường đại học) nên dùng being P2.
Đáp án D being admitted.
Tạm dịch: Điểm thi kém khiến Jimmy không được nhận vào trường.
Question 8. Đáp án C
Kiến thức được hỏi: Bị động với các động từ chỉ ý kiến.
Vì đề bài có “last year”, với các hành động xảy ra trong quá khứ ta dùng cấu trúc: S+ be+ believed (/said/
thought...)+ to have P2
Đáp án: C. to have lost
Tạm dịch: Người ta tin rằng năm ngoái công ty đã mất rất nhiều tiền.
Mở rộng: Với các hành động xảy ra ở hiện tại hoặc tương lai, ta dùng cấu trúc: S+ be + believed (/said/
thought...)+ to V.
Ex: The company is believed to lose a lot of money next year. (Người ta tin rằng năm tới công ty sẽ mất
rất nhiều tiền.)
Question 9.
Kiến thức được hỏi: Rút gọn mệnh đề dùng P2.
Câu có 2 mệnh đề cùng chủ ngữ là Mr.Smith nên ta có thể lược bớt chủ ngữ ở mệnh đề phía trước và do ý
nghĩa của câu có yêu tố bị động nên dùng P2.
Câu bị động (When/ once...) S+ be+P2, S+V (S giống nhau)
→ (When/ once...) P2, S+ V
Đáp án: B - appointed
Tạm dịch: Khi được bổ nhiệm làm giám đốc mới của công ty, ông Smith biết rằng mình có rất nhiều thứ
cần làm.

Trang 9
Mở rộng: Câu chủ động: (When/ once...) S + V, S + V (S giống nhau)
→ (When/once...) Ving, S + V.
Ex: When I feel tired, I go to bed early → When feeling tired, I go to bed early.
Question 10. Đáp án D
Kiến thức được hỏi: Liên từ
Because + mệnh đề: bởi vì, do
Because of + danh từ/cụm danh từ: bởi vì, do
Even though + mệnh đề: dù cho, mặc dù
In spite of + danh từ/cụm danh từ: dù cho, mặc dù
Câu dùng cụm danh từ “his poor English” với ý mặc dù nên đáp án là D.
Tạm dịch: Mặc dù tiếng Anh kém, anh ta đã trao đổi vấn đề của mình một cách rõ ràng.
Question 11. Đáp án B
Kiến thức được hỏi: Câu điều kiện
Dùng câu điều kiện loại 2 với những việc không có thật ở hiện tại, phần cần điền là động từ ở vế If, phần
động từ ở vế chính đang là would take nên ta suy ra dùng câu điều kiện loại 2.
If+ S+ Ved, S+ would/ could + Vo.
→ Chọn đáp án B
Tạm dịch: Nếu tôi thấy một cái ví trên đường, tôi sẽ đem đến cho cảnh sát.
Question 12. Đáp án B
Kiến thức được hỏi: Giới từ
To be acquainted with: làm quen với, biết
To be responsible for: chịu trách nhiệm cho. Ta chọn đáp án: B - with/for
Tạm dịch: Tội cho rằng bạn đã làm quen với chủ đề này vì bạn chịu trách nhiệm soạn thảo tài liệu kèm
theo.
Question 13. Đáp án C
Kiến thức được hỏi: Loại từ
Vị trí chỗ trống cần một trạng từ để bổ sung ý nghĩa cho động từ phía sau, Adv+ P2
office (n): văn phòng
officer (n): nhân viên, viên chức
officially (adv): một cách chính thức official (adj): chính thức
Vì thế đáp án: C. officially
Tạm dịch: Bài hát đã chính thức được chọn cho World Cup 2018 được tổ chức tại Nga.
Question 14. Đáp án C
Kiến thức được hỏi: Lựa chọn từ.
obtain (v): đạt được, giành được, thu được
remain (v): nguyên vẹn, duy trì trạng thái như thế nào ( + adj)

Trang 10
maintain (v): giữ, duy trì
(v): đạt, tới, đạt tới.
Vậy phù hợp với nghĩa của câu nhất ta chọn đáp án C. maintain the ecosystem: duy trì hệ sinh thái, không
dùng B. remain vì sau đó cần tính từ.
Tạm dịch: Với duy trì hệ sinh thái mà chúng đang sống, và làm cho nhiều loài khác có thể sống sót trong
những môi trường đó.
Question 15. Đáp án A
Kiến thức được hỏi: Lựa chọn từ
A. reforestation (n): tái trồng rừng
B. rebuilding (n): tái xây dựng
C. replenishment (n): tái bổ sung
D. reconstruction (n): tái kiến thiết
Vậy phù hợp với nghĩa của câu nhất ta chọn đáp án A.
Tạm dịch: Sau trận hỏa hoạn, chính phủ cam kết thi hành một chương trình tái trồng rùng trên toàn vườn
quốc gia bị tàn phá.
Question 16. Đáp án B
Kiến thức được hỏi. Collocation - Cụm từ cố định
Ta có cụm: In recognition of + N: để công nhận, đáp án B.
Tạm dịch: Anh ấy được trao huân chương để công nhận sự phục vụ của anh ấy với đất nước.
Các phương án còn lại:
A. gratitude (n): lòng biết ơn
C. knowledge (n): kiến thức
D. in response to (n): phản hồi lại
Question 17. Đáp án C
Kiến thức được hỏi. Thành ngữ - cụm từ cố định
Ta có cụm: in one’s capable hands: trong quyền giải quyết, đáp án C.
Các phương án còn lại:
A. on your guards: cẩn thận đề phòng
B. up to your eyes: rất bận
D. in the care of you: dưới sự quan tâm của ai
Tạm dịch: Tôi sẽ đi công tác trong một tuần, vì vậy tôi sẽ để anh giải quyết mọi thứ.
Question 18. Đáp án A
Kiến thức được hỏi: Phrasal Verb - Ngữ động từ
Dựa vào nghĩa của câu ta chọn đáp án A (to) wipe out: tàn phá, càn quét, giết chết
Các phương án còn lại:
B. (to) lay off: sa thải

Trang 11
C. (to) put aside: để dành
D. (to) take down: tháo xuống, viết xuống
Tạm dịch: Các nhà khảo cổ học cho rằng những trận lũ lụt lớn có thể đã quét sạch loài khủng long.
Question 19. Đáp án D
Lỗi sai ở sự hài hòa chủ vị.
Chủ ngữ của câu là “the assumption”- chủ ngữ số ít, phần theo sau “that smoking has bad effects on our
health” là bổ ngữ kèm theo, vì thế vị ngữ phải là dạng số ít. have been proved -ì has been proved.
→ Ta chọn được đáp án D
Tạm dịch: Giả định rằng hút thuốc lá có những ảnh hưởng xấu đến sức khỏe của chúng ta đã được chứng
minh.
Question 20. Đáp án A
Lỗi sai ở: so many → so much
Do: “luggage” là danh từ không đếm được, vì thê' ta không thể dùng “many” mà phải dùng “much”. Chọn
đáp án A.
Tạm dịch: Cô ta có nhiều hành lý đến mức không có đủ chỗ trong xe cho nó.
Question 21. Đáp án A
Lỗi sai về dùng từ vựng. Phải sửa: Alike → Like Alike (adj): giống nhau.
Chú ý: Không dùng “Alike” để đứng trước một danh từ.
Ex: They look alike: Họ trông giống nhau.
Like+ N: giống như.... She looks like a princess.
Vậy đáp án là A.
Tạm dịch: Giống như sóng ánh sáng, sóng cực ngắn có thể phản chiếu và có tính tập trung.
Question 22. Đáp án A
Dịch câu đề bài: “Tôi đã không làm vỡ cái điện thoại di động đó”, Lan nói.
Dịch các phương án:
A. Lan phủ nhận việc đã làm vỡ cái điện thoại di động.
B. Lan thừa nhận việc đã làm vỡ cái điện thoại di động.
C. Lan ngăn chúng tôi không làm vỡ cái điện thoại di động.
D. Lan hứa sẽ làm vỡ cái điện thoại di động.
Đáp án A hợp với nghĩa câu gốc nhất. Đây là dạng câu chuyển từ câu trực tiếp sang gián tiếp, khi làm để
chọn đáp án đúng hãy nhìn kĩ các động từ chuyển thể của từng phương án cho phù hợp với nghĩa câu gốc
nhất.
Question 23.
Dịch câu đề bài: “ Chiếc túi quá nặng, nên chúng tôi không thể mang nó theo”.
Dịch các phương án:
A. Nếu chiếc túi không quá nặng, chúng tôi đã mang nó theo.

Trang 12
B. Câu viết sai ngữ pháp (Unless không dùng “not” phía sau.)
C. Câu viết sai ngữ pháp (Không viết If had mà phải là If the bag had not been heavy...)
D. Nếu chiếc túi không quá nặng, chúng tôi đã mang nó theo.
Tình huống đưa ra ở thì quá khứ, định hướng sẽ viết về câu điều kiện loại 3, không có thật trong quá khứ.
Loại A vì viết điều kiện loại 2; loại B, C do sai ngữ pháp.
Đáp án D là câu điều kiện loại 3 viết ở dạng đảo ngữ nên hoàn toàn chính xác.
Question 24. Đáp án B
Dịch câu đề bài: Anh ta là một người phá đám ở bữa tiệc tối nay!
Chú ý: wet blanket (n): người phá đám, người khiến người khác mất vui.
Dịch các câu phương án.
A. Anh ta khiến mọi người trong bữa tiệc bị ướt đẫm.
B. Anh ta làm hỏng niềm vui của người khác tại bữa tiệc.
C. Anh đã mua một tấm chăn ướt cho bữa tiệc.
D. Anh ta bị ướt khi đi vế nhà từ bữa tiệc.
Phù hợp với nghĩa câu gốc nhất là đáp án B.
Question 25. Đáp án D
Dịch câu đề bài: “ Anh ta bị nghi ngờ đã đánh cắp những chiếc thẻ. Cảnh sát đã thẩm vấn anh ta nhiều
ngày.”
Chú ý: be supected of Ving: bị nghi ngờ làm gì
Dịch các phương án:
A. Anh ta bị thẩm vấn nhiều ngày rồi, bị nghi ngờ đã đánh cắp những chiếc thẻ.
B. Nghi ngờ đã đánh cắp những chiếc thẻ, anh ta đã bị thẩm vấn trong nhiều ngày
C. Đã bị nghi ngờ đánh cắp những chiếc thẻ, anh ta bị thẩm vấn trong nhiều ngày.
D. Bị nghi ngờ đánh cắp những chiếc thẻ, anh ta bị thẩm vấn trong nhiều ngày.
Đây là dạng câu rút gọn hai câu có cùng chủ ngữ ở dạng bị động → loại câu B và C (dạng chủ động). Loại
câu A vì rút gọn sai vị trí (thường đặt động từ ở đầu câu).
(tham khảo kiến thức câu 9.) → Đáp án D đúng.
Question 26.
Dịch câu đề bài: “ Cô gái quấn cái lọ hoa bằng mút xốp mềm. Cô ấy không muốn nó bị vỡ trong lúc gửi
đi”.
Dịch các phương án:
A. Cô gái quấn cái lọ hoa bằng mút xốp mềm để mà nó sẽ không bị vỡ trong lúc gửi đi.
B. Cô gái quấn cái lọ hoa bằng mút xốp mềm do đó nó đã không bị vỡ trong lúc gửi đi.
C. Sai ngữ pháp, không có so as+ clause
D. Cô gái quấn cái lọ hoa bằng mút xốp mềm vì nó không bị vỡ trong lúc gửi đi.

Trang 13
Hai câu đề cho có quan hệ: Làm gì với mục đích gì/ để mà gì nên câu A sử dụng “so that”, (với mục đích
là, để mà) là hợp lý và chính xác nhất.
Các phương án khác bị loại do viết không thể hiện được ý chỉ mục đích giống câu gốc.
Question 27. Đáp án A
Dịch câu: David và Cathy đang nói chuyện về buổi tiệc tuần tới.
David. “ Tớ muốn mời cậu tới dự buổi tiệc vào chủ nhật tuần tới”.
Cathy. “_____________”.
Dịch các phương án:
A. Cảm ơn cậu. Buổi tiệc lúc mấy giờ?
B. Rất hân hạnh được gặp bạn. (chỉ dùng trong lần đầu gặp gỡ)
C. Bạn hoàn toàn được chào đón
D. Mình thấy vui vì bạn thích nó.
Giải thích. Để đáp lại một lời mời theo cách lịch sự thì đáp án A là phù hợp nhất.
Question 28. Đáp án A
Dịch câu. Tom và Mary gặp lại nhau vào buổi họp lớp.
Tom. “ Kiểu tóc của cậu trông thật tuyệt Mary à!”
Mary. “_____________”.
Dịch các phương án:
A. Cảm ơn cậu. Lời khen rất tuyệt.
B. Tại sao cậu lại nói như thế?
C. Xin lỗi. Tớ không thích nó.
D. Tớ cũng nghĩ vậy.
Đáp lại lời khen ngợi một cách lịch sự thì ta nên cảm ơn. Do đó: A là đáp án phù hợp nhất.
Question 29. Đáp án B
Dữ liệu của câu I strongly recommend: tôi thực lòng khuyên - bạn nên “take out an insurance policy in
the house” lưu ý key word insurance: bảo hiểm, nghĩa cả cụm: “mua bảo hiểm cho ngôi nhà”
Lời khuyên mua bảo hiểm cho ngôi nhà để mà?
Quan sát cụm được gạch chân for your own peace of mind, có peace: sự thanh thản, đoán được nghĩa của
cụm: yên tâm, yên lòng.
Dịch các phương án:
A. để dừng việc ngủ
B. để không phải lo lắng
C. để dừng việc suy nghĩ
D. để dừng việc tin tưởng
Vậy peace of mind: sự yên tâm, yên lòng = to stop your worrying. Đáp án B
Tạm dịch: “Tôi thực lòng khuyên bạn nên mua bảo hiểm cho ngôi nhà để cảm thấy yên tâm hơn”.

Trang 14
Question 30. Đáp án C
Dữ liệu của câu: Hàng năm, tổ chức từ thiện này takes on những người tình nguyện viên để giúp đỡ cho
những người khốn khó và người nghèo.
Suy nghĩ theo hướng: các tổ chức từ thiện muốn hoạt động được đều cần sự giúp sức của những người
tình nguyện viên. Nên so sánh với các phương án để chọn ra nghĩa hợp nhất: (hoặc bạn có thể thay từng
phương án lên trên để chọn)
Dịch các phương án:
A. đuổi việc
B. tạo ra
C. tuyển thêm, tiếp nhận
D. phỏng vấn
Nghĩa hợp nhất: recruit = take on (phrasal verb): tuyển dụng, nhận vào làm.
Đáp án: C
Tạm dịch: Hàng năm, tổ chức từ thiện này lại tiếp nhận thêm những người tình nguyện viên để giúp đỡ
người khốn khó và người nghèo.
Question 31.
Dữ liệu của câu: Tôi phải có một chiếc đồng hồ đeo tay bởi vì punctuality là rất cần thiết trong công việc
mới của tôi.
Ta có thể đoán được nghĩa của từ gạch chân thông qua dữ liệu “phải có một chiếc đồng hồ đeo tay” vì “sự
đúng giờ” là rất quan trọng.
Dịch các phương án:
A. đúng giờ
B. vui vẻ
C. hiệu quả
D. chậm trễ, trễ giờ
punctuality /ˌpʌŋktʃuˈæləti/ (n): sự đúng giờ trái nghĩa với D-being late: trễ giờ.
Tạm dịch: Tôi phải có một chiếc đồng hồ đeo tay bởi vì sự đúng giờ là rất cần thiết trong công việc mới
của tôi.
Question 32.
Dữ liệu của câu: Jose đã có một thời gian khó khăn khi so sánh iPhone với Samsung vì đối với anh ấy
chúng apples and oranges.
Ta có thành ngữ apples and oranges: hoàn toàn khác nhau, so sánh với các phương án:
A. containing too many technical details: chứa quá nhiều chi tiết kỹ thuật
B. very similar: rất tương đồng
C. completely different: hoàn toàn khác nhau
D. very complicated: rất phức tạp

Trang 15
Câu yêu cầu tìm từ trái nghĩa nên apples and oranges (hoàn toàn khác nhau) trái nghĩa với B-very similar
(tương đồng).
Tạm dịch: Jose đã có một thời gian khó khăn so sánh iPhone với Samsung vì đối với anh ấy chúng rất
khác nhau.
Question 33: Đáp án C
Ta có cụm từ: career opportunities: cơ hội việc làm/ nghề nghiệp. Đáp án C.
Trích bài: The number of people unemployed is still relatively high, and the number of new career
opportunities isn’t nearly enough to put people in jobs they are seeking.
Tạm dịch: Số lượng những người thất nghiệp vẫn còn tương đối cao và số lượng cơ hội việc làm mới thì
gần như là chưa đủ để có thể đưa người lao động vào làm những công việc mà họ đang tìm kiếm. Các
phương án còn lại:
A. responsibilities (n): trách nhiệm.
B. activities (n): hoạt động.
D. possibilities (n): khả năng, tiềm năng.
Question 34.
Who: Đại từ quan hệ ( ĐTQH) chỉ người, đứng chức năng làm chủ ngữ trong MĐQH. Chỗ trống cần điền
đứng sau danh từ chỉ người “workers” và đứng trước động từ “ have been out of the workforce” vì thế ta
cần một ĐTQH thay thế cho danh từ chỉ người làm chủ ngữ → who. Ta chọn đáp án A
Các phương án còn lại:
B. where: nơi mà (dùng sau các trạng từ chỉ nơi chốn).
C. whose: thể hiện ý sở hữu, sau whose + N.
D. which: ĐTQH chỉ vật, có thể làm chủ ngữ hoặc làm tân ngữ trong MĐQH.
Trích bài: As a job seeker, you’re competing with a high number of experienced workers who have been
out of the workforce for a while and are also seeking work.
Tạm dịch: Là một người tìm việc, bạn đang phải cạnh tranh với số lượng lớn những người khác đã có
kinh nghiệm, những người chỉ đang thất nghiệp tạm thời và cũng muốn tìm cho mình một công việc.
Question 35. Đáp án D
Đây là một câu hỏi về liên từ, chọn liên từ đúng liên kết ý câu trước và câu sau của bài.
Ta chọn đáp án D-However: Tuy nhiên, mặc ; dù vậy, liên từ chỉ sự đối lập.
Do câu phía trên đang nêu ra các khó khăn khi phải cạnh tranh với nhiều người có kinh nghiệm và cũng
đang tìm kiếm việc làm. Tuy nhiên ý câu dưới là khi bạn học đại học thì vẫn có thể có được việc làm tốt.
Các phương án còn lại:
A. Otherwise: Nếu không thì, liên từ chỉ điểu kiện giả định.
B. Moreover: Hơn nữa, ngoài ra, liên từ dùng để thêm thông tin.
C. Therefore: Do đó, liên từ chỉ kết quả.

Trang 16
Trích bài: As a job seeker, you’re competing with a high number of experienced workers who have been
out of the workforce for a while and are also seeking work. However, when you have a higher education,
it generally equips you for better job security.
Tạm dịch: Là một người tìm việc, bạn đang phải cạnh tranh với số lượng lớn những người khác đã có
kinh nghiệm, những người chỉ đang thất nghiệp tạm thời và cũng muốn tìm cho mình 1 công việc.Tuy
nhiên, khi bạn học đại học thì đó chính là hành trang để bạn kiếm được 1 công việc tốt.
Question 36. Đáp án D
Đây là một câu hỏi về từ vựng, lựa chọn từ đúng. Ta dùng cụm: received sth (v): nhận được cái gì đó.
Đáp án D
Trong bài: received a higher education (đây là một colllocation): nhận được sự giáo dục đại học/ học đại
học.
Các phương án còn lại:
A. permitted (v) sb to V: cho phép ai làm gì.
B. refused (v) to V: từ chối làm gì.
C. applied (v): ứng dụng, ứng tuyển,...
Trích bài: Generally speaking, employers tend to value those who have completed college than those who
have only completed high school and are more likely to replace that person who hasn’t received a higher
education.
Tạm dịch: Nói chung thì các ông chủ thường có xu hướng trân trọng những người đã học xong đại học
hơn là những người mới chỉ học xong cấp ba và sẽ có thể thay thế người mà chưa học đại học.
Question 37.
Đây là câu hỏi về loại từ, chúng ta lựa chọn loại từ đúng vào chỗ trống khi mà 4 phương án đều xoay
quanh một nghĩa gốc.
Nhận thấy rằng chỗ trống đang đứng sau tính từ “educated” và ta có luôn có cụm a/ an+adj+ N (noun)
nên khẳng định được chỗ trống cần N số ít đếm được. Vậy loại A - employment (n): việc làm, sự thuê làm
(N không đếm được) và D - unemployed (adj): thất nghiệp.
Có 2 danh từ số ít đếm được là employer: ông chủ/ người sử dụng lao động hoặc employee: người lao
động
Nhưng phù hợp nhất về văn cảnh phải là educated employee : người lao động có học vấn. Đáp án C.
Trích bài: Furthermore, some companies even go so far as to pay your tuition because they consider an
educated employee to be valuable to their organization.
Tạm dịch: Ngoài ra, một vài công ty còn chịu trả thêm phí học tập cho bạn bởi họ coi những người lao
động có học vấn cao sẽ rất có ích cho tổ chức của họ.
Các phương án còn lại:
READING 1

Trang 17
Đoạn 1: Mở bài: Giới thiệu 2 nguồn năng lượng có thể tái tạo được rất tiềm năng là năng lượng mặt trời
và năng lượng địa nhiệt.
Đoạn 2 và 3: Thân bài: phần tích kĩ hơn về cách khai thác và ích lợi từ 2 nguồn năng lượng này:
+ Đoạn 2: năng lượng mặt trời
+ Đoạn 3. năng lượng địa nhiệt
Đoạn 4: Kết bài: Nhấn mạnh sự hữu ích và khuyến khích nên đầu tư phát triển và khai thác các nguồn
năng lượng sạch.
Question 38. Đáp án B
Câu hỏi: Nội dung/ Chủ để chính của bài là gì?
A. Những ích lợi vượt trội hơn của năng lượng mặt trời và năng lượng gió so với các nguồn năng lượng
truyền thống.
B. Hai nguồn năng lượng sạch nên được khai thác nhiều hơn nữa.
C. Cách mà những nguồn năng lượng được lấy ra từ tự nhiên.
D. Các ví dụ về cách sử dụng các nguồn năng lượng trên khắp thế giới.
Nhìn vào cấu trúc bài viết, nội dung bài xoay quanh việc phân tích ích lợi khi sử dụng 2 nguồn năng
lượng sạch và cách thức khai thác chúng. Đáp án B.
Không chọn A do trong bài không nêu rõ việc so sánh này.
Không chọn C do đây chỉ là một ý của bài mà thôi.
Không chọn D do bài chỉ nêu ra ít ví dụ về khai thác năng lượng mặt trời và địa nhiệt.
Chú ý: Với câu hỏi chủ đề chính của bài, đáp án đúng phải nếu được nội dung xuyên suốt của toàn
bài, các phương án chỉ nêu một ý nhỏ hay một ý nào đó xuất hiện ở từng đoạn đơn lẻ không phải là
đáp án đúng. Có thể để lại câu hỏi này làm sau cùng, khi đã đọc bài nhiều lần và hiểu rõ nội dung của
bài.
Question 39. Đáp án A
Câu hỏi: Từ nào đồng nghĩa/ có thể thay thế tốt nhất từ “abundant”?
A. a lot: nhiều
B. scarce: hiếm
C. little: quá it
D. enough: đủ
Đáp án A. abundant: dồi dào, phong phú = a lot: nhiều.
Chú ý: đây là câu hỏi từ vựng, nếu biết nghĩa của từ đang được hỏi sẽ rất thuận lợi, nhưng dù có biết hay
không thì luôn phải đặt từ cần tìm vào câu và ngữ cảnh của nó để dùng thông tin luận ra nghĩa.
“Solar energy, which reaches the earth through sunlight, is so abundant that it could meet the needs of
worldwide energy consumption 6,000 times over.”
Năng lượng mặt trời, nguồn năng lượng mà tới trái đất thông qua ánh nắng, là nguồn năng lượng cực dồi
dào đến mức mà nó có thể đáp ứng được nhu cầu sử dụng năng lượng của toàn thế giới gấp tận 6000 lần.

Trang 18
Tips: Cũng có thể thay lần lượt từng phương án lên thay cho từ cần tìm nghĩa để phán đoán.
Question 40. Đáp án A
Câu hỏi. Từ “it” nằm ở đoạn 2 thay thế cho từ nào?
Các phương án khác.
A. năng lượng mặt trời
B. trái đất
C. ánh nắng
D. mức tiêu thụ năng lượng.
Đây là câu hỏi suy luận về đại từ dùng để thay thế cho danh từ đã xuất hiện phía trước nhưng không
muốn nhắc lại danh từ đó. Ta xem lại câu chứa từ “it”:
Solar energy, which reaches the earth through sunlight, is so abundant that it could meet the needs of
worldwide energy consumption 6,000 times over. Năng lượng mặt trời, nguồn năng lượng mà tới trái đất
thông qua ánh nắng, là nguồn năng lượng cực dồi dào đến mức mà nó có thể đáp ứng được nhu cầu sử
dụng năng lượng của toàn thế giới gấp tận 6000 lần.
“Nó” ở đây là năng lượng mặt trời, đáp án A.
Question 41. Đáp án C
Câu hỏi: Theo như bài viết, cách tạo ra năng lượng mặt trời giống với cách tạo ra năng lượng địa nhiệt
như thế nào?
A. Chúng đều cần sử dụng máy phát điện.
B. Chúng đều sử dụng nhiệt từ bề mặt trái đất.
C. Chúng đều cần công nghệ khá đơn giản
D. Chúng đều là năng lượng thông thường và khá đắt đỏ.
Thông tin chúng ta tìm ở đoạn 3 dòng 5 và 6. “... and as with solar power, the technology needed to
utilize geothermal energy is fairly simple.”: và cũng như năng lượng mặt trời, công nghệ cần có để khai
thác năng lượng địa nhiệt là khá đơn giản, phù hợp nhất với đáp án C.
Các phương án khác không đúng theo như ý của bài.
Question 42. Đáp án B
Câu hỏi. Câu nào miêu tả mục đích của tác giả trong bài viết này?
A. Để cảnh báo con người về mối nguy hại của việc sử dụng năng lượng hóa thạch.
B. Để thuyết phục con người về những lợi ích của các nguồn năng lượng thay thế đang phát triển.
C. Để nói vể ưu điểm và nhược điểm của việc sử dụng nguồn năng lượng thay thế.
D. Để kể ra các vấn đề và các giải pháp liên quan đến việc nóng lên của toàn cầu.
Rõ ràng ta thấy bài văn tập trung nói về ích lợi của 2 nguồn năng lượng sạch vừa dồi dào, bất tận vừa thân
thiện môi trường. Phần kết của bài tác giả cũng khẳng định lại những ưu điểm của năng lượng sạch vượt
trội hơn so với năng lượng thông thường và nhấn mạnh chúng ta cần đầu tư hơn nữa vào sự phát triển
những nguồn năng lượng này trên toàn cầu → Đáp án B.

Trang 19
READING 2
Đoạn 1. Các cách để chọn ra người lãnh đạo
+ Trong gia đình, bạn bè không thông qua tuyển chọn chính thức.
+ Trong nhóm lớn, tổ chức: tuyển chọn chính thức.
Đoạn 2. Không có mẫu lãnh đạo chung cho mọi nhóm.
Đoạn 3,4. Phân tích, so sánh kĩ vai trò và cách thức lãnh đạo của 2 loại, “expressive leader” và
“instrumental leader”
+ “expressive leader”: kiểu lãnh đạo coi trọng tình cảm, sự hòa thuận trong nhóm, động viên khuyến
khích các thành viên tiến lên và nỗ lực giành chiến thắng.
+ “instrumental leader: kiểu lãnh đạo coi trọng kỉ luật, nội quy, dùng các phương pháp, kỉ luật để ép các
thành viên đạt được mục tiêu chung của nhóm.
Question 43. Đáp án B
Câu hỏi: Bài viết tập trung thảo luận vấn đề gì?
A. Cách thức mà sự lãnh đạo khác nhau trong các nhóm nhỏ và nhóm lớn.
B. Vai trò của người lãnh đạo trong các nhóm xã hội.
C. Các vấn đề mà các nhà lãnh đạo phải đối mặt.
D. Cách thức mà các nhóm xã hội tìm ra ai là người lãnh đạo họ.
Nội dung cả bài xoay quanh thảo luận về người lãnh đạo trong nhóm, vai trò và cách thức lãnh đạo. Đáp
án đúng là B.
Loại A và D vì chỉ là ý nhỏ của đoạn 1; loại C do không đề cập nhiều trong bài
Question 44. Đáp án B
Câu hỏi: Bài viết đề cập tất cả các cách sau để người ta có thể trở thành người lãnh đạo
TRỪ____________
A. tuyển dụng
B. sự huấn luyện lãnh đạo riêng biệt
C. kiểu mẫu văn hóa truyền thống
D. tiến trình bầu cử trang trọng
Đây là câu hỏi có chứa từ phủ định. Thông tin tìm ở đoạn 1, dùng phương pháp loại trừ, cách thức nào
không được nhắc trong bài sẽ là đáp án chọn. Loại các phương án A, C, D do chúng lần lượt xuất hiện
trong các câu:
In the family, traditional cultural patterns (phương án C) confer leadership on one or both of the parents.
In larger groups, leaders are usually chosen formally through election (phương án D) or recruitment
(phương án A). Trong gia đình, cách thức lãnh đạo truyền thống chọn lãnh đạo có thể là bố, mẹ hoặc cả
hai. Trong các nhóm lớn hơn, những người lãnh đạo thường được chọn một cách chính thống thông qua
bầu cử hoặc tuyển dụng.
Question 45. Đáp án C

Trang 20
Câu hỏi: Phát biểu nào dưới đây về sự lãnh đạo mà có thể được suy ra từ đoạn 2:
A. Một người có thể học làm người lãnh đạo hiệu quả tốt nhất bằng việc học các nghiên cứu về lĩnh vực
lãnh đạo.
B. Hầu hết mọi người đều khát khao trở thành lãnh đạo nhưng có quá ít bằng chứng về năng lực.
C. Một người có thể là lãnh đạo tốt của nhóm này nhưng chưa chắc đã là lãnh đạo hiệu quả ở một nhóm
khác.
D. Có quá ít người thành công trong việc chia sẻ sự lãnh đạo với một người khác.
Đoạn 2 nhấn mạnh không có những phẩm chất chung cho tất cả các nhà lãnh đạo của các nhóm “there is
no set of personal qualities that all leaders have in common”; Mỗi nhóm khác nhau lại có yêu cầu riêng
và người lãnh đạo của nhóm ấy phải hội tụ đủ các phẩm chất riêng biệt ấy để dẫn dắt nhóm của mình
“any person may be recognized as a leader if the person has qualities that meet the needs of that particular
group”
Xâu chuỗi các ý lại ta có thể suy ra phương án C là phù hợp nhất với thông tin ở đoạn 2.
Question 46. Đáp án B
Câu hỏi: Bài viết chỉ ra rằng người lãnh đạo “instrumental leaders” thường tập trung vào_____________
A. Chia sẻ trách nhiệm với các thành viên khác trong nhóm.
B. Đạt được mục tiêu.
C. Đảm bảo một mối quan hệ hòa thuận giữa các thành viên trong nhóm.
D. Xác định ra những người lãnh đạo mới.
Thông tin có thể được tìm thấy rải rác ở đoạn 3 và 4 nhưng có thể thấy rõ ở đoạn 4 những dòng đầu
“Instrumental leaders are likely to have a rather secondary relationship to other group members. They
give orders and may discipline group members who inhibit attainment of the group’s goals.”
Người lãnh đạo “instrumental leaders” có tình cảm xa cách hơn với các thành viên khác trong nhóm. Họ
đưa ra mệnh lệnh và nhiều kỉ luật với các thành viên để có thể đạt được mục tiêu chung của nhóm.
→ Chọn đáp án B
Question 47. Đáp án B
Câu hỏi: Từ “collective” ở đoạn 3 gần nghĩa nhất với từ nào?
A. cần thiết B. nhóm
C. cụ thể D. điển hình
Câu hỏi từ vựng: đoán nghĩa của từ, ta đặt từ vào văn cảnh và câu chứa nó để đoán nghĩa. Expressive
leadership, on the other hand, is leadership that emphasizes the collective well-being of a social group’s
member. Người lãnh đạo kiểu “Expressive” mặt khác lại tập trung vào sự hạnh phúc của mỗi thành viên
trong cả nhóm → Chọn đáp án B
Question 48. Đáp án D
Câu hỏi: Có thế hiếu rằng_____________
A. Có rất nhiều căng thẳng và mâu thuẫn trong sự lựa chọn người lãnh đạo trong gia đình.

Trang 21
B. Thường có sự tuyển chọn người lãnh đạo trong gia đình cũng như trong các nhóm lớn hơn.
C. Người ta thường nói rằng phải có một tiêu chuẩn các phẩm chất năng lực chung của những người lãnh
đạo.
D. Người lãnh đạo được chọn ra một cách trang trọng hoặc không trang trọng.
Đáp án D là đúng vì chúng ta có 2 cách để chọn ra người lãnh đạo, trang trọng ở các nhóm, tổ chức lớn;
không trang trọng ở gia đình và nhóm bạn bè.
A. Bài không nhắc tới.
B. Sai, do gia đình không cần thông qua “election: tuyển chọn” người lãnh đạo.
C. Đoạn 2 khẳng định không có tiêu chuẩn chung cho mọi nhà lãnh đạo.
Dùng phương pháp loại trừ ta chọn được đáp án D.
Question 49. Đáp án B
Câu hỏi: Từ “resolve” ở đoạn 4 đồng nghĩa với từ nào?
A. Nói về
B. Tìm ra cách giải quyết cho
C. Tránh sự lặp lại
D. Tránh nghĩ về
Câu hỏi về từ gần nghĩa nhất, đặt vào câu văn chứa từ resolve ta có.
They offer sympathy when someone experiences difficulties or is subjected to discipline, are quick to
lighten a serious moment with humor, and try to resolve issues that threaten to divide the group.
Họ thường có sự thông cảm với những thành viên đang gặp khó khăn hoặc những người đang phải kỉ luật,
họ nhanh chóng làm giảm nhẹ đi những tình huống căng thẳng với sự hài hước và cố gắng tìm cách giải
quyết cho những vấn đề đe dọa gây chia rẽ nhóm.
→ Chọn đáp án B
Question 50. Đáp án C
Câu hỏi. Đoạn 3 và 4 tổ chức thảo luận về sự lãnh đạo theo khía cạnh nào?
A. Các ví dụ để minh họa cho vấn đề
B. Sự trần thuật các sự kiện
C. Sự so sánh và đối lập
D. Phân tích nguyên nhân và ảnh hưởng.
Đoạn 3 và 4 là sự so sánh cũng như đối chiếu về hai phong cách lãnh đạo theo kiểu “expressive” hay
“instrumental leadership”.
Đọc đoạn 3,4 ta luôn tìm thấy sự so sánh, đối lập liên tiếp khi thể hiện 2 phong cách lãnh đạo này:
Instrumental leadership is leadership that emphasizes the completion of tasks by a social group. Group
members look to instrumental leaders to “get things” done. Expressive leadership, on the other hand, is
leadership that emphasizes the collective well-being of a social group’s member.

Trang 22
Tạm dịch: Lãnh đạo theo kiểu Instrumental là kiểu lãnh đạo tập trung vào việc hoàn thành các nhiệm vụ
của nhóm. Các thành viên dựa vào người lãnh đạo để đảm bảo các công việc được hoàn thành. Lãnh đạo
kiểu Expressive thì ngược lại là kiểu lãnh đạo tập trung vào sự hạnh phúc của cả nhóm.
Instrumental leaders are likely to have a rather secondary relationship to other group members. They give
orders and may discipline group members who inhibit attainment of the group’s goals. Expressive leaders
cultivate a more personal or primary relationship to others in the group. They offer sympathy when
someone experiences difficulties or is subjected to discipline, are quick to lighten a serious moment with
humor, and try to resolve issues that threaten to divide the group.
Tạm dịch: Người lãnh đạo “instrumental leaders” có tình cảm xa cách hơn với các thành viên khác trong
nhóm. Họ đưa ra mệnh lệnh và nhiều kỉ luật với các thành viên để có thể đạt được mục tiêu chung của
nhóm. Lãnh đạo “expressive” lại có tình cảm thân thiết, gần gũi hơn. Họ thường có sự thông cảm với
những thành viên đang gặp khó khăn hoặc những người đang phải kỉ luật, họ nhanh chóng làm giảm nhẹ
đi những tình huống căng thẳng với sự hài hước và cố gắng tìm cách giải quyết cho những vấn đề đe dọa
gây chia rẽ nhóm.
Vì vậy C là đúng nhất.

Trang 23

You might also like